Você está na página 1de 366

ELEMENTOS DA

ME C ANICA1
DINÂMICA | ENERGIA
QUANTIDADE DE MOVIMENTO
Elementos da Física

Mecânica I

Marcelo Rufino de Oliveira

1a Edição
2018

Belém-PA

Editora Ximango
Copyright © 2018 by marcelo rufino de oliveira

Todos os direitos desta edição estão reservados


à Marcelo Rufino de Oliveira
Belém — Pará — Brasil
E-mail: marcelorufino@hotmail.com

RONALDO CORRÊA
Ilustração da Capa

LOUDES PACHECO
Ficha Catalográfica

GRÁFICA PRINT SOLUTION


Impressão

XIMANGO EDUCACIONAL
Editora

O48a

Elementos da Física: Mecânica I / Oliveira, Marcelo Rufino, -


1 ed. - Belém-PA; 2017; 300 p.

ISBN:

1. Física. 2. Mecânica 3. Dinâmica. 4. Energia. 5. Quantidade de


Movimento. Título.

CDD: 530
índice

1. Cálculo 1
Limite 1
Derivada 3
Integral 8

2. Princípios da Dinâmica 14
Mecânica Clássica .... 14
1a Lei de Newton 14
O Conceito de Massa 15
O Conceito de Força . . . 16
Força Resultante 17
Decomposição de Forças 17
2a Lei de Newton 18
Força Peso . 19
Dinamômetro 19
3a Lei de Newton 20
Força de Reação Normal .... 22
Aplicações das Leis de Newton 23
Vínculos Geométricos 28
Exemplos........................ 30
Referenciais não Inerciais 45
Exemplos 47
Exercícios 51

3. Força de Atrito 74
Reação Normal e Força de Atrito 74
Atrito Estático 74
Atrito Cinético 75
Aceleração sobre um Plano Inclinado Áspero 78
Força de Atrito em Duas Dimensões 80
Exemplos 81
Exercícios 92
4. Força Centrípeta..................................... 115
Forças no Movimento Circular Uniforme 115
Forças no Movimento Circular Acelerado 117
Movimento Pendular................................. 118
Forças no Movimento Não Retilíneo 120
Outras Expressões.......................... 120
Força Centrífuga.............................. 121
Exemplos.......................................... 122
Exercícios.......................................... 133

5. Trabalho e Potência 151


Energia............................................ 151
Trabalho de uma Força Constante 151
Trabalho de Forças Variáveis . . . . 143
Trabalho Total............................... 154
Trabalho da Força Peso.................... 155
Exemplos . 156
Trabalho da Força Elástica............. 158
Trabalho da Força de Atrito Cinético 160
Exemplos......................................... 162
Exercícios............................................ 179

6. Energia Mecânica................................................ 196


Energia Potencial de um Sistema...................... 196
Energia Potencial Gravitacional.......................... 196
Energia Potencial Elástica................................. 197
Forças Conservativas e Não Conservativas . . . 198
Forças Conservativas e Energia Potencial .... 199
Energia Mecânica................................................ 199
Conservação da Energia Mecânica................. 200
Energia Mecânica e Forças Não Conservativas 201
Exemplos............................................................. 202
Exercícios............................................................. 216
7. Impulso e Quantidade de Movimento 238
Impulso..................................................... 238
Força Média............................................ 238
Centro de Massa.................................... 238
Corpo Rígido........... 242
Sistema de Partículas 245
O Movimento do Centro de Massa 246
Conservação da Quantidade de Movimento 247
Sistema de Massa Variável........................ 251
Exemplos 253
Exercícios 266

8. Colisão Mecânica 288


Conceitos Fundamentais 288
Colisão Central Direta 289
Coeficiente de Restituição 292
Tipos de Colisão............. 293
Colisão Central Oblíqua . . 297
Exemplos.......................... 299
Exercícios........................... 312

9. Dinâmica da Rotação......... 341


Energia Cinética de Rotação 341
Teorema dos Eixos Paralelos 342
Torque................................................... 343
Trabalho, Energia Cinética e Potência 345

10. Gabaritos 347



Este livro é voltado para alunos de ensino médio, fundamentalmente os que esteja
estudando para concursos militares, como ITA, IME, Escola Naval, AFA, EsPCEx e EFOMM.
Entende-se também que os conceitos aqui apresentados podem ser importantes para os alunos
que estejam se preparando para participar de olimpíadas científicas de física de ensino médio.
Apesar de ser voltada para o ensino médio, para entender melhor esta obra é necessário ter
domínio dos conceitos mais básicos de cálculo, fundamentalmente da derivada e integral. Estes
conceitos serão apresentados neste primeiro capítulo, restringindo ao estudo das funções de uma
variável.
Cabe ressaltar que para um estudo mais rigoroso o aluno deve consultar livros de nível
superior de cálculo ou análise matemática. Neste primeiro capítulo livro serão estudados alguns
artifícios que serão muito úteis na determinação de derivadas e integrais em problemas apenas de
interesse físico. Assim, não serão trabalhados neste capítulo alguns tópicos de cálculo que não
possuem aplicação na física contida neste livro.

LIMITE

O limite de uma função f(x), quando existe, para um determinado argumento x que tende a
x0, é igual a L se a função f(x) se aproxima de L quando x se aproxima de Xq. É importante frisar
que a função f(x) não precisa estar definida em x0. Simbolicamente escreve-se:

lim f(x) = L
X-»Xq

Caso f(x) esteja definida em Xo e seu gráfico não apresenta descontinuidades nem
oscilações muito fortes segue que o limite da função quando x tende a Xo é igual a f(x0):

lim f(x) = f(x0)


x->x0

Por exemplo, sabe-se que a função f(x) = 2X2 - x + 1 é contínua para todo x e IR. Assim,
pode-se afirmar que

Iim2x2 -x + 1 = f(2) = 2.22-2 + 1 = 7


x->2

Situações como esta, em que a função é contínua no ponto em que se deseja calcular o
limite, não costumam apresentar dificuldade. O problema reside nas situações em que a função
não é definida no ponto em que se deseja calcular o limite. Isto pode parecer, em princípio, uma
situação distante do interesse da física, mas é exatamente o contrário. Por exemplo, se x(t) é a
função horária do espaço, a definição de velocidade no instante to é

x(t)-x(t0)
v(t0)= lim
t->tg t-to

x(t)-x(t0)
Perceba que a função f(t) = não é definida para t = to. Isto não impede a
t-to
determinação de v(t0), todavia, seu cálculo não é uma mera substituição de valores. Além disso,
as operações algébricas necessárias para calcular este tipo de limite dependem da natureza da
função, ou seja, se é polinomial, exponencial, logarítmica, trigonométrica ou mesmo se envolve
termos de natureza distinta.
Por exemplo, suponha que uma partícula se movimente no espaço de acordo com a
equação horária do espaço x(t) = t3, onde todas unidades são medidas no SI. Qual a velocidade
no instante t = 1 s? Pela definição de velocidade instantânea segue que:

1
Elementos da física - Mecânica I - Cálculo
X(t)-x(1) t3 -1
v(1) = lim = lim------ -
t-1 t-»i t-1

t3 — 1
Claramente a função f(t) = ——— não é definida para t = 1. Porém, como já foi explicado, o
mais importante na determinação de um limite não é valor da função no ponto especificado e sim
como a função se comporta nas vizinhanças de t = 1. Para tanto, observe a tabela abaixo.

x f(x)
0,97 2,9109
0,98 2,9404
0,99 2,9701
1,01 3,0301
1,02 3,0604
1,03 3,0909

t3 — 1
Percebe-se claramente que, apesar da função f(t) = ——y- não estar definida para t = 1, a
medida que nos aproximamos de t = 1 a função se aproxima, tanto pela esquerda quanto pela
t3 -1
direita, de 3. Desta forma, pode-se suspeitar que o limite f(t) =-------, quando t tende a 1, vale 3,
t-1
porém a tabela não pode ser encarada como uma demonstração. Para lidar com situações como
essa, criou-se uma definição de limite onde o que acontece exatamente no ponto em que se
deseja calcular o limite não é importante, importando apenas o que ocorre nas vizinhanças desse
ponto. Isso permite o cancelamento dos fatores comuns no numerador e no denominador.

... .. t3-1 .. fP-^íftt2+t + 1)


v 1) = hm-------= hm ———-------- Hm t2 +1 +1 = 12 +1 +1 = 3 m/s
t-i t-1 t-i txí

Considere agora que uma outra partícula se movimenta de modo que sua função horária
do espaço seja x(t) = Vt, com todas as unidades medidas no SI. A velocidade no instante t = 16 s
pode ser determinada como segue abaixo:

v(16) = lim
x(t)-x(16) Vt-2
..hm-------- lim------- ------------------------ = |jm 1
t-16 t-16 t-16 t-16 t-16 )(^/t + 2)(Vt + 4) t-16 (4/t + 2)(Vt + 4)
1
v(16) = -j=---- !-=-------=--------- 11
-------- = — m/s
(VÍ6 +2)(VÍ6 +4) (2 + 2)(4 + 4) 32

Você agora deve estar se perguntando se toda vez que quiser determinar uma velocidade
vai ter que resolver um limite? A resposta é: não necessariamente. A definição apresentada para
velocidade através de limite está totalmente correta, porém é possível usar um outro artifício
x(t) — x(t )
matemático para calcular limites do tipo lim ———492 mais rapidamente do que ter que executar
t—>to t —10
todas as fatorações necessárias. Este artifício recebe o nome de derivada a será explanado no
próximo item.

2
Elementos da Física - Mecânica I - Cálculo
DERIVADA

A derivada de uma função f(x) em um certo ponto Xo é definida como

df f(x)-f(x0)
lim
dx x=x0
X->Xo x-x0

Outra maneira de escrever a derivada de uma função é trabalhar com o acréscimo


infinitesimal Ax = x - x0. Deste modo, pode-se reescrever a derivada de uma função f(x) em um
certo ponto x0 como

df = |im f(x0+Ax)-f(x0)
dx AX->0 Ax

Outra maneira de simbolizar a derivada de uma função de uma variável no ponto x0 é f'(Xo).
Vamos agora aplicar a definição de derivada por limite para calcular a derivada de algumas
funções conhecidas.

Derivada da função constante


Se f(x) = k então a derivada de f(x) em um ponto genérico x0 vale:

_ 0+Ax)-f(x
||m f(x k-k _ Qn
0) _ |..|m --------
----- x_oz
f'(x0)
Ax-»0 Ax Ax—>0 Ax

Derivada da função polinomial


Se f(x) = A.xn então a derivada de f(x) em um ponto genérico x0 vale:

n n
A Xo 1Ax + xS'2Ax2 + ... +Axn’1
.. i A(x0 +Ax)n -Axg 1 2
f'(x0)= lim = lim —
Ax-»O Ax Ax—>0 Ax
n n
A^óí x"0-1 + xjj 2Ax + ... +Axn
1 2 n n
= lim A Xo xJJ~2Ax + ... +Axn~2
Ax—>0 1 2
f'(xo) = AnXg

Obs: Este resultado é válido para qualquer n real, não apenas para n natural

Derivada da função sen x


Se f(x) = sen x então a derivada de f(x) em um ponto genérico x0 vale:

|jm A(x0 + Ax)n - AXp = |im sen(x0 + Ax)-senx0 = |.m senx0.cosAx + senAx.cosx0 -senx0
f'(x0)
AX->0 Ax Ax->0 Ax Ax—>0 AX

Para Ax pequeno sabe-se que sen Ax » Ax e cos Ax ® 1. Substituindo obtém-se:

f'(Xo) = COS Xo

Derivada da função cos x


Se f(x) = cos x então a derivada de f(x) em um ponto genérico Xo vale:

X I- A(x0+Ax) -Ax£ = |jm COS(X0 4-Ax) — COSXq = cos x0. cos Ax - sen Ax. sen x0 - cos x0
f (xn)= lim 0 —--------- 1
U Ax—>o ax AX->0 AX Ax—>0 Ax

3
Elementos da Física - Mecânica I - Cálculo
Para Ax pequeno sabe-se que sen Ax « Ax e cos Ax « 1. Substituindo obtém-se:

f'(x0) = - sen x0

Poderiamos continuar determinando as derivadas de outras funções elementares, porém


este não é o objetivo deste livro. Abaixo está uma tabela com as derivadas das funções mais
utilizadas na física. O leitor pode ficar à vontade para tentar fazer as respectivas demonstrações
por meio da definição de derivada.

f(x) f’(x)
k ________0
Axn Anxn~1
sen x cos x
cos x - sen x
ex ex
ax _____ ax.ln a_____
In x ~.... ~ 2
___________ X___________
1
loga x
______ x.lna______
tg x sec2 x
sec x sec x. tg x
cotg x - cossec2 x
cossec x - cossec x. cotg x

Propriedades das Derivadas

i) Se f(x) = k.g(x) então f'(x) = k.g '(X)


Demonstração:
Se f(x) = k.g(x):
f(x)-f(x0) |..|m k.g(x)-k.g(x
--------- 0)
U/ ..
_ ||m k . g(x)-g(x0) k. Iim 9W-9(x°.L k.g'(x)
f'(x) = lim
X->Xq x-x0 X->X0 X - Xo X—>Xq x-x0 x->x0 X - xo

ii) Se f(x) = g(x) + h(x) então f'(x) = g '(X) + h '(X)


Demonstração:
Se f(x) = g(x) + h(x):
HimfW^.) = |jm g(x)+h(x)-g(x0)-h(x0) g(x)-g(x0) + h(x)-h(x0)
f'(x) lim
x->x0 x - xo x-»x0 X - Xo x->x0 x — x0 x - x0
g(x)-g(x0) h(x)-h(x0)
lim + lim g'(x) + h’(x)
x—>x0 x-x0 x->x0 X-Xo

iii) Se f(x) = g(x).h(x) então f'(x) = g ’(x).h(x) + g(x). h '(x)


Demonstração:
Se f(x) = g(x).h(x):
f(x)= lim fW-f(xo). |im g(x)h(x)-g(x0)h(x0) = |jm g(x)h(x) - g(x0 )h(x0) - h(x)g(x0) + h(x)g(x0) =
x->xo X - Xo x—>x0 x - xo X->XO x - xo

= |im [g(x) - g(x0 )]h(x) 4- g(x0 )[h(x) - h(x0)] g(x)-g(x0) h(x)-h(x0)


lim h(x) + g(x0)
x-»Xo X - Xo x->«o x-x0 x — x0

g(x)-g(x0) h(x)-h(x0) g(x)-g(x0) h(x)-h(x0) =


lim h(x) + lim g(x0) h(x0) lim + g(x0) lim
X->Xq x-x0 X->Xq x-x0 x-»x0 x-x0 X->Xq x-x0 J
h(x0)g'(x0) + g(x0)h'(x0)

4
_______ __________________________________________________ Elementos da Física - Mecânica I - Cálculo
Como Xo pode assumir qualquer valor real (dentro dos domínios de definição das funções f, g e h),
segue que f'(x) = g '(x).h(x) + g(x). h '(x)

g'(x).h(x) - g(x).h'(x)
iv) Se = então f'(x) =
Ih(x)]2
Demonstração:
Se f(x) = ^:
h(x)
g(x) g(x0)
)im f(x)-f(x0) = )jm h(x) h(x0) = |jm g(x)h(x0)-g(x0)h(x)
f'(x)
x-»x0 x-xo x->x0 X-Xo x->Xo h(x)h(x0)(x-x0)
= |im g(x)h(x0)- g(x0 )h(x) + g(x0 )h(x0)- g(x0)h(x0) =
x->x0 h(x)h(x0)(x-x0)
[g(x) - g(x0 )]h(x0) - g(x0 )[h(x) - h(x0)] g(x)-g(x0) h(x)-h(x0)
= lim lim h(x0)- g(x0) =
X->Xq h(x)h(x0)(x-x0) x->x0|_h(x)h(x0)(x-x0) h(x)h(x0)(x-x0)

g(x)-g(x0) h(x)-h(x0)
= lim h(x0) - lim g(x0)
x-*0Lh(x)h(x0)(x-x0) x-»x0[h(x)h(x0)(x-x0)

h(x0) g(x)-g(x0) -«-lim -h(x)-h(x0) g'(Xo).h(xo)-g(xo).h'(xo)


=----- — lim ■
h(x0)h(x0)x-x0 _ x-x0 h(x0)h(x0) x->x0 x-x0 [h(x0)]2
Como Xo pode assumir qualquer valor real (dentro dos domínios de definição das funções f, g e h),
. g'(x).h(x)-g(x).h’(x)
segue que f (x) = ■— -—~-——-
[h(x)]2

v) Se f(x) = h(g(x)) então f’(x) = h'(g(x)).g’(x)


Demonstração:
Sendo u = g(x), tem-se Au = g(x + Ax)-g(x). Então, Audepende de Ax e quando Ax—>0 tem-se
Au —> 0. Assim, g(x + Ax) = g(x) + Au = u + Au e pode-se escrever:
r h(g(x)) = h(u) e h(g(x +Ax)) = h(u +Au).
Ay .. h(u + Au)-h(u)
Logo, lim — = lim —--------- ------ — . Suponha que Au * 0. Então:
ax—>0 Ax ax—>o Ax
|jm Ay |jm h(u + Au)-h(u) |jm h(u + Au)-h(u) Au =
Ax—>0 Ax Ax—>o Ax Ax-»O AX Au
= lim h(u + Au)-h(u) Au=|jm h(u + Au)-h(u) g(x + Ax)-g(x)
ax->o Au Ax ax->o Au Ax
Quando Ax -> 0, tem-se Au -> 0:

lim — = h'(u).g'(x) = h'(g(x)).g'(x), o que completa a prova no caso em que Au # 0.


ax—>oAx

Exemplos:

1) Determine as derivadas das seguintes funções:


a) x3.cos x; b) (x3-3x2+1)5; c) sen 2x;
d) 2cos 2x; e) sen x2; f) Vx;
x3
g) ^x2 + 4x - 5 ; i) esenx;
cosx
10x5.sen3x
j) (x5 + sen e2x)10; k)

5
Elementos da Física - Mecânica I - Cálculo
Solução:

a) Sejam f(x) = x3 e g(x) = cos x


Sabe-se que (f.g)' = f.g + f.g' = 3x2.cos x + x3(- sen x) = x2(3cos x - x.sen x)

b) Sejam h(x) = x5 e g(x) = x3 - 3x2 + 1


Se f(x) = h(g(x)) => f'(x) = h'(g(x)).g’(x) = 5(x3 - 3x2 + 1)4(3x2 - 6x)

c) Sejam h(x) = sen x e g(x) = 2x


Se f(x) = h(g(x)) => f'(x) = h'(g(x)).g'(x) = (cos 2x).2 = 2.cos 2x

d) Como 2cos2 x = 1 + cos2x , pode-se fazer h(x) = cos 2x e g(x) = 2x


f(x) = 1 + h(g(x)) => f ’(x) = h'(g(x)).g'(x) = (- sen 2x)2 = - 2.sen 2x
Outra maneira é fazendo f(x) = 2cos x e g(x) = cos x
(f.g)' = f .g + f.g' = 2(- sen x)cos x + (2cos x)(- sen x) = - 4.sen x.cos x = - 2.sen 2x

e) Sejam h(x) = sen x e g(x) = x2


Se f(x) = h(g(x)) => f'(x) = h'(g(x)).g'(x) = (cos x2)(2x) = 2x.cos x2

f) Sabe-se que se f(x) = Axn tem-se f '(x0) = Anxn 1


A _1_-| A
1 _1 d
Assim, se f(x) =Vx então f'(x) =—x2 = —x
=— X 2 = 2-J=
2

g) Sejam h(x) = x1/3 e g(x) = x2 + 4x - 5


1 --1 2x + 4
Se f(x) = h(g(x)) => f'(x) = h'(g(x)).g'(x) =-(x2 + 4x - 5)3 ,(2x + 4) =
3 3(x2 + 4x - 5)2'3

h) Sejam g(x) = x3 e h(x) = cos x


Sabe-se que se f = | então f' = g'h-g.h' 3x2. cos x - x3(- sen x) 3x2 cosx + x3 senx
h2 (cosx)2 (cosx)2

i) Sejam h(x) = ex e g(x) = sen x


Se f(x) = h(g(x)) => f'(x) = h'(g(x)).g'(x) = eslienx.cosx

j) Sejam h(x) = x10 e g(x) = x5 + sen e2x


Perceba, porém, que a 2a parcela de g(x) também é uma função composta.
Se f(x) = h(g(x)) => f'(x) = h'(g(x)).g'(x) = 10(x5 + sen e2x)9[5x4 + (cos e2x)2e2x]

k) Sejam g(x) = 10x5.sen 3x e h(x) = e.Ví


Ví 1
Assim: g’ = 50x4.sen 3x + 10x5.(cos 3x).3 = 10x4(5.sen 3x + 3x.cos 3x) e h' =e
2\/x
g'h gh' 10x4 (5 sen3x + 3x.cos3x)e'/x - 10x5 senSx.e^
Sabe-se que se f = — então f' =
h h2 e2'1*

6
Elementos da Física - Mecânica I - Cálculo
Interpretação Geométrica da Derivada

Considere uma função continua y(x). No gráfico desta função são destacados 4 pontos, de
coordenadas (x0, y0), (xn, yi), (x2, y2) e (x3, y3), como indicado na figura abaixo. Tome o ponto de
coordenadas (Xo, yo) como referência e trace segmentos de retas ligando-o aos demais pontos
destacados.

u
yi -

</2--

IJ3 - ■
.'/() - -

"1 r- T“
.r
•?’o •1’3 ■í’2 ■1'1

Ayi y1 - y0 y2-y0 = tg02 e Ay3 y3-y0,


Observe que: tSe,. tg%
Ax4 X, - x0 x2 -x0 AX3 X3 -x0

A partir dessa construção, fica óbvio que, à medida que Ax diminui, a razão Ay/Ax vai se
aproximando de tan 0, onde 0 é o ângulo formado entre a reta tangente à curva y(x), passando
pelo ponto de coordenadas (x0, y0), e o eixo x. Então, tem-se que

y’(x0) = lim — = tg0


° AX->0 ax

Essa interpretação da derivada tem inúmeras utilidades. Por exemplo, dada a equação de
dx
uma trajetória unidimensional, x = x(t), a velocidade é dada por v = —. Graficamente, isto

significa que a velocidade é a tangente da curva no gráfico de x versus t. Dada a velocidade em


função do tempo, v = v(t), a aceleração é definida através da derivada a = —, que pode ser
dt
interpretada como a tangente da curva no gráfico de v versus t. Na verdade, se uma grandeza
física A for definida como a derivada da grandeza B, tem-se que A é a inclinação da reta tangente
ao gráfico de B.
No estudo da cinemática, é conhecido que a equação horária do espaço do movimento
at2
retilíneo uniformemente variado é x(t) = Xo + vot +—-, onde Xo é a posição inicial, v0 é a velocidade

inicial e a é a aceleração. Como a velocidade é definida como a derivada do espaço no tempo,


segue que

.f
d xo+vot + —
at2
dx
v(t) = v0 + at
dt dt

Deste modo, ao traçar o gráfico x x t do MRUV, tem-se que a reta tangente ao gráfico em
um determinado instante to possui inclinação dada por v0 + ato.

7
Elementos da Física - Mecânica I - Cálculo
INTEGRAL

Dada a equação horária x = x(t), já vimos que é possível obter a velocidade instantânea v(t)
tomando a derivada de x em relação a t, isto é, v(t) = . Frequentemente, temos que resolver
dt
o problema inverso: dada a velocidade v = v(t), precisamos calcular o espaço percorrido entre um
instante inicial t, e um instante final tf, isto é x(tf) - x(t,) = xf -x,. Esse problema tem uma solução
gráfica muito simples, que conduz ao conceito de integral. Vamos considerar o gráfico de v contra
t indicado na figura

i Aí? ' A/3 I


r.3 I \

I Af^|
1 *
1

1
1

ti = tf)
-i- ^2
■I

Tj tf = t.[
t

Se o gráfico de v contra t fornecesse a velocidade média vm(t a solução do problema


seria trivial; nesse caso o espaço percorrido seria dado por

Xf-Xi= vm(Wf)(tf-ti)

Contudo, a velocidade média não é conhecida. No entanto, se o intervalo tf - t, fosse bem


pequeno, a velocidade poderia ser considerada constante e igual a velocidade média vm(t t()
nesse intervalo. Usualmente, o intervalo entre ti e tf não é pequeno, mas sempre pode ser dividido
em um certo número de subintervalos. Na figura, foram escolhidos apenas quatro intervalos
menores, de comprimentos Atf = ti - t0, At2 = t2 - t,, At3 = t3 - t2 e At» = L - t3. Em um subintervalo
genérico (entre t2 e t3, por exemplo), a velocidade média é definida como

x(t3)-x(t2) = x3-x2
^m(t2->t3) => x3 x2 - vm(t2_,,t3)At3
t3 —12 At3

Fazendo vm(t^t3) = Vm3 segue que

x3 — x2 = vm3At3

Generalizando para um intervalo de tempo qualquer:

Xf - Xj = (X4 - x3) + (x3 - x2) + (x2 - x,) + (x, - x0) = Vm4At4 + vm3At3 + vm2At2 + Vm1Ati,

onde X| = Xo e Xf = X4. Essa expressão pode ser escrita numa forma bem mais compacta utilizando
a notação de somatório:

8
Elementos ila Física - Mecânica I - Cálculo

xf-xi = EvmjAti
>1

Ao invés de considerar apenas 4 subdivisões, poderiamos ter subdividido o intervalo t( - ti


em N subintervalos bem menores. Nesse caso teríamos

N
xf-xi=EvmjAtj
j-1

Note que esta expressão é exata. O problema é que não conhecemos as velocidades
médias vmj. Porém, no limite de N muito grande e quando o maior Atj tende a zero, vmj tende a
velocidade instantânea Vj = v(tj), e a soma das inúmeras parcelas, que se denomina integral
definida, costuma ser escrita na forma

N N rt
iAti =l!™0£viAtj =ít'v(t)dt
j=i j-1

Perceba que no lugar de um índice j que assume valores discretos, há uma variável de
integração contínua em t (a velocidade instantânea v, é substituída pela velocidade instantânea
v(t), e Atj passa a ser um “intervalo infinitesimal” dt). À medida que N aumenta, é fácil perceber
N
que a soma ^VjAtj corresponde cada vez mais fielmente à área sob a curva do gráfico de v
J=1
versus t. Nesse limite a soma, ou melhor, a integral definida, corresponde exatamente à área sob
a curva da função v = v(t) entre t = t, e t = tf. Dessa forma, a integral é o caso particular de um
limite, em que as parcelas de uma soma tendem a zero, mas o número de parcelas tende a infinito.

Propriedades das Integrais Definidas

1) J(‘'f(t)dt = jS(t)dt+£f(t)dt

2) Jabf(t)dt = -Jbaf(t)dt

3) £ f(t)dt = O

4) ^Af(t)dt = Aj‘'f(t)dt

5) Jt‘f [Af(t) + Bg(t)J dt = Aj*’ f(t) dt + BJ*' g(t) dt

Teorema Fundamental do Cálculo


"Se a função F(x) for dada por F(x)= fxf(t)dt , onde a é uma constante arbitrária, então
Ja
d[F(x)]
= f(x). A função F(x) se chama primitiva de f(x)."
dx

Pelo teorema, conclui-se que a integral corresponde a uma operação inversa da derivação.

9
Elementos da Física - Mecânica I - Cálculo
Integrais Indefinidas

Ax^ Bx2 ~
Para cada valor da constante k na expressão G(x) = —— + —— + Cx + k temos uma função
G(x) diferente. Cada uma dessas funções é chamada primitiva da função f(x) = Ax2 + Bx + C, pois
^ = f(x) para qualquer valor de k. Essas primitivas formam uma família de funções que são
dx
normalmente simbolizadas como G(x) = jf(x)dx, sem a preocupação de especificar os limites de
dG
integração. Isso é o que se chama integral indefinida. Como — = f(x), é claro que
dx

F(x)= (xf(x)dx = G(x)-G(a),


Ja

dF
com — = = f(x). O teorema fundamental do cálculo pode então ser reescrito na forma
dx

G(x)-G(a) = J*f(t)dt,

em que G(x) é uma primitiva genérica de f(x). Note que a constante aditiva k, distinguindo as
diferentes primitivas, desaparece quando se faz a diferença G(x) - G(a).

Na tabela abaixo registramos algumas integrais indefinidas razoavelmente simples que vão
aparecer em problemas de física. Note que a e k são constantes arbitrárias.

f(x) G(x) = í f(x)dx


Ax + k

Axn —— xn+1 + k
n + 1________
1
sen Ax —cos(Ax) + k

cos Ax — sen(Ax) + k

eta ----- + k
A
2 In x + k
x
In x
2
X

Cálculo das Integrais Definidas

A partir do teorema fundamental do cálculo

F(x) = G(x)-G(a) = J*f(t)dt,

onde G(x) é uma primitiva genérica de f(x). Então, para x = b, vem

G(x)-G(a)= íbf(t)dt
Ja

10
___________________________________________________ Elementos da Física - Mecânica l - Cálculo
Para calcular uma integral definida basta achar uma primitiva G(x) e encontrar os seus
valores nos extremos do intervalo de integração. É comum utilizar a notação

a rb
G(b)-G(a) = bG(x) = £ dG

Com essa notação:

rb b
[ f(x)dx = G(x) = G(b)-G(a)
J'a
a a

Exemplos:

2
1) Calcular a área compreendida entre o gráfico da função f(x) = —5- e o eixo do x, para o intervalo
x2
1 <x<2.
Solução:
Pela interpretação geométrica de integral, a área pedida é igual a
S= f2f(x)dx= í2-|-dx
J!
2
2 2
Sabe-se que a primitiva de f(x) —= é G(x) = — + k. Note que
x x
1 no cálculo das integrais definidas pode-se omitir a constante k,
que vai ser sempre cancelada na diferença G(b) - G(a), onde a e
0.5 -■
b são os limites de integração. Então:
+ s = 21 _2x- 22 2
=1
1
1

2) Calcular o trabalho executado pela força f(x) = - 4x + x2 no percurso entre X1 = - 1 m ex2 = + 1


m. Suponha todas as unidades medidas no sistema internacional de unidades.
Solução:
Neste livro, especificamente no capítulo no trabalho e potência,, você verá que a definição de
«2
trabalho é W1->2 = J f(x)dx. Nessas condições, o trabalho solicitado é dado por:
«1
x2 +1 +1
W. >2 = j f(x)dx = j (-4x + x2 )dx =
W,_ -2x2 + 3 = í-2 + — -2-11=22 J
X, -1 l 3 3 3“

3) E equação que rege as transformações gasosas de um gás ideal é PV = nRT, onde P é a


pressão, V é o volume, n o número de rnols, R é uma constante e T é a temperatura. Determine o
trabalho realizado sobre n rnols de um gás, quando o mesmo passa por uma transformação a
temperatura constante T, de um volume VA para um volume VB.
Solução:
VB

A definição de trabalho realizado sobre uma transformação gasosa é WA_,B = J PdV.


VA
VB VB nRT

Logo: WA^B = j PdV= I---- -dV , porém, note que o produto nRT é constante para uma
vA va v

transformação a temperatura constante e por isso pode ser colocado fora da integral:
VB Vn
W,a_b= J—dV = nRTjVAVdv
W nRT lnV = nRT(lnVB-lnVA) = nRTIn-^-
vA vA

11
Elementos tia Física - Mecânica 1 - Cálculo
Integral de Uma Derivada

Do “teorema fundamental do cálculo” decorre imediatamente que

r»d[F(x)] dx = bF(x) = F(b)-F(a),


Ja dX à

uma vez que segundo este teorema o integrando é a derivada da primitiva.

Esta é uma maneira ligeiramente diferente de escrever o TFC. É claro que o TFC garante
que o integrando é sempre a derivada de uma função. Porém, nem sempre esta função pode ser
escrita em termos de combinações de funções simples. Se conseguirmos escrever o integrando
como uma derivada de uma função simples, a integral se torna trivial.

Se a integral for indefinida temos simplesmente

rd[F(x)]
dx = F(x),
•' dx

onde omitimos, como de costume, a constante de integração.

Mudança de Variável de Integração

Uma das técnicas mais versáteis para calcular integrais é a mudança de variável de
integração. Ela é baseada na igualdade

y(fa) b
f f(y)dy = Jf(y(x)) Éywidx
y(a) a
dx

Esta igualdade pode ser usada tanto da esquerda para a direita quanto da direita para a
esquerda, conforme foi mais conveniente, observando os seguintes critérios:

i) da direita para a esquerda: Será usado quando for fácil ver que o integrando tem a forma
dy
f(y)— e identificar y(x). Neste caso, após alterar os limites de integração, devem ser feitas as
dx
substituições

d[y(x)j
y(x)->y e dx dy
dx

ii) da esquerda para a direita: Neste caso, é necessário identificar uma função y(x) que leve a
uma integral mais simples. Além disto, é necessário ser capaz de inverter a função y(x) escolhida
e expressar x em função de y para determinar os limites de integração da integral em x que
aparece no lado direito da equação. Após os limites de integração serem alterados, devem ser
feitas as substituições

y->y(x) e dy-> ÉXWldx


dx

12
Elementos da Física - Mecânica I - Cálculo
Exemplos:

1) Calcule a integral J sen3 x.cos x dx.


Solução:
d(senx)
Observe que cosx = . Isto sugere colocar y(x) = sen x e fazer as substituições sen x -> y
dx
e cos x dx -> dy. Assim:
J sen3 x. cos x dx = j y3 dy = yl+k= sen4 x + k
4 4~

1
1
2) Calcule a integral J dy.
1/2 71~y2
Solução:
Uma boa escolha é y(x) = sen x porque a igualdade 1 - sen2 x = cos2 x permite simplificar o
denominador. A função y(x) = sen x pode ser invertida: x = arc sen y e os limites de integração da
integral em x são arc sen 1/2 = tt/6 e arc sen (1) = n/2. Desta forma: y -> sen x e dy-> cos x dx
1 */2 1 n!2 it/2 x/2 71 7t
[ —dy = f —, -cosxdx= f-------- cosxdx= f dx = X =—
1/2 a/1 - y2 n/6 "Vi - sen2 X n/6 COS X «/6 h/6 2 6 3

5
1
3) Calcule a integral J dy.
3
y.lny
Solução:
Fazendo y(x) = ex tem-se x = In y e os limites de integração serão In 3 e In 5.
Fazendo y -> ex e dy -> ex dx a integral fica da forma:
5 a In5 a In5 a In 5
f------ dy= inJí
J3y|ny 3eX-x
í—
----- exdx= ln3X
dx = Inx = In(ln5)-ln(ln3)
In3

13
MECÂNICA CLÁSSICA

As leis que regem os movimentos dos corpos foram definidas por Isaac Newton, em 1686
no seu livro “Philosophiae Naturalis Principia Mathematica” (Princípios Matemáticos da Filosofia
Natural). Newton introduziu o conceito de força e com este conceito foi possível determinar as
interações entre partículas e suas vizinhanças. O conceito de massa foi introduzido de modo a
descrever o fato de que partículas diferentes, na mesma situação e mesma vizinhança, possuem
comportamentos diferentes.

É interessante ressaltar que pode-se aplicar a Mecânica Clássica somente aos casos em
que as velocidades das partículas sejam muito menores que a da luz c = 3.10® m/s (em geral, v <
0,1c). Porém, esta limitação na aplicação da Mecânica Clássica não constitui um impedimento
grande em sua aplicabilidade, uma vez que os movimentos aos quais estamos habituados na
nossa vida diária possuem velocidades compatíveis com a restrição.

Leis de Newton
As três leis de Newton formam a base de toda a mecânica clássica e permitem descrever
todos os movimentos dos objetos à nossa volta.

PRIMEIRA LEI DE NEWTON: LEI DA INÉRCIA

Todo corpo mantém o seu estado de repouso ou de movimento uniforme segundo uma
linha reta, se não for compelido a mudar o seu estado por forças nele impressas.

Os projéteis continuam no seu movimento, a menos que sejam retardados pela resistência
do ar ou impelidos para baixo pela força da gravidade. Um pião, cujas partes, pela sua coesão,
são continuamente desviadas dos seus movimentos retilíneos, não cessa de rodar se não for
retardado pelo ar. Os corpos maiores—planetas e cometas—encontrando menos resistência nos
espaços livres, continuam os seus movimentos, retilíneos ou circulares, por tempo muito maior.

Um sistema de referência em que se verifique a lei da inércia, é designado por referencial


inercial. Consideremos, por exemplo, uma esfera colocada em repouso sobre uma mesa
horizontal, no interior de um vagão de trem, observada por duas pessoas, o passageiro que
colocou a esfera na mesa e uma pessoa que está sentada na estação por onde está a passar o
trem. Em relação à pessoa que está na estação, a esfera poderá estar em repouso, se o trem
estiver parado, ou em movimento se o trem estiver em movimento. Nos dois casos a esfera
manterá o seu estado, de repouso ou de movimento uniforme; se o trem estiver em movimento,
com velocidade uniforme e em linha reta, a esfera acompanhará o movimento da mesa no trem,
estando assim em repouso em relação ao passageiro no trem. Se a velocidade do trem não for
uniforme, a esfera, que mantém a sua velocidade uniforme, rodará para trás, se o trem estiver
acelerando, ou para a frente, se o comboio estiver freando.

Assim, do ponto de vista do passageiro, a bola apenas manterá o seu estado inicial de
repouso se o trem estiver parado ou em movimento retilíneo uniforme. Desta forma, o comboio em
repouso ou com movimento retilíneo uniforme constitui um referencial inercial, mas o trem em
movimento não uniforme não será um referencial inercial. Se a velocidade do trem for uniforme,
mas o movimento for ao longo de uma curva, a esfera rodaria para alguns dos lados da mesa e o
trem não seria um referencial inercial.

14
Elementos da Física - Mecânica I - Princípios da Dinâmica
O CONCEITO DE MASSA

Inércia pode ser definida como a resistência à mudança do estado de repouso ou


movimento retilíneo uniforme de um corpo. Assim, se um força ou conjunto de forças é aplicada a
um determinado corpo, quanto maior a inércia deste corpo, menor deve ser a taxa temporal de
variação de sua velocidade e, consequentemente, a sua aceleração. A medida da inércia de um
corpo é realizada por meio da sua massa inercial (ou, simplesmente, massa).

Consideremos uma situação em que uma mesma força F é aplicada sobre duas partículas
A e B, de massa mA e mB, respectivamente. Suponha que aA e aB são os módulos das acelerações
assumidas por A e B, respectivamente, devido à aplicação da força F. Como a inércia mede a
resistência à taxa temporal de variação da velocidade da partícula, pode-se escrever que:

me _aA

mA aB

Experimentalmente verifica-se que se for aplicada uma força F', diferente de F, sobre as
partículas A e B, a razão das acelerações de A e B é a mesma obtida sob ação da força F:

a\ aA

aá aB

e conclui-se que a razão das massas de duas partículas é independente da força usada.

Se assumirmos a partícula A como padrão e atribuirmos a ela a massa unitária, podemos


determinar que a massa da partícula B, relativamente à da A, é:

aA
mB =7^-
aB

isto é, pode-se definir a massa de uma partícula como a o inverso da razão das acelerações
adquiridas por ela e uma partícula padrão quando sujeitas à ação de uma mesma força.

O padrão de massa adotado é o quilograma padrão, que é um cilindro de platina


conservado cuidadosamente no Bureau de Pesos e Medidas de Paris. Assim, quando afirmar-se
que a massa de uma partícula é m, tem-se que a massa dessa partícula é um múltiplo m do
quilograma padrão. A unidade de massa no SI é o quilograma (kg).

O instrumento mais usado para a medição da massa de um corpo é a balança. Ao longo


da história vários tipos de balanças foram construídas. Pode-se classificar as balanças como
analógicas (baseadas no equilíbrio dos corpos em duas alavancas) e digitais (baseadas na
aplicação da célula de carga, que capta a intensidade de compressão e transforma essa energia
mecânica recebida em pulso elétrico).

Balança Analógica Balança Digital

15
Elementos da Física - Mecânica I - Princípios da Dinâmica
O CONCEITO DE FORÇA

Utilizando a primeira lei de Newton e o conceito de referenciais inerciais, pode-se definir


força como uma interação entre dois corpos ou entre um corpo e seu ambiente. Esta é a causa de
sempre nos referimos à força que um corpo ou o meio exerce sobre um segundo corpo. Força é
uma quantidade vetorial. Possui magnitude (a intensidade, ou módulo da força), direção, sentido e
respeitando as regras da soma vetorial.

Quando uma força implica contato direto entre dois corpos, como as mãos de uma pessoa
empurrando um bloco, a denominamos força de contato. Na figura abaixo, um corpo está se
movimentando sobre um solo horizontal rugoso. Pode-se identificar três tipos comuns de forças de
contato. A força normal N é exercida sobre o objeto por qualquer superfície com a qual está em
contato. O adjetivo normal significa que a força sempre atua perpendicularmente à superfície de
contato, sem importar o ângulo dessa superfície. A força de fricção Fat exercida sobre um objeto
por uma superfície atua paralelamente à superfície, na direção oposta ao deslizamento. A força F,
exercida pelas mãos de uma pessoa que empurra o objeto

N
——►
/at

Além das forças de contato, também existem forças de campo, que atuam sobre corpos
que estão separados. A força entre dois imãs é um exemplo deste tipo de força, assim como a
força gravitacional. A Terra atrai contra si qualquer objeto que se deixa cair, inclusive quando não
há contato direto entre o objeto e a Terra. A força de atração gravitacional que a Terra exerce
sobre um corpo se chama peso do corpo.

Por exemplo, considere o movimento da Lua em torno da Terra. Apesar de não haver
contato entre a Lua e a Terra, a Terra exerce uma força de atração sobre a Lua, assim como a
Luz exerce uma força de atração sobre a Terra, força essa responsável, entre outras
consequências, pelo movimento das marés.

Flt Ftl O
-► ◄—

Flt é a força que a Lua exerce sobre a Terra e FTL é a força que a Terra exerce sobre a
Lua. Futuramente será demonstrado que FLT + FTL = 0.

16
Mementos da Física - Mecânica I - Princípios da Oinâmica
FORÇA RESULTANTE

Considere um corpo de massa m e suponha que mais de uma força atua sobre este corpo.
Neste caso o movimento do corpo é o mesmo que o efeito produzido pela ação de uma força
denominada força resultante FR, que é obtida pela soma vetorial de todas as forças atuantes no
corpo. Na figura abaixo é possível identificar um exemplo com cinco forças atuantes.

Logo, para este exemplo, tem-se que a força resultante no corpo vale:

Fr = Ê, + F2 + F3 + F4 + Fs

De um modo geral, quando n forças Fj,F2 Fn estão atuando em um corpo, o vetor força
resultante é igual a:

Fr =Fi +F2 +- + Fn

DECOMPOSIÇÃO DE FORÇAS

Considere uma força F e um sistema de eixos bidimensional xy. Esta força F pode ser
decomposta em duas componentes ortogonais Fx e Fy , respectivamente paralelas aos eixos x e y,
de modo que quando somadas vetorialmente resultam na força original F. Esta decomposição é
realizada aplicando as relações métricas e trigonométricas em um triângulo retângulo:

|FX |=|F|.cos0
|Fy|=|F|.senO

tg9 = Ül
|FX|
Fy
I F|= 7|Fx |2 +|Fy |2
>
X F = Fx+Fy
fx

Na realidade, a decomposição que foi feita sobre a força F pode ser realizada com
qualquer grandeza física que seja vetorial, tal como velocidade, aceleração, quantidade de
movimento bastando definir sobre qual sistema cartesiano o vetor deve ser decomposto.

17
______________________________________________ Elementos da Física - Mecânica I - Princípios da Dinâmica
2a LEI DE NEWTON: FORÇA E ACELERAÇÃO
A segunda lei de Newton pode ser encarada como o modelamento matemático do conceito
de força, a definindo em função do efeito que produz sobre os corpos em que atua. O texto
original do livro de Newton é:

A mudança na quantidade de movimento é proporcional à força motora impressa e faz-se


na direção da linha reta segundo a qual a força motora é aplicada. Se uma força gera uma
quantidade de movimento, uma força dupla gerará uma quantidade de movimento dupla, uma
força tripla gerará uma quantidade de movimento tripla, quer a força seja impressa de uma vez e
imediatamente, quer seja impressa gradual e sucessivamente. E se o corpo já então se movia, a
nova quantidade de movimento (sempre dirigida na direção da força atuante) é adicionada ou
subtraída à quantidade de movimento inicial, conforme sejam concordantes ou opostas uma da
outra; ou juntas obliquamente de forma a produzir uma nova quantidade de movimento composta
pela determinação das duas.

Antes de enunciar essa lei, Newton já tinha definido previamente no seu livro a quantidade
de movimento, que na nossa linguagem vetorial moderna corresponde a um vetor p, igual ao
produto entre a massa da partícula, m, e a sua velocidade: p = mv. A quantidade de movimento
também costuma ser designada de momento linear.

A “mudança da quantidade de movimento”, referida no enunciado da lei, é a quantidade de


movimento final, p2, menos a quantidade de movimento inicial, pv Na frase “quer a força seja
impressa de uma vez e imediatamente, quer seja impressa gradual e sucessivamente” Newton
está a referir-se ao integral da força em função do tempo. Consequentemente, em notação vetorial
a segunda lei de Newton equivale à seguinte equação

jFdt = p2-p1
ti

onde F se refere à força resultante atuante na partícula. Esta equação pode ser escrita também
de modo diferencial

F=—
dt

e escrevendo a quantidade de movimento em função da velocidade obtém-se,

Como a força resultante é a soma vetorial de todas as forças atuantes em um determinado


corpo de massa m:

d(mv)
F1+F2+... + Fn =
dt

Se a massa do corpo for constante, a derivada acima será igual ao produto da massa pela
derivada da velocidade, ou seja, igual à massa vezes a aceleração:

F, + F2 +... + Fn = m.ã

Esta é a forma mais habitual de escrever a segunda lei de Newton.


A unidade de força no Sistema Internacional (SI) de unidades é o newton, N. Uma força de
1 N é a força que produz a aceleração de 1 m/s2 em um corpo com massa de 1 kg.

18
Elementos da Física - Mecânica I - Princípios da Dinâmica
FORÇA PESO

Conforme já foi referido no capítulo sobre cinemática, no vácuo todos os objetos em queda
livre são acelerados com a aceleração da gravidade, que na superfície terrestre, ao nível do mar,
tem um valor constante g aproximadamente igual a 9,81 m/s2.

Assim sendo, de acordo com a segunda lei de Newton o peso de qualquer objeto (força da
gravidade exercida pela Terra) é diretamente proporcional à sua massa: P = mg, em que g é um
vetor constante na direção vertical, com sentido de cima para baixo e módulo igual à aceleração
da gravidade g. Por exemplo, um corpo com massa de 2 kg na superfície terrestre terá um módulo
da força peso igual a:

| P |= m. |g 1=2.9,81 = 19,62 N

Se o mesmo corpo estivesse na superfície da Lua, onde a aceleração da gravidade é igual


a 1,62 m/s2, a sua massa seria a mesma mas o seu peso seria menor, devido a aceleração da
gravidade ser na Lua ser menor do que na Terra:

|PL l= m.|gL |=2.1,62 = 3,24 N

Igual a 1a lei de Newton, a 2a lei é válida apenas em referenciais inerciais. Dois referencias
inerciais podem ter uma velocidade relativa não nula, mas essa velocidade relativa deverá ser
constante, ou seja, a aceleração relativa de um referencial inercial em relação aos outros deverá
ser nula. Como tal, a aceleração de um objeto deverá ser a mesma em relação a qualquer
referencial inercial. As velocidades medidas em diferentes referenciais inerciais podem ser
diferentes, mas a sua derivada no tempo (aceleração) será igual em todos. Newton acreditava na
possibilidade de determinar a aceleração absoluta de um objeto, em relação ao espaço absoluto,
e na equação F = m.ã interpretava ã como a aceleração absoluta.

DINAMÔMETRO

O instrumento utilizado para a medição de força é o dinamômetro. Existem dois tipos de


dinamômetro, o manual e o digital. No dinamômetro manual uma mola é usada para dar a
indicação da força aplicada na extremidade livre da mola. O funcionamento do dinamômetro
eletrônico é bastante similar ao da balança digital, onde a força aplicada no gancho do
dinamômetro é transformado em pulsos elétricos por sensores.

3
I339 |

V
SI 6

G
lil
■n
a

Dinamômetro Manual Dinamômetro Eletrônico

19
_________________________________________ Elementos da Física - Mecânica I - Princípios da Dinâmica
3a LEI DE NEWTON: LEI DA AÇÃO E REAÇÃO

A toda ação opõe sempre uma reação de mesma intensidade, mesma direção e sentido
oposto. Isto é, as ações mútuas de dois corpos um sobre o outro são sempre iguais e opostas.

Por exemplo, se uma pedra é pressionada com um dedo, o dedo é igualmente pressionado
pela pedra. Se um cavalo puxar uma pedra por meio de uma corda, o cavalo será puxado para
trás igualmente em direção à pedra, pois a corda esticada tanto puxa o cavalo para a pedra como
puxa a pedra para o cavalo, tanto dificulta a progressão do cavalo como favorece a progressão da
pedra.

Esta terceira lei enunciada por Newton é conhecida como lei de ação e reação. Considere
o exemplo proposto por Newton: um cavalo que arrasta um bloco pesado por meio de uma corda.
A corda exerce a mesma força sobre o bloco e sobre o cavalo, mas em sentidos opostos.

É conveniente analisar separadamente as forças que atuam no bloco e no cavalo, como


mostra a figura. Se a velocidade com que o cavalo arrasta o bloco for constante, a segunda lei de
Newton implicará que a soma das forças que atuam sobre o bloco e sobre o cavalo será nula.

O peso do bloco Pb atua no centro de gravidade do bloco. A corda puxa o bloco na direção
em que está esticada, com uma força T, como se mostra no lado esquerdo da figura. A resultante
do peso e da força da corda é um vetor que aponta para baixo e para a direita. Uma vez que a
resultante das forças no bloco é nula (aceleração nula), o chão deverá exercer uma força Fb para
cima e para a esquerda, força essa devida ao contato entre as superfícies do bloco e do chão.

20
________________________________________ Elementos da Física - Mecânica I - Princípios da Dinâmica
A corda puxa o cavalo para trás, com a força -f oposta á força que atua no bloco. Nas
duas ferraduras do cavalo que estão em contato com o chão haverá duas forças de contato, F. e
F2, que apontam para cima e para a frente. A resultante dessas duas forças, mais o peso do
cavalo e a tensão na corda, deverá ser nula.

As forças exercidas pelo chão são as três: Fb , F, e F2 . Essas três forças de contato com o
chão contrariam a tendência a cair do bloco e do cavalo, travam o movimento do bloco e a
empurram o cavalo para a frente. A corda trava o movimento do cavalo e ao mesmo tempo puxa o
bloco para a frente, com a mesma força, em módulo, com que está travando o cavalo.

Sobre o chão atuam em total 5 forças de reação. As reações aos pesos do bloco e do
cavalo, -Pb e -Pc, são as forças de atração gravitacional do bloco e do cavalo sobre a Terra.
Essas forças atuam no centro de gravidade da Terra, mas foram representadas perto do chão na
figura. As outras três forças são as forças exercidas sobre o chão pelo bloco e pelo cavalo. Se a
velocidade do cavalo for constante, a soma dessas 5 forças será nula.

-Pb -Pc

Se o cavalo estivesse acelerando a soma das forças sobre o cavalo e o bloco seria uma
força que apontaria para a direita. A soma das 5 forças que atuam sobre o chão seria a reação
dessa força, o que provocaria, sobre a Terra, uma tendência desta se deslocar para a esquerda.
No entanto, como a massa da Terra é muitas ordens de grandeza superior à massa do cavalo e
do bloco, a aceleração da Terra para a esquerda seria imperceptível em comparação com a
aceleração para a direita do cavalo e do bloco. Como salienta Newton, o resultado dessas forças
sobre o cavalo mais o bloco e sobre o chão não seria o de produzir velocidades iguais e de
sentidos contrários, mas sim quantidades de movimento iguais e de sentido contrário.

21
Elementos da Física - Mecânica I - Princípios da Dinâmica
FORÇA DE REAÇÃO NORMAL

Suponha que um corpo está apoiado sobre a superfície horizontal de uma mesa. O corpo
exerce sobre a mesa uma força que será identificada como a ação. De acordo com a terceira lei
de Newton a superfície da mesa exerce uma força sobre o corpo, que neste caso será identificada
como a reação. Como a reação é perpendicular à superfície de contato, ela é denominada reação
normal ou simplesmente normal (em matemática, um vetor perpendicular a uma superfície é
denominados vetor normal).

[ J

P< r

7///////////////////7//////7/7///Z,

Sempre que um corpo estiver apoiado sobre uma superfície horizontal seu peso irá
'omprimir a superfície, logo a reação normal terá módulo igual ao peso do corpo:

|N|=|P|

Na situação ilustrada na figura seguinte uma barra está apoiada em um solo horizontal e
em uma parede vertical.

N2

77/77777/777.'

N,

//////////777/77/7//s
No solo horizontal tem-se o contato de uma quina da barra com uma superfície plana,
fazendo com que a reação normal N2 seja perpendicular à superfície do solo. Por outro lado, no
contato da barra com a parede vertical, a quina da parede está pressionando a superfície da barra.
Deste modo, a reação normal N2 é perpendicular à superfície da barra. Na verdade, pode-se
assegurar que toda vez que uma quina pressionar uma superfície reta a direção da reação normal
é perpendicular à superfície reta.

22
Elementos da Física - Mecânica I - Princípios da Dinâmica
Roldana

Roldana, também conhecida como polia, é um instrumento utilizado para alterar a direção
de uma força aplicada em um fio ou um cabo. Se forem considerados desprezíveis tanto a massa
da roldana quanto o atrito proveniente do contato do eixo da roldana com sua superfície interna,
diz-se que a roldana é ideal. Um fio, cabo ou corda é ideal quando sua massa for desprezível e
não esticar (também denominada de inextensível). Quando roldana e fios forem ambos ideais
segue que as intensidades das forças aplicadas nos seus extremos serão iguais.

Para exemplificar esta situação considere um corpo de peso P preso a um fio que passa
por uma roldana fixa, conforme indica a figura. No lado esquerdo da roldana, devido à gravidade,
o fio se encontra na direção vertical. Suponha que no lado direito da roldana o fio se encontra na
direção horizontal. Considerando fio e roldana como ideais conclui-se que |T |=|T'|=T. Perceba
que apenas os módulos destas forças são iguais, uma vez que o vetor f é horizontal e o vetor f'
é vertical, impossibilitando a igualdade entre T e T'.

Pl

APLICAÇÕES DAS LEIS DE NEWTON

Dois corpos ligados por uma corda

Considere dois corpos com massas M, e M2, ligados por uma corda sem massa, podendo
deslizar sobre uma mesa sem atrito. Existe ainda uma força horizontal F agindo sobre M2, como
indicado na figura. Deseja-se determinar os módulos da tensão na corda e a aceleração do
sistema. Como a corda tem massa desprezível, ela simplesmente transmite a força. Decompondo
as forças nos corpos:

T = M-ia-i
F - T = M2.a2

Mi NL
T T F

O sistema possui um vínculo geométrico de forma tal que os corpos são obrigados a andar
juntos e assim a, = a2 = a. Logo, somando as duas equações:

F
F - M,a = M2a a =---------
M,F
T = M,a =
M,+M2

23
Elementos da Física - Mecânica I - Princípios da Dinâmica
Corpos em Contato

Uma força F é aplicada sobre um corpo de massa m-, que está em contato com outro
corpo de massa m2, como mostra a figura (a) abaixo. Ambos estão colocados sobre uma mesa
perfeitamente horizontal sem atrito e pretende-se determinar a aceleração do sistema e a força de
contato entre os corpos 1 e 2.
F
m2

a)

■’2
F

m2
m2g

b) c)

Nas figuras (b) e (c) estão indicadas as decomposições de forças no corpos 1 e 2. A força
P21 é a força que o corpo 2 realiza no corpo 1. A força P12 é a força que o corpo 1 aplica no corpo
2. Estas duas forças constituem uma aplicação da 3a Lei de Newton:

‘"21 _ P12 IP21 HP,2I=fc

Perceba que os dois corpos se movimentam sempre juntos, implicando que a aceleração
dos dois corpos é igual. Aplicando a 2a Lei de Newton em cada um dos corpos:

F - Fc = m,a
Fc = m2a

Somando as duas equações obtém-se:

F
F = (m1+m2)a => a =-----------
m1 +m2

Substituindo o valor da aceleração na 2a equação:

m2F
Fc
md +m2

e assim vemos que este resultado é similar ao do caso em que os dois corpos estão ligados pela
corda.

24
Elementos da Física - Mecânica I - Princípios tia Dinâmica
Máquina de Atwood

Uma máquina de Atwood consiste em dois blocos de massas m, e m2, ligados às


extremidades de uma corda que passa por uma roldana circular fixa, como ilustra a figura abaixo.

A
w

"'2g

Suponhamos, sem perda de generalidades, que m, < m2, ou seja, abandonando o sistema
a partir do repouso o corpo 1 sobe e o corpo 2 desce. A decomposição das forças em cada corpo
está indicado na figura acima. Como a corda é suposta de massa desprezível e inextensível,
conclui-se que os módulos das acelerações dos corpos 1 e 2 são iguais bem como os módulos
das trações exercidas sobre cada corpo. Pela 2a Lei de Newton segue que:

Para o bloco T — rr^g = nr^a


Para o bloco m2: m2g - T = m2a

Somando as duas equações:

m2 -m,
m2g - m4g = (m, + m2 )a a = —?----- ig
m, +m2

Esta aceleração no corpo 1 está dirigida de baixo para cima e no corpo 2 de cima para
baixo. Note que se mi = m2 a aceleração dos corpos é nula, porém isto não significa que os
corpos fiquem parados uma vez que o sistema pode se mover com velocidade constante.

Substituindo o valor encontrado para a na 1a equação:

2 -m
m9 1 2m2m1
mi9 (m, + m2 + m2 -m1) = —
T = m1(g + a) = m1 g + — —Lg = —Lg
m., + m2 J m, +m2 m, + m2

25
Elementos tia Física - Mecânica I - Princípios da Dinâmica
Plano Inclinado Liso

Um objeto muito utilizado no estudo da dinâmica é o plano inclinado, que nada mais é que
uma superfície em forma de rampa, apresentando uma inclinação 0, 0 < 0 < 90°, com relação à
horizontal. O plano inclinado é liso quando pode-se desprezar o atrito do corpo em relação à
superfície inclinada do plano. Suponha que um corpo de massa m é colocado sobre a superfície
de um plano inclinado.

■6 /

17 N
m

e( —
¥77S77/7/77/////7/77/7/7?77J'
k _e/

Na 1a figura ao lado um corpo de massa m é colocado sobre um plano inclinado liso. Na 2a


figura está a decomposição de forças que atuam no corpo de massa m. Verifica-se que apenas
duas forças atuam no corpo: o peso P e a reação normal N em relação ao plano inclinado.

Uma vez que o corpo somente pode se movimentar ao longo do plano inclinado, sabe-se
que seu vetor aceleração resultante deve ser paralelo à superfície inclinada da rampa. Como a
reação normal N é perpendicular à superfície do plano, a força resultante no corpo é igual à
projeção do peso P na direção da superfície da rampa. Na decomposição das forças tem-se que
esta projeção é igual a:

|FR|=|P|.sen0 => FR = m.g.sen 0

Como a força resultante é igual ao produto da massa pela aceleração resultante:

Fr = m.g.sen 0 = m.aR aR = g.sen 0

Como o corpo não pode se mover na direção perpendicular ao plano inclinado nesta
direção existe equilíbrio entre as forças atuantes no corpo. Desta forma, o módulo da reação
normal é igual ao módulo da projeção do peso na direção perpendicular ao plano inclinado:

|N|=|P|.cos0 => N = m.g.cos 0

Por exemplo, tome um plano inclinado liso cuja inclinação com a horizontal é 30°. Um
corpo de massa m = 10 kg é colocado sobre sua superfície inclinada. Adotando g = 10 m/s2, a
aceleração adquirida pelo corpo em seu movimento de descida vale:

a = g.sen0 => a =10.0,5 => a = 5,0 m/s2

A reação normal ao plano inclinado é igual a:

V3
N = m.g.cos 0 => N = 10.10.— => N=50>/3N

26
Elementos Da Física - Mecânica I - Princípios da Dinâmica
Força Elástica

Em algumas situações dinâmicas quando uma partícula é deslocada de sua situação de


equilíbrio surge um força que faz a partícula ter a tendência de voltar à posição de equilíbrio. Este
retorno depende de propriedades intrínsecas do corpo e da intensidade e tempo de atuação da
força. A força que atua no sentido de devolver o corpo para a sua condição original de equilíbrio é
chamada de força restauradora ou força de restituição.

Agora será analisado um sistema dinâmico muito comum, denominado sistema massa-
mola. Uma mola tem presa uma extremidade em uma parede e na outra extremidade é fixado um
corpo de massa m que pode se mover sobre uma superfície horizontal lisa. Suponha que o
comprimento da mola quando nenhuma força é imposta sobre ela vale Lo. Neste caso afirma-se
que a mola está relaxada. Quando sobre a mola atua uma força o comprimento da mola varia. Se
a força for de compressão o comprimento final da mola é menor que Lo. Se a força for de
estiramento o comprimento final da mola é maior que Lo. Uma mola sempre reage à tentativa de
alteração de seu comprimento. Se uma força for aplicada em uma mola de modo à comprimi-la ou
estirá-la, surge como reação na mola uma força que tende a restaurar sem comprimento original.
Experimentalmente verifica-se que, dentro de certos limites, a força restauradora é proporcional à
deformação da mola.

F = — kx

Uma mola pode apresentar uma resistência maior ou menor à mudança de comprimento.
Esta resistência está associada à dureza da mola e se relaciona com uma grandeza denominada
constante elástica, representada pela letra k. O sinal negativo indica que a força é sempre
contrária ao deslocamento da mola em relação à sua posição de equilíbrio. Isto pode ser
verificado nas 3 figuras seguintes, onde em (a) tem-se uma mola em sua posição de equilíbrio
(sem distensão ou compressão), em (b) a mola comprimida e em (c) a mola distendida.

r< o

S
F>0

x=o X <0 X> D


(a)
(b) (c)

Em cada uma das figuras estão indicadas também a força de mola e a deformação em
cada caso. O deslocamento x do corpo é sempre medido em relação à posição de equilíbrio da
mola. Perceba que o sentido da força elástica é sempre oposto ao sentido do deslocamento x, o
que justifica o sinal negativo em F = - kx.

Enquanto a mola obedecer a esta relação ela está obedecendo a uma lei denominada Lei
de Hooke e em um regime denominado regime elástico. Se a mola for deformada além deste
ponto ela entra em um regime denominado plástico e não retornará mais para seu tamanho (forma)
original.

27
Elementos da Física - Mecânica I - Princípios da Dinâmica
VÍNCULOS GEOMÉTRICOS

Em determinados sistemas dinâmicos, envolvendo mais de um corpo, é possível identificar


um vínculo geométrico entre os movimentos de corpos pertencentes ao sistema. O vínculo
geométrico ocorre quando é possível determinar uma relação entre os deslocamentos de corpos
do sistema. Este vínculo, quando existir, permite que seja determinada uma relação entre as
acelerações dos corpos envolvidos no vínculo. Observe os exemplos seguintes para entender
melhor a determinação dos vínculos geométricos.

Polia Fixa

; X, t Considere dois blocos ligados por


T|—; 1 ■- um fio inextensível, conforme indica a 1a
1 -------------
figura. O bloco 1 está sobre uma mesa
horizontal e o bloco 2 está pendendo na
vertical. Suponha que é dado ao bloco 2
2
o X2
um deslocamento x2. Como o fio é
inextensível, segue que o deslocamento x,
do bloco 1 deve necessariamente ser igual
LÜ ao deslocamento do bloco x2: x, = x2

Considerando que o movimento dos sistema iniciou a partir do repouso:


a,!2 a2t2
x, = x2 <» ~2 2“ — 82

Uma Polia Móvel e Outra Fixa


'/S^SSSS/^T/S, '// ’///////^///, No sistema apresentado na 1a figura o
corpo 1 está preso no centro de uma polia móvel
e o corpo 2 está preso ao fio que passa por uma
polia fixa e pela polia móvel onde está preso o
corpo 1. Devido à polia onde está preso o corpo 1
X1 d ser móvel, fazendo o corpo 1 subir uma distância
Xn tem-se que um comprimento 2xt do fio passa
para o lado direito da polia fixa. Este comprimento
x2 é igual à altura que desce o corpo 2. Assim,
conclui-se que x2 = 2xv Supondo que o sistema
inicia seu movimento a partir do repouso:
a2t2
x2 = 2x, <=> <=> a2 = 2ai
2 2

Uma Polia Móvel e Duas Fixas


/////////////////; '/////////////////, No sistema da figura ao lado um corpo
(1) está preso ao centro de uma roldana móvel.
Um fio, que passa pela roldana móvel, possui
presos em suas extremidades os corpos (2) e
(3). Suponha que o corpo (1) desce uma
distância x1f o corpo (2) sobe x2 e o (3) sobe x3.
LJ X, Deste modo, um comprimento x2 + x3 do fio
X3 passa de suas extremidades para o centro,
X2
onde a polia móvel puxa para baixo o fio de
m uma distância 2x,:
2ait2 _a2t2 , a3t2 __
2x, = x2 + x3 =>
2 2 2
2ai — a2 + a3

28
Elementos da Física - Mecânica I - Princípios fla Dinâmica
Plano Inclinado Móvel

(D

X (2) X y * :
Um bloco (1) é colocado sobre a superfície de um plano inclinado (2) que pode se deslocar
sobre um solo perfeitamente horizontal. Suponha que o plano inclinado se desloca uma distância
x2 (logicamente horizontal) enquanto que o bloco se deslocai na horizontal uma distância Xi e na
vertical uma distância y. Na 2a figura pode-se concluir que tg0 = —-—. Se a2 é a aceleração do
X.| + x2
plano inclinado, a1x a componente horizontal da aceleração do bloco, a1y a componente vertical da
aceleração do bloco 1 e supondo que todos os movimentos iniciam a partir do repouso:

a-|yt2

2 aiy
tg0 = — => tgO = => tg0 =
x1 + x2 aixt2 , a2t2 aix+a2
2 2

Note que as forças (Peso e Normal) que geram movimento no sistema sao todas forças
constantes. Logo, a força resultante em cada corpo é constante. Deste modo as acelerações a1y,
aly
a1x e a2 são acelerações constantes. Logo, tg0 = é constante, implicando que a trajetória
aix + a2
do bloco é uma reta.

Conclusão

Alguns sistemas possuem vínculos geométricos que permitem obter relações entre os
deslocamentos dos corpos que compõe o sistema. De maneira geral, quando existe um ou mais
vínculos geométricos em um sistema, é possível encontrar relações do tipo k,x, + k2x2 + ... + knxn
= 0, onde ki, k2 kn são constantes e Xi, x2 xn são os deslocamentos dos corpos 1, 2, .... n
do sistema. Desde que é possível assumir que todos os corpos iniciam seus movimentos a partir
at2
de uma posição de repouso e sabe-se que x, =-^-, onde a, é a aceleraçao do coro i (1 < i < n),
segue que kia, + k2a2 + ... + knan = 0. Estas expressões, em conjunto com as expressões obtidas
pela aplicação da 2a Lei de Newton, permitem determinar as acelerações de todos os corpos que
participam do sistema.

29
Elementos da Física - Mecânica I - Princípios da Dinâmica
Exemplos:

1) (ITA-OO) Uma pilha de seis blocos, de mesma massa m , repousa sobre o piso de um elevador,
como mostra a figura. O elevador está subindo em movimento uniformemente retardado com uma
aceleração de módulo a. O módulo da força que o bloco 3 exerce no bloco 2 é dada por:

_2
_3
A.
.5.
a3m(g + a) b)3/n(g-a) c)2m(g + a) d)2m(g-a) e)m(2g-a)
Solução: Alternativa D

2m A força que o bloco 3 exerce no bloco 2 é exatamente a força que faz com que
os blocos 1 e 2 subam. Aplicando a 2a Lei de Newton para o sistema formado
pelos blocos 1 e 2:
F| j2mg 2mg - F = 2ma F = 2m(g - a)

2) (lTA-07) Equipado com um dispositivo a jato, o homem-foguete da figura cai livremente do alto
de um edifício até uma altura h, onde o dispositivo a jato é acionado. Considere que o dispositivo
forneça uma força vertical para cima de intensidade constante F. Determine a altura h para que o
homem pouse no solo com velocidade nula. Expresse sua resposta como função da altura H, da
força F, da massa m do sistema homem-foguete e da aceleração da gravidade g, desprezando a
resistência do ar e a alteração da massa m no acionamento do dispositivo.

F \

-y
h

7/////////7 solo
Solução:
A velocidade atingida pelo homem-foguete a uma altura h do solo, quando sob a ação exclusiva
do peso é dada por v = ^2g(H - h).
Aplicando a 2a Lei de Newton para o sistema homem-foguete:
Fr = P - F => m.a = m.g - F => a = g - F/m
De acordo com a equação de Torricelli:
0 = v2 + 2a.h => 0 = 2gH-2gh + 2ah-^^ => h=^
m

30
________________________________________ Elementos da Física - Mecânica I - Princípios da Dinâmica
3) (ITA-12) Um elevador sobe verticalmente com aceleração constante e igual a a. No seu teto
está preso um conjunto de dois sistemas massa-mola acoplados em série, conforme a figura. O
primeiro tem massa m, e constante de mola k-,, e o segundo, massa m2 e constante de mola k2.
Ambas as molas tem o mesmo comprimento natural (sem deformação) f. Na condição de
equilíbrio estático relativo ao elevador, a deformação da mola de constante k! é y, e a da outra, x.
Pode-se então afirmar que (y - x) é

a) [(K2-k1)m2 + k2m1](g - a)/K1k2. fcj


b) [(K2+k1)m2+k2m1](g-a)/K1k2.
mi
c) [(K2-k1)m2 + k2m1}g + a)/K1k2.
d) [(K2 +k1)m2 + k2m1](g + a)/K1k2-21
e) [(^2 -ki)m2 +k2m1](g + a)/K1k2 +2t
7712

Solução: Alternativa C
Como os corpos m, e m2 estão em equilíbrio relativo, a aceleração
F4 mi
ÍL
rri2
resultante em cada um é igual à aceleração do elevador. Deste modo,
escrevendo a 2a Lei de Newton para cada corpo:
Fz 4- Pi
Jk,y-m1g-k2x = m1a y = m2(g + a) c (m, +m2)(g + a)
,r P2 x= e y=
[k2x - m2g = m2a k2 k,
[(k2-k,)m2 +k2m,](g + a)
Subtraindo: y-x
k,k2

4) (ITA-12) No interior de um carrinho de massa M mantido em repouso, uma mola de constante


elástica k encontra-se comprimida de uma distância x, tendo ima extremidade presa e a outra
conectada a um bloco de massa m, conforme a figura. Sendo o sistema então abandonado e
considerando que não há atrito pode-se afirmar que o valor inicial da aceleração do bloco relativa
ao carrinho é
x

m M

iniuin^iiniiiiiiiiÍTnfiiiiiiiii

a) kx/m. b) kx/M. c) kx/(m + M). d) kx (M - m)/mM. e) kx (M + m)/mM.


Solução: Alternativa E
Escrevendo a 2a Lei de Newton para cada corpo
Fe
Fe [kx = ma1 kx kx
N, ( => a, = — e a, = —■
[kx = Ma2 m M
m9Jui Como as acelerações dos corpos possuem sentidos
contrários, a aceleração relativa é dada pela soma dos
módulos das acelerações de cada corpo:
kx kx kx(M + m)
ar=a1+a2=—- + tt
m M Mm

5) (ITA-11) Sobre uma mesa sem atrito, uma bola de massa M é presa por duas molas alinhadas,
de constante de mola K e comprimento natural l0, fixadas nas extremidades da mesa. Então, a
bola é deslocada a uma distância x na direção perpendicular à linha inicial das molas, como
mostra a figura, sendo solta a seguir. Obtenha a aceleração da bola, usando a aproximação
(1 + a)“ = 1 + aa.
a) a = - kx/M b) a = - kx2/2M/0 c) a = - kx2/Mí0 d) a = - kx3/2Mí20 e) a = - kx3/M£2o

31
Elementos da Física - Mecânica I - Princípios da Dinâmica
Solução: Alternativa E
Pelo Teorema de Pitágoras segue que:
F ÍKF
IX x j r x2 i zxX2 x2
:x 4 + AX = + x? = Á> J1 + = 41 + ■—= 4 + — => Ax = —-
I 'o io °l 2C2 I 0 o"
24 oi
2lo
A força resultante na bola vale:
x x2 X 2kx3
| Fr |= 21F | sen0 = M | ã | => M | ã |= 2kAx = 2k — x2” => |a|=
4+Ax 2l0 M(2í£ +x2)
4 +—
° 2/?0
kx3
Uma vez que 4 » x então: a
Mí20

6) (IME-86) Três molas, a, b e c, tem comprimento natural Ca = 0,5 m, 4 = 0,6 m e 4 = 0,7 m, e


constante elástica ka = 10 N/m, kb = 15 N/m e kc = 18 N/m, respectivamente. Elas são ligadas
entre si e estiradas entre duas paredes distantes 2,0 metros uma da outra, onde as extremidades
são fixadas, conforma figura abaixo.
Qual o comprimento de cada uma das molas estiradas, em equilíbrio?
. n H

2,0 m

Solução:
Como as molas estão ligadas em série as forças em cada uma delas é igual:
Fa = Fb=Fc => kaAxa = kbAxb = kcAXc => 10(xa - 0,5) = 15(xb - 0,6) = 18(xc - 0,7) =>
10xa - 5 = 15xb-9 = 18Xc-12,6 => xa = 1,8xc - 0,76 e xb = 1,2xc- 0,24
Na figura pode-se observar que a soma dos comprimentos finais das molas é 2,0 m:
xa + xb + Xc = 2,0 => 1,8Xc-0,76 + 1,2xc-0,24 + Xc = 2,0 => 4xc = 3 => xc = 0,75 m =>
xb = 0,66 m e xa = 0,59 m

7) (ITA-11) Um corpo de massa M, inicialmente em repouso, é erguido por uma corda de massa
desprezível até uma altura H, onde fica novamente em repouso. Considere que a maior tração
que a corda pode suportar tenha módulo igual a nMg, em que n > I.Qual dever ser o menor tempo
possível para ser feito o erguimento desse corpo?
I 2H ' 2nH ' nH ' 4nH ' 4nH
a) b) c) d) e)
(n-i)g (n-l)g 2(n-1)2g (n-2)g (n-i)g

Solução: Alternativa B
Seja a o maior valor que suporta a corda. Neste caso o corpo
v“
deve ser erguido aplicando uma aceleração igual a a, durante
v*- um tempo t’, para depois ser freado até atingir a altura H. Seja v'
P
I a velocidade máxima atingida pelo corpo. Pela 2a Lei de Newton:
I T - P = Ma nMg - Mg = Ma => a = (n - 1 )g
I
I Como a aceleração a é constante: v’ = at’ = g(t -1’) => t = nt’
"r t *t
No gráfico acima a área é numericamente igual ao espaço
v'.t at'.t = (n - 1)gt2 r~2nH~
percorrido: H = Área = => t =
2 2 2n (n-1)g

32
___________________________________ Elementos da Física - Mecânica I - Princípios da Dinâmica
8) (ITA-81) A figura (a) representa um plano inclinado cujo ângulo de inclinação sobre o horizonte
é a. Sobre ele pode deslizar, sem atrito, um corpo de massa M. O contrapeso tem massa m, e
uma das extremidades do fio está fixa ao solo. Na figura (b) o plano inclinado foi suspenso, de
modo a se poder ligar as massa m e M por meio de outro fio. Desprezando os atritos nos suportes
dos fios, desprezando a massa do fio e sendo a aceleração da gravidade g, podemos afirmar que:
fig.b

fig.a -V

B
3
a

a) No caso (a) a posição de equilíbrio estático do sistema ocorre somente se M sen a = m;


b) Tanto no caso (a) como no caso (b) o equilíbrio se estabelece quando e somente quando M =
m;
c) No caso (b), o corpo m é tracionado em A por uma força TA = (m + M.sen a)g;
d) No caso (b), a aceleração do corpo M é g(M.sen a - m)/(M + m) no sentido descendente;
e) No caso (a) não há nenhuma posição possível de equilíbrio estático.
Solução: Alternativa D
No caso (a) a posição de equilíbrio estático ocorre quando M.sen a > mg M.sen a > m
Logo, as alternativas A, B e E estão incorretas.
a Na figura ao lado estão indicadas, para o caso (b), as decomposições de
—Ü— forças no corpos m e M. Como existem dois fios distintos ligando os
corpos as trações em cada fios são diferentes.
mg Jf? Os fios inextensíveis implicam que os dois corpos possuem mesmo
módulo de aceleração. Note que o sistema (b) pode se mover tanto no
sentido horário quanto no sentido anti-horário.
Aplicando a 2a Lei de Newton para cada corpo (adotando o sentido horário):
T2 - T-] + Mgsen a = Ma (1)
T1-T2-mg = ma (2)
_ . . _ .., . (Msena-m)q
Somando as duas equações: Mgsen a - mg = (M + m)a => a = ------------------ —
M+m
Se o movimento de M é descendente tem-se T2 = 0. Substituindo o valor de a em (2):
(Msena-m)g mg(M + m + Msena-m) mM(1 + sena)g
T, = m(g + a) = m g +
M+m M+m M+m

9) (ITA-96) Dois blocos de massa M estão unidos por um fio de massa desprezível que passa por
uma roldana com um eixo fixo. Um terceiro bloco de massa m é colocado suavemente sobre um
dos blocos, como mostra a figura. Com que força esse pequeno bloco de massa m pressionará o
bloco sobre o qual foi colocado?

a) 2.M.m.g/(2M+m)

b) m.g

c) (m - M).g

d) m.g/(2M+m)
□ m
e) Outra expressão.

Solução: Alternativa A

33
_______________ Elementos da Física - Mecânica I - Princípios da Dinâmica
2a Lei de Newton para o corpo M da esquerda:
T N T - Mg = Ma (1)
II) 2a Lei de Newton para o corpo M da direita:
M M Mg + N-T = Ma (2)
Mg^ III) 2a Lei de Newton para o corpo m:
Mgj; mg - N = ma (3)
N_
Somando (1) e (2): N = 2Ma => a (4)
2M
mN mg = N| 1 + — 2mMg
Aplicando em (4) em (3): mg - N =-----
2M M l 2M 2M + m

10) Determine a aceleração do corpo 2 no arranjo abaixo se a massa do corpo 2 é n vezes maior
que a massa do corpo 1 e o ângulo de inclinação do plano inclinado com relação à normal é a.
Todos os atritos, bem como as massas das polias e dos fios podem ser negligenciados.

| Q2
a

Solução:
A polia móvel faz com que a aceleração do corpo 1

I
seja o dobro da aceleração do corpo 2.
2T Suponha que o corpo 2 desça e,
2T, consequentemente, o corpo 1 suba.
N Como a polia móvel não possui massa a força
resultante nela deve ser igual a zero. Isto ocorre
mg
quando a tração no fio que liga o corpo 1 à polia
móvel for o dobro da tração no-fio que liga o corpo
I Ynrng 2 ao plano inclinado.
Aplicando a 2a Lei de Newton para cada corpo:
2T - mgsen a = m.(2a) => 2T - mgsen a = 2ma (1)
nrng - T = nma => 2nmg - 2T = 2nma (2)
(2n - sen q)g
Somando as equações (1) e (2): 2nmg - mgsen a = 2m(n + 2)a => a =
2(n + 2)
Se sen a < 2n (o enunciado não afirma que n é inteiro) o corpo 1 sobe e se sen a > 2n o corpo 1
desce. Se sen a = 2n o sistema permanece parado ou se movimenta com velocidade constante.

11) (OBF-12) Um homem está sentado sobre uma prancha e se puxa para cima em um plano
inclinado de 30° como mostra a figura. Se o peso do homem e da prancha é de 1200N, determine
a aceleração se o homem exerce uma força de 200N. Despreze todas as formas de atrito e
considere ideais as roldanas e os cabos, (sen 30° = 0,5).

30°

34
Elementos da Física - Mecânica I - Princípios da Dinâmica
Solução:
Seja mi a massa do homem e m2 a massa da prancha.
Suponha que m = m, + m2.
Aplicando a 2a Lei de Newton:
N T, F T - F - nrngsen 0 = nr^a
*F ÍL 2T + F - m2gsen 0 = m2a
Somando as duas expressões:
ma^-
3T - (rm + m2)gsen 0 = (m, + m2)a =>
*30° 3T-mgsen0 = ma => 3.200 - 1200.0,5 = 120a =>
a=0

12) Sobre uma plataforma de um elevador de massa m encontra-se um homem de massa M. Uma
corda que está amarrada no elevador passa por uma roldana e vem às mãos do homem. A corda
e a roldana são ideais. O homem puxa a corda e sobe junto com o elevador com aceleração
constante a. Calcule a aceleração a e a força exercida pelo homem sobre a plataforma.

~F
Solução:

T Pelo contato dos pés do homem com o piso da plataforma segue que
T
a acelerações dos dois corpos é igual. Pela 2a Lei de Newton:
T - mg — N = ma
T + N - Mg = Ma
Somando as duas equações: 2T - (M + m)g = (M + m)a =>
N T (M + m)(g + a)
2
mg N

13) Uma barra pode mover-se sem atrito tanto para baixo como para cima, entre dois suportes
fixos. A massa da barra é igual a m. O extremo inferior da barra toca a superfície lisa da cunha de
massa M. A cunha está situada em uma mesa horizontal lisa. Determinar a aceleração da cunha.

m
M
\a

Solução:

35
r
Elementos ila Física - Mecânica I - Princípios da Dinâmica
Na decomposição de forças ao lado note que não é
possível que a barra esteja em contato com todas os
suportes. A atuação da força F na barra faz com que
esta tenha a tendência de girar no sentido horário,
n2 fazendo com que tenha contato apenas com o
suporte superior direito e o suporte inferior esquerdo.
Escrevendo a 2a Lei de Newton para cada corpo:
Mg mg - F.cos a = may (1)
mg
F.sen a = Max (2)
Suponha que a barra desce uma distância y
enquanto a cunha se desloca horizontalmente de
uma distância x. Pelo vínculo geométrico entre os
movimentos da barra e da cunha conclui-se que:

m tg a = — => tg
tgaa=— =s ay = ax.tg a (3)
x ax
Multiplicando (1) por sen a:
m.g.sen a - F.sen a.cos a = m.ay.sen a (4)
x Multiplicando (2) por cos a:
F.sen a.cos a = M.ax.cos a (5)
Somando as equações (4) e (5): m.g.sen a = m.ay.sen a + M.ax.cos a =>
m.g.tg a = m.ay.tg a + M.ax (6)
m.g.tg a
Substituindo (3) em (6): m.g.tg a = m.ax.tg2 a + M.ax => ax
m.tg2a + M

14) Que força horizontal deve ser constantemente aplicada a M = 21 kg para que rm = 5 kg não se
movimente em relação a m2 = 4 kg? Despreze os atritos e considere g = 10 m/s2.

m1 I I---------- ~c\

F M
m2

Solução:
Como todos os blocos possuem a mesma aceleração: F = (M + m, + m2)a (1)
N
T No corpo rm existe equilíbrio vertical (N = m,g) enquanto que a força resultante é
igual à tração: FR1 = T = mia
l m,g Como a aceleração é horizontal pode-se aplicar o teorema de Pitágoras no
triângulo utilizado para determinar a força resultante em m2:
T2 = FR2 + (m2g)2 => (mia)2 = (m2a)2 + (m2g)2 => a2(m,2 - m22) = m22g2
m2g
a=
yjm2 - m.'22
m2g
(M + m, +m2)m2g
Substituindo em (1): F
i/m,2 -m22
30.4.10
Substituindo os valores numéricos: F = 400 N
-725-16

15) (IME-84) Determinar a massa necessária ao bloco A para que o bloco B, partindo do repouso,
suba 0,75 m ao longo do plano inclinado liso, em um tempo t = 2,0 s. Desprezar as massas das
polias e dos tirantes e as resistências passivas ao movimento. A massa do bloco B vale 5,0 kg e a
aceleração da gravidade deve ser considerada igual a 10 m/s2.

36
_________ Elementos da Física - Mecânica I - Princípios da Dinâmica

Solução:
A aceleração de B pode ser determinada por:
AdB = aBt2/2 => 0,75 = aB.4/2 => aB = 0,375 m/s2
Suponha que o corpo A desça uma distância x. Este corpo puxa o fio
para baixo uma distância 3x, distância esta que veio do movimento de
subida do corpo B. Assim, conclui-se que se o corpo A descer uma
distância x o corpo B sobe uma distância 3x.
Assim, tem-se que: aB = 3aA => 0,375 = 3aA => aA = 0,125 m/s2
B
Escrevendo a 2a Lei de Newton para cada corpo:
60°
/3
T - mBg sen 0 = mBaB T- 5.10.—— = 5.0,375
L mAg-3T = mAaA
10mA -3T = 0,125.mA

T-25V3 = — T = 45,176 N
8 45,176 =
79mA
79mA mA= 13,72 kg
10mA -3T = ^ T= 24
24
8

16) (IME-85) Os dois blocos da figura deslizam sobre o plano horizontal sem atrito. Sabendo-se
que os pesos dos blocos A e B são, respectivamente, 250 N e 375 N, determinar a aceleração
relativa entre os blocos e a tensão no cabo. Adotar g = 10 m/s2.

100 N B
-«----- A
n—.......
Solução:
Seja T a tração na corda. A 2a Lei de Newton para cada corpo é:

aA
F —2T = mAaA 100-2T = 25aA 25 => aA + aB = 4 m/s2
3T = mBaB 3T = 37,5aB 2T
.25 -3b

x
Suponha que o bloco A anda uma distância x
para a esquerda, puxando assim um
comprimento 2x do fio para a esquerda. Este
comprimento 2x veio do lado direito do sistema,
onde o fio passa pelo bloco B em 3 pontos.
Logo, como o comprimento do fio é constante segue que o corpo B anda uma distância 2x/3
também para a esquerda, ou seja:
2 2 5 95
aB= —aA => aA+ —aA=4 => =4
— aA=4 => aA = 2,4 m/s => aB=1,6m/s
A aceleração relativa é dada por: aR = aA - aB = 0,8 m/s2
Assim: T = 12,5aB = 12,5.1,6 = 20 N

37
__________ _____ _____________________________ Elementos da Física - Mecânica I - Princípios da Dinâmica
17) (Olimpíada Iberoamericana de Física-98) Um corpo de massa m = 1 kg está inicialmente
suspendido em um carrinho de massa M = 11 kg mediante o sistema de polias mostrado. As
polias e os fios são de massa desprezível e também desprezam-se todas as forças de atrito.

ni

<1

- ...I 0
- I © I
0
Se o corpo é solto quando está a uma altura h = 4,9 m acima da base do carrinho, depois de
quando tempo o corpo atingirá a base do carrinho? Qual terá sido o deslocamento do carrinho
nesse tempo?
Solução:

.. I
x

y F

r*j T

Mg» N

De início podemos notar que o carrinho e o corpo possuem sempre o mesmo deslocamento
horizontal, ou seja, a = ax (1)
Escrevendo a 2a Lei de Newton para cada corpo:
2T - F = Ma (2)
F = max (3)
mg - T = may => 2mg - 2T = 2may (4)
Suponha que é dado um deslocamento x do carrinho para a direita. Este deslocamento do
carrinho faz com que um comprimento y = 2x do fio passe para a região do sistema onde está o
corpo de massa m. Assim, segue que ay = 2a (5)
Somando as expressões (2), (3) e (4): 2mg = Ma + max + 2may (6)
Substituindo (1) e (5) em (6):
2mg 4mg
2mg = Ma + ma + 4ma = (M + 5m)a => d —----------------- => ay =
M + 5m M + 5m
'h(M + 5m)
h = ^l => t = ^ = 2s
2 2mg 2.1.9,8
at2 mg t2 1.9,8
X =----- = 4 = 2,45m
2 M + 5m 16

18) (IME-02) Sejam M, m, e m2 as massas dos blocos homogêneos dispostos conforme a figura
abaixo, inicialmente apoiados sobre uma placa horizontal. Determine a aceleração do bloco de
massa m,, em relação à roldana fixa, após a retirada da placa, sabendo que M = rri! + m2 e m, <
m2. Considere que não há atrito no sistema e despreze o peso das polias e das cordas que unem
os blocos.

38
_________ Elementos da Física - Mecânica I - Princípios da Dinâmica
//////////

M
A
—1
m.
m2

Placa
Solução:
Nos exemplos anteriores as equações envolvendo os vínculos
geométricos foram determinadas analisando como os corpos
dos sistema se movimentam em função do deslocamento x de
um dos corpos do sistema. Neste exemplo, as expressões do
y yP vínculo geométrico serão determinadas baseadas no fato de
y. que os comprimentos dos fios são constantes. Assim, serão
determinadas relações de como os corpos se movimentam em
y2
M ■ relação a um referencial fixo. Estas expressões permitem
T relacionar as acelerações dos corpos do sistema.
Adotando o referencial na polia identificamos as posições dos
corpos de massas M, m, e m2 como y, y, e y2 e a posição da
m.
polia móvel como yp. Sejam também t, e í2 os comprimentos
m2
dos fios 1 e 2 e a,, a2, a3 as acelerações dos corpos mb m2 e
M, respectivamente, temos:
y + yp = e y, - yp + y2 - yp = í2 => y, + y2 - 2yp = í2
Derivando estas duas equações em função do tempo obtemos:
a3 + ap = 0 e a! + a2 = 2ap => a, + a2 + 2a3 = 0 (1)
Separando os 3 corpos e aplicando a 2a Lei de Newton temos
(2) T, = 2T
(3) Mg - 2T = Ma3
(4) m,g -T = mia,
(5) m2g - T = m2a2
2T T T
Isolando as acelerações nas expressões: a3=g------- (6), a,=g--------- (7) e a2=g---------- (8)
T r 3 ° M mi m2
Substituindo estes valores em (6), (7) e (8) em (1):
T T „ 4T -rí 1 4
14]^
g---+ g--- + 2g- —= 0 > T — + —+ — =4g => T = _____________ g (9 )
m, m2 M 4m1m2 + m,M + m2M
l^m, m2 M
MJ
4m2M 4m1m2 + m,M + m2M - 4m2M
Substituindo (9) em (7): a, = g - g= g =>
4m1m2 +m1M + m2M 4m1m2 +m,M + m2M
4m1m2 + m,M - 3m2M [4(M - m2 )m2 + (M - m2 )M - 3m2M]
a, = g= g
4m1m2 + m,M + m2M 4m1m2 + M(m, + m2)
4Mm2 - m2 + M2 - Mm2 - 3Mm2 M2 - 4m2
a1------------------------------------- z— ------------- g a.
4m,m2 +M M2 + 4m,m2

39
___________ __________________________________ Elementos da física - Mecânica I - Princípios da Dinâmica
19) Um bloco de massa m, que está conectado a uma parede através de um fio ideal, é
abandonado em repouso sobre um carrinho de massa M = 2m. Se o sistema pode deslizar sem
atrito, determinar as acelerações adquiridas pelo bloco e pelo carrinho.
sen a = 0,6 cos a = 0,8

m
ã
M

I
Solução:

T a
* ■*»——

O
N
o 1
▼ Mg

Pela decomposição das forças pode-se concluir, pela 2a Lei de Newton, que:
T - T.cos a + F.sen a = Ma => T - 0,8.T + 0,6.F = 2ma => T + 3F = 10ma (1)
mg - F.cos a - T.sen a = may => mg - 0,8.F - 0,6T = may => 5mg - 4F - 3T = 5may (2)
T.cos a - F.sen a = max => 0,8.T - 0,6.F = max => 4T-3F = 5max (3)
Suponha que o corpo M se movimente uma distância x para a direita. Úm comprimento x do fio
passa para o lado do plano inclinado, fazendo com que o corpo m ande uma distância x ao longo
do plano inclinado. Dos vínculos geométricos conclui-se que:
ay = a.sen a => ay = 0,6a => 5ay = 3a (4)
ax = a - a.cos a => ax = a - 0,8a => 5ax = a (5)
Substituindo (4) em (2): 5mg - 4F - 3T = 3ma (6)
Substituindo (5) em (3): 4T - 3F = ma (7)
Somando as expressões (1) e (7): 5T=11ma => T = 2,2ma (8)
Substituindo (8) em (1): 2,2ma + 3F = 10rna => 3F = 7,8ma => F = 2,6ma (9)
5mg - 4F - 3T = 3ma => 5mg - 10,4ma - 6,6ma = 3ma => 5mg = 20ma => a = g/4
As componentes da aceleração de m são dadas por
ax = a/5 = g/20 e ay = 3a/5 = 3g/20

40
_____________________________________________ Elementos da Física - Mecânica I - Princípios da Dinâmica
20) Os blocos A, B e C tem massas de 1kg, 2kg e 3kg, respectivamente. Calcular as suas
acelerações. Desprezar atritos.

-s
C
7\
A

Solução:

Xb

c

T
i ,I EH _
EEZ ZEHÇ^
7
id
I.
T O O
xA
I i
A
I !
mAg^

Pela 2a de Lei de Newton:


2T
2T = mcac => ac=—- 0)
mc
2T
2T = mBaB =“
(2)

mAg - T = mAaA => aA=g--±- (3)


mA
Suponha que, após um intervalo de tempo, o bloco A se deslocou xA, o bloco B se deslocou xB e o
bloco C se deslocou xc. Neste processo, um comprimento 2xB + 2xc do fio passa da região onde
estão B e C para a região onde está A. Deste modo, pode-se afirmar que xA = 2xB + 2xc.
Assim, pode-se afirmar que aA = 2aB + 2ac (4)
T 4T 4T T 4T
— + — + —=g
4T
Substituindo (1), (2) e (3) na expressão (4): g------- = — + —
m.
mA m,B m
m m,c mA mB mc
T(4mAmB + 4mAmc + mBmc) mAmBmcg
=g (5)
mAmBmc 4m'AAm
1 B + 4mAmc + mBmc
T =_ mBmcg_____ 4mA(mB +mc)g
aA
mA 4mAmB + 4mA‘am1 c + mBmc 4m A mB + 4mAmc + mBmc
o 2T 2mAmcg
dp — —
mB 4m‘aA1mB + 4mAmc + mBmc
= 2T =_____ 2mAmBg_____
ac
mc 4mAmB + 4mAmc + mBmc
Substituindo os valores numéricos mA = 1 kg, mB = 2 kg e mc = 3 kg obtém-se:
10g 3g 2g
aA 13 B 13 c 13

41
Elementos da Física - Mecânica I - Princípios da Dinâmica
21) A cunha mostrada na figura pode deslizar sobre o plano horizontal sem atrito. A massa da
cunha é M e seu ângulo se elevação é a = 30°. Um corpo de massa m pode deslizar sem atrito
sobre a superfície inclinada da cunha. A trajetória do corpo faz um ângulo <p = 60° com a horizontal.
Determine a razão m/M.

Solução:

\ a i
NYí
'S * I
'X / ; y'
mg
a
Q-lXi—1
N’ ,,Mg '''
7/^//////^
Aplicando a 2a Lei de Newton em cada um dos corpos:
Ncosa
Ncos a ...
mg-Ncosa = may => ay=g----- —— (1)
m
Nsena
N.sen a = max => ax = (2)
m
Nsena
N.sen a = Ma => a =----------- (3)
M
y . ay
Pelo vínculo geométrico do sistema tem-se que: tg a = => tga = ——
x, + x2 ax+a
Ncos a Nsena Nsena
ay = (ax + a)tg a g--------- -----------+ tga
m m M
mg-Ncosa Nsena(M + m)
tga => Mmg - NMcos a = N(M + m)tga.sen a =>
X M/TÍ
_________ Mmg_________
NfMcos a + (M + m)sen a.tg a] = Mmg => N= (4)
Mcosa + (M + m)sena. tga
. . . ... ... Mgcosa (M + m)gsena.tga
Substituindo (4) em (1): a = g - —----- ---- ---- -— = ——---- —-------- -—
’ Mcosa + (M + m)sena.tga Mcosa + (M + m)sena.tga
(M + m)gsen2 a (M + m)gsen2a
ay = ay = (5)
Mcos2 a + (M + m)sen2 a M + msen2 a
Mg sen a. cos a Mg sen a. cos a
Substituindo (4) em (2): ax = => ax (6)
Mcos2 a + (M + m)sen2 a M + msen2 a
ay M+m m
Dividindo as expressões (5) e (6): tga => tg<p= 1 + — tga =>
M

tg60° = íl + ^|tg30° (. m)y3 3=1+- => m=2


= 1+—
t My l M 3 M M

42
________________________________________ Elementos da Física - Mecânica I - Princípios da Dinâmica
22) Determinar as acelerações dos corpos de massas m2 e m3 do sistema mecânico da figura.
Despreza-se o atrito entre todas as superfícies de contato.

m2

'^ZZZZZZZZZZZZZZZZZZZZZZZZZ^
mi

Va
Solução:
K
I x,

êT
jrn,g
___________
Q'/ZZZZZZZZZZZZZZZZZZZZZZZAZX
m2g

Como m, está ligado ao plano inclinado através de um fio inextensível que passa por uma polia
fixa tem-se que as acelerações de m, e m2 possuem mesmo módulo.
Escrevendo a 2a Lei de Newton para cada corpo:
m1g-T = m1a (1)
T + F.sen 0 = m2a (2)
F. cos a
m3g - F.cos 0 = m3ay => ay = g - (3)
m3
F. sen 0
F.sen 6 = m3ax => ax = (4)
m3
rr^g + F.senO
Somando as equações (1) e (2): n^g + F.sen 0 = (rm + m2)a => a = (5)
m1 + m2
Do vínculo geométrico entre os movimentos de m2 e m3 segue que:]
tg0= 3y => (ax + a)tg 0 = ay (6)
ax+a
F.sen 0 m,g + F. sen 0 F. cosa
Substituindo (3), (4) e (5) na equação (6): tgO = g-
IDg + rD2 m3
F(m., + m2)sen2 0 + m1m3g sen 0 + Fm3 sen2 0 m3gcos0 - Fcos2 0
(m1+m2)rjc^'
F^ + m2)sen2 0 + m1m3g sen 0 + Fm3 sen2 0 = (m1 + m2 )m3gcos 0 - F(m1 + m2 )cos2 0 =>
2 r/ x zxi r- [(m< + m9)cos 0 - m« sen 0]g
F[m1 + m2 + m3sen 0] = ma^rr^ + m2)cos 0 - rrhsen 0]g => F = ■ 3 1------—------------ |-------— (7)
rri! + m2 + m3 sen 0
Substituindo (7) em (3), (4) e (5):
F.cosa (m, +m2)gcos2 ©-rr^gsenOcos©
(3): ay = g- g-
m3 m, + m22 + m3 sen2 0

43
____________ Elementos tia Física - Mecânica I - Princípios da Dinâmica
[m, + m2 + m3 sen2 0 - (m, + m.i2) cos2 0 + m, sen 0 cos 0]g
ay - ■
m, + m2 + m3 sen2 0
[(m, + m2 )(1 - cos2 0) + m3 sen2 0 + m, sen 0 cos 0]g
y m, + m2 + m3 sen2 0
[(m, + m2)sen2 0 + m3 sen2 0 + m, sen0cos0]g
y m, + m2 + m3 sen2 0
[(m, +m2 + m3)sen2 0 + m, sen0cos0]g
y m, + m2 + m3 sen2 0
F.sen0[(m, +m2)sen0cos0-m,sen2 0]g
(4) ax
m3 m, + m2 + m3 sen2 0
m3[(m, + m2)cos0-m, sen0]g
’9 — • — • — sen2 0
(5) a-mi9 + F sen0 - m,+m2+m3
m, + m2 01,+ m2
(m2 + 01,012 + nyj+j-sen2^ + m,m3 sen 0 cos 0 + m2m3 sen 0 cos 0 - nynj-sen2"© )g
(m, + m2 + m3 sen2 0)(m, + m2)
[m,(m, +m2) + m3(m, +m2)sen0cos0]g Jrnr^rnJííni, +m3 sen0cos0)g
a= => a
(m, + m2 + m3 sen2 0)(m, + m2) (m, + m2 + m3 sen2 0)^rrir+-rrÇ5
(m, + m3 sen 0 cos 0)g
m, + m2 + m3 sen2 0

44
Elementos da Física - Mecânica I - Princípios da Dinâmica
REFERENCIAIS NÃO INERCIAIS

Em muitas situações é importante comparar descrições de um dado movimento feitas por


observadores que estão em movimento relativo de translação entre si. Por conveniência, vamos
considerar que (S) é o referencial de um observador em repouso e (S') é o referencial de um
observador móvel
(S’)

A
A figura mostra que os vetores posição da partícula vistos pelos dois observadores se
relacionam através de

r'=r-rR

onde fR é o vetor posição da origem do referencial móvel, em relaçao ao referencial fixo. A


derivação da expressão conduz diretamente à relação existente entre as velocidades da partícula
segundo os dois observadores:

V' = V-VR

e derivando novamente obtém-se a relação entre as acelerações:

ã’ = ã-ãR

A última expressão tem uma consequência importante: se o movimento relativo entre os


observadores for retilíneo e uniforme, ãR será zero e ambos os observadores determinam a
mesma aceleração para o objeto móvel. Se no referencial (S) se verificar o Princípio da Inércia,
que diz que um corpo livre de interações mantém constante a sua velocidade, e se não existir
aceleração de (S’) em relação a (S), será pela equação ã' = ã = 0, ou seja, o Princípio da Inércia
verifica-se também em (S’). Chamamos referencial inercial a um sistema de eixos em que seja
verificado o Princípio da Inércia. Podemos agora concluir que se (S) for um referencial inercial,
qualquer outro referencial que tenha em relação a (S) um movimento de translação retilíneo e
uniforme será também um referencial de inércia. Por esse motivo, designam-se por referenciais
equivalentes dois sistemas de eixos com movimento relativo de translação retilíneo e uniforme.

Um referencial que sofra uma aceleração não pode ser um referencial inercial. Um carro
que freia (isto é, desacelera) é um bom exemplo de um referencial não inercial. Um objeto
abandonado a si mesmo tende a manter o seu movimento inalterado (Principio da Inércia) e por
isso quando o carro freia esse objeto tende a acelerar em relação ao referencial do carro. Se
conseguirmos identificar um referencial inercial poderemos testar os outros referenciais
verificando se têm aceleração em relação ao primeiro. Na prática, interessa-nos que o referencial
com que trabalhamos seja “suficientemente inercial” para estudarmos o movimento de que nos
ocupamos. Se quisermos estudar a queda de uma maçã, a superfície da Terra está
suficientemente em repouso. Já o movimento do planeta Mercúrio será difícil de descrever e

45
______________________________________________ Elementos da Física - Mecânica I - Princípios da Dinâmica
explicar se tomarmos a Terra como referencial, como verificaram os astrônomos anteriores a
Copérnico (séc. 16) que usavam um sistema geocêntrico para o Sistema Solar.

Quando a observação de um movimento é feita de um referencial não inercial (acelerado),


as leis de Newton deixam de ser válidas, isto é, a força sobre o corpo não obedece a relação
dv
F = m—. Como a lei de força neste caso fica bastante difícil de ser escrita, principalmente porque
dt
ela depende da posição momentânea do corpo, será introduzida uma força extra no sistema, que
é equivalente ao efeito produzido pelo fato do referencial ser não inercial. Com a adição destas
forças fictícias, chamadas de forças inerciais, a 2a lei de Newton passa a ser novamente válida.
Note que as forças inerciais simulam o efeito de uma força real, porém elas não são exercidas por
nenhum elemento do sistema.

Considere um vagão acelerado como mostrado na figura dentro do qual encontra-se um


observador. Se um corpo cair a partir do repouso, para um observador externo, a trajetória é tal
que a única força agindo sobre o corpo é Mg. Para um observador no interior do vagão acelerado,
a trajetória do corpo é tal que indica a existência de uma força -Mã, de forma que a força total
calculada por ele é:

F = Mg-Mã

onde o termo -Ma é a força inercial.

-Ma
Mg

Por outro lado, se o corpo estiver preso por uma corda no teto do vagão, um observador
externo verá o corpo acelerado tal que:

T + Mg = Mã (observador em repouso)

Para um observador no interior do vagão, o corpo nao está acelerado e, portanto, para ele,
a equação de forças é:

f + Mg - Mã = 0 (observador acelerado ou não inercial)

Neste caso, afirma-se que o corpo está em equilíbrio relativo ao observador, uma vez que
a aceleração relativa entre os dois é nula.

Em referenciais não inerciais o termo -Mã é conhecido como força de inércia ou força de
Einstein.

Outro exemplo de aplicação de referenciais não inerciais está apresentado na figura


seguinte, onde dois corpos, de massas M e m, em formas de prismas triangulares são
empurrados sobre uma mesa horizontal lisa. A força aplicada sobre corpo de massa M é suficiente
para que não exista movimento relativo entre os prismas.

46
Elementos da Física - Mecânica 1 - Princípios da Dinâmica

—-►

, mg

Como o sistema se move como um todo único, pode-se afirmar que:

F
F = (M + m)a => a =--------
M+m

Será adotado, a partir de agora, um sistema não inercial de eixos que se desloca para a
direita com mesma aceleração ã do sistema formado pelos corpos. Neste sistema surge no corpo
de massa m uma força de inércia F, = -mã, como indicado na figura. Neste sistema não inercial a
aceleração do corpo de massa m é nula, ou seja, neste sistema não inercial o corpo de massa m
está em equilíbrio. Na direção paralela à inclinação do plano inclinado este equilíbrio é obtido
igualando as componentes de e mg:

F
F!.cos0 = m.g.sen0 jrí.a. cos 0 = jtí.g. sen 0 cos0 = g.sen0 F = (M + m)g.tg 0
M+m

Deste modo, se uma força de módulo F = (M + m)g.tg 0 for aplicada ao sistema, o bloco de
massa m fica em equilíbrio em relação ao bloco de massa M.

Exemplos:

1) Um vagão de desce um plano inclinado liso com ângulo de inclinação 0. No teto do vagão está
preso um fio em cuja outra extremidade se encontra uma esfera. Determine o ângulo a formado
pelo fio e a direção perpendicular ao teto do vagão.

Solução:
Como o atrito pode ser desprezado, a aceleração do sistema vagãd+esfera vale
a = g.sen 0, paralelamente ao plano inclinado. Adotando um referencial não
a
inercial que se desloca com esta aceleração surge na esfera uma força de inércia
de módulo ma = mgsen 0. Neste referencial a esfera está em equilíbrio, uma vez
mg que o referencial não inercial mede uma aceleração nula da esfera.
A figura ao lado ilustra o triângulo de forças que atuam na esfera no referencial
não inercial. Aplicando a Lei dos Senos neste triângulo:
90“-e^
mg mgsen©
^mã —---- ----- = —2---- sen a = sen 0.cos (a - 0) =>
sen[9O°-(a-0)] sen a
2sen a = 2sen 0.cos (a - 0) => 2sen a = sen a + sen (20 - a) =>
sen a = sen (20 - a) => a = 20-a => a = 0

47
________________________________ Elementos da Física - Mecânica I - Princípios da Dinâmica
2) (ITA-88) Uma pessoa de massa m1 encontra-se no interior de um elevador de massa m2.
Quando na ascensão, o sistema encontra-se submetido a uma força de intensidade Fresuilante = F.
o assoalho do elevador atua sobre a pessoa com uma força de contato dada por:
. miF . . m.F . m.F (m, + m2 )F m2F
a)------ 1------++m,g
01,9 b)------ 3--------m,g
m.g c)------ 3— d) e)
m1+m2 m,+m2 m1+m2 m2 m, +m2

Solução: Alternativa A

__ t fy'
|m,a

m2a
Tn

|n
m,g
m2g

F
Como o elevador e a pessoa possuem mesma aceleração: F = (mi m2)a => a =------------
m, + m2
Adotando um referencial x’y’ não inercial que possui a mesma aceleração do sistema
elevador+pessoa tem-se que tanto o elevador e a pessoa estão em equilíbrio neste referencial. No
referencial x’y’ surgem forças de inércia, de módulos m,a e m2a, na pessoa e no elevador,
respectivamente. Analisando a decomposição das forças na pessoa:
., m.F
N = m,a + m,g =•----- 1----- + m,g
m, +m2

3) Sobre um prisma triangular se coloca uma corrente flexível e homogênea de modo que seu
ponto médio fique sobre a aresta superior do prisma. Este se apóia em um plano horizontal
perfeitamente liso. Determinar a aceleração horizontal que deve ser comunicada ao prisma para
que a corrente fique em equilíbrio, sendo g a aceleração da gravidade.

Solução:
T Suponha que massa da corrente seja 2m, ou
N, seja, cada metade da corrente possui massa
----- ã m. Adotando a > 0, a tendência da corrente é
ma descer sobre o plano de inclinação a. Deste
------ S mg
mg modo, para equilibrar a corrente deve-se
acelerar o prisma para a esquerda de uma
aceleração ã. Adotando um referencial não inercial x’y’ que se move com aceleração â as duas
metades estão em equilíbrio em relação a x’y’. Surgem forças de inércia em cada metade da
corrente, de intensidades ma, como indicado na decomposição de forças.
Na corrente da esquerda: mgsen a = macos a + T (1)
Na corrente da direita: mgsen p + macos p = T (2)
Logo: mgsen a = macos a + mgsen p + macos p => g(sen a - sen p) = a(cos a + cos p) =>
„ ía-B} a>- . (a-0
2gsen = 2a cos ----- - cos a = gtgl
l 2 J 2

48
_____________________________________________ Elementos da Física - Mecânica I - Princípios da Dinâmica
4) Um bloco (1) de massa m repousa sobre um plano inclinado (2) de massa M, em um local onde
a aceleração da gravidade é g. Despreze todos os atritos. Determine a aceleração dos dois corpos.
(DO^

x &
777/7/77/77Z//7///77/
Solução:
Esta situação já foi resolvida anteriormente adotando um referencial inercial. Porém, quando um
referencial não inercial for adotado, a determinação das velocidades dos corpos se torna um
pouco mais simples.
Sejam, no referencial inercial: ã = aceleração do plano inclinado; ãx = aceleração horizontal do
bloco; ãy = aceleração vertical do bloco. Adotando, a partir de agora, um referencial x’y’ não
inercial que se desloca para a direita com aceleração ã (mesma do plano inclinado no referencial
inercial), neste referencial não inercial o plano inclinado está em repouso.
\ a |
—>ã NXI
Ma
x

N’ ,,Mg
No referencial não inercial x’y’ surgem forças de inércia nos corpos, de módulos ma e Ma,
conforme indica a decomposição de forças. Como, em relação a x’y’, o plano inclinado está em
equilíbrio, pode-se afirmar que: x
M.a = N.sen a (1)
Além disso, em x’y’ o corpo (1) desce o plano inclinado (que está parado), ou seja, sua aceleração
resultante a’r é paralela à rampa do plano inclinado:
N + m.a.sen a = m.g.cos a => N = m.g.cos a-m.a.sen a (2)
m.a.cos a + m.g.sen a = m.a'r => a'r = a.cos a + g.sen a (3)
Substituindo (2) em (1): M.a = m.g.sen a.cos a - m.a.sen2 a =>
... 2 , m.sen a. cos a ...
a(M + m.sen a) = m.g.sen a.cos a => a =--------------- ;—g (4)
M + m.sen a
Substituindo (4) em (3):
m.g. sen a. cos2 a m.g. sen a. cos2 a + Mg. sen a + m.g. sen3 a
a'r
M +m.sen2 a M +m.sen2 a
(M + m)sena
g
r M + m.sen2a
Projetando a’r (que é paralelo à rampa do plano inclinado) nas direções horizontal e vertical,
determina-se as componentes das acelerações, em relação a x’y’, do bloco (1):
(M + m)senacosa , , (M + m)sen2a
a'x =a'r .cosa --$------- 9 e ga'e =a' . senra
a y’r=a' .sena
= = —— g
M + m. sen a---------------------------------------- M + m.sen a
Em relação ao referencial inercial:
(M + m)sen2a_
ay=a'.y g
M +m.sen2 a
(M + m)sena.cosa m.sen a, cos a _ M.sena.cosa
ax a’x-a g
M +m.sen2 a M +m.sen2 a x M +m.sen2 a

49
_____ ___ __________________________ Elementos da Física - Mecânica I - Princípios da Dinâmica
5) Um bloco de massa m, que está conectado a uma parede através de um fio ideal, é
abandonado em repouso sobre um carrinho de massa M = 2m. Se o sistema pode deslizar sem
atrito, determinar as acelerações adquiridas pelo bloco e pelo carrinho.
sen a = 0,6 cos a = 0,8

ã
Il
Solução:

T
------- >ã
fP
m

a
M I
I (J
N
"17
▼ Mg

Em um referencial inercial seja ã a aceleração do carrinho. Adotando um referencial não inercial


que se desloca com a mesma aceleração ã do carrinho tem-se o carrinho com aceleração nula.
Uma vez que no referencial inercial a aceleração da parte horizontal do fio é zero, no referencial
não inercial a aceleração do fio é -ã. Como neste referencial o carrinho está parado, a o módulo
da aceleração do bloco será igual ao do fio. No referencial não inercial surgem nos corpos forças
de inércia, como indicados na figura, de módulos Ma e ma.
Desde que o carrinho está em equilíbrio:
T + N.sen a = T.cos a + Ma => T + 0,6N = 0,8T + 2ma => T + 3N = 10ma (1)
Escrevendo a 2a Lei de Newton na referencial não inercial:
Eixo x’: ma - T.cos a + N.sen a = m.a.cos a => ma - 0,8T + 0,6N = 0,8ma =>
4T - 3N = ma (2)
Eixo y’: mg - N.cos a - T.sen a = m.a.sen a mg - 0,8N - 0,6T = 0,6ma =>
5mg - 4N - 3T = 3 ma (3)
11ma
Somando as equações (1) e (2): 5T = 11 ma T (4)
5
13ma
Substituindo (4) em (1) obtém-se N = (5)
5
52ma 33ma
Substituindo (4) e (5) em (3): 5mg -
—5 5
= 3ma

.
=> 5mg = 20ma

g 3 3g
=>
a=f
Ainda no referencial não inercial: a'x a. cos a 91=9-, a' =a.sena = —- = —
45 5 ’ y 4 5 20
Como a aceleração ã do referencial não inercial é horizontal, no referencial inercial tem-se:
3g
ay a '
y 20 ‘

Por outro lado: ax = a'x-a g_9=_g_


5 4 20

50
Elementos da Física - Mecânica I - Princípios da Dinâmica

Exercícios de Embasamento não alterar-se devido à aplicação dessa


aceleração.
e) De acordo com lei da ação e reação, o
E1) (Udesc-15) Com relação às Leis de
objeto pode ser acelerado sem que haja uma
Newton, analise as proposições.
força atuando sobre ele. A aceleração pode
I. Quando um corpo exerce força sobre o outro,
ser causada pela variação de velocidade que
este reage sobre o primeiro com uma força de
será, então, responsável pela força que surge
mesma intensidade, mesma direção e mesmo
no objeto.
sentido.
II. A resultante das forças que atuam em um
E3) (UCPel-14) Conta a lenda que Sir Isaac
corpo de massa m é proporcional à aceleração
Newton, um dos físicos mais conhecidos da
que este corpo adquire.
história, teve sua ideia sobre o comportamento
III. Todo corpo permanece em seu estado de
de corpos em um campo gravitacional, quando
repouso ou de movimento retilíneo uniforme, a
estava cochilando embaixo de uma macieira e
menos que uma força resultante, agindo sobre
foi atingido, na cabeça, por uma maçã.
ele, altere a sua velocidade.
Assinale a opção correta em relação à
IV. A intensidade, a direção e o sentido da
segunda Lei de Newton F = m.a.
força resultante agindo em um corpo é igual à
a) Quanto maior for a massa de um objeto,
intensidade, à direção e ao sentido da
menor a força necessária para acelerá-lo, já
aceleração que este corpo adquire.
que a aceleração é inversamente proporcional
Assinale a alternativa correta.
à massa.
a) Somente as afirmativas ui e IV são
b) Um objeto é primeiramente acelerado e,
verdadeiras.
então, surge nele uma força que é proporcional
b) Somente as afirmativas I e IV são
à sua massa.
verdadeiras.
c) Pode-se medir a massa de um objeto,
c) Somente as afirmativas I e II são
conhecendo-se apenas a aceleração com a
verdadeiras.
qual ele se desloca.
d) Somente as afirmativas II e III são
d) Um objeto, quando nele existe uma força
verdadeiras.
resultante nula, pode ter uma aceleração
e) Todas as afirmativas são verdadeiras.
constante.
e) Um objeto, quando recebe um puxão ou um
E2) (UCPel-15) Considere as Leis de Newton e
empurrão, desenvolve uma aceleração que é
assinale a opção correta relativa a um objeto
inversamente proporcional à sua massa.
que está apoiado em um plano horizontal sem
qualquer tipo de atrito.
E4) (UCPel-14) Um astronauta, que pesquisa o
a) Uma força surge no objeto devido a
solo lunar, necessita carregar 100 kg de
mudanças em sua aceleração. Quando dois
equipamento em uma mochila presa a suas
objetos de mesma massa possuem a mesma
costas. A aceleração da gravidade na
aceleração, eles estão, consequentemente,
superfície da lua é de, aproximadamente, 2
sujeitos à ação de forças de mesma
m/s2. Assinale a alternativa correta em relação
intensidade. A trajetória de um objeto
ao treinamento do astronauta, que foi realizado
acelerado pode ser tanto curvilínea, quanto
na Terra, onde a aceleração gravitacional pode
retilínea.
ser considerada, para simplificar os cálculos,
b) Uma aceleração surge nesse objeto,
como 10 m/s2.
quando ele está submetido à ação de uma
a) Uma mochila de 20 kg na Terra causa ao
força. Consequentemente, a velocidade do
astronauta a mesma sensação. A inércia não
objeto também varia e sua trajetória pode ser
conta nesse caso devido à velocidade do
tanto curvilínea, quanto retilínea.
astronauta na superfície da Lua ser muito
c) O objeto pode estar acelerado sem que haja
baixa.
uma força atuando sobre ele, desde que ele se
b) O astronauta em treinamento na Terra
encontre em movimento. Entretanto, quando
precisou carregar uma mochila que tem a
há uma força atuando sobre um corpo, a
mesma massa de 100 kg, pois o fato de as
aceleração será inversamente proporcional a
acelerações gravitacionais serem diferentes é
essa força.
irrelevante.
d) Uma aceleração no objeto faz com que surja
c) O astronauta teve de carregar na Terra uma
nele uma força que é diretamente proporcional
mochila de apenas 20 kg para sentir o mesmo
à sua massa. Quanto maior a massa, maior a
efeito que sentirá na Lua. A massa da mochila
aceleração. A velocidade do objeto pode ou

51
Elementos da Física - Mecânica I - Princípios da Dinâmica
não é o que importa nesse caso, mas, sim, o Sabendo que o ângulo entre o cabo e a vertical
peso. vale 0, que sen 0 = 0,6, cos 0 = 0,8 e g =
d) Uma mochila de 60 kg na Terra compensa a 10m/s2, a intensidade da força de resistência
diferença entre as acelerações gravitacionais. do ar que atua sobre o recipiente vale, em N,
Somente o efeito da força peso é importante.
e) Ele precisa treinar, na Terra, com uma
mochila de 20 kg para sentir o mesmo efeito
devido à força peso. Contudo, por causa da
MRU
inércia, ele terá mais dificuldade de mudar sua
direção de movimento na Lua.

E5) (PUC/PR-12) A respeito das grandezas


massa e força peso, pode-se afirmar
CORRETAMENTE:
a) Massa é o mesmo que força peso, mas a) 500. b) 1250. c)1500. d) 1750. e) 2000.
medida em unidades diferentes.
b) Massa é uma característica intrínseca do E8) (Unesp-11) No gráfico a seguir são
corpo, enquanto a força peso representa a apresentados os valores da velocidade v, em
interação gravitacional entre o corpo e o m/s, alcançada por um dos pilotos em uma
planeta no qual este se encontra. corrida em um circuito horizontal e fechado,
c) Ao levar um bloco de um lugar a outro no nos primeiros 14 segundos do seu movimento.
Universo, seu peso permanece inalterado Sabe-se que de 8 a 10 segundos a trajetória
enquanto sua massa se altera. era retilínea. Considere g = 10 m/s2 e que para
d) Não é possível medir a massa de um corpo completar uma volta o piloto deve percorrer
na Lua, porque lá não existe gravidade. uma distância igual a 400 m.
e) O que nos mantém “presos” a Terra é sua v(m/s)

atmosfera.
100

E6) (PUC/PR-10) Julgue as assertivas a seguir 60

a respeito das leis de Newton. 40

I. É possível haver movimento na ausência de


- —í——4-------- 1 I I I I I I 1------- 1 1 *-- —•
uma força. 1 2 3 4 5 6 7 8 9 10 11 12 13 14 tW

II. É possível haver força na ausência de A partir da análise do gráfico, são feitas as
movimento. afirmações:
III. A força que impulsiona um foguete é a força I. O piloto completou uma volta nos primeiros 8
dos gases de escape que saem da parte segundos de movimento.
traseira do foguete, à medida que o foguete II. O piloto demorou 9 segundos para
expele os gases para trás. completar uma volta.
IV. Um par de forças de ação e reação sempre III. A força resultante que agiu sobre o piloto,
atuam no mesmo corpo. entre os instantes 8 e 10 segundos, tem
Assinale a alternativa CORRETA: módulo igual a zero.
a) Apenas as assertivas I e II são verdadeiras. IV. Entre os instantes 10 e 12 segundos, agiu
b) Apenas a assertiva I é verdadeira. sobre o piloto uma força resultante, cuja
c) Apenas as assertivas I, II e III são componente na direção do movimento é
verdadeiras. equivalente a três vezes o seu peso.
d) Todas as assertivas são falsas. São verdadeiras apenas as afirmações
e) Apenas a assertiva IV é verdadeira. a) I e III. b) II e IV. c) III e IV.
d) I, III e IV. e) II, III e IV.
E7) (Unesp-12) Em uma operação de resgate,
um helicóptero sobrevoa horizontalmente uma E9) (UFPE-97) Uma locomotiva puxa 3 vagões
região levando pendurado um recipiente de de carga com uma aceleração de 2,0 m/s2.
200kg com mantimentos e materiais de Cada vagão tem 10 toneladas de amassa.
primeiros socorros. O recipiente é transportado Qual a tensão da barra de engate entre o
em movimento retilíneo e uniforme, sujeito às primeiro e o segundo vagões, em unidades de
103N? (Despreze o atrito com os trilhos).
forças peso (P ), de resistência do ar horizontal
( F ) e tração ( T ), exercida pelo cabo
inextensível que o prende ao helicóptero.

52
Elementos da Física - Mecânica I - Princípios tia Dinâmica
bloco B fica sujeito a uma força de intensidade
y I fi. Em uma segunda experiência, aplica-se a
força de intensidade F, de direção horizontal,
com sentido para a esquerda sobre o bloco B,
e observa-se que o bloco A fica sujeito a uma
E10) (Mackenzie-02) No sistema a seguir, o força de intensidade f2. Sendo o valor da
atrito é desprezível, o fio e a polia são ideais e massa do bloco A o triplo do valor da massa
a mola M, de massa desprezível, tem do bloco B, a relação vale
constante elástica 200 N/m. Quando o corpo B K f?
A B A B
é seguro, a fim de se manter o conjunto em
equilíbrio, a mola está deformada de ..... e, 1a Experiência 2a Experiência
depois do corpo B ter sido abandonado, a a) 3 b)2 c) 1 d) 1/2 e) 1/3
deformação da mola será de

B E13) (Udesc-13) A Figura 1 mostra um plano


4,0 kg com inclinação 0. Sobre o plano inclinado, há
um bloco de massa M que está preso, em uma
M
das extremidades, a um dinamômetro D. A
outra extremidade está presa ao bloco de
A massa m, por meio de um fio que passa por
1,0 kg
uma polia. Despreze as massas da polia e as
g = 10 rrV#2 do fio, assim como o atrito entre o bloco de
As medidas que preenchem correta e massa Meo plano inclinado.
respectivamente as lacunas, na ordem de
leitura, são: - D
a) 2,5 cm e 3,0 cm. c) 5,0 cm e 6,0 cm.
M
e) 10,0 cm e 12,0 cm. b) 5,0 cm e 5,0 cm.
d) 10,0 cm e 10,0 cm.

E11) (Mackenzie-08) No sistema ao lado, o fio


e a polia são considerados ideais e o atrito 0
entre as superfícies em contato é desprezível.
Abandonando-se o corpo B a partir do repouso,
no ponto M, verifica-se que, após 2s, ele passa
pelo ponto N com velocidade de 8m/s.
Sabendo-se que a massa do corpo A é de 5kg, Sabendo que o sistema está em equilíbrio
0-
a massa do corpo B é estático, assinale a alternativa que representa
a força lida no dinamômetro.
a) F= (M+m)gsen0 b) F = Mgsen0 + mg
c) F= (M+m)g d) F = mgsenO + Mg
N e) F = Mgcos0 + mg

A E14) (FEI-97) O corpo A, de massa mA = 1kg,


sobe com aceleração constante de 3m/s2.

\ 37° Sabendo-se que o comprimento da mola é L =
1m e a constante elástica da mola é K = 26
Dados: g = 10m/s2 N/m. (ver imagem). Considere g = 10 m/s2.
cos37° = 0,8
sen37° =0,6
a)1kg b) 2kg c) 3kg d) 4kg e) 5kg

E12) (Mackenzie-10) Os blocos A e B abaixo


repousam sobre uma superfície horizontal
perfeitamente lisa. Em uma primeira
0 !
experiência, aplica-se a força de intensidade F,
de direção horizontal, com sentido para a
direita sobre o bloco A, e observa-se que o A massa do corpo B vale aproximadamente:

53
Elementos da Física - Mecânica I - Princípios da Dinâmica
a) 1,0 kg b) 1,45 kg c) 1,58 kg Gráfico 1
d) 1,67 kg e) 1,86 kg L(m)x

E15) (Unesp-08) Dois corpos, A e B, atados


por um cabo, com massas mA = 1kg e mB = 0,20
2,5kg, respectivamente, deslizam sem atrito no
solo horizontal sob ação de uma força, 0,12 -
também horizontal, de 12N aplicada em B.
Sobre este corpo, há um terceiro corpo, C, >
com massa mc = 0,5kg, que se desloca com B, 0 tempo
sem deslizar sobre ele. A figura ilustra a Gráfico 2
situação descrita. m(kg)A
C
1,16-
B F = 12 N

¥¥¥¥¥¥
Calcule a força exercida sobre o corpo C.
0,20
------------------------------- 1----------- ►
0 t' tempo
E16) (Unesp-15) O equipamento representado Analisando as informações, desprezando as
na figura foi montado com o objetivo de forças entre a água que cair no recipiente B e
determinar a constante elástica de uma mola o recipiente R e considerando g = 10 m/s2, é
ideal. O recipiente R, de massa desprezível, correto concluir que a constante elástica k da
contém água; na sua parte inferior, há uma mola, em N/m, é igual a
torneira T que, quando aberta, permite que a a) 120. b) 80. c) 100. d) 140. e) 60.
água escoe lentamente com vazão constante e
caia dentro de outro recipiente B, inicialmente E17) (Fuvest-10) Uma pessoa pendurou um fio
vazio (sem água), que repousa sobre uma de prumo no interior de um vagão de trem e
balança. A torneira é aberta no instante t = 0 e percebeu, quando o trem partiu do repouso,
os gráficos representam, em um mesmo que o fio se inclinou em relação à vertical. Com
intervalo de tempo (t’), como variam o auxílio de um transferidor, a pessoa
comprimento L da mola (gráfico 1), a partir da determinou que o ângulo máximo de inclinação,
configuração inicial de equilíbrio, e a indicação na partida do trem, foi 14°. Nessas condições,
da balança (gráfico 2). Dados: tg14° = 0,25; g = 10m/s2.

mola
Lt
R

B
¥
nh
a) represente, na figura da página de resposta
(abaixo), as forças que agem na massa presa
ao fio.
b) indique, na figura da página de resposta
[ ~|
—I □□
(abaixo), o sentido de movimento do trem.
c) determine a aceleração máxima do trem.

E18) (Fuvest-12) Um mobile pendurado no teto


tem três elefantezinhos presos um ao outro por
fios, como mostra a figura. As massas dos
elefantes de cima, do meio e de baixo são,
respectivamente, 20g, 30g e 70g.

54
Elementos da Física - Mecânica I - Princípios da Dinâmica
Os valores de tensão, em newtons, nos fios
superior, médio e inferior são, respectivamente,
iguais a 00 m
NOTE E ADOTE a) T + ma. b) T + 2ma
Desconsidere as massas dos fios. c) 2T + 2ma. d) 2T + ma.
Aceleração da gravidade g = 10m/s2.
E21) (AFA-02) Para levantar um pequeno
motor até determinada altura, um mecânico
dispõe de três associações de polias:
I II ni

a) 1,2; 1,0; 0,7. b) 1,2; 0,5; 0,2.


c) 0,7; 0,3; 0,2. d) 0,2; 0,5; 1,2.
e) 0,2; 0,3; 0,7.

E19) (Fuvest-13) O pêndulo de um relógio é Aquela(s) que exigirá(ão) MENOR esforço do


constituído por uma haste rígida com um disco mecânico é (são) somente
de metal preso em uma de suas extremidades. a) I. b) II. c) I e III. d) II e III.
O disco oscila entre as posições A e C,
enquanto a outra extremidade da haste E22) (ITA-61) A plataforma P da figura está
permanece imóvel no ponto P. A figura abaixo fora do campo gravitacional de Terra. Ela é
ilustra o sistema. acelerada para a esquerda, com uma
p aceleração ã em relação a um observador
inercial B, fora da plataforma. Seja A um
g observador solidário à plataforma. Sejam M e
M, dois corpos quaisquer, sendo que o
primeiro está ligado à plataforma por meio de
uma mola K e o segundo está apenas apoiado
sobre a plataforma. Os observadores
A observam os dois corpos, de modo que:
B m k M
A força resultante que atua no disco quando
ele passa por B, com a haste na direção a
vertical, é
Note e adote: g é a aceleração local da p
gravidade. a) Para A a massa M, está em repouso e M
a) nula. acelerada com uma aceleração - ã .
b) vertical, com sentido para cima. b) Para o observador B, M e M, têm
c) vertical, com sentido para baixo. aceleração ã.
d) horizontal, com sentido para a direita. c) Para A, M, tem aceleração - ã e M tem
e) horizontal, com sentido para a esquerda.
aceleração ã.
d) Para o observador B, M, está parada e M
E20) (AFA-02) Um avião reboca dois
tem aceleração ã.
planadores idênticos de massa m, com
velocidade constante. A tensão no cabo (II) é T. e) Nenhuma das afirmações acima é correta.
De repente o avião desenvolve uma
E23) (ITA-70) Um corpo C, cujo peso é Q, está
aceleração a. Considerando a força de
suspenso, como mostra a figura, por um
resistência do ar invariável, a tensão no cabo (I)
passa a ser sistema constituído por fios flexíveis e
inextensíveis que deslizam, sem atrito, sobre

55
Elementos da Física - Mecânica I - Princípios da Dinâmica
as polias A e B. Sabendo-se c"~ —massas
que as ---------- a = (m, - m2)g/(m1+ m2)
dos fios e das polias são desprezíveis e que o c) T = (m, - m2)g; a = (m, - m^g/tm, + m2)
sistema todo está em equilíbrio, pode-se d) T = (m, - m2)g; a = (rm - m2)g/m1
afirmar que: e) T = (m, + m2)g; a = (rm + m2)g/m1

E26) (ITA-77) Uma partícula se move sobre


uma reta e seu movimento é observado de um
referencial inercial. A diferença V2 - Vi das
velocidades desta partícula, nos instantes t2 e
t, respectivamente:
cM a) irá depender exclusivamente dos valores
das forças que agem sobre a partícula nos
a) F = Q/2 instantes h (inicial) e t2 (final).
b) F = Q/4 b) irá depender exclusivamente do impulso da:
c) F = Q força aplicada a partícula no intervalo t,, t2 e dai
D) o corpo Q é homogêneo velocidade inicial.
e) o corpo Q não é homogêneo c) irá depender exclusivamente do valor médiO'
da força no intervalo de tempo ti, t2.
E24) (ITA-70) Dois dinamômetros, A e B, estão d) será igual a a(t2 - h) onde a é o valor médio
ligados como mostra a figura. Sejam F, e F2 as da força no intervalo de tempo ti, t2.
leituras nos dinamômetros A e B, e) Nenhuma das respostas acima é correta.
respectivamente, quando se aplica uma força
F, na extremidade livre do dinamômetro B. E27) (ITA-80) Um vagão desloca-se
Valem as seguintes relações: horizontalmente, em linha reta, com uma
aceleração 2 constante. Um pêndulo
'A/
simples está suspenso do teto do vagão. 0
/ pêndulo não está oscilando e nessa posição
/ 'S de equilíbrio forma um ângulo 0 com a vertical.
/ Calcular a tensão F no fio do pêndulo.
a) F = mg cos 0 b) F = m.a.sen 0
a) F = F, + F2 = 2 F,; b) F = F, + F2 = 3 F,; c) F = m \/a2 + g2 d) F = m(g.cos 0-a.sen0)
c) F = F2 = 2 F,; d) F = F, = F2; e) F = m(g.sen 0 + a.cos 0)
e) F = F1 = 2F2.

E25) (ITA-76) No sistema esquematizado são


desprezíveis: o atrito, o momento de inércia da
Exercícios de Fixação
roldana e a massa do fio que liga as massas
m, e m2. Sabe-se que mt > m2 e que a F1) (Mackenzie-06)
aceleração da gravidade local é g . A tensão T 3,0 kg 3,0 kg

no fio e a aceleração ã da massa m , são , S.Okg0 1b0° \ D _ 60’ 3 S.Ok;


respectivamente, dadas por:
Em um laboratório de ensaios mecânicos, foi
necessário compor um sistema conforme a
ilustração acima. As polias e os fios são
considerados ideais, o atrito entre as
superfícies em contato e a massa do
dinamômetro D são desprezíveis e o módulo
da aceleração gravitacional local é 10m/s2.
Quando o sistema está em equilíbrio, a
indicação do dinamômetro é:

3
a) T = (2.m1.m2.g)/(m1 + m2);
a) 24N b) 35N c) 50N d) 65N e) 76N

F2) (Mackenzie-06) O conjunto abaixo é


constituído de polias, fios e mola ideais e não
há atrito entre o corpo A e a superfície do
a = (m, - m2)g/(m1 + m2) plano inclinado. Os corpos A e B possuem a
b) T = (m1.m2.g)/(m1 + m2); mesma massa. O sistema está em equilíbrio

56
Elementos tia Física - Mecânica I - Princípios da Dinâmica
quando a mola M, de constante elástica 2000 um plano inclinado que forma um ângulo a
N/m, está deformada de 2 cm. com a horizontal e sustenta o bloco de massa
I | mB.
4
S M
! B
Adote:
g = lOm/s2
cosa = 0,8
a sen a = 0,6
massa de cada um desses corpos é:
a) 10kg b) 8kg c) 6kg d) 4kg e) 2kg

F3) (UFPE-02) Um pequeno bloco de 0,50 kg


desliza sobre um plano horizontal sem atrito, Assinale a alternativa que apresenta o valor de
sendo puxado por uma força constante F = mB capaz de fazer com que o sistema
10,0 N aplicada a um fio inextensível que permaneça em equilíbrio, desprezando todas
passa por uma roldana, conforme a figura as forças de atrito,
abaixo. Qual a aceleração do bloco, em m/s2, a) mB = mA cos(a) b) mB = mA sen(a)
na direção paralela ao plano, no instante em c) mB = 2mA d) mB = 2mA sen(a)
que ele perde o contato com o plano? e) mB = 2mA cos(a)
Despreze as massas do fio e da roldana, bem
como o atrito no eixo da roldana. F6) (UEM-11) Dois corpos, A e B, estão
//////////////////////// conectados por um fio inextensível que passa
por uma polia, como ilustra a figura que segue:

////////////////////////
a) 12,4 b) 14,5 c) 15,2 d) 17,3 e)18,1

F4) (Fuvest-98) Duas cunhas A e B, de


massas MA e MB respectivamente, se
descolam juntas sobre um plano horizontal
sem atrito, com aceleração constante ã , sob a
ação de uma força horizontal F aplicada à
cunha A, como mostra a figura. A cunha A Os corpos A e B estão inicialmente em
permanece parada em relação à cunha B, repouso e possuem massas mA e mB,
apesar de não haver atrito entre elas. respectivamente. Com base nessas
informações, despreze as massas do fio e da
polia e o atrito entre as superfícies em contato,
analise as alternativas e assinale o que for
F ,
A g correto.
9 01) Se mA = mB, os corpos A e B continuarão
B em repouso.
02) Se mA = 2mB1 B, oO módulo do vetor
a) Determine a intensidade da força . . . 2g-gsen0
F aplicada à cunha A. aceleraçao e dado por a = —-—--- , em
b) Determine a intensidade da força N, que a que g é a aceleração gravitacional e 0 é o
cunha B aplica à cunha A. ângulo entre o plano inclinado e a superfície
c) Sendo 0 o ângulo de inclinação da cunha B, horizontal.
determine a tangente de 0. 04) Quando os corpos A e B estão em repouso,
nenhuma força atua sobre esses corpos.
F5) (Udesc-12) A Figura mostra dois blocos de 08) As forças de tração que o fio exerce no
massa mA e mB conectados por um fio corpo A e no corpo B têm o mesmo módulo.
inextensível e de massa desprezível, que 16) Na direção normal à superfície do plano
passa por duas polias também de massa inclinado, a força resultante que atua sobre o
desprezível. O bloco de massa mA está sobre corpo B é nula.

57
[lementos Ha Física - Mecânica I - Princípios da Dinâmica
Podemos afirmar corretamente que nessa
F7) (UFC-03) A figura abaixo mostra dois situação o elevador está
blocos de massas m = 2,5 kg e M = 6,5 kg, A) descendo com velocidade constante.
ligados por um fio que passa sem atrito por B) subindo aceleradamente.
uma roldana. Despreze as massas do fio e da C) subindo com velocidade constante.
roldana e suponha que a aceleração da D) descendo aceleradamente.
gravidade vale g = 10 m/s2.
F9) (UECE-10) Um elevador parte do repouso
com uma aceleração constante para cima com
relação ao solo. Esse elevador sobe 2,0 m no
primeiro segundo. Um morador que se
encontra no elevador está segurando um
pacote de 3 kg por meio de uma corda vertical.
Considerando a aceleração da gravidade igual
111 a 10m/s2, a tensão, em Newton, na corda é
A) 0. B) 12. C) 42. D) 88.

F10) (Uespi-06) A figura ilustra um rapaz de


massa m = 80 kg que está de pé sobre uma
balança fixa no piso de um elevador.
M Considere que o módulo da aceleração da
P gravidade no local é g = 10 m/s2. Nestas
I I circunstâncias, qual é a marcação da balança,
O bloco de massa M está apoiado sobre a se o elevador desce em movimento retardado
plataforma P e a força F aplicada sobre a com aceleração constante de módulo a = 3
roldana é suficiente apenas para manter o m/s2?
bloco de massa m em equilíbrio estático na ///////////////////////
posição indicada. Sendo F a intensidade dessa
força e R, a intensidade da força que a
plataforma exerce sobre M, é correto afirmar
que: ü
a) F = 50 N e R = 65 N. A
b) F = 25 N e R = 65 N.
c) F = 25 N e R = 40 N. rapaz

d) F = 50 N e R = 40 N. &
e) F = 90 N e R = 65 N.
lg
F8) (UECE-10) Duas massas diferentes estão
penduradas por uma polia sem atrito dentro de
um elevador, permanecendo equilibradas uma Balança
em relação à outra, conforme mostrado na
figura a seguir. A)1040 N B)800 N C) 560 N
D) 240 N E) zero

F11) (Uespi-12) A figura a seguir ilustra duas


pessoas (representadas por círculos), uma em
cada margem de um rio, puxando um bote de
massa 600 kg através de cordas ideais
paralelas ao solo. Neste instante, o ângulo que
cada corda faz com a direção da correnteza do
rio vale 0 = 37°, o módulo da força de tensão
em cada corda é F = 80 N, e o bote possui
aceleração de módulo 0,02 m/s2, no sentido
contrário ao da correnteza (o sentido da
correnteza está indicado por setas tracejadas).
Considerando sen(37°) = 0,6 e cos(37°) = 0,8,

) 58
Elementos da Física - Mecânica I - Princípios da Dinâmica
qual é o módulo da força que a correnteza a) 1,20kg b) 1,32kg
b)1,32kg c) 2,40kg
exerce no bote? d) 12,0kg e)13,2kg.
r^> pessoa
F14) (Mackenzie-97) No conjunto a seguir, de
correnteza
fios e polias ideais, os corpos A, B e C estão
inicialmente em repouso. Num dado instante
esse conjunto é abandonado, e após 2,0 s o
corpo B se desprende, ficando apenas os
corpos A e C interligados. O tempo gasto para
1D pessoa que o novo conjunto pare, a partir do
A) 18 N B) 24 N C) 62 N desprendimento do corpo B, é de:
D)116 N E)138 N

F12) (Mackenzie-96) Num local onde a


aceleração gravitacional tem módulo 10m/s2,
dispõe-se o conjunto a seguir, no qual o atrito
é desprezível, a polia e o fio são ideais. Nestas
condições, a intensidade da força que o bloco
Ikg [A]
A exerce no bloco B é: [c|2 kS
3 kg B

a) 8,Os b) 7,6s c) 4,8s d) 3,6s e) 2,Os.

F15) (Unesp-08) Um rebocador puxa duas


barcaças pelas águas de um lago tranqüilo. A
primeira delas tem massa de 30 toneladas e a
segunda, 20 toneladas. Por uma questão de
economia, o cabo de aço l que conecta o
Dados: rebocador à primeira barcaça suporta, no
mA = 6,0 kg; mB = 4,0 kg; mc = 10 kg; máximo, 6.105 N, e o cabo II, 8.104 N.
sen 0 = 0,8; cos 0 = 0,6
a) 20 N b) 32 N c) 36 N d) 72 N e) 80 N 20 ton ii 30 ton

F13) (Mackenzie-97) Um bloco de 10 kg Desprezando o efeito de forças resistivas,


repousa sozinho sobre o plano inclinado a calcule a aceleração máxima do conjunto, a
seguir. Esse bloco se desloca para cima, fim de evitar o rompimento de um dos cabos.
quando se suspende em P2 um corpo de
massa superior a 13,2 kg. Retirando-se o F16) (UFSCar) A polia e os fios da figura são
corpo de P2, a maior massa que poderemos considerados ideais, sem inércia. O fio é
suspender em P, para que o bloco continue perfeitamente flexível e não há atritos a
em repouso, supondo os fios e as polias ideais, considerar. Considere g = 10 m/s2. Dadas as
deverá ser de: massas mA = 40 kg e mB = 24 kg. Determine as
acelerações a (do corpo A) e (3 (do corpo B)
Dados: quando:
g= 10 m/s2 a) Q = 400 N; b) Q = 720 N; c)Q = 1200N.
cos 8 = 0,8
Q
sen 8 = 0,6
□ b.2
8 n |A

P1

59
Elementos tia Física - Mecânica I - Princípios da Oinãmica
F17) (FEI) Os blocos representados na figura
abaixo possuem, respectivamente, massas mi
= 2,0 kg e m2 = 4,0 kg; a mola AB possui
ir
massa desprezível e constante elástica K = 50
N/m. Não há atrito entre os dois blocos e nem g
entre o bloco maior e o plano horizontal.

A B F
rri]
rri2 —►

F20) (AFA-95) Uma corda homogênea,


inextensível, flexível, com 2,20m de
Aplicando ao conjunto a força F constante e comprimento, colocada sobre o tampo de uma
horizontal, verifica-se que a mola experimenta mesa, mantendo inicialmente 5cm pendente.
uma deformação de 20 cm. Qual a aceleração Se entre a mesa e a corda não existe atrito, o
do conjunto e qual a intensidade da força F? comprimento, em centímetros, da parte
pendente, no instante em que a aceleração da
F18) (FEI-95) O sistema abaixo está corda for igual a 4,1 m/s2, é
acelerado. Podemos afirmar que: a) 45 b) 60 c) 85 d) 90

F21) (AFA-88) Considere a figura abaixo:

"■.... I
<_______________
c

As massas de A, B e C são, respectivamente,


8 iguais a 15kg, 20kg e 5kg. Desprezando os
a) não existe atrito. atritos, a aceleração imprimida ao conjunto,
b) a aceleração do corpo B é o dobro da
quando abandonado a si próprio, é, em m/s2.
aceleração do corpo A. Dado: g = 10m/s2; sen 0 = 0,8; cos 0 = 0,6
c) a força normal do corpo A é o dobro da força a) 0,25 b)1,75 c) 2,50 d) 4,25
normal em B.
d) a força que o fio exerce no corpo A é o F22) (AMAN) Na figura abaixo as massas dos
dobro da força que o fio exerce no corpo B. corpos A, B, e C são respectivamente 2 kg, 3
e) a aceleração do corpo B é a metade da kg e 5 kg. Calcule:
aceleração do corpo A. a) a aceleração do sistema;(adote g = 10 m/s2)
b) a tração no fio que liga B e C;
F19) (FAAP) Mediante fios e polias ideais, o c) a tração no fio que liga A e B.
peso suspenso de massa 100 g solicita
horizontalmente o carrinho de comprimento 2
m e massa 3,9 kg, que está sobre a mesa. No
instante indicado na figura, o carrinho é solto e
começa a gotejar líquido dentro dele à taxa de
180 gotas por minuto. Sendo o volume de cada
gota igual a 0,1 cm3, calcule o volume máximo
de líquido armazenado pelo carrinho.
Despreze todos os atritos, assim como a
massa das gotas em comparação com a
massa do carrinho. Adote g = 10 m/s2. F23) (ITA-69) Considere o sistema ilustrado na
figura abaixo. Supondo-se que tanto a massa
da barra AB como a da polia não desprezíveis,
podemos afirmar que AB está em equilíbrio se:

60
Elementos da Física - Mecânica I - Princípios tia Dinâmica
Í2 F27) (ITA-78) Três corpos A, B e C, com
<- X > massas respectivamente iguais a 4,0 Kg, 6,0
a r B Kg e 8,0 Kg, acham-se apoiados sobre uma
superfície horizontal, sem atrito. Estes corpos
ml
acham-se ligados por intermédio de molas de
massas desprezíveis, e são abandonados a
partir da posição indicada na figura, quando as
tensões nas molas AB e BC forem
m3 respectivamente 1,00 x 10N e 1,50 x 10N.
a) m, e, = (m2 + m3) í2 Pode-se afirmar que as acelerações “aAB” (do
sistema constituído pelos corpos A e B) e “a"
b) nri! (m2 + m3) ti = 4 m2 m3 í2 (do sistema constituído pelos três corpos A, B
c) mi (m2 + m3) í, = 2 m2 m3 t2 e C) serão dadas por:

d) 2 m, (m2 + m3) £1 = m2 m3 t2
0- -víííSimr-i
60'
B
e) m, t2 = (m2 + m3) £1

F24) (ITA-72) Três forças de direções


constantes são aplicadas num ponto material
de massa m = 2,0 kg, formando os ângulos da C
figura (a), todos iguais entre si. Essas forças
variam linearmente com o tempo na forma a) aAB = 1,75 m/s2 , a = 0,97 m/s2;
indicada no gráfico (b). (Os sentidos indicados b) aAB = 1,5 m/s2 , a = 0 (nula);
em (a) são considerados como os sentidos c) aAB =1,0 m/s2 , a = 0,81 m/s2;
positivos das forças). No instante t = 4s o d) aAB = 1,75 m/s2 , a = 0,81 m/s2;
módulo da resultante vale : e) aAB = 1,0 m/s2, a = 0,97 m/s2;
7A(»)
F28) (ITA-80) No instante dinâmico
representado abaixo, são desprezíveis todos
2 —T, os atritos e o peso do fio que liga os blocos A
e B. Calcular a tensão no fio, sendo m a
ÍQ massa de cada bloco e g a aceleração da
1 rt.>
?2
q 7 gravidade.

<•) A 0>)

a)6N

a)0m/s2 b) /3
b)4 N c) 2 N d) 0 N

F25) (ITA-72) Na questão anterior, o módulo


da aceleração do ponto para t = 0, vale :
m /s 2 c) >/2 / 2 m/s2
í/3 m/s
d) 2 m/s2 e) 3 m /s2
e) 3 N

í
a) T
b) T
= mg(1
= mg(1
V,
+ sen a)/2
+ sen2 a)/(1 + sen a)
a

c) T = mg
F26) (ITA-72) Com relação à questão anterior, d) T = mg sen a
podemos afirmar: e) T = mg tg a
a) A resultante das forças é um vetor
constante. F29) (ITA-82) O plano inclinado da figura 4 tem
b) A aceleração do ponto material nunca se massa M e sobre ele se apoia um objeto de
anula. massa m. O ângulo inclinação é a e não há
c) A resultante das forças tem direção atrito nem entre o plano inclinado e o objeto,
constante. nem entre o plano inclinado e o apoio
e) Para t = 4s a velocidade do ponto material horizontal. Aplica-se uma força F horizontal ao
é nula. plano inclinado e constata-se que o sistema
e) Nenhuma das afirmações acima é correta. todo se move horizontalmente sem que o
objeto deslize em relação ao plano inclinado.

61
Elementos da física - Mecânica I - Princípios da Dinâmica
Podemos afirmar que, sendo g a aceleração b) o elevador deve estar subindo e F = 99 N
da gravidade local: c) o elevador pode estar subindo ou descendo
e F = 81 N
d) o elevador pode estar subindo ou descendo
m. e F = 99 N
F
e) N. D. R. A.
M
F33) (IME-69) O corpo A, pesando 2 kgf, está
' ZZZZZZZZZZZZZ/ZZZ/ZZZZZZZZZZZZZ/Sz
em repouso sobre uma superfície
a) F = mg perfeitamente polida, sustentando por uma
b) F = (M + m)g mola, de constante elástica 20 kgf/m e por um
c) F tem que ser infinitamente grande fio de massa desprezível, que passa por uma
d) F = (M + m)g tg a roldana ideal. Até chegar ao repouso, a mola
e) F = Mg sen a foi distendida de 10 cm. A reação da superfície
sobre o corpo A é nula. Calcule o peso do
F30) (ITA-86) Na figura a seguir, as duas corpo B em kgf.
massas m, = 1,0 kg e m2 = 2,0 kg, estão
ligadas por um fio de massa desprezível que
passa por uma polia também de massa
desprezível, e raio R. Inicialmente m2, é
colocada em movimento ascendente, gastando
0,20 segundos para percorrer a distância d =
1,0 m indicada. Nessas condições m2 passará F34) (IME-98) Na figura a seguir os objetos A e
novamente pelo ponto “O” após B pesam, respectivamente, 40 N e 30 N e
aproximadamente: Adotar g = 10,0 m.s-2. estão apoiados sobre planos lisos, ligados
entre si por uma corda inextensível, sem peso,
que passa por uma polia sem atrito.
Determinar o ângulo 0 e a tensão na corda
d Jl ,0 m quando houver equilíbrio.

m.

Ll
□ m2
A B

l30° e

F31) (ITA-67) Duas partículas P e Q estão


inicialmente em repouso e separadas de uma F35) O carrinho da figura desliza no plana
distância de 1,00 m. P tem uma massa de 0,10 horizontal com aceleração de 8 m/s2. O corpo
kg e Q uma massa de 0,30 kg. P atrai Q e Q A possui 4 kg de massa e não há atrito entre o
por sua vez atrai P, com uma força constante corpo e os planos de apoio. Determine a força
de 1,0 x 1O'Z newtons. Nenhuma força externa horizontal que a parede vertical exerce no
atua sobre o sistema. A que distância, a partir corpo considerando-o em repouso em relação
da posição original de P, as partículas irão ao carrinho. Considere g = 10 m/s2.
colidir?
a) 0,75 m b) 0,40 m c) 0,30 m d) 1,33 m
E) Nenhum dos valores acima. a

F32) (ITA-76) Um bloco de 10,0 kg está


apoiado no piso de um elevador que se 7////////////////^
desloca verticalmente com uma aceleração
constante ã = -^-, ondeg é a aceleração da F36) Na figura seguinte, os pesos da polia, do
fio e da mola são desprezíveis e assume-se g
= 10 m/s2. Sendo as massas de A e B mA = 40
gravidade local, sendo g = 9,0 m /s 2 . O piso
kg e mB = 24 kg, a deformação da mola igual a
do elevador exerce sobre o bloco uma força F.
50 cm e a intensidade de F igual a 720 NI,
Pode-se afirmar que:
determinar:
a) o elevador deve estar descendo e F = 81 N

62
Elementos da Fisica - Mecânica I - Princípios da Dinâmica

“ F
A
50 cm
C m J-40 cm
2m
150 cm

Balança
| A | B B B
Figura 1 Figura 2 Figura 3
a) a constante elástica da mola em N/m; a) esticada de 20 cm
b) o módulo das acelerações de A, de B e do b) comprimida de 20 cm
eixo da polia; c) esticada de 30 cm
c) a indicação da balança sobre a qual d) comprimida de 30 cm
repousam, inicialmente, os dois blocos. e) comprimida de 40 cm

F37) Uma força F puxa um sistema constituído F39) Um indivíduo de massa m = 50kg está
por um plano inclinado de massa M, que forma sobre uma balança de molas, a qual está fixa
com a horizontal um ângulo 0, e por um corpo num carrinho B que desce por uma rampa sem
de massa m, ligado a M por uma mola ideal de atrito, como mostra a figura. Sao dados: g =
constante elástica k, conforme a figura. 10m/s2 e sen 0 = 0,20. Determine a marcação
Sabendo que m não se move em relação a M da balança, supondo que seu mostrador esteja
e que todos os atritos são desprezados, calibrado em newtons.
determine a deformação da mola.
Dados: F = 20 N, k = 100 N/m, M = 3 kg, m = 1
kg, sen 0 = 0,6 e g = 10 m/s2.

,r
F
M F40) (OBF-13) No sistema de polias mostrado
)0
cr n na figura ao lado, qual deve ser a força
aplicada na extremidade livre da corda para
levantar o objeto de 2kg?
a) comprimida de 10 cm
b) comprimida de 20 cm
c) esticada de 20 cm
d) esticada de 5 cm
e) esticada de 10 cm

F38) Na figura 1, tem-se uma mola ideal de 2,0


m de comprimento, não deformada, de
constante elástica k = 50 N/m, amarrada entre
os pontos A e B na vertical. Na figura 2, corta- Fi
se a mola no ponto C, sendo que AC = 50 cm.
Na figura 3, coloca-se um corpo de altura 40
cm e massa 8 kg amarrado às novas molas AC
e CB. Sabendo que g = 10 m/s2, determine a
deformação da mola AC. F41) Uma corda uniforme, de comprimento L,
repousa em equilíbrio sobre um prego fixado
na parede em um local onde a aceleração da
gravidade é g. Uma pequena perturbação faz a
corda entrar em movimento. Determine a
aceleração da corda no instante em que o

63
Elementos da Física - Mecânica I - Princípios da Dinâmica
comprimento do lado da corda que está g(M,-M2) g(M,-M2)
a) b)

rn
descendo é x. M, +M2 M, + 2M2
g(2M,-M2) g(M,-2M2)
c) d)
4M, + M2 M, + 4M2

I e)
g(2M,-M2)
2M, + M2

A3) (ITA-89) A figura representa um sistema


mecânico com as seguintes características: ré
uma roldana de massa desprezível que pode
girar sem atrito; S é um balde de massa m e P
é um peso de massa M tal que m = 0,8/W; 8 e
Exercícios de Aprofundamento Psão ligados por uma corda apoiada em rmas
que não escorrega sobre a roldana; R é um
A1) (FMTM-03) A figura mostra um carrinho A, reservatório que contém água e uma torneira T
é acionada quando o balde toca nela; o balde
com massa mA, que pode se mover sem atrito
sobre outro carro, no qual está fixa uma por sua vez possui uma válvula que abre em
roldana. O carrinho A está ligado por um fio contato com o solo permitindo a saída de toda
ideal, passando pela roldana, a um corpo B de a água; o balde cheio tem massa mc = 1,2M. A
massa 3 kg. Quando o conjunto todo está sob amplitude do movimento é h = 4,0 m. Sabendo
que as operações de enchimento e
uma aceleração a, o carrinho A e o corpo B
não se movem em relação ao carrinho maior e esvaziamento do balde demoram um tempo A t
a parte do fio entre o corpo B e a roldana = 5,0 s cada uma, e que o movimento só se
forma um ângulo de 53° com a horizontal. processa com o balde cheio ou vazio, calcule o
Nestas condições, a vale, em m/s2, período completo desse movimento periódico.
Dados: g = 10 m/s2, sen 53° = 0,8 e cos 53° =
0,6
A R

1
a m
B h
V
.53-
B
M P
O O
a) 2,5. b) 3. c) 5. d) 7,5. e) 10.

A2) (ITA-66 adaptado) Determine a aceleração A4) No sistema dinâmico representado na


do corpo de massa M, no esquema abaixo. figura temos dois corpos, de massas 3M e M,
Considere que os fios são inextensíveis, que unidos por um fio de massa desprezível que-
as polias são ideais e que o corpo Mi está passa por uma polia também de massa,
descendo. desprezível. O corpo de massa M está
suspenso, enquanto que o corpo de massa 3W
Í2222222222222222;
pode deslizar sobre um plano inclinado que
possui dois ângulos de inclinação. O corpo de
g massa 3M é inicialmente liberado do repouso,,
a uma distância / da polia fixa que marca a
passagem de um ângulo de inclinação para o
outro. Considerando que o fio se mantém
M, paralelo ao plano em ambas inclinações e que
todos os atritos são desprezíveis, calcular e
período de oscilação do sistema. Dado sen a =
1/2 e sen 0 = 1/4. Obs: Considere que a
passagem de inclinação é suave, e seu

64
Elementos da Física - Mecânica I - Princípios da Dinâmica
comprimento é pequeno comparado com o A7) Numa corda, colocada em uma roldana,
movimento dos corpos. penduram-se os pesos com massas m, e m2.
A roldana, estando imóvel, (os pesos não se
movem), foi equilibrada numa balança como se
vê na figura. Em quanto será necessário variar
o peso no prato da direita, para que, ao liberar-
se a roldana e consequentemente moverem-se

7l
os pesos, o equilíbrio se mantenha.
-------- ZT

m.
A5) Determine as acelerações dos pesos com I—I rri2
massas m2, m3 e a tensão das cordas no
sistema desenhado abaixo se: mi = m2 + m3. A8) Um corpo pode deslizar sem atrito a partir
As massas das cordas e roldanas são muito do vértice A ao longo dos lados A e AC de um
menores que as massas dos pesos. triângulo ABC. Qual o ângulo <p necessário
para que o corpo, sem velocidade inicial, faça
os percursos AB e AC no mesmo tempo?
A

mi||

EJm2 B
(9—£\c
A6) Um aparato que serve para estudar as leis
do movimento uniformemente acelerado A9) Supõe-se que no diagrama da figura a
consta de dois pesos de massas iguais a M, bola é inicialmente solta, sem velocidade inicial,
suspensos entre si por um fio inextensível que desde a altura h. Supondo que não há perdas
passa por uma polia fixa. No instante inicial o de energia no sistema, a bola ficará
peso do lado esquerdo toca o solo e o da executando um movimento contínuo de subida
direita está a uma altura H sobre este. Em e descida dos planos inclinados. O período do
cima do peso da direita se coloca uma movimento executado pela bola será:
sobrecarga de massa meo sistema começa a
mover-se. Quando o peso da direita se
encontra uma altura h sobre o solo, a
sobrecarga m se engancha em um suporte fixo
e se livra do sistema. Em quanto tempo, desde h
que começou o movimento, chegará ao solo a
corpo da direita?

a) 21/2h/g(l/sena + l/sen0)
b) 2^2h / g (1 / cos a +1 / cos 0)

m c) 2A/2h / g [1 / sen(a + 0)]


d) 2^/2h / g[ 1 / cos(a + 0)]
e) nda
H
h A10) Um homem puxa-se para cima de um
M| plano inclinado pelo método mostrado na
figura. Se o peso combinado do homem e do
carrinho é de 100 N, determine a aceleração
do carrinho, enquanto o homem exerce uma

65
Elementos da Física - Mecânica I - Princípios da Dinâmica
tração de 250 N no cabo. Despreze todo o
atrito e a massa do fio, polias e pinos de
articulação.

L>

l>
A13) No sistema representado na figura, os
)30° fios e as polias são ideais, não há atrito e as
massas dos blocos A, B e C são
a) 7 m/s2 b) 15 m/s2 c) 65 m/s2 respectivamente iguais a 15kg, 10kg e 24kg. A
d) 70 m/s2 e) 75 m/s2 aceleração da gravidade tem módulo 10m/s2.
Sendo aA, aB e ac os módulos das acelerações
A11) Ao suspendermos um corpo de massa m dos blocos A, B e C, respectivamente,
a uma mola de constante elástica k, conforme determine:
a figura I, no equilíbrio, a mola sofre uma
B
deformação x. Se, a seguir, tomamos 1/3 da ST
mola, amarramos a dois fios , e os 2/3
restantes a outros fios, e pendurarmos a
massa m, no equilíbrio temos a figura II, onde
sen 0 = 0,8 e sen p = 0,6. Determine o valor da
relação entre x e a soma das deformações das 1
a) a relação entre aA, aB e acl
molas na figura II.
Figura I Figura 11
b) os valores de aA, aB e ac;
c) o módulo da tração no fio que está ligado ao
bloco A.

A14) Determine a tensão nos cabos ligados a


l 2^/3 #3 cada bloco e a aceleração de cada um deles.
0
Despreze a massa das polias e dos cabos.
P
Adote g = 9,8 m/s2.

m m

a) 3/2 b) 2/3 c) 1 d) 1/2 e) 4/3

A12) Um sistema consiste de duas roldanas


com eixos fixos e uma roldana móvel. Através
das roldanas colocou-se uma corda nos B 6 kg
extremos da qual foram pendurados os pesos
com massas m( e m3 e no eixo da roldana
móvel pendurou-se um peso de massa m2. As A
partes da corda, que não se encontram na A 8 kg
roldana, estão situadas na posição vertical.
l
Determinar a aceleração do bloco de massa
m2 se as massas das roldanas e da corda e A15) Dois blocos que são mostrados na figura
também a fricção podem ser desprezadas. estão originalmente em repouso. As massas
das polias e o todos os atritos podem ser
desprezados. Adote g = 9,8 m/s2. Determine:
a) a aceleração de cada bloco;
b) a tensão no caso.

66
Elementos da Física - Mecânica I - Princípios da Dinâmica

A18) Considere o sistema representado na


30 kg figura, onde os fios e as polias são ideais. A
aceleração da gravidade tem módulo 10m/s2 e
A
as massas de A e B são respectivamente
iguais a 3,0 kg e 2,0kg. Sendo aA e aB os
módulos das acelerações dos blocos A e B,
determine:
a) relação entre aA e aB
b) os valores de aAe aB
c) os módulos da tração no fio ligados ao bloco
B A.
25 kg

A16) No arranjo da figura a massa do corpo 1 A


é r] (r| > 2) vezes maior que a massa do corpo
2. No início o corpo 1 está a uma altura h do
solo, em um local onde a aceleração
gravitacional é g. As massas das polias e das
cordas podem ser desprezadas. Em
determinado momento o sistema é liberado do
repouso. Suponha que o corpo 1 se mantém A19) Determinar a tensão das cordas, nas
parado após o choque com o solo. Qual é a quais estão pendurados os pesos no sistema
máxima altura alcançada pelo corpo 2? esquematizado no sistema abaixo. As massas
’///////////////<
das cordas e das roldanas podem ser
desprezadas. Não há atrito. As massas dos
corpos m1t m2, m3, m4, m5, m6, m7 e m8 são
conhecidas.

A17) Determinar a aceleração dos pesos no


sistema desenhado na figura abaixo. As
massas das roldanas, da corda e o atrito
podem ser desprezados.

Jfíf /Hj .7?^ r7?j.T^ .7?t ««T^

A20) Determinar a aceleração do corpo de


massa m4 no sistema esquematizado na figura
seguinte. As massas das cordas e das
roldanas podem ser desprezadas. Não há
atrito. As massas m1t m2, m3 e m4 são
conhecidas.

mi

m2|

67
Elementos da Física - Mecânica I - Princípios da Dinâmica
b) Qual o maior valor de f possível?

A23) (OBF-08) Duas placas de madeira são


interligadas em uma extremidade de maneira a
permanecerem sempre perpendiculares. São
presas polias nestas placas e por elas passam
um fio leve, flexível e inextensível que em suas
| | m4 = 6 kg extremidades sustentam massas iguais a 30
kg que escorregam sobre as placas. As placas
podem girar em torno do ponto O conforme o
| | m3 = 3 kg
desenho abaixo. Para que ângulo(s) 0 a

â
aceleração dos corpos é máxima? Determine a
tensão no fio para cada caso. Despreze a
inércia das polias e o atrito das massas com as
m2 = 2 kg placas.
□ m, = 1 kg

A21) (OBF-04) Um garoto, dispondo de uma


mola de constante elástica k, montou a
experiência ilustrada na figura abaixo para
determinar a massa mB de um bloco.
k m

M
n
['llli llll llllj
___I A24) (OBF-12) Uma pessoa gostaria de se
cy o
pesar mas dispõe de uma balança com uma
capacidade limitada para 60 kg e um
Fr dinamômetro. Resolve, então, montar o arranja
A massa do carrinho é M e a massa do bloco de cabos e roldanas ideais mostrado na figura.
ligado a mola é m. Os atritos entre todas as Com isso, o dinamômetro marca 50N e a
superfícies e na polia são desprezíveis. balança 550N. Qual é o peso do homem?
Determine a massa mB em função das outras
duas massas e da compressão x da mola,
quando o sistema é liberado.

A22) (OBF-04) A figura representa dois baldes


de massas M, e M2, contendo cada um uma
quantidade de areia de massa M.

r balança

M.O
1 2
A25) Um bloco A se encontra sobre uma
prancha C, a uma distância d = 5 m de um de
seus extremos. A e C estão ligados pelas
extremidades de um mesmo fio ao bloco B,
Considere a polia e os fios ideais. Supondo conforme indica a figura. Despreze todos os
que a massa M2 seja ligeiramente maior que a atritos e massas dos fios e polias. Sabendo-se
massa Mi que mA = mB = 2 kg e mc = 6 kg, determine o
a) Qual a quantidade m de areia que deve ser tempo que o bloco A leva para percorrer a
transferida do balde de massa M, para o balde distância d.
de massa M2 para que a aceleração do
sistema aumente de um fator f ?

68
Elementos da Física - Mecânica I - Princípios da Dinâmica
g B Via/s* do mesmo material, como representado na
d
figura.

------------------<Sí d/2

7
3c/

dl 1
A26) Na situação abaixo o sistema pode ser
considerado livre de atritos. As polias e os fios
têm massas desprezíveis. As massas dos
blocos A e B são iguais e valem 25 kg. A
aceleração da gravidade no local vale 10 m/s2. Na ausência de atrito entre as superfícies dos
blocos e entre a base da cunha e o plano
A B
----- ► 300 N horizontal, encontre o tempo de queda do
paralelepípedo até tocar o plano horizontal na
forma , onde i e j são números inteiros e
25 kg 25 kg vjg
g é a aceleração da gravidade.
O bloco B está sob a ação de uma força
A29) Sabe-se que na figura 1 as massas m1f
externa para a direita, de módulo igual a 300 N.
m2 e m3 movem-se com a mesma aceleração a.
Determine a aceleração do bloco A e a
Retirando-se a massa m3, conforme a figura 2,
aceleração do bloco B, em m/s2.
o sistema passa a se mover com aceleração
A27) Um bloco de massa m é colocado em 6a/5. Colocando-se a massa m3 sobre a
massa mi, conforme a figura 3, o sistema
repouso na parte mais alta de um plano
passa a se mover com aceleração 2a.
inclinado de massa 2m, que descansava em
repouso sobre uma superfície plana horizontal. Determine o valor de tg a.
A altura do plano inclinado é h e seu ângulo de m3
inclinação com a horizontal é 30°. Considere m2 m2
que não existe atrito entre o bloco e o plano
inclinado e nem entre o plano inclinado e o
plano horizontal. A aceleração local é g.
Determine:
a) As acelerações do bloco e do plano
inclinado.
b) A velocidade com que o bloco atinge a base
I m7
F” Figura 1 m.
1 |

l
Figura 2

do plano inclinado. —/r m2


c) O tempo que o bloco demora para atingir a m3
base do plano inclinado. n Figura 3
mi
.m

h
2m A30) Um corpo está preso a um fio, cuja outra
3O°7^^ extremidade está atada a um carrinho. O
carrinho se movimenta com aceleração a
sobre um plano inclinado de ângulo de
A28) (OBF-04) Um bloco em forma de inclinação a, em um local onde a aceleração
paralelepípedo de arestas d, d e d/2 é da gravidade é g. Determine o ângulo 0 que o
colocado na parte superior de um outro bloco fio forma com a vertical.
em forma de cunha, de arestas d, 3d e 4d, feito

69
Elementos da Física - Mecânica I - Princípios da Oinâmlcj

(llj)

a ('

A
Vm,

A31) Uma cunha de massa m encontra-se


apoiada sobre um bloco de massa M e uma A34) Na figura, todas as superfícies são
parede imóvel como indica-se na figura. consideradas com atrito desprezível.
Determinar a aceleração de cada um dos Determine a aceleração do bloco B.
blocos. Despreza-se o atrito e considere g A B
como sendo a aceleração da gravidade local.
&

1
m m
s>-
(*)..... ~........ .
m f
//////////////////////
0 /I
M
a) 2g/13 b) 3g/26 c) 25g/26
d) g/6 e) g/8
SS/////////////////////;
A35) Um plano inclinado, de altura H e ângulo
A32) Uma corda de massa desprezível de inclinação a, está repousando sobre um
repousa sobre uma cunha de inclinação a. solo horizontal, em um local onde a aceleração
Uma das extremidades da corda está fixa à da gravidade é g. Um pequeno bloco está
parede e na outra se encontra uma partícula localizado próximo ao plano inclinado. Qual a
de massa m. Se a cunha é puxada com aceleração horizontal que deve ser imposta ao
aceleração constante ã na horizontal, plano inclinado de modo que o bloco atinja o
determine a aceleração da partícula em topo do plano inclinado em um tempo t? Os
relação ao solo. atritos são todos desprezíveis

ã 4H 2H
a) b)
t2 sen 2a t2 sen a
c) g , 2H 2H
d) gsena + -5------
' cos a t2 sen2 a cos a t sena
A33) Dois blocos de massas m, e m2 estão e) 9t9“ + ?^
acoplados conforme a figura, sobre rampas
dotadas das referidas inclinações. O fio e a
polia são ideais, e não há qualquer atrito entre A36) A figura abaixo ilustra três blocos de
as superfícies. A aceleração da gravidade local massa mA = 2 kg, mB = 4 kg e mc = 10 kg,
é g. Que aceleração horizontal a deve ser inicialmente em repouso em uma região onde
dada ao sistema de modo que os blocos g = 10 m/s2. O bloco C (cuja seção transversail
permaneçam estacionários em relação à forma um triângulo equilátero) está fixo e o
estrutura A? sistema é liberado. Qual a intensidade da força
normal que a superfície horizontal exerce
sobre mc?

70
Elementos da Física - Mecânica I - Princípios da Dinâmica
vale g. Podemos afirmar que a força de A em
C, a força de B em D e a força sobre o teto,
devido ao sistema blocos-polia-fios valem,
respectivamente:

Teto

A37) Considere o dispositivo da figura abaixo,


onde todos os atritos podem ser desprezados,
a polia é ideal, o fio é inextensível e todas as
superfícies são perfeitamente horizontais ou m m
C D
verticais. Adote que a aceleração da gravidade
local é g. Calcule, em função de m1t m2, M e g, 2m A B 3m
as acelerações dos três corpos.
a) 4mg/3; mg/3; 7mg/4
mi b) mg/7; 2mg/7; 23mg/7
c) 8mg/7; 6mg/5; 27mg/7
d) 6mg/7; 4mg/7;42mg/5
mj e) 8mg/7; 6mg/7;48mg/7
M
A40) O sistema mostrado é abandonado a
partir do repouso. Sabe-se que mA = mB = m.
Desprezando todos os atritos, determine a
7//////////////////^^^ velocidade do corpo de massa 2m quando o
corpo B chegar ao piso? Considere g =10 m/s2.
A38) Os corpos A e B do esquema
apresentado a seguir têm massas A
respectivamente iguais a 40 e 5 kg. A
aceleração local da gravidade vale g = 10 m/s2,
os fios são ideais e as polias têm inércia
desprezível. O atrito entre os fios e as polias,
bem como entre o corpo A e o plano horizontal B 2m
de apoio, é admitido também desprezível.
0,5 m
Determine a aceleração do ponto C da corda
em relação ao referencial inercial.
C
A F = 60N
a) m/s b) m/s c) m/s
^/W////7ZWW
d) m/s e) m/s

E A41) No sistema da figura seguinte o corpo A


está apoiado sobre um corpo C, que possui
a) 2,5 m/s2 b) 4 m/s2 c) 5 m/s2 uma polia fixada. Um fio inextensível,
d) 7 m/s2 e) 8 m/s2 passando por duas polias, liga o corpo A ao
corpo B. Despreze todo tipo de atrito.
Inicialmente todos os corpos estão em repouso
A39) Um sistema mecânico de corpos é
e a distância entre o corpo A e a polia fixada
montado para o estudo da máquina de Atwood.
em C vale L. Suponha que 4mA = 2mB - mc.
A polia tem massa e momento de inércia
Determine o tempo que se passa desde o
desprezíveis. Os fios têm massa desprezível e
instante mostrado na figura e o instante que o
comprimentos constantes. O bloco A tem
corpo A se choca com a polia fixada em C.
massa 2m, B massa 3m, C massa m e D
Suponha que g é a aceleração da gravidade
massa m. A aceleração da gravidade no local
local.

71
Elementos da física - Mecânica I - Princípios da Dinâmica
- A44) Na figura, não existe atrito entre
quaisquer das superfícies. As massas da

Í3 BR I8 roldana A e da corda podem ser desprezadas.


Sabendo que nm = m, m2 = 2m e m3 = 3m,
determine a aceleração de m,.
!♦
0
I
L

m3
) a(\

a) JF
V 3ê
c)

V 3g
0
A42) Desde o ponto A, situado no extremo
A45) Um aluno resolveu determinar a aceleração de
superior do diâmetro vertical de certa
circunferência, por dois canais, AB e AC, um vagão. Amarrou uma mola ideal de constante
colocados ao longo de duas cordas distintas, elástica k e comprimento natural L no chão e no teto
são abandonados simultaneamente dois do vagão em dois pontos pertencentes a uma
corpos. Uma das cordas possui um ângulo a mesma reta vertical, como na figura 1. A seguir,
com relação à vertical e a outra corda possui cortou a mola a uma distância de 0,4L do teto e
um ângulo 0 com relação à vertical, de modo amarrou uma massa m, de dimensões desprezíveis,
que a > 0. Determine a relação entre os às duas novas molas, acelerando então o vagão.
tempos, tAB e tAC, que os corpos demoram para Após atingido o equilíbrio, figura 2, determinou os
atingir a circunferência. valores de a e 0. Sabendo que cos a = sen 0 = 0,6
e sendo g a aceleração da gravidade, calcule o
a) tAB = tAC
módulo da aceleração do vagão.
b) tABsen a = tACsen 0
c) tABcos a = tACcos 0
d) tABtg a = tACtg 0 F:
L
e) tABsec a = tACsec 0

A43) Um bloco de massa m, que está


TJ
I
Figura 1
conectado a uma parede através de um fio a
ideal, é abandonado em repouso sobre um
carrinho de massa M = 2m. Se o sistema pode
deslizar sem atrito, determinar a aceleração a
adquirida pelo sistema.
sen a = 0,6 cos a = 0,8
:•
Figura 2
m A)|g B)^g C) 18g D) ^g
a i 4 u y
E) os dados fornecidos são insuficientes

A46) Na figura, temos um sistema de massas,


polias e fios preso ao teto de um elevador. O
elevador tem aceleração para cima de
a) g/2 b) g/ 3 c) g/4 d) g/5 e) g/6 5 m/s2. A polia E e os fios são ideais. As
massas de A, B e C são respectivamente
iguias a 1 kg, 2 kg e 1 kg. A massa da polia D

72
Elementos tia Física - Mecânica 1 - Princípios da Dinâmica
é de 3 kg. Em relação ao piso do elevador, o zzzzz
sistema é abandonado a partir do repouso.
Sabendo que g = 10 m/s2, determine o vetor o
aceleração de A em relação a B. ui?

E Ia 1111 1113

DJ ///

000 A49) Considere uma máquina de Atwood


infinita, conforme mostra a figura. Uma corda
passa por cada polia, que em uma das
extremidades está conectada a uma massa e
na outra extremidade a uma polia. Todas as
A47) Na figura a seguir, os elevadores A e B massas são iguais ame todas as polias e
são iguais e têm massa 2m cada um. Dentro cordas são ideais. O sistema está em repouso
deles, há dois gêmeos C e D (de mesma inicialmente. Considerando-se a gravidade
massa m cada um) sobre duas balanças iguais local igual a g, determine a aceleração da
e de massas desprezíveis. Ligando os dois primeira massa (mais à esquerda na figura)
elevadores há uma corda de massa 3m, que quando o sistema for liberado.
passa por uma polia ideal, cujo raio é muito <zzzzzz>
pequeno se comparado com o comprimento da X
corda. No início, o sistema estava em equilíbrio, P)
com o mesmo comprimento de corda de cada
lado. Após uma leve perturbação, o elevador B mL
começa a descer. Determine a fração da corda
pendente do lado direito quando um gêmeo lê m
na balança um valor 50% maior que o outro
gêmeo, no mesmo instante. m
m S ;

E A50) Um bloco A, de massa m, repousa na


superfície horizontal de um prisma B, de
massa M, que se encontra também em
repouso sobre uma rampa inclinada de ângulo

A) 0,6
A

B) 0,7
2E B

C) 0,8
0, com a horizontal, conforme a figura a seguir.
Abandonando-se o sistema do repouso,
determine a aceleração adquirida pelo bloco A
D) 0,9 E) 1,0
(direção, sentido e intensidade). Desconsidere
todos os atritos e adote a gravidade g.
A48) O sistema da figura está em repouso.
Depois de abandoná-lo, determine o módulo
da aceleração do bloco 1. O bloco 2 é vazado, A
na forma de um túnel vertical, capaz de facilitar
o movimento do bloco 1. Não existe atrito em
todas as superfícies de contato. As polias são
ideais e o fio que une os blocos 1 e 3 é
inextensível.

73
REAÇÃO NORMAL E FORÇA DE ATRITO

No exemplo do cavalo que arrasta um bloco do capítulo anterior já foi referida a existência
de forças de contato entre duas superfícies. Essas forças podem apontar em qualquer direção,
mas o sentido é sempre no sentido em que as duas superfícies tendem a se afastar. É habitual
separar essas forças de contato em duas componentes, uma componente perpendicular às
superfícies em contato, chamada reação normal e outra componente tangente às superfícies,
denominada força de atrito.

> l —

R,

F,
l r
A força de contato entre superfícies é uma força distribuída em vários pontos da superfície.
A resultante de todas essas forças será representada em um ponto da superfície, separando as
componentes normal e tangencial. A reação normal, Rn terá sempre o sentido que faz separar os
dois corpos em contato. A força de atrito, Fa, pode ter qualquer um dos dois sentidos na direção
tangencial.

ATRITO ESTÁTICO

Quando não existe movimento relativo entre as duas superfícies em contato, a força de
atrito designa-se de atrito estático. A força de atrito estático pode ser nula, ou pode estar orientada
em qualquer dos dois sentidos na direção tangente às superfícies em contato.

No exemplo do cavalo e o bloco as forças de atrito nas ferraduras do cavalo são atrito
estático. A força de atrito estático faz possível colocar um veículo em movimento ou fazer com que
trave. É também a força que nos permite caminhar: empurramos com os nossos pés o chão e a
reação do chão no sentido oposto faz-nos avançar.

Mas se o chão estivesse coberto por gelo, os pés escorregavam para trás e não se
conseguia avançar para a frente. Isso acontece porque o módulo da força de atrito estático não
pode ultrapassar um valor máximo, que é proporcional à reação normal:

Fe — Pc^n

em que pe é uma constante própria do tipo de superfícies em contato, chamada coeficiente de


atrito estático. O coeficiente de atrito estático costuma ser menor que 1. Em termos da força de
contato completa, isso implica que a a força de contato costuma estar perto da direção normal,
com desvio máximo de menos de 45°.

Considere-se um exemplo: as forças entre a estrada e os pneus de uma bicicleta. As


forças de atrito entre os dois pneus e a estrada são ambas forças de atrito estático, porque as
rodas não escorregam. Na roda traseira a força de atrito aponta para a frente, na direção do

74
____________________________________________________ Elementos da Física - Mecânica I - Força de Atrito
movimento da bicicleta, como resultado da reação da estrada à ação que o pneu exerce sobre a
estrada no sentido oposto.

A força de atrito na roda da frente é no sentido oposto ao movimento, porque nessa roda
não é exercida nenhuma tração pelo ciclista. Para manter essa roda em rotação, contrariando o
atrito no eixo da roda, é preciso que a estrada atue com força de atrito no sentido oposto à
velocidade da bicicleta.

—-'—

Se a velocidade da bicicleta for constante, o módulo da força de atrito no pneu traseiro


deverá ser igual à soma dos módulos da força de atrito no pneu da frente e da resistência do ar.

ATRITO CINÉTICO

Quando as duas superfícies em contato deslizam entre si, a força de atrito designa-se de
atrito cinético. No exemplo do cavalo e o bloco a força de atrito que atua no bloco é atrito cinético.
A força de atrito cinético é sempre oposta ao movimento e tem módulo constante que depende da
reação normal:

Fc = UcRn

Em que pc é o coeficiente de atrito cinético, que costuma ser menor que o coeficiente de
atrito estático entre as mesmas superfícies. Por ser oposta ao movimento, a força de atrito cinético
faz sempre diminuir o valor da velocidade relativa entre as superfícies, mas nunca pode inverter o
sentido da velocidade. No instante em que a velocidade seja nula, a força de atrito cinético
também será nula.

Considere um experimento onde uma força F variável é aplicada sobre um corpo de massa
M, inicialmente em repouso sobre uma superfície áspera, como esquematizado na figura. Se F é
relativamente pequena, o corpo continua em repouso e neste caso, F = Fal. Note que se F = 0, Fal
= 0, indicando que a força de atrito só existe se houver tendência ao deslizamento. Se
continuarmos a aumentar F, esta atinge um valor máximo para o qual o corpo se encontra
iminência de deslizar. Neste ponto define-se o coeficiente de atrito estático como Fmax = peN. A
partir daí, o corpo entra em movimento e qualquer incremento em F contribui exclusivamente para
acelerar o corpo.

75
Elementos da Física - Mecânica I - Força de Atrito

F
M

Na situação de movimento, a força de atrito é Fal = pdN, onde pd é chamado de coeficiente


de atrito dinâmico. Assim, no regime estático Fat < peN e no regime dinâmico Fat = pdN, sendo pd<
pe (verificado experimentalmente).

Fat
iminência de deslizamento
HeN .
pdN-
deslizamento

45°

F
Como exemplo do cálculo de força de atrito, tomemos um corpo de massa M sobre um
plano inclinado, como mostra a figura seguinte. Da 2a lei de Newton:

N - Mg cos 0 = 0 e Mg sen 0 - Fat = Ma

No caso do corpo estar na iminência de deslizamento, a = 0 e Fat = peN. Desta forma,


segue que pe = tg 0.

F.

A análise apresentada sobre força de a atrito neste capítulo é uma representação


macroscópica aproximada de um fenômeno complexo. No nível microscópico, as forças de atrito e
a normal se devem à forças intermoleculares (fundamentalmente elétricas) entre duas superfícies
ásperas nos pontos onde entram em contato. Ao deslizar uma caixa sobre um piso, se formam e
se rompem encaixes entre ambas superfícies, e o número total de encaixes varia. Portanto, a
força de atrito cinética não é perfeitamente constante.

76
Elementos da Física - Mecânica I - Força de atrito

Bloco

-e-
Piso

Vista ampliada

Abaixo é possível verificar uma tabela com os valores médios dos coeficientes de atrito
estático e cinético entre algumas superfícies.

coeficiente de atrito estático coeficiente de atrito dinâmico


Interface
_____________ Pcstatico_____________ _____________ Pdinâmico_____________
__________ 0,5__________
_____pneu - asfalto seco_____ ___________ 03___________
pneu - asfalto molhado __________ 0,3__________ ___________ 0,15___________
pneu - paralelepipedo seco __________ 0,6__________ ___________ 03___________
pneu - paralelepipedo molhado __________ 03__________ ___________ 03___________
_____ madeira-madeira_______________ 0,54__________ ___________0,34___________
gelo-aço _________ 0,027_________ ___ _______0,014__________
_________ aço- aço_________ __________ 0,15__________ 0,12
_______ teflon-teflon_______ __________ 0,04__________
_______ nylon-nylon_______ __________ 0,15__________
_____aluminio-aluminio_____ __________ 13__________
borracha - papelão 0,6

Perceba que na maioria dos casos tem-se os coeficientes de atrito pertencentes ao


intervalo 0 < p < 1, porém nada impede que um coeficiente de atrito seja maior que 1. Por mais
lisa e polida que sejam duas superfícies de contato, na prática é impossível que um coeficiente de
atrito seja igual a 0.

77
______ ____________________________________________ Elementos da Física - Mecânica I - força de Atrito
ACELERAÇÃO SOBRE UM PLANO INCLINADO ÁSPERO

1) Corpo Descendo

Uma partícula de massa m é colocada sobre um plano inclinado áspero, cuja inclinação
com a horizontal é 0 e comprimento L. Suponha que o coeficiente de atrito estático pe é tal que
Pe < tg 0, ou seja, a partícula entra em movimento descendente quando colocado sobre o plano
inclinado.

Fat

p
L

Seja pc o coeficiente de atrito cinético entre o plano inclinado e a partícula. A partícula é


então colocada com velocidade nula no ponto de altura máxima sobre o plano inclinado. Pela
decomposição de forças, na direção perpendicular ao plano inclinado tem-se que:

N = P cos 0 => N = mg cos 6

Escrevendo a 2a Lei de Newton na direção paralela ao plano inclinado:

P sen 0 - Fat = ma => mg sen 0 - Npc = ma =>


mg sen 0 - mgpc cos 0 = ma => a = g(sen 0 - pc cos 0)

De posse do módulo da aceleração da partícula é possível determinar o módulo da


velocidade com que a partícula atinge a base do plano inclinado. Como esta aceleração é
constante pode-se aplicar a equação de Torricelli:

■> y° i---------------------------------
^f=)rQ +2aL => vf =A/2g(sen0-pccos0)L

O tempo que a partícula leva para percorrer todo o plano inclinado pode ser determinado
aplicando a equação horária do espaço:

’tX ' 2L
L= ^descida
2 sen0-pc cosô

78
Elementos da Física - Mecânica I - Força de atrito
2) Corpo Subindo

Considere agora que a partícula de massa m é lançada sobre o plano inclinado, de


inclinação 0 com a horizontal e comprimento L, com uma velocidade inicial v0, como indicado na
figura. Adote que o coeficiente de atrito estático pe entre a superfície do plano inclinado e a
partícula satisfaz pe < tg 0. Suponha que a velocidade v0, imposta à partícula na base do plano
inclinado, é suficiente para a partícula atingir o ponto mais alto do plano inclinado.

L
Fat
N Vo

e
P

Pelo equilíbrio de forças na direção perpendicular ao plano inclinado:

N = P cos 0 N = mg cos 0

Escrevendo a 2a Lei de Newton na direção paralela ao plano inclinado:

Psen0 + Fat = ma => mg sen 0 + Npc = ma =>


mg sen 0 + mgpccos 0 = ma => a = g(sen 0 + pccos 0)

Esta aceleração, que é paralela ao plano inclinado, está no sentido contrário ao movimento
da partícula, fazendo com que o módulo de sua velocidade diminua. Assim, o correto seria afirmar
que a = - g(sen 0 + pccos 0). De posse da aceleração da partícula é possível determinar o módulo
da velocidade com que a partícula atinge a base do plano inclinado. Como esta aceleração é
constante pode-se aplicar a equação de Torricelli:

Vf=v§+2aL => vf = -2g(sen0 + pc cos0)L

Assim, a velocidade inicial que deve ser imposta à partícula de modo que esta alcance o
topo do plano inclinado com velocidade nula é determinada fazendo v( igual a zero:

v0 = ^gísen 0 + p.c cos 0)L

Neste caso, o tempo que a partícula leva para percorrer todo o plano inclinado, atingindo
seu topo com velocidade nula, vale:

' 2L
^subida
2 sen0 + pccos0

79
Elementos da Física - Mecânica I - Força de fttrit»
FORÇA DE ATRITO EM DUAS DIMENSÕES

Suponha que um corpo de massa m é colocado sobre um plano inclinado áspero de modo
que pe > tg 0, ou seja, o corpo fica em repouso sobre o plano inclinado. Em determinando
momento uma força F, paralela à base do plano inclinado, é aplicada sobre o corpo, de modo que
este fique na iminência de deslizar sobre a superfície do plano inclinado.

y
Faty
\x
F
Fatx
\ P.sen 0
e

Na decomposição de forças indicada na figura destacam-se dois atritos. Fatx é uma força
de atrito estático, com sentido contrário ao da força F e se opõe à tendência de movimento ao
longo do eixo x. Analisando o equilíbrio de forças no eixo x segue que Fatx = F. Analogamente,
Faty é uma força de atrito estático e devido ao equilíbrio no eixo y tem-se que Faty = P.sen 0. A
força de atrito resultante é dada por:

FatR =7(Fatx)2 + (Faty)2 => FatR =-Jf2+(mgsen0)2

Note que a aplicação da força F não altera o valor da reação normal do plano inclinado
sobre o corpo, cujo módulo vale N = mg cos 0. Como o corpo está na iminência de movimento a
força de atrito resultante é igual ao produto do módulo da reação normal pelo coeficiente de atrito
estático:

FatR = N.pe = y]F2 +(mgsen0)2 => mgpecos0 = -jF2 +(mgsen0)2


F2j-r
(mgpe cos0)2 =c2 mr>oXxr,a\22
+(mgsen0) => F2 =m2g2(p2 cos2 0-sen2 0) =>
Fmin =mg7pe cos2 0-sen2 0

Assim, aplicando ao corpo de massa m uma força F de módulo mg-Jp2 cos2 0-sen20 o
corpo fica na iminência de movimento sobre o plano inclinado. Note que o valor dentro da raiz
quadrada é sempre positivo desde que foi convencionado que pe > tg 0.

80
Elementos da Física - Mecânica I - Força de Atrito
Exemplos:

1) (IME-82) Um corpo que repousa sobre uma superfície rugosa horizontal, recebe um impacto
horizontal e desliza sobre a referida superfície durante 5 segundos, quando pára tendo percorrido
25 m. Determine o coeficiente de atrito entre o corpo e a superfície horizontal. Nota: Considere g =
10 m/s2.
Solução:
Seja Ax o espaço percorrido pelo corpo ao longo do plano:
at2 „c a.25 „ . 2
Ax — => 25 =------ => a = 2 m/s
2 2
Como a única força horizontal que aplicada ao corpo é a força de atrito:
Fr = Fat = ma => mgp = ma => a = gp => 2 = 10p => p = 0,2

2) (UFPB-05) Um bloco de 1 kg está apoiado sobre uma prancha de 4 kg, como mostra a figura. O
bloco é puxado por uma força F horizontal. Os coeficientes de atrito estático e dinâmico entre o
bloco e a prancha são 0,8 e 0,6, respectivamente.

F ♦
C I
Considerando-se que o atrito entre a prancha e o solo é desprezível, então é correto afirmar que a
maior aceleração da prancha será:
a) 1,0 m/s2 b) 1,2 m/s2 c) 1,5 m/s2 d) 1,6 m/s2 e) 2,0 m/s2
Solução: Alternativa E
Analisando a decomposição de forças no bloco tem-se:
N,
F Na direção vertical: Nt = mg
Na direção horizontal: F - Fat = mai
Fat mg
Analisando a decomposição de forças na prancha tem-se:
Na direção vertical: N2 = Ni + Mg = mg + Mg = (M + m)g
N,| Fat N2 Fat
Na direção horizontal: Fat = Ma2 => a2 =-----
M

Mg Deste modo, a2 será máximo quando Fat for máximo. O máximo


valor de uma força de atrito ocorre quando existe iminência de
movimento, ou seja, no caso da força de atrito estático máxima:
Fatmáx N-iPest _ mgpest _ 1.10.0,8 2 o m/s2
máx
M MM 4

3) (ITA-90) A figura abaixo representa três blocos de massas M, = 1,00 kg, M2 = 2,50 kg e M3 =
0,50 kg, respectivamente. Entre os blocos e o piso que os apóia existe atrito, cujos coeficientes
cinético e estático são, respectivamente, 0,10 e 0,15, e a aceleração da gravidade vale 10,0 m/s2.
Se ao bloco M-, for aplicada uma força F horizontal de 10,00 N, pode-se afirmar que a força que
bloco 2 aplica sobre o bloco 3 vale:

F
------- * i 2 3

a) 0,25N b) 10.00N c)2,86N d) 1,25N e) Nenhuma das anteriores.


Solução:
A força de atrito estático máximo é dada por:
Fatest máx = NT.pest = ("li + m2 + m3)gpest = (4).(1 0).(0,1 5) => Fat est máx - 6,00 N
Como F>Fatest => os corpos se movimentam
Assim, aplicando a 2a Lei de Newton ao sistema formado pelos três corpos:

81
___________________________________________________ Elementos da Física - Mecânica I - Força de Atrito
F - Fat cin tolai = mTa => F - (m, + m2 + m3)gnan = (m, + m2 + m3)a =>
10,00 - (4)(10)(0,10) = 4a => a = 3/2 m/s2
A decomposição de forças, na direção do movimento, no corpo 3 está indicada abaixo. A força F2.3
é a força que o corpo 2 aplica no corpo 3. Escrevendo a 2a Lei de Newton no corpo 3:
F2-3 — Fat3 = m3a => F2.3 - m3gnCin = rn3a =>
^2-3 3 F2.3 - (0,50)(10)(0,10) = (0,50)(3/2) =>
Fat 3 F2.3 - 0,50 = 0,75 => FM = 1,25 N

4) (EN-89) Os coeficientes de atrito entre o bloco de massa igual a 20 kg e o carrinho de massa


igual a 10 kg são: pE = 0,20 e pc = 0,12. Considere uma força constante F de módulo igual a 100
N aplicada no carrinho. Desprezando-se o atrito entre o carrinho e o solo e usando g = 10 m/s2,
aceleração (em m/s2) do bloco em relação ao carrinho tem módulo igual a
a) 1,2
20 kg
b) 7,6
c) 6,4 10 kg
d) 10
e) zero w//////////^
Solução: Alternativa C

N, Note que existe uma diferença percentual razoável entre pE e pc-


Isso faz com que suposição automática de que os blocos
possuem movimento relativo entre si possa ser falsa. Isso ocorre
Fat mg quando a força resultante no bloco estiver no intervalo entre a
força de atrito cinético e a força de atrito estática máxima.
Fat
N„ N2
Suponha que o bloco e o carrinho não possuam movimento
relativo. Se isto realmente ocorre a alternativa correta seria E.
F
Neste caso tem-se:
F = (M + m)a => 100 = 30a => a = 10/3 = 3,33... m/s2
Mg A força resultante no bloco seria F, = ma = 200/3 = 66,66... N
A força de atrito estática máxima vale:
Fatestmax = N,pE = mgpE = 20.10.0,20 = 40 N
Encontrou-se uma contradição, uma vez que se nao existisse movimento relativo entre os corpos
deveria ocorrer FateStmax > F,. Assim, na realidade, o bloco desliza sobre o carrinho e a força de
atrito entre o bloco e o carinho é cinética.

Analisando a decomposição de forças no bloco tem-se:


Na direção vertical: N, = mg
Na direção horizontal: Fat = ma, => mgpc = ma. => a! = gpc = 10.0,12 = 1,2 m/s2

Analisando a decomposição de forças no carrinho tem-se:


Na direção horizontal: F - Fat = Ma2 => F-mggc = Ma2 => 100 - 20.10.0,12 = 10a2 =>
100-24 = 10a2 => a2 = 7,6m/s2

Logo, a aceleração relativa entre os corpos é aR = a2 - a! = 7,6 - 1,2 = 6,4 m/s2

5) (ITA-97) Um antigo vaso chinês está a uma distância d da extremidade de um forro sobre uma
mesa. Essa extremidade, por sua vez, se encontra a uma distância D de uma das bordas da mesa,
como mostrado na figura. Inicialmente tudo está em repouso. Você apostou que consegue puxar o
forro com uma aceleração constante a (veja figura) de tal forma que o vaso não caia da mesa.
Considere que ambos os coeficientes de atrito, estático e cinético, entre o vaso e o forro tenham
valor p e que o vaso pare no momento que toca na mesa. Você ganhará a aposta se a magnitude
da aceleração estiver dentro da faixa:

82
Elementos da Física - Mecânica I - Força de Atrito

d—

u r
------- D-

a) a < ^Hg b)a>^pg


b) a > c) a > pg d) a > -ng e) a >
d D -d
Solução: Alternativa E
Note que a força resultante no vaso é exatamente a força de atrito cinético entre as superfícies do
vaso e do forro. Logo:
Fal = m.av => m.g.p = m.av => av = gp
De forma que o vaso não caia da mesa, a distância percorrida pelo vaso em um tempo t deve
menor que D - d enquanto o forro percorre, no mesmo intervalo de tempo t, uma distância D:
Ax(orro = D = at2^ => t2^ = D/a
Axvaso < D - d => a„t2/2 < D - d => gpD/a < D - d => a > Dgp/(D - d)

6) (ITA-07) A partir do nível P, com velocidade inicial de 5m/s, um corpo sobe a superfície de um
plano inclinado PQ de 0,8 m de comprimento. Sabe-se que o coeficiente de atrito cinético entre o
plano e o corpo é igual a 1/3. Considere a aceleração da gravidade g = 10m/s2, sem 0 = 0,8, cos 0
= 0,6 e que o ar não oferece resistência. O tempo mínimo de percurso do corpo para que se torne
nulo o componente vertical de sua velocidade é.
Q

4S

/////////////////z p
V
X
a) 0,20 s. b) 0,24s. c) 0,40s. d) 0,44s. e) 0,48s.
Solução: Alternativa D
No trajeto PQ a força resultante no corpo é dada por:
FR = P, + Fat => maR = mgsen 0 + mgpcos 0 => aR=10m/s2
Aplicando a equação de Torricelli pode-se determinar a velocidade com que o corpo chega em Q:
vQ2 = vP2 - 2aAe => vQ = 3,0 m/s
O tempo que o corpo leva para percorrer a distância PQ vale:
vQ = vP - a.At, => AÍt = 0,20 s
A partir de Q o corpo é lançado obliquamente. Neste caso, analisando a componente vertical da
velocidade do corpo é possível determinar o tempo que o corpo demora para chegar no ponto de
altura máxima:
vQ.sen 0 = g.At2 => At2 = 0,24 s
Logo: AtT = At, + At2 = 0,44 s

7) (ITA-98) Um caixote de peso W é puxado sobre um trilho horizontal por uma força de
magnitude F que forma um ângulo 0 em relação à horizontal, como mostra a figura. Dado que o
coeficiente de atrito estático entre o caixote e o trilho é p, o valor mínimo de F, a partir de qual
seria possível mover o caixote, é:

W| 0

. 2W Wsen0 pWsen0 pWsec0


a) ------ b) --------------
b) c) d) e) (1 - ji tan 0)W
1-p 1-|itan0 1-ptan0 1-ptan0
Solução: Alternativa D

83
Elementos da Física - Mecânica I - Forca de Atrito.
Na direção vertical tem-se:
4N . N = W + Fsen 0;
Na direção horizontal:
Fcos 0 = Fat = Np => Fcos 0 = Wp + Fpsen 0 =>
w p_ Wp p_Wpsec0
=>
cos0-psen0 1-ptg0

8) (ITA-08) Na figura, um bloco sobe um plano inclinado, com velocidade inicial Vo. Considere h o
coeficiente de atrito entre o bloco e a superfície. Indique a sua velocidade na descida ao passar
pela posição inicial.

'sen0-psen0 sen 0 - p cos 0 sen0 + pcos0


a) Vo, b) Vo,> -------- ------- c) Vo,
COS0-pCOS0 y sen0 + pcos0 sen 0 - p cos 0
'psenO + cos© /psen0 -cos0
d) Vo, e) Vo,1 —-----------------
psen0cos0 y psen0cos0

Solução: Alternativa B
Na subida o módulo da aceleração é dado por: a, = g(sen 0 + p.cos 0)
Na descida o módulo da aceleração é dado por: a2 = g(sen 0 - p.cos 0)
Na subida: v,2 = Vo2 - 2a,L => v02 = 2a,L
Na descida: v22 = v,2 + 2a2L => v22 = 2a2L
v2 a sen0-p.cos0
Dividindo as duas equações: = — => V2=V0
v02 a, sen0 + p.cos0

9) (ITA-12) Considere uma rampa plana, inclinada de um ângulo 9 em relação à horizontal, no


inicio da qual encontra-se um carrinho. Ele então recebe uma pancada que o faz subir até umai
certa distância, durante o tempo ts, descendo em seguida até sua posição inicial. A “viagem”
completa dura um tempo total t. Sendo p o coeficiente de atrito cinético entre o carrinho e a rampai,
a relação í/ísé igual a
a) 2 b) 1 + ,y(tan0 + p)/tan0-p
c) 1 + 7(c°s0 + p)/cos0-p d) 1 + A/(sen0 + p)/cos0-p
e) 1 - 0 + p) / tan 0 - p

Solução: Alternativa B
Sabe-se que no movimento de subida a aceleração resultante é as = g(sen 0 + pcos 0), enquanto
que no movimento de descida a aceleração resultante é as = g(sen 0 - pcos 0)
Í2í
Deste modo pode-se determinar os tempos de subida e de descida utilizando a relação t = J—

l),--jf-4(sene240o'^; ">•- _ Í2£ = | 21


yad y g(sen0-pcos0)
Logo: l = k±k = 1 + k = /sen0 + pcos0 'tge+p
1+-3-
t, ts tts8 ysenO-pcos0 tg0-p

84
______________________________________________ Elementos da Física - Mecânica I - Força de Atrito
10) (IME-76) Um de peso P repousa sobre uma superfície horizontal. O coeficiente de atrito
estático entre o bloco e a superfície horizontal é p. Empurra-se o bloco com uma força F que
forma um ângulo 0 com a horizontal conforme esquematizado na figura. A partir destes dados,
estabeleça uma expressão para o ângulo tfalém do qual não é possível mover o bloco, por maior
que seja a força F.
F

e
X

Solução:
Note que aumentando o valor de 0 (0 < 0 < 90°) aumenta-se o módulo de N,
N provocando um aumento do módulo da força de atrito estático máximo. Por
outro lado, aumentando o valor de 0 diminui-se a componente de F na direção
horizontal (F.cos 0) que faz o corpo ter a tendência de se movimentar. Assim,
para qualquer valor de 0 acima do ângulo em que a força necessária para
Fat movimentar o bloco tende para infinito, o sistema não se movimentará.
P
Equilíbrio na direção vertical: N = P + F.sen 0
Equilíbrio da direção horizontal: F.cos 0 = Fat = N.p =>
p
F.cos 0 = (P + F.sen 0)u. => F.cos 0 - Fusen 0 = P => F =---------
cos0-psen0
F tende para infinito quando 0 tende para zero, ou seja: cos 0 - psen 0 = 0 => tg0 = —
P
1
Logo, para um ângulo 0 > arc tg— o bloco não se movimentará, independentemente do módulo da
F
força F.

11) (ITA-03) Na figura, o carrinho com rampa movimenta-se com uma aceleração constante Ã.
Sobre a rampa repousa um bloco de massa m. Se jiéo coeficiente de atrito estático entre o bloco
e a rampa, determine o intervalo para o módulo de Ã, no qual o bloco permanecerá em repouso
sobre a rampa.

Solução
A força resultante no corpo de massa m é dada por FR = m.A
Adotando um referencial não inercial que move com a mesma aceleração
 do sistema, neste sistema o bloco está em repouso sobre o carrinho.
Neste referencial não inercial surge uma força de inércia -m no bloco,
como indicado na decomposição de forças.
A normal com o plano inclinado pode ser calculada pelo equilíbrio de
forças, no referencial não inercial, que existe na direção perpendicular ao plano inclinado:
N = m.g.cos a - FR.sen a = m(g.cos a - A.sen a)
No referencial não inercial existe equilíbrio na direção paralela à rampa do carrinho:
mA.cos a + P( = Fat => m.A.cos a + m.g.sen a = N.p =>

85
_____________ _____________________________________ Elementos da Física - Mecânica I - Forçade iltrft
m.A.cos g + m.g.sen g = m.p(g.cos g-A.sen g) => A(cos g + p.sen g) = g(p.cos a - sen a) =
g(p.cosg- sen a)
cosg + p.seng
Como este é o valor máximo do módulo de Â, então o intervalo procurado é:
- . a(p.cosg - seng)
0 < A < ——-------------------- -, onde p.cos g > sen g
cos g + p. sen g

12) (IME-88) Um carro de peso Q, provido de uma rampa fixa e inclinada de ângulo a, suporta uirr
bloco de peso P. O coeficiente de atrito estático entre o bloco e a rampa vale p. Não há atritc
entre o carro e o chão. Determine: a) o maior valor da aceleração com a qual o carro pode sei
movimentado sem que o corpo comece a subir a rampa, b) a intensidade F da força horizontal
correspondente.

5^

Solução:
a) Adotando um sistema de eixos que se movimenta com a mesmia
aceleração ã do carro, tem-se que o bloco fica em equilíbrio relativo ao carro.
Neste referencial surge no bloco uma força de inércia cujo módulo F, é igual
ao produto da massa do bloco pela aceleração do referencial. Na direção
perpendicular ao plano inclinado:
N = P.cos g + Fj.sen g
Na direção paralela ao plano inclinado:
Fj.cos g = P.sen g + N.p => Fj.cos g = P.sen g + p(P.cos g Fj.sen a) =>
p P(seng + pcosg)
Fj(cos g - p.sen g) = P(sen g + p.cos g) => Fj=—a
g cosg-pseng
a sen g + p cos g
cosg-pseng
P+Q (sen g + p COS g)(P + Q)
b)F -------- a => F=
g cosg-pseng

13) (ITA-04) Um atleta mantém-se suspenso em equilíbrio, forçando as mãos contra duas paredes
verticais, perpendiculares entre si, dispondo seu corpo simetricamente em relação ao canto e
mantendo seus braços horizontais alinhados, como mostra a figura. Sendo m, a massa do corpo
atleta e p o coeficiente de atrito estático interveniente, assinale a opção correta que indica □
módulo mínimo da força exercida pelo atleta em cada parede.

~ifp2
a) ^1 - b) + c) p* -1 d)
2 +1
e) n.d.a.
2 +J ' 2 l p2 -1 p2 +1 ' 2 1 p2 -1

86
Flementos da Física - Mecânica I - Força de Atrito
Solução: Alternativa B
Seja Fh a força horizontal exercida pelo homem sobre a parede. Como seus braços estão na
horizontal, a única direção possível para a força que o homem, intuitivamente, tenta fazer para
evitar que caia, é a direção horizontal. Entretanto, o peso do homem é transmitido, através de
seus braços, para a região de contato das mãos com a parede. Inicialmente, para facilitar a
visualização da figura,vamos dividi-la ao meio.

Note que, devido à simetria da figura, a força FH faz um ângulo de 45°


com a parede. Decompondo as forças, encontramos que:
F -fh^ F _rng
rat1 - 2 e rat2 ~ 2

> Como a força de atrito resultante é igual ao produto do módulo da


N P/2 normal com o coeficiente de atrito:
Fal2
N.H = 7(Fati)2+(Fat2)2 FhV2 Fh2 (mg)2
p=
2 2 4
FH2|i2 _ Fh2 , (mg)2 R mg
2’2 4
Fh = 1

que é o valor da força horizontal exercida pelo atleta sobre a parede.


O valor da resultante total que o atleta aplica da parede é:

FK=J(FH)2 +
mg)
\2
' (mg)2 (mg)7 _ mg fe2_+ 1
p _ “'a
2 J 2(p2-1) 4 R
R o
2 Vn2-1

14) (ITA-05) Considere uma rampa de ângulo 0 com a horizontal sobre a qual desce um vagão,
com aceleração ã , em cujo teto está dependurada uma mola de comprimento /, de massa
desprezível e constante de mola k, tendo uma massa m fixada na sua extremidade. Considerando
que l0 é o comprimento natural da mola e que o sistema está em repouso com relação ao vagão,
pode-se dizer que a mola sofreu uma variação de comprimento A/ = /-10 dada por
a) A/ = mg sen 0/k.
b) A/ = mg cos 0 / k. l/t
c) A/ = mg/k. m* ! ã
d) A/ = m^a2 -2agcos0 + g2 /k.
0
e) A/ = mja2 - 2ag sen 0 + g2 /k.

Solução: Alternativa E
Adotando-se o referencial não inercial e representando a força inercial, obtemos o seguinte
diagrama de forças.
^xpa Para um observador dentro do vagão a massa m está em repouso, por isso o
90°^e" triângulo de forças é fechado.
mg Aplicando a lei dos cossenos no triângulo, obtemos:
(kAí)2 =m2g2 + m2a2 - 2m2agcos(90° -0)
kAf
Logo, Aí = msja2 -2agsen0 + g2 /k

Observação: Na opinião do autor deste livro, com todo o respeito aos elaboradores das questões
do vestibular do ITA, nesta questão perdeu-se uma ótima oportunidade de elaborar uma questão
muito superior à que foi proposta. Note que a resposta correta foi encontrada pela simples
aplicação de um teorema dos cossenos. Porém, sendo p o coeficiente de atrito entre o vagão e a
superfície da rampa, sabe-se que a aceleração resultante do sistema vale a = g(sen 0 - pcos 0).
Substituindo o valor da aceleração na expressão encontrada para Aí:

87
_______ __________________________________ Elementos da Física - Mecânica I - Força de fttrlti
mV(g(sen0-pcos0)2 -2g2(sen0-pcos6)sen0 + g2
Aí = =>
__________________ k__________________________________
mg^sen2 0-^isen-Ocósfí + p2 cos2 0-2sen2 0 + 2psen^cõsl3 +1
-

mgVcos2 0 + p2 cos2 0 mgcos0V1 + p2


Aí k => Aí
— k
Perceba que bastava ter informado o coeficiente de atrito cinético que a expressão da elongação
da mola se torna bem mais simples que a original.

15) (ITA-12) A figura mostra um sistema formado por dois blocos, A e B, cada um com massa m,
O bloco A pode deslocar-se sobre a superfície plana e horizontal onde se encontra. O bloco B
está conectado a um fio inextensível fixado à parede, e que passa por uma polia ideal com eixo
preso ao bloco A. Um suporte vertical sem atrito mantém o bloco B descendo sempre paralelo a
ele, conforme mostra a figura. Sendo p o coeficiente do atrito cinético ente o bloco A e a superfície.
g a aceleração da gravidade, e 0 = 30° mantido constante, determine a tração no fio após o
sistema ser abandonado do repouso.

A
e
s B|

iiiiiiiiiiiiiniiiiiiiiiiiiiiiil)lliililll)inniln))l)lllllll)

SOLUÇÃO IDEAL:
2a Lei de Newton no bloco A em x: T.cos 30° - pN - N’ = ma (I)
T Equilíbrio das forças no bloco A em y: N = mg + T - T.sen 30° (II)
N’ Substituindo (II) em (I):
TI —► T.cos 30° - p(mg + T - T.sen 30°) - N’ = ma =>
+ N’ TVã ( TT, .
mg N mg ——pl mg + — l-N =ma (III)
2
Fat 2a Lei de Newton no bloco B em x: N’ = ma (IV)
Substituindo em (IV) em (III):
TV3 , T. , 2m(pg + 2a)
—pl mg + — -ma ma => T-V3-2mgp-Tp = 4ma => T (V)
2 V3-p
Existe um vínculo geométrico nos movimentos dos corpos:
* a ono at2 ay‘2 ^3 2a
Ax = Ay.cos30° => — = —--------- =>=> a„=-7=
ay (VI)
2 2 2 y Vã
2a Lei de Newton no bloco B em y: mg-T = may => T Vã
2a = gVã- — (VII)
m
2m (pg + gVã _T 2mg(Vã + p)
Logo, substituindo (VII) em (V), segue que: T '3
>/3-p m 3Vã-p

16) (IME-97) Um corpo de 4kg é puxado para cima por uma corda com a velocidade constante
igual a 2 m/s. Quando atinge a altura de 7 m em relação ao nível da areia de um reservatório, a
corda se rompe, o corpo cai e penetra no reservatório de areia, que proporciona uma força
constante de atrito igual a 50 N. É verificado que o corpo leva 4 s dentro do reservatório até atingir
o fundo. Faça um esboço gráfico da velocidade do corpo em função do tempo, desde o instante
em que a corda se rompe (Po) até atingir o fundo do reservatório (P2), indicando os valores para
os pontos Po, Pi e P2, sendo P, o início do reservatório.

88
Elementos da Física - Mecânica I - Força de Atrito
Dado: g = 10 m/s2
-= 2m/s
Po TA
1 corpo

7m

Pi

P2
reservatório de areia
Solução:
Será adotado um eixo y vertical com sentido de baixo para cima.
Neste eixo a velocidade v2 do corpo ao atingir o reservatório de areia é dada por:
| v21= ^vf +2gh =V4 + 2.10.7 =12 m/s => v2 = -12m/s
O tempo ti que o corpo leva para atingir a areia é:
v2 = Vt - gt 1 => - 12 = 2 - 10t, => t-j = 1,4 s
Assim, desde o instante em que o corpo é lançado até o momento que atinge a areia a equação
horária da velocidade do corpo é:
v(t) = Vi - gt => v(t) = 2 - 10t para 0 < t < 1,4
No interior da areia a aceleração do corpo vale:
F-P = ma => 50-40 = 4.a => a = 2,5 m/s2
Deste modo, a equação horária da velocidade do corpo no interior da areia é:
v’(t) = v2 + a(t - ti) => v’(t) = - 12 + 2,5(t - 1,4) => v’(t) = - 15,5 + 2,5t para 1,4 í t < 5,4 s
v(m/s)
2
\1.4

-2

-12

17) Qual a máxima aceleração que pode ser dada ao carro, mantendo o corpo (1) parado em
relação ao carro? O coeficiente de atrito entre o corpo (1) e o carro é p.
m ------------------------- 7—x

a
m

Solução:

89
Elementos da Física - Mecânica I - Força dejtrii»

N Na figura ao lado estão destacadas as


decomposições de forças que atuam nos
blocos 1 e 2, bem como a soma vetorial
Fat mg T mg
que gera o vetor resultante no corpo 2.
a
(2) ma Analisando as forças que atuam no corpo 1
"Hng conclui-se que:
N = mg
Fat-T = ma => Np-T = ma =>
mgp-T = ma => T = m(gp-a)
Na soma vetorial de forças no bloco 2: T = ^/(mg)2 +(ma)2 => => T = rn^/g2 + a2

Logo: gp-a=Vg2+a2 g(p2-D


=> g2p2 - 2agp + a2 = g2 + a2 => gn2 — Za,A = g => a=
2p
Observação: Note que a situação proposta somente é possível se p > 1.

18) Um vagão de trem de massa M desce um plano inclinado de inclinação 6, tendo preso ao seu
teto um fio que sustenta um corpo de massa m. O coeficiente de atrito entre o vagão e o plano
inclinado é n (p < tg 0) e a aceleração da gravidade é g. Determinar o ângulo a que o fio forma
com o eixo vertical, estando o corpo em equilíbrio relativamente ao vagão.

Solução:
Na figura ao lado estão indicadas as forças que atuam no corpo de massa
m e sua relação com a geometria do sistema. A força peso deve ser
vertical, a tração deve estar ao longo do direção do fio e a força resultante
deve ser paralela ao plano inclinado, que justifica a figura ao lado. Note na
a’
figura que FR=P + T . As projeções de P nas direções paralela e
Pcos 0 perpendicular ao plano inclinado geram forças de intensidades P.sen 6 e
P.cos 0, respectivamente. Sabe-se que a aceleração resultante no corpo m
p é dada por:
T
aR = gsen 0 - gpcos 0
Logo, multiplicando esta expressão por m:
Fr maR = mgsen 0 - mgpcos 0 => FR = Psen 0 - F, onde F = Ppcos 0
Psen 0 Assim, na geometria da figura, a distância entre o corpo mea força de
módulo P.cos 0 equivale à força F = Ppcos 0.
r, , , . ,n . F P.p.cos0 tg0-tga
Deste modo: tg(0-a) =---- = —---- ==pp => => —2---z— = M
n —n
P.COS0 P.cos0 1 + tg0.tga
tg 0-tg a = p + p.tg 0.tg a => (1 + p.tg 0)tg a = tg 0 - p tga = tge-p
1 + ptg0

90
____________________________________________________ Elementos da Física - Mecânica I - Força de atrito
19) Um engenheiro está projetando a construção do segundo andar de uma fábrica, onde será
instalado um depósito de produtos. Neste andar será construída uma rampa que será utilizada
para transportar caixas com os produtos até os caminhões que fazem a distribuição. O engenheiro
conhece a distância ( (fixa) entre o prédio e o ponto em que os caminhões param para carga,
tomada em relação à parte traseira do caminhão. Considere conhecido o coeficiente de atrito p
entre cada caixa e a rampa e a aceleração da gravidade g local.
a) Determine sob que ângulo a com a horizontal esta rampa deve ser construída de modo que o
tempo que cada caixa leva para ir do segundo andar até o caminhão seja mínimo.
b) Determine este tempo mínimo.

n n

□n a

◄----------- (----------- ►

Solução:
a) Sabe-se que a aceleração de descida das caixas vale a = g(sen a - p.cos a). O comprimento x
(
da rampa é dado por x =-------- . Assim, o tempo para cada caixa descer a rampa vale:
cosa

At
\ a
I
\ gcosa(sena-pcosa)
2e (1)
Desta forma, o tempo At será mínimo quando cos a(sen a - p.cos a) for máximo:
, . 2 sen2a p(cos2a + 1) sen2a - pcos2a - p
cos a(sen a - p.cos a) = sen a.cos a - p.cos a = —-—■ - —---- - ------- =------------- -----------

7l + p2 sen 2a. __ 1 = - cos 2a. p


-M
a/i + M2: + M2 ?
2
2
1 P
Como + = 1 então existe um arco p, do 1o quadrante do ciclo trigonométrico,
.71 + P2 ,
1 P
tal que cosp = e senp
V1 + P2 V^ + P2
_-Jl + p2 (sen2a.cosp-cos2a.senp)-p ^1 + p2. sen(2a - p) - p
Assim: cos a(sen a - p.cos a) _ = _
Logo, cos a(sen a - p.cos a) será máximo quando sen (2a - P) for máximo, que ocorre quando
sen (2a - p) = 1, ou seja:

2a-p=| 2a = P + - => tg2a = tgí p+-| j = -cotgp => tg 2a =- 2


2
I 2'
l
b) Substituindo cos a(sen a - p.cos a) = ——----- — em (1): At = 2
g(V1 + m2 - m)

91
Elementos da Física - Mecânica I - Força de fltritl

Exercícios de Embasamento é a aceleração da gravidade, dada em metros


por segundo ao quadrado.
E1) (PUC/PR-12) Sobre um bloco de 15 kg em 04) O coeficiente de atrito cinético entre s
superfície e o bloco é maior que X/(mg), err
repouso, em uma superfície plana e horizontal,
que g é a aceleração da gravidade, dada emn
aplica-se uma força F formando um ângulo 0 metros por segundo ao quadrado.
com a horizontal. Os coeficientes de atrito 08) No S.I., tanto os coeficientes de atrito
estático e cinético entre a superfície e o bloco cinético e estático são dados em newtons.
valem, respectivamente, 0,3 e 0,2. Sabendo 16) A força de atrito estático é sempre maior
que a força tem intensidade de 100 N, qual é, que a força de atrito cinético.
aproximadamente, a aceleração adquirida pelo
bloco? E4) (Mackenzie-01) Duas pequenas caixas
Considere: sen 0 = 0,8 e cos 0 = 0,6. cúbicas idênticas são empurradas,
A) 10 m/s2 B) 3,1 m/s2 C) 6 m/s2
simultaneamente, uma contra a outra, sobre
D) 3,6 m/s2 E) 4,4 m/s2
uma reta horizontal, a partir dos pontos A e B,
com velocidades de módulos respectivamente
E2) (PUC/PR-13) Um bloco de 10 kg encontra- iguais a 7,2 km/h, em relação à reta. O choque
se em repouso sobre um plano inclinado, frontal entre elas ocorre no ponto C, médio dle
conforme figura a seguir. Sabe-se que os AB, com a velocidade de uma das caixas igual
coeficientes de atrito estático e dinâmico entre a 7,2 km/h, em relação à outra. Considerandlo
o corpo e a superfície são, respectivamente,
que apenas o atrito cinético, de coeficiente pC ,
0,75 e 0,70. Para esse plano também
entre as caixas e o plano de deslocamento foi
considere sen 0 = 0,6 e cos 0 = 0,8.
o responsável pela variação de suais
velocidades, podemos afirmar que:
2,0 m

h H c

Instante do empurrão

9 I
A partir do exposto, pode-se marcar como
CORRETA qual das alternativas a seguir? o°o
(Adote g = 10 m/s2) a;
— iB
A) O corpo desce acelerado. O módulo da
aceleração é de 6 m/s2. Instante do choque
B) Faltam dados para calcular a situação em
que o corpo se encontra. a) mc = 0,05 b) Mc = 0,1 C) Mc = 0,15
C) O corpo desce em movimento uniforme. As d) mc = 0,2 e) Mc = 0,3
forças que atuam no corpo se anulam.
D) A força de atrito torna-se superior à E5) (Mackenzie-03) Um corpo de peso P sobe
intensidade do componente tangencial ao o plano inclinado com movimento acelerado,
plano da força Peso. Sendo assim, mesmo que
devido à ação da força horizontal F , de
muito lentamente, o corpo irá subir o plano.
intensidade igual ao dobro da deu seu peso. O
E) O corpo permanece em repouso. A situação
atrito entre as superfícies em contato tem
será de iminência de movimento.
coeficiente dinâmico igual a 0,40. O valor da
aceleração do corpo é: (Dados: g = 10 m/s2;
E3) (UEM-12) Supondo que um bloco de
cos a = 0,8; sen a = 0,6)
massa m kg esteja sobre uma superfície plana
e horizontal e que para mover esse bloco uma
força ligeiramente maior que X N é necessária,
assinale o que for correto.
01) A força de atrito estático máxima é igual a
X N.
________
02) O coeficiente de atrito estático entre a
superfície e o bloco é igual a X/(mg), em que g a) 3,5 m/s2 b) 3,0 m/s2 c) 2,5 m/s2
d) 2,0 m/s2 e) 1,5 m/s2

92
Elementos da Física - Mecânica I - força de atrito

E6) (Mackenzie-04) Os corpos A e B da figura


são idênticos e estão ligados por meio de um
fio suposto ideal. A polia possui inércia
desprezível, a superfície I é altamente polida e
o coeficiente de atrito cinético entre a
superfície II e o corpo B é p = 0,20. Em
determinado instante, o corpo A está descendo
com velocidade escalar 3,0 m/s. Após 2,0 s,
sua velocidade escalar será:
í í

Dados:
A B
sen a = 0,6
cosa = 0,8
g = 10m/s2
SUPERFÍCIE l/ \SUPERFÍCIE II
60>\ a) 0,25 e 16,00m b) 0,50 e 8,00m
Abo’
c) 0,25 e 8,00m d) 0,50 e 16,00m
e) 0,20 e 16,00m
a) 0 b)1,0m/s c) 2,0 m/s
d) 3,0 m/s e) 4,0 m/s E9) (Mackenzie-09) Um bloco A, de massa 6kg,
está preso a outro B, de massa 4kg, por meio
E7) (Mackenzie-04) O bloco da figura é de uma mola ideal de constante elástica
abandonado do repouso no ponto A e atinge o 800N/m. Os blocos estão apoiados sobre uma
ponto B, distante 3,00 m de A, após 1,41 s. Se superfície horizontal e se movimentam devido
não existisse atrito entre o bloco e a superfície à ação da força F horizontal, de intensidade
de apoio, o bloco iria de A para B em 1,00 s. O 60N. Sendo o coeficiente de atrito cinético
coeficiente de atrito cinético entre o bloco e a entre as superfícies em contato igual a 0,4, a
superfície plana inclinada é: distensão da mola é de
F
B .ÍWO, A
A
3,00 m z Dado:
✓ g = 10 m/s2

a) 3cm d) 6cm
b) 4cm e) 7cm
c) 5cm
-----------------
a) 0,375 b) 0,420 c) 0,475 E10) (Mackenzie-10) Um corpo de peso 30N
d) 0,525 e) 0,575 repousa sobre uma superfície horizontal de
coeficiente de atrito estático 0,4. Por meio de
E8) (Mackenzie-09) Certo corpo começa a uma mola de massa desprezível, de
deslizar, em linha reta, por um plano inclinado, comprimento natural 20cm e constante elástica
a partir do repouso na posição x0 = 0. 20N/m, prende-se esse corpo em uma parede
Sabendo-se que após 1,00s de movimento, ele como mostra a figura.
passa pela posição x, = 1,00m e que, com
mais 3,00s, ele chega à posição x21 o
coeficiente de atrito cinético entre as
superfícies em contato (pc) e a posição x2 são,
respectivamente, iguais a 'UT

A máxima distância a que podemos manter


esse corpo da parede e em equilíbrio será de
a) 26cm b) 40cm c) 80cm
d) 90cm e) 100cm

93
_____ Elementos da física - Mecânica I - Força de Atrita
E14) (Mackenzie-14) Na figura abaixo, a mola
E11) (Mackenzie-12) Um corpo de 5kg está em M, os fios e a polia possuem inércia
movimento devido à ação da força F, de desprezível e o coeficiente de atrito estática
intensidade 50N, como mostra a figura abaixo. entre o bloco B, de massa 2,80 kg, e o plana
inclinado é m = 0,50. O sistema ilustrado se
encontra em equilíbrio e representa o instante
em que o bloco B está na iminência de entrar
em movimento descendente.

S M

O B
cos37° = 0,8 e sen37° = 0,6
rp
O coeficiente de atrito cinético entre a EE
superfície de apoio horizontal e o bloco é 0,6 e
a aceleração da gravidade no local tem módulo 9
igual a 10m/s2. A aceleração com a qual o
corpo está se deslocando tem intensidade Dados: g = 10 m/s2, sen 0 = 0,80 e cos 0 = 0,60
a) 2,4m/s2 b) 3,6m/s2 c) 4,2m/s2 Sabendo-se que a constante elástica da mola
d) 5,6m/s2 e) 6,2m/s2 é k = 350 N/m, nesse instante, a distensão da
mola M, em relação ao seu comprimento
E12) (Mackenzie-13) Certo menino encontra- natural é de
se sentado sobre uma prancha plana e desce a) 0,40 cm b) 0,20 cm c) 1,3 cm
por uma rampa inclinada, conforme ilustração d) 2,0 cm e) 4,0 cm
abaixo. O coeficiente de atrito cinético entre a
prancha e a rampa é pc = 0,25, cos 0 = 0,8, E15) (Mackenzie-15)
sen 0 = 0,6 e g = 10m/s2. . v (m/s)

5.0

A; 4,0 ■

3.0-
B ;
2.0-
fa___ t3 __01
í 2,4 m i.o-
Sabe-se que o conjunto, menino e prancha,
* t (s)
possui massa de 50 kg e que ao passar pelo 0.0 2.0 4.0 6.0 8,0 ] 0.0
ponto A, sua velocidade era 1,0 m/s. A Um corpo de massa 2,0 kg é lançado sobre um
variação de quantidade de movimento sofrida plano horizontal rugoso com uma velocidade
por esse conjunto entre os pontos A e B foi inicial de 5,0 m/s e sua velocidade varia com o
a) 100N.S d)400N.s tempo, segundo o gráfico acima. Considerando
b) 200N.S e) 500N.S a aceleração da gravidade g = 10,0 m/s2, o
c) 300N.S coeficiente de atrito cinético entre o corpo e o
plano vale
13) (Mackenzie-13) Um aluno observa em a) 5,0.10’2 b) 5.0.10-1 c) 1,0. 10~1
certo instante um bloco com velocidade de d) 2,0.10'1 e)2,0.10-2
5m/s sobre uma superfície plana e horizontal.
Esse bloco desliza sobre essa superfície e E16) (UFPE-96) A figura mostra um bloco que
para após percorrer 5m. Sendo g = 10m/s2, o escorrega, a partir do repouso, ao longo de um
coeficiente de atrito cinético entre o bloco e a plano inclinado. Se a atrito fosse eliminado, o
superfície é bloco escorregaria na metade do tempo. Dê o
a) 0,75 b) 0,60 c)0,45 d) 0,37 e) 0,25 valor do coeficiente de atrito cinético, entre o
bloco e o plano.

94
Elementos da Física - Mecânica I - Força de Atrito

I g

E17) (UFPR-03) Dois blocos de massas iguais E19) (UFPE-10) Considere dois blocos
a 2,0 kg e 4,0 kg estão presos entre si por um empilhados, A e B, de massas mA = 1,0 kg e
fio inextensível e de massa desprezível. Como mB = 2,0 kg. Com a aplicação de uma força
representado abaixo, o conjunto pode ser horizontal F sobre o bloco A, o conjunto move-
puxado de duas formas distintas sobre uma se sem ocorrer deslizamento entre os blocos.
mesa, por uma força F paralela à mesa. O O coeficiente de atrito estático entre as
coeficiente de atrito estático entre os blocos e superfícies dos blocos A e B é igual a 0,60, e
a mesa é igual a 0,20. O fio entre os blocos não há atrito entre o bloco B e a superfície
pode suportar uma tração de até 10 N sem se horizontal. Determine o valor máximo do
romper. Com base nesses dados, é correto módulo da força F, em newtons, para que não
afirmar: ocorra deslizamento entre os blocos.
-F ------------
U. 2.0 kg 4,0 kg ■>
A

Figura 1 B

4,0 kg
2,0 kg

Figura 2 E20) (UFF-12) ímãs são frequentemente


(1) Se o conjunto for puxado pelo bloco de utilizados para prender pequenos objetos em
maior massa, como na figura 2, o fio que une superfícies metálicas planas e verticais, como
os blocos arrebentará. quadros de avisos e portas de geladeiras.
(2) Se o conjunto for puxado pelo bloco de Considere que um ímã, colado a um grampo,
menor massa, como na figura 1, o fio que une esteja em contato com a porta de uma
os blocos arrebentará. geladeira. Suponha que a força magnética que
(3) O conjunto da figura 1 será acelerado se a o ímã faz sobre a superfície da geladeira é
força F tiver módulo maior que 12 N. perpendicular a ela e tem módulo FM. O
(4) No conjunto da figura 2, as forças de atrito conjunto imã/grampo tem massa m0. O
que atuam em cada um dos blocos têm o coeficiente de atrito estático entre a superfície
mesmo módulo. da geladeira e a do ímã é pe- Uma massa M
está pendurada no grampo por um fio de
E18) (UFPE-99) Na figura abaixo, não há atrito massa desprezível, como mostra a figura.
entre o bloco B e a superfície horizontal sobre
a qual está apoiado. O bloco B tem 20,0 kg e
aceleração constante de 2,0 m/s2 para a
direita. O bloco A está em repouso, preso a
uma parede por uma mola de constante IV
elástica k = 200 N/m. A mola está alongada
em 1,0 cm, devido ao atrito cinético entre os ô
blocos A e B. Calcule o módulo da força
horizontal F, em newtons, que puxa o bloco B
para a direita.
IM
a) Desenhe no diagrama as forças que agem
sobre o conjunto ímã/grampo (representado

95
Elementos da Física - Mecânica I - Força de Atrito
pelo ponto preto no cruzamento dos eixos x e y b) Calcule o coeficiente de atrito dle
na figura), identificando cada uma dessas deslizamento entre o corpo e o plano inclinado.
forças.
b) Qual o maior valor da massa M que pode E24) (Unesp-11) As figuras 1 e 2 representam
ser pendurada no grampo sem que o conjunto dois esquemas experimentais utilizados para a
caia? determinação do coeficiente de atrito estático
entre um bloco B e uma tábua plana, horizontal
E21) (UESC-06) A figura representa um bloco ////
de peso 500,0 N apoiado em uma parede !fio A
vertical por uma prensa que aplica uma força
de intensidade igual a 1.0.104 N. Is fio A
tábua

0
Considerando-se o bloco e a parede
///.'
Figura 1
indeformáveis e sabendo-se que o coeficiente
de atrito estático entre o bloco e a parede é Figura 2
igual 0,4, pode-se afirmar que o número No esquema da figura 1, um aluno exerceu
máximo de blocos iguais ao primeiro, que
uma força horizontal F no fio A e mediu o
poderão ser colocados sobre o primeiro bloco,
valor 2,0 cm para a deformação da mola,
é
quando a força F atingiu seu máximo valor
possível, imediatamente antes que o bloco B
se movesse. Para determinar a massa do
bloco B, este foi suspenso verticalmente, com
< 3 0 o fio A fixo no teto, conforme indicado na figura
2, e o aluno mediu a deformação da mola igual
a 10,0 cm, quando o sistema estava em
equilíbrio. Nas condições descritas,
desprezando a resistência do ar, o coeficiente
01)8 02)7 03)6 04)5 05)4 de atrito entre o bloco e a tábua vale
A) 0,1. B) 0,2. C) 0,3. D) 0,4. E) 0,5.
E22) (Udesc-09) Calcule a aceleração do
sistema abaixo quando o corpo de massa M é E25) (Unesp-14) Em um trecho retilineo e
puxado por uma força F que forma um ângulo horizontal de uma ferrovia, uma composição
a com a horizontal. Sabendo-se que entre a constituída por uma locomotiva e 20 vagões
superfície e o corpo existe um coeficiente de idênticos partiu do repouso e, em 2 minutos,,
atrito cinético p. atingiu a velocidade de 12 m/s. Ao longo de
F todo o percurso, um dinamômetro ideall
acoplado à locomotiva e ao primeiro vagão
indicou uma força de módulo constante e iguall
a 120000 N.
dinamômetro 7

BI iiiiiiiiaiiiiiinii luiiinaiiiiiiiiii i—l


M Considere que uma força total de resistência
ao movimento, horizontal e de intensidade
média correspondente a 3% do peso do
Dados: F = 10 N; M = 2 kg; a = 60°; p = 0,1; conjunto formado pelos 20 vagões, atuou
cos 60° = 0,5; sen 60° = 0,9 e g = 10 m/s2. sobre eles nesse trecho. Adotando g = 10 m/s2,
calcule a distância percorrida pela frente dai
E23) (Unesp) Um bloco de massa m = 5,0 kg locomotiva, desde o repouso até atingir a
está apoiado sobre um plano, inclinado de 30° velocidade de 12 m/s, e a massa de cada
em relação à horizontal. Se uma força F, vagão da composição.
paralela ao plano inclinado, é aplicada ao
bloco com sentido para cima, o bloco desliza 26) (Unifesp-15) Um abajur está apoiado sobre1
para baixo com velocidade v = (2t) m/s. Se a a superfície plana e horizontal de uma mesai
mesma força F é aplicada para baixo, o corpo em repouso em relação ao solo. Ele é1
desliza com velocidade v’ = (3t) m/s. acionado por meio de um cordão que pende
a) Calcule F; verticalmente, paralelo à haste do abajur,
conforme a figura 1. Para mudar a mesa de

96
Elementos da física - Mecânica I - Forca de Atrito
posição, duas pessoas a transportam inclinada,
fio
em movimento retilíneo e uniforme na direção B
horizontal, de modo que o cordão mantém-se
vertical, agora inclinado de um ângulo 0 = 30°, superfície horizontal
constante em relação à haste do abajur, de
acordo com a figura 2. Nessa situação, o
abajur continua apoiado sobre a mesa, mas na A
iminência de escorregar em relação a ela, ou
seja, qualquer pequena inclinação a mais da
mesa provocaria o deslizamento do abajur.
Figurai Figura 2
Desenho Ilustrativo
cordão---- -I I' vertical a) 0 N b) 30 N c) 40 N
P d) 50 N e) 80 N

rt
Calcule:
E29) (EsPCEx-14) Um trabalhador da
construção civil tem massa de 70 kg e utiliza
uma polia e uma corda ideais e sem atrito para
a) o valor da relação N-i/N2, sendo N, o módulo transportar telhas do solo até a cobertura de
da força normal que a mesa exerce sobre o uma residência em obras, conforme desenho
abajur na situação da figura 1 e N2 o módulo abaixo. O coeficiente de atrito estático entre a
da mesma força na situação da figura 2. sola do sapato do trabalhador e o chão de
b) o valor do coeficiente de atrito estático entre
concreto é pe = 1,0 e a massa de cada telha é
a base do abajur e a superfície da mesa.
de 2 kg. O número máximo de telhas que
podem ser sustentadas em repouso, acima do
E27) (Unicamp-93) Um caminhão transporta
solo, sem que o trabalhador deslize,
um bloco de ferro de 3.000 kg, trafegando
permanecendo estático no solo, para um
horizontalmente e em linha reta, com
ângulo 0 entre a corda e horizontal, é:
velocidade constante. O motorista vê o sinal
Dados: g = 10 m/s2, cos 0 = 0,8 e sen 0 = 0,6
(semáforo) ficar vermelho e aciona os freios,
aplicando uma desaceleração de 3,0 m/s2. O polia n
bloco não escorrega. O coeficiente de atrito
estático entre o bloco e a carroceria é 0,40.
Adote g = 10 m/s2:
corda
a) Qual a força que a carroceria aplica sobre o
bloco durante a desaceleração?
b) Qual é a máxima desaceleração que o
telhas ->□
caminhão pode ter para o bloco não
escorregar?
a) 30 b) 25 c) 20 d) 16 e) 10
E28) (EsPCEx-10) Dois blocos A e B, de
E30) (AFA-04) Um bloco de massa m é
massas MA = 5 kg e MB = 3 kg estão dispostos
arrastado, à velocidade constante, sobre uma
conforme o desenho abaixo em um local onde
superfície horizontal aplicada a uma corda,
a aceleração da gravidade vale 10 m/s2 e a
conforme o esquema da figura abaixo. Sendo
resistência do ar é desprezível. Sabendo que o
p o coeficiente de atrito entre as superfícies, o
bloco A está descendo com uma velocidade
módulo da força abaixo é
constante e que o fio e a polia são ideais,
podemos afirmar que a intensidade da força de
atrito entre o bloco B e a superfície horizontal é
de

////////////
a) p(T - mg) b) p(mg + Tsen0)
c) Tcos0 d) Tsen0

97
Elementos da Física - Mecânica I - Força de fttrita
E31) (AFA-09) Na situação de equilíbrio abaixo, horizontais. Pressentindo um perigo iminente o
os fios e as polias são ideais e a aceleração da maquinista freia bruscamente, travando todas
gravidade é g. Considere pe o coeficiente de as rodas da composição. Assim fazendo o
atrito estático entre o bloco A, de massa mA, e trem para num intervalo de 100m. Para que
o plano horizontal em que se apóia. isso ocorra, o coeficiente de atrito dinâmico
oferecido pelos trilhos deve ser (g = 10 m.s ):
A) 1,5 B) 0,20 C)1,0 D) 0,10 E) 0,02

a------- E35) (ITA-77) Um corpo cai na água e após


%
Y alguns segundos atinge uma velocidade
E praticamente constante (chamada velocidade
A maior massa que o bloco B pode ter, de limite) de 5,0 m/s. Sabendo-se que:
modo que o equilíbrio mantenha, é - a massa do corpo é 8 g;
a) pemA b)3pemA c) 2ps mA d) 4pe mA - a força exercida pela água sobre o corpo é
dissipativa, oposta ao movimento do corpo e
E32) (EN-10) Um pequeno bloco de massa m proporcional à velocidade do mesmo, isto é
está, devido ao atrito, em repouso sobre uma F = -av;
superfície cilíndrica numa posição que forma - quando o corpo atinge a velocidade limite a
um ângulo 0 com a vertical, conforme indica a força total sobre o corpo é nula;
figura. Os coeficientes de atrito estático e Calcule o coeficiente a, que será:
cinético entre o bloco e a superfície são, A) 16 Ns/m
respectivamente, iguais a pe e Mc- B) 1,6 x 10’2 kg/s
Considerando o bloco como uma partícula, C) 1,6 x 10'3 kgf/s
quanto vale o módulo da força de atrito entre o D) 1,6x10’3 Ns/m
bloco e a superfície? E) Nenhum dos valores acima.

E36) (IME-89) Um carro de corrida de Fórmula


1 parte do repouso, atinge a velocidade de 216
km/h, freia e pára no tempo total de 30
segundos. O coeficiente de atrito entre as
rodas e a estrada, que é explorado ao limite
durante a frenagem, é p = 0,5. Sabendo que
as acelerações, no período de velocidade
a) mg sen 0 b) mg cos 0 c) pemg crescente e no período de frenagem, são
d) mmg sen 0 e) pcmg cos 0 constantes, determine:
a) a aceleração durante o período em que a
E33) (ITA-73) Na figura temos um bloco de velocidade está aumentando;
massa igual a 10 kg sobre uma mesa que b) a distância total percorrida ao longo dos 30
apresenta coeficiente de atrito estático de 0,3 e segundos.
cinético de 0,25. Aplica-se so bloco uma força Dado: g = 10 m/s2
F de 20 N. Utilize a lei fundamental da
dinâmica (2a lei de Newton) para assinalar
abaixo o valor da força de atrito (Fa) no
sistema indicado (g = 9,8 m/s2).
Exercícios de Fixação
N

t F1) (Udesc-11) A Figura 1 representa um bloco


de massa m que se encontra sobre um plano
F
inclinado, sob a ação de uma força constante F,
cuja direção e sentido são os indicados na
Figura 1. Sendo pe e pc os coeficientes de
«s atrito estático e cinético, respectivamente,
a) 20 N b) 24,5 N c) 29,4 N entre o bloco e a superfície inclinada do plano;
d) 6,0 N e) Nda “a” a aceleração e “v” a velocidade do bloco;
"g” o valor da aceleração local da gravidade:
E34) (ITA-74) Uma composição ferroviária com
massa total de 100 toneladas, corre à
velocidade de 20 m/s sobre trilhos retos e

98
_____ Elementos da Física - Mecânica I - Força de atrito
mola, e, ainda, considerando g = 10 m/s2 e os
fios leves e flexíveis:

Assinale a alternativa incorreta. 4 kg


a) a = gsenO - F/m - pegcos0 = 0 ; 3 kg
v=0
30° E
b) a = gsenO - pcgcosô - F/m = 0 ;
v = constante
c) a = gsenO - F/m - pcgcos0 0 ; Os fios que chegam à polia C são paralelos ao
v = constante plano inclinado. Desprezando-se as massas
d) a = - gsenO - pcgcos0 + F/m = 0 ; das polias e o atrito nos seus eixos e a massa
v = constante da mola, e ainda, considerando g = 10 m/s2 e
e) a = F/m - gsen0 - Pcgcos0 0; os fios leves e flexíveis, calcule as trações nos
v # constante fios AC e BDE e a força de atrito no bloco A
quando este encontra-se em equilíbrio.
F2) (Unesp-97) Dois blocos, A e B ambos de
massa m, estão ligados por um fio leve e F4) (UEM-13) Um bloco de 6 kg está em
flexível, que passa por uma polia de massa repouso sobre um plano inclinado de 30° com
desprezível, que gira sem atrito. O bloco A a horizontal. Considerando g = 9,8 m/s2,
está apoiado sobre um carrinho de massa 4m, analise as alternativas abaixo e assinale o que
que pode se deslocar sobre a superfície for correto.
horizontal sem encontrar qualquer resistência. 01) O módulo da componente da força peso do
A figura a seguir mostra a situação descrita. bloco na direção perpendicular ao plano
a
| m |— ---------------------------- z-s inclinado é aproximadamente 51 N.
L~~ ] ,/í 02) A força de atrito estático que o plano
inclinado exerce sobre o bloco está na direção
paralela ao plano inclinado e no sentido
ascendente.
fmlB
04) Sendo o coeficiente de atrito estático entre
o plano inclinado e o bloco igual a 0,7, a força
Quando o conjunto é liberado, B desce e A se de atrito que o plano inclinado exerce sobre o
desloca com atrito constante sobre o carrinho, bloco é de aproximadamente 48 N.
acelerando-o. Sabendo que a força de atrito 08) O módulo da força normal que o plano
entre A e o carrinho, durante o deslocamento, inclinado exerce sobre o bloco é de
equivale a 0,2 do peso de A (ou seja, Força de aproximadamente 29 N.
atrito = 0,2mg) e fazendo g - 10 m/s2, 16) A força resultante que atua sobre o bloco é
determine: a própria força de atrito que o plano inclinado
a) a aceleração do carrinho; exerce sobre o bloco, pois é ela a responsável
b) a aceleração do sistema constituído por A e por manter esse bloco em repouso.
B.
F5) (UFPR-08) Uma caixa se movimenta sobre
F3) (UFGO) A figura abaixo mostra um bloco A uma superfície horizontal e, quando sua
de 4 kg apoiado num plano inclinado de 30° velocidade tem módulo 10 m/s, passa a subir
com a horizontal sendo puxado por outro bloco uma rampa, conforme indicado na figura.
E de 3 kg. O plano inclinado é rugoso e por Sabendo que o coeficiente de atrito entre o
isso existe uma força de atrito agindo no bloco bloco e o material da rampa é 0,75, calcule até
A. Uma mola de constante elástica K = 100 que altura, em relação à superfície horizontal,
N/m mantém o bloco A em equilíbrio sobre o a caixa irá subir nessa rampa.
plano quando encontra-se esticada de 0,3 m.
Os fios que chegam à polia C são paralelos ao
plano inclinado. Desprezam-se as massas das
polias e o atrito nos seus eixos e a massa da

99
Elementos da Física - Mecânica I - Força de atrito
F8) (UESC-11) Considere uma força de
intensidade constante sendo aplicada a uma
caixa de massa m que se encontra sobre uma
superfície plana e horizontal. Sabendo-se que
6m
10 m/s a direção da força é paralela à superfície, o
coeficiente de atrito estático entre a caixa e a
superfície é igual a p, o módulo da aceleração
da gravidade local é igual age que a caixa
8m
está na iminência de movimento, é correto
afirmar que a resultante das forças de contato
F6) (UFU-01) Um garoto realizou o seguinte
que a caixa recebe da superfície tem módulo
experimento: arrumou uma balança, colocou-a
igual a
sobre um carrinho de madeira com pequenas
01) mg 02) pmg
rodas, de forma que ele deslizasse numa p2)1/2
03) (1 + p)mg 04) mg(1
rampa inclinada sem atrito, subiu na balança e
05) (mg)~1(1-p2)1/2
deslizou plano abaixo. Considerando que o
garoto "pesa" 56 kg e que a leitura da balança
F9) (UFRJ-11) Um bloco de massa 2,0 kg está
durante a descida era de 42 kg, analise as
sobre a superfície de um plano inclinado, que
afirmativas abaixo e responda de acordo com
está em movimento retilíneo para a direita,
o esquema que se segue.
com aceleração de 2,0 m/s2, também para a
I- O ângulo de inclinação da rampa é 0 = 30°.
direita, como indica a figura a seguir. A
II- A força de atrito sobre os pés do garoto é
inclinação do plano é de 30° em relação à
horizontal e para a esquerda.
horizontal.
III- A força normal sobre os pés do garoto é
igual ao seu peso.
2,0 m/s2

300
. o ~O O O O O O O o
Balan a Suponha que o bloco não deslize sobre o
plano inclinado e que a aceleração da
gravidade seja g = 10 m/s2. Usando a
aproximação V3=1,7 , calcule o módulo e
indique a direção e o sentido da força de atrito
exercida pelo plano inclinado sobre o bloco.

F10) (UFPB-07) Dois blocos A e 8 de massas


A) I e III são corretas. m^ = 6kg e mB = 4kg, respectivamente, estão
B) II e III são corretas. apoiados sobre uma mesa horizontal e
C) Apenas I é correta. movem-se sob a ação de uma força F de
D) I e II são corretas. módulo 60N, conforme representação na
figura abaixo.
F7) (UESC-09) O motorista de um caminhão
socorreu um carro que parou na estrada por
causa de problemas mecânicos, utilizando
F
uma corda de massa e dimensões -------- ► A
desprezíveis para unir os veículos, que foram B
acelerados pela ação da força motora de
intensidade F, aplicada pelo caminhão. Considere que o coeficiente de atrito dinâmico
Desprezando-se os atritos de rolamento, entre o corpo A e a mesa é ^ = 0,2 e que o
admitindo-se que o coeficiente de atrito coeficiente entre o corpo 8 e a mesa é
cinético de deslizamento é constante e que a /zs = 0,3. Com base nesses dados, o módulo
massa do carro é metade da massa do da força exercida pelo bloco A sobre o bloco'
caminhão, a intensidade da força aplicada na 8 é:
corda para puxar o carro é igual a a) 26,4 N c) 32,4 N e) 48,4 N'
01)2F/3 02) 3F/2 03) 2F b) 28,5 N d) 39,2 N
04) F/2 05) F/3

100
Elementos da Física - Mecânica I - Força de Atrito
F11) (UFPB-08) A superfície de uma mesa é
constituída de dois materiais distintos, A e 6.
Um bloco de metal com massa igual a 2,0kg é
lançado sobre essa mesa com velocidade
inicial de 5,0m/s. Inicialmente, o bloco desliza
sobre o material A e, a seguir, passa a
deslizar sobre o material S. Os coeficientes
de atrito cinético entre o bloco e os dois
materiais são, respectivamente, pAc = 0,35 e
pBc = 0,25 e estão, representados no gráfico a a) Faça um esquema de todas as forças que
seguir, em função da posição d. agem sobre a caixa e identifique claramente a
origem de cada uma delas. Escreva o valor,
em N, da resultante dessas forças.
i b) Qual o valor da força de atrito entre a caixa
e o plano?
c) Qual o valor mínimo do coeficiente de atrito?
5?õ 4”) F14) (Aman-84) Um móvel se desloca com
movimento retilíneo e uniforme sobre uma
Nesse contexto, a distância percorrida pelo
superfície horizontal, sob a ação de uma força
bloco até atingir o repouso é:
de intensidade 30N, paralela ao plano. Quando
a) 2,0 m c) 1,0 m e) 3,0 m
a intensidade da força é duplicada, o móvel
b) 4,0 m d) 5,0 m
adquire aceleração de intensidade igual a 2,0
m.s-2. Sendo g = 10m.s~2 e supondo que o
F12) (UFPB-09) Sobre um bloco com massa
coeficiente de atrito cinético entre o corpo e a
1,0 kg, apoiado sobre uma mesa horizontal
superfície seja constante, o seu valor é:
(figura ao lado), existe uma força dada pela
A) 0,25 B) 0,20 C) 0,28 D) 0,15 E) 0,32
equação cartesiana F = 1/ + 3£, expressa no
Sistema Internacional de Unidades (S.I.). F15) (Ciaba-90) O bloco B se apoia no bloco A
e está ligado à parede por uma corda
horizontal BC. Que força F é necessário para
tomar iminente o movimento de A?

Dados: Coeficiente de atrito: pAB = ; Fad =

Pesos: PA = 30 kgf; PB = 20 kgf

I
777777777 - ' .' /777Zz .. 7777777/
Considerando que o coeficiente de atrito
cinético entre o bloco e a mesa é 0,2 e
admitindo que, inicialmente, foi fornecida ao
bloco uma velocidade de 4,0w/r ao longo do B
eixo x , é correto afirmar que o bloco, até E» ----------- A
parar, percorreu uma distância de:
a) 16 m 77777777777777777777/777777%
c) 32 m e) 80 m
b) 20 m d) 40m D
a) 15,0 kgf d) 5,0 kgf
F13) (Fuvest-96) Tenta-se, sem sucesso,
b) 21,7 kgf e) 12,5 kgf
deslocar uma caixa de peso P = 50 N, em
c) 11,6 kgf
repouso sobre um plano horizontal com atrito,
aplicando-lhe uma força F = 200 N, na direção F16) (AFA-06) Os blocos A e B , de massas
da haste. Despreza a massa da haste.
iguais a 2 kg e 3 kg, respectivamente, ligados
por um fio ideal, formam um sistema que
submetido a ação de uma força constante f
de intensidade 15 N, desloca-se com
aceleração de 1 m/s2, conforme a figura abaixo.
Se a tração no fio que liga os blocos durante o

101
Elementos da Física - Mecânica I - força de Atrito
deslocamento é de 9 N, pode-se afirmar que a a) 4 b) 2^2 c) 2 d) Vã
razão entre os coeficientes de atrito dos blocos
A e B com a superfície vale F19) (EN-01) Um bloco de massa igual a 6,0
Iz° J | * I [ kg sobe um plano inclinado de 30°, sob a ação
de uma força F de módulo igual a 40 N,
paralela à reta de maior declive do plano.
a)
£ c)
2
d)1 Existe atrito entre o bloco e o plano. Sabe-se
que no intervalo de tempo de 2,0 segundos, o
bloco percorre 4,0 metros no plano, em M.R.U.,
F17) (AFA-07) Três blocos, cujas massas mA =
mB = m e mc = 2m, são ligados através de fios e que, no instante t = 2,0 segundos, a força F
e polias ideais, conforme a figura. Sabendo-se é retirada. A distancia adicional, em
que C desce com uma aceleração de 1 m/s2 e centímetros, que o bloco ainda percorre plano
que 0,2 é o coeficiente de atrito entre S e a acima é de:
superfície S, pode-se afirmar que o coeficiente a) 30 b) 35 c) 38 e) 40
de atrito entre A e S vale
F20) (ITA-93) Um corpo de peso P desliza
lUUlh

__EH3
s
sobre uma superfície de comprimento t,
inclinada com relação a horizontal de um
ângulo a. O coeficiente de atrito cinético entre
o corpo e a superfície é p e a velocidade inicial
0 do corpo é igual a zero. Quanto tempo demora
o corpo para alcançar o final da superfície
inclinada?
a) 0,10 b) 0,20 c) 0,30 d) 0,40
a) V^/g
18) (AFA-14) Um bloco, de massa 2 kg, desliza b) V3€/[g(sena + pcosa)J
sobre um plano inclinado, conforme a figura
seguinte. c) ^/2//[g(sena + pcosa)]

d) >/3í/[g(sena-pcosa)]

<$> e) A/2f/[g(sena - p cos a)]

F21) (EN-13) O bloco B, de massa 10,0 kg,


está sobre o bloco A, de massa 40,0 kg,
ambos em repouso sobre um plano inclinado
377 X que faz um ângulo 0 = 30° com a horizontal,
conforme a figura. Há atrito, com coeficiente de
O gráfico v x t abaixo representa a velocidade
atrito estático 0,600, entre o bloco B e o bloco
desse bloco em função do tempo, durante sua
A, não havendo atrito entre o bloco A e o plano
subida, desde o ponto A até o ponto B.
inclinado. A intensidade mínima da força F,
db v(m/s) em newtons, aplicada ao bloco A e paralela ao
4 plano inclinado, para que o sistema permaneça
em repouso, é (Dado: g = 10,0 m/s2)

0 0,5
Considere a existência de atrito entre o bloco e
o plano inclinado e despreze quaisquer outras
formas de resistência ao movimento. Sabendo
a) 250 b) 225 c) 200 d) 175 e) 150
que o bloco retorna ao ponto A, a velocidade
com que ele passa por esse ponto, na descida,
F22) (EN-15) Observe a figura a seguir.
em m/s, vale

102
_____ Elementos da Física - Mecânica I - Força de Atrito
F24) (ITA-95) Dois blocos de massas m, = 3,0
kg e m2 = 5,0 kg deslizam sobre um plano,
inclinado de 60° com relação à horizontal,
encostados um no outro com o bloco 1 acima
do bloco 2. Os coeficientes de atrito cinético
entre o plano inclinado e os blocos são p1c =
0,40 e p2c = 0,6 respectivamente, para os
Um caixote pesando 50 N, no instante t = 0, se blocos 1 e 2. Considerando a aceleração da
encontra em repouso sobre um plano muito gravidade g = 10 m/s2, a aceleração do
longo e inclinado de 30° em relação à bloco 1 e a força F12 que o bloco 1 exerce
horizontal. Entre o caixote e o plano inclinado, sobre o bloco 2 são respectivamente, em N:
o coeficiente de atrito estático é 0,20 e o a) 6,0 m/s2; 2,0 b)0,46m/s2; 3,2
cinético é 0,10. Sabe-se que a força F , c) 1,1 m/s2; 17 d) 8,5 m/s2; 26
paralela ao plano inclinado, conforme indica a e) 8,5 m/s2; 42
figura acima, tem intensidade igual a 36 N. No
instante t = 9 s, qual o módulo, em newtons, da F25) (IME-64) Um carrinho A, apresenta a
força de atrito entre o caixote e o plano? Nesse superfície superior plana e horizontal com
mesmo instante, o bloco estará subindo, comprimento de 4,5 m, sendo a sua massa
descendo ou permanece em repouso sobre o igual a 100 kg. Sobre essa superfície superior
plano inclinado? colocamos um bloco B, de 10 kg, em uma das
a) 14 e descendo. extremidades. O coeficiente de atrito cinético
b) 11 e permanece em repouso. entre o bloco e a superfície superior do
c) 9,0 e subindo. carrinho é de 0,1. Calcular o tempo que leva o
d) 8,5 e permanece em repouso. bloco B para atingir a outra extremidade do
e) 4,5 e subindo. carrinho A, e a distância percorrida pelo
carrinho durante esse tempo, quando se
F23) (EN-15) Observe a figura a seguir. exerce, sobre o carrinho uma força horizontal
constante de 507,8 N, que começa a atuar a
partir do repouso. Suponha g = 9,8 m/s2.

F26) (IME-79) Os blocos A e B da figura tem


pesos iguais. Determine o coeficiente de atrito
mínimo para manter o sistema em equilíbrio.
Despreze o peso da corda e o atrito na
roldana.

f2 V
Na figura acima, o bloco de massa m = 2,0 kg
que está encostado na parede é mantido em
repouso devido à ação de duas forças, F, e F2,
cujos módulos variam no tempo segundo as
F27) Um cubo de massa m repousa em uma
respectivas equações F, = Fo + 2,0t e F2 = Fo +
superfície horizontal cujo coeficiente de atrito é
2,0t, onde a força é dada em newtons e o
tempo, em segundos. Em t = 0, o bloco está na p. Determine o ângulo a em relação a
iminência de entrar em movimento de descida, horizontal na qual é possível aplicar a menor
sendo o coeficiente de atrito estático entre o força necessária para mover o corpo, e o valor
bloco e a parede igual a 0,6. Em t = 3,0 s, qual dessa força F, sendo g a aceleração da
o módulo, em newtons, a direção e o sentido gravidade.
da força de atrito? Dado: g = 10 m/s2.
a) 7,5 e vertical para cima. F28) Na figura o corpo de massa m2 = 10 kg
b) 7,5 e vertical para baixo. escorrega sobre uma mesa sem atrito. Os
c) 4,5 e vertical para cima. coeficientes de atrito estático e cinético entre
d) 1,5 e vertical para cima. m2 e m, = 5 kg são = 0,6 e & = 0,4. a) Qual
e) 1,5 e vertical para baixo. a aceleração máxima de m(? b) Qual é o valor
máximo de m3 quando m, desloca-se, sem

103
Elementos da física - Mecânica 1 - Força de ntrito
escorregar, com m2? c) Se m3 - 30 kg, F32) O sistema está na iminência de
determinar a aceleração de cada corpo e a deslizamento, provocado pela força
tensão no cabo. F horizontal. Os coeficientes de atrito entre os
m'| I corpos e entre o corpo e a mesa valem p =
0,25. Determine o valor do ângulo 0.
m2 ------------- z
z
z
2M
z
z
z 4M --------
F29) Determine o valor da força mínima F que //////////////'
deve-se aplicar a um corpo de massa m, que i M
se encontra sobre um plano inclinado, com i
i
ângulo de inclinação 0, para que o corpo suba i
o plano inclinado com velocidade uniforme. O
coeficiente de atrito entre o corpo e o plano
inclinado é p. F33) Um bloco de massa m, é lançado de
baixo para cima, ao longo de um plano
inclinado. O coeficiente de atrito cinético entre
F o plano e o bloco é pc = 0,5 e o ângulo de
0 í inclinação do plano, 0 = 45°, é suficiente para
permitir o deslizamento do bloco de volta ao
ponto de lançamento. Sendo ts o tempo de
F30) Duas bolas estão unidas entre si por um subida e td o tempo de descida do bloco, tem-
fio sem peso, que passa por uma polia se a seguinte relação:
também sem peso, e uma delas se submerge
em um recipiente com líquido. Com que
velocidade estacionária v se moverá as bolas
se se sabe que a velocidade estacionária de
cada uma bola independente mo mesmo
líquido é igual a v0. A força de resistência do
líquido é proporcional a velocidade. A
densidade do líquido é rf( e a do material das
bolas é d.

F31) Uma fina corrente metálica se encontra


parcialmente dependurada de uma mesa, a) ^- = 1
como mostra a figura. Se o coeficiente de atrito b) — = 3 c) ^- = 73
‘s ts ts
estático entre a corrente e a mesa for m, qual é
2
a fração mínima do comprimento da corrente
que deve ser mantida sobre a mesa para que a
corrente não escorregue.
d)t 3
e) — = -3=
ts V3

F34) (OBF-04) Em um plano inclinado cujo


coeficiente de atrito cinético é p, colocam-se
dois blocos de massas m e M, dispostos
conforme a figura abaixo, tais que, ao serem
abandonados, o bloco de massa M desce.

a) ~ b)-^ c)/-
p+1 p+1 1-p
e)^-
1-M 1+p

104
ElementosdaFísIca—Mecânica I - Força de Atrito
Considerando os fios e a polia ideais,
determine a velocidade dos blocos quando o F39) No sistema da figura, o bloco 1 tem
bloco de massa M chegar ao solo. massa de 10 kg e seu coeficiente de atrito
estático com o plano inclinado é 0,5. Entre que
F35) Um bloco de 1,0 kg está sobre outro de valores mínimo e máximo pode variar a massa
4,0 kg que repousa sobre uma mesa lisa. Os m do bloco 2 para que o sistema permaneça
coeficientes de atrito estático e cinemático em equilíbrio?
entre os blocos valem 0,60 e 0,40. A força F
aplicada ao bloco de 4,0 kg é de 25 N e a
aceleração da gravidade no local é
aproximadamente igual a 10 m/s2. A força de
atrito que atua sobre o bloco de 4,0 kg tem a
intensidade de: 2

I.Okg
F = 25N 45°
4,0kg ---------- >
A) 5,0N B)4,0N C) 3,0N D) 2,0N E)1,0N F40) Os dois blocos que se mostram na figura
se encontram originalmente em repouso. Se
F36) Na configuração abaixo, o coeficiente de são desprezadas as massas das polias e o
atrito entre os blocos A e B é p, = 0,10 e entre efeito de atrito nestas e se supõe que as
o bloco B e a superfície horizontal é p2. Sendo componentes de atrito entre o bloco A e a
PA = 20 N, PB = 80 N e Pc = 60 N, e sabendo- superfície horizontal são pe = 0,25 e pc = 0.20,
se que o sistema está na iminência de determine
deslizamento, o coeficiente de atrito p2 vale: a) a aceleração de cada bloco;
z b) a tensão no cabo.

r"
z

z
A
th B
z
I \ B
//////h/////// 30 kg | i

c
A) 0,20 B) 0,28 C) 0,30 D) 0,10 E) 0,58
I I
F37) Um bloco está numa extremidade de uma
prancha de 2 m de comprimento. Erguendo-se 1
lentamente essa extremidade, o bloco começa B
a escorregar quando ela está a 1,03 m de 25 kg
altura, e então leva 2,2 s para deslizar até a
outra extremidade, que permaneceu no chão.
Qual é o coeficiente de atrito estático entre o
bloco e a prancha? Qual é o coeficiente de
atrito cinético?

F38) Um bloco é lançado para cima, com


velocidade de 5 m/s, sobre uma rampa de 45°
de inclinação. O coeficiente de atrito cinético
entre o bloco e a rampa é 0,3.
a) Qual é a distância máxima atingida pelo
bloco ao longo da rampa?
b) Quanto tempo leva o bloco para subir a
rampa?
c) Quanto tempo leva para descer a rampa?
d) Com que velocidade final chega ao pé da
rampa?

105
Elementos tia Física - Mecânica I - Força de atrito
Exercícios de Aprofundamento
A1) (FAAP) Qual a força horizontal capaz de AT
tornar iminente o deslizamento de cilindro, de I
peso 50 kgf, ao longo do apoio em V, mostrado I
na figura? O coeficiente de atrito estático entre
p B
—t
o cilindro e o apoio vale 0,25. 4,80 m
piscina

[desenho ilustrativo-fora de escala |

30° 30° a) 4073 N.s b) 6073 N.s


c) 70^3 N.s d) 180>/3 N.s
e) 240x/3 N.s

A4) (EN-12) Na figura abaixo, tempos 0 bloco


B de massa igual a 4,0 kg e um recipiente
A2) (UEL-12) Uma pessoa, de massa 80,0 kg, (massa desprezível) cheio de areia,
consegue aplicar uma força de tração máxima interligados por um fio (inextensível e de
de 800,0 N. Um corpo de massa M necessita massa desprezível) que passa por uma polia
ser levantado como indicado na figura a seguir. ideal. Os coeficientes de atrito estático e
O coeficiente de atrito estático entre a sola do cinético entre o bloco B e a reta de maior
sapato da pessoa e o chão de concreto é pe = declive do plano inclinado valem,
1,0. Faça um esboço de todas as forças que respectivamente, 0,050.73 e 0,040.73 . 0
atuam em todo o sistema e determine qual a recipiente possui um pequeno orifício no fundo,
maior massa M que pode ser levantada pela por onde a areia pode sair. No instante t = 0, a
pessoa sem que esta deslize, para um ângulo massa da areia no recipiente é de 1,7 kg. A
9 = 45. partir do instante t = 0, com a areia saindo do
orifício, 0 módulo da maior aceleração (em
e m/s2) adquirida pelo bloco B é (Dado g = 10
m/s2)

Areia Ll
M 30“

a) 4,2 b)4,4 c) 5,0 d) 5,5 e) 5,8


A3) (EsPCEx-15) Em um parque aquático, um
menino encontra-se sentado sobre uma A5) (EN-14) Considere uma força horizontal F
prancha e desce uma rampa plana inclinada aplicada sobre a cunha 1, de massa mi = 8,50
que termina em uma piscina no ponto B, kg, conforme mostra a figura abaixo. Não há
conforme figura abaixo. O conjunto menino- atrito entre a cunha e o chão, e o coeficiente
prancha possui massa de 60 kg, e parte do de atrito estático entre a cunha e o bloco 2, de
repouso do ponto A da rampa. O coeficiente de massa m2 = 8,50 kg, vale 0,200. O maior valor
atrito cinético entre a prancha e a rampa vale de F, em Newtons, que pode ser aplicado à
0,25 epéo ângulo entre a horizontal e o plano cunha, sem que o bloco comece a subir a
da rampa. Desprezando a resistência do ar, a rampa é
variação da quantidade de movimento do Dados: g =10 m/s2; sen 0 = 0,600; cos 0 =
conjunto menino-prancha entre os pontos A e 0,800
B é de Dados: intensidade da aceleração da
gravidade g = 10 m/s2 considere o conjunto
menino-prancha uma partícula
Dados: cos p = 0,8 e sen p = 0,6)

106
______Elementos da física - Mecânica I - Força de Atrito
A8) (IME-12) A figura 1 mostra dois corpos de
massas iguais a m presos por uma haste rígida
F de massa desprezível, na iminência do
movimento sobre um plano inclinado, de
ângulo 0 com a horizontal. Na figura 2, o corpo
inferior é substituído por outro com massa 2m.
a) 85,0 b) 145 c) 170 d) 190 e) 340 Para as duas situações, o coeficiente de atrito
estático é p e o coeficiente de atrito cinético é
A6) (ITA-84) A figura representa uma mesa p/2 para a massa superior, e não há atrito para
horizontal de coeficiente de atrito cinético pi, a massa inferior. A aceleração do conjunto ao
sobre a qual se apoia o bloco de massa MI2- : longo do plano inclinado, na situação da figura
Sobre ele está apoiado o objeto de massa m, 2é
sendo p o coeficiente de atrito cinético entre
eles. M2 e m estão ligados por cabos
horizontais esticados, de massa desprezível.
Desprezando-se a resistência do ar e o atrito
nas roldanas, podemos afirmar que m se
deslocará com velocidade constante em Figura 1 Figura 2
relação a um observador fixo na mesa, se Mi a) (2gsen 0)/3 b) (3gsen 0)/2
for tal que:
c) (gsen 0)/2 d) g(2sen 0 - cos 0)
e) g(2sen 0 + cos 0)

«2 A9) Dois blocos de massas de 4 kg e 8 kg


..... ;
'77777777777777777/77777777777777777777777777777777777777 estão conectados por um fio e deslizam por um
plano inclinado de 30°. O coeficiente de atrito
cinético entre o bloco de 4 kg e o plano é de
Fig. 2 0,25 e entre o bloco de 8 kg e o plano é de
0,35.
8.00
A) M, = pm kg
B) Mj = p1(M2 + m) + 2 um
C) M-, = |i1M2 + pm
D) M-, = 2pm + 2|j1(M2 + m) 4.00
E) M-, = p1(M2 + m)

A7) (ITA-86) Da posição mais baixa de um


plano inclinado, lança-se um bloco de massa =
5,0 kg com velocidade de 4,0 m/s no sentido 30-
ascendente. O bloco retorna a este ponto com
uma velocidade de 3,0 m/s. O ângulo do plano a) Calcule a aceleração de cada bloco.
inclinado mede 0 = n/6. Calcular a distância “d” b) Calcule a tensão no fio.
percorrida pelo bloco em sua ascensão. c) O que ocorre se os blocos forem invertidos,
Obs. Adotar para g = 10,0 m/s 2. de maneira que o bloco de 4 kg esteja acima
do bloco de 8 kg?

A10) Sobre uma mesa horizontal lisa descansa


um prisma de massa M, com ângulo de
inclinação a, e sobre ele descansa outro
prisma de massa m. Sobre o prisma menor
6 atua uma força horizontal F,e nestas condições
ambos os prismas se movem como se forem
A) 0,75 m D) 2,0 m um todo único (não variando sua disposição
B) 1,0 m E) NDA mútua). Determinar a força de atrito entre os
C) 1,75m prismas.

107
Elementos da Física - Mecânica I - Força de Atrito
M3 estão ligados por cabos esticados, de
massa desprezível, que passam por uma
E * m roldana de massa desprezível, assim como M2
M está ligado a massa suspensa M,. Calcular a
_________ aceleração do sistema. Dados: = 3M2, M2 =
2AÍ3, sen a = 4/5, p2 = 2/3 e p = 1/2

A11) (Olimpíada de Moscow) Um bloco pode-


se mover-se ao longo de um plano inclinado
em várias direções. Se ele recebe uma m3
velocidade v direcionada para baixo ao longo
m2
do plano inclinado, seu movimento passa a ser
uniformemente desacelerado, e ele para a
depois de percorrer uma distância /,. Se a
mesma velocidade é imposta ao corpo para M1
cima do plano inclinado, ele para depois de
percorrer uma distância l2. Na base do plano
inclinado é colocada uma guia horizontal. A14) Um bloco A de massa m está localizado
Determine a distância / percorrida pelo corpo sobre um outro bloco B de massa 2m. O bloco
no plano inclinado ao longo da guia, se a A está ligado por um fio ao bloco C, de massa
mesma velocidade v é imposta ao corpo, m, enquanto que o bloco B está ligado ao
agora na direção horizontal. bloco D, de massa 2m, através de outro fio. 0
coeficiente de atrito cinético entre os blocos A
e B é igual a p. Suponha que não existe atrito
entre o bloco B e o piso horizontal. Supondo o
sistema em movimento acelerado, determine a
razão entre as acelerações dos corpos C e D.
7//Z
m A
A12) (OBF-03) A boca de um copo é coberta
B 2m
com um cartão circular, e sobre o cartão
coloca-se uma moeda (vide figura a seguir).
£
Os centros do cartão e da moeda são
coincidentes com o centro da boca do copo.
Considere como dados deste problema: o raio
CÉ] V. 4
[zrnjp
do cartão, R, o raio da boca do copo, r, e o
coeficiente de atrito entre a moeda e o cartão,
p. O raio da moeda por ser desprezado. Move- a)^ b)^£ c)
se o cartão horizontalmente, em trajetória 1-H
retilínea e com aceleração constante. 2(1-P) 2(1-m)
Determine o valor da menor aceleração do d) 2-p e)
R
cartão, ac, para que a moeda ainda caia
dentro do copo, quando o cartão é retirado por A15) A figura mostra duas caixas iguais
completo. conectadas entre si através de fio e polia
cartão "X
ideais. O coeficiente de atrito entre cada bloco
i.---- R- .—r—. e a superfície prismática vale p = 0,5 e a
I

x moeda
gravidade local vale g. Determine a maior
aceleração a com que o sistema pode ser
acelerado horizontalmente para a direita, sem
que os blocos escorreguem em relação ao
prisma.
Dado sen a = 0,6 cos a = 0,8.
A13) A figura representa um plano inclinado de
coeficiente de atrito cinético p2 sobre a qual se
apoia o bloco de massa M2. Sobre ele está
apoiado o objeto de massa M2, sendo p o
coeficiente de atrito cinético entre eles. M2 e

108
Elementos da Física - Mecânica I - Força de Atrito
M | m
3m

a M
2m

a.

b) 2g / 3 c)g/2
mgx/37 mgVn mgV26
a)g a) b) c)
d) 3g / 4 e) 3g / 2 6 3 5
mg-i/FF mgVJó
d) e)
A16) Sobre um plano inclinado, com ângulo de 4 6
inclinação 30°, coloca-se uma prancha plana
de massa 10 kg e sobre ela um corpo de A19) O corpo A de massa m.ia em um plano
massa 5 kg. O coeficiente de atrito entre o inclinado de ângulo 0 está ligado ao corpo B
corpo e a prancha é 0,15 e entre a prancha e o de massa mb por uma corda que passa por
plano é 0,3. Determine a aceleração da uma roldana, conforme figura abaixo. O corpo
prancha. B está sobre um plano horizontal sem atrito.
Uma mola submetida a uma distensão x
A17) No teto de um vagão, presa por uma prende o corpo B a uma parede vertical. Ao ser
haste rígida, encontra-se uma polia ideal. Pela liberado a partir do repouso, o corpo A passa a
polia, passa um fio ideal. Nas extremidades do subir o plano inclinado com uma aceleração
fio, estão presos uma pequena esfera de inicial a.
massa m e um bloco de massa M = 28m. A
esfera encontra-se suspensa e o bloco
encontra-se em repouso em relação ao vagão,
em contato com o piso do vagão. Devido ao
fato de o vagão estar acelerado, com uma
aceleração de módulo a = 3g/4, a parte do fio
que passa pela polia e prende a esfera não se
encontra na vertical. Com base nessas
///////////////////
informações, faça o que se pede. Determine:
a) a expressão da aceleração a
b) a deformação x' no momento em que a
â resultante das forças sobre o corpo A for nula.
Dados:
• coeficiente de atrito entre a massa ma e o
plano inclinado: pa
M • constante elástica da mola: k
• aceleração da gravidade: g
a) Determine o ângulo de inclinação do fio que Observação: a corda e a roldana são ideais
prende a esfera, em relação à vertical.
b) Determine a força de atrito estático que age A20) Uma barra de massa M está situada num
sobre o bloco. plano horizontal liso, no qual move-se sem
c) Determine o valor mínimo do coeficiente de atrito. Sobre a barra está colocado um corpo
atrito estático entre o piso do vagão e o bloco, de massa m. O coeficiente de atrito entre o
para que o bloco permaneça em repouso em corpo e a barra é fi Para que valor de F, que
relação ao vagão. atua na barra na direção horizontal, o corpo
começa a deslizar sobre a mesma? Decorrido
A18) No sistema mostrado na figura abaixo o quanto tempo o corpo cairá da barra, se o
bloco de massa m não se move com relação comprimento da mesma é /?
ao bloco de massa 3m. O coeficiente de atrito nm
dinâmico entre o bloco de massa 3m e o solo
vale 0,25. Determine o valor da força que M F
exerce o bloco de massa m no bloco de massa -<-------------- /---------------------- ►

3m.

109
Elementos fla Física - Mecânica I - Força de atrito
A21) Um corpo de massa m é puxado plano
inclinado acima com movimento uniforme. A
inclinação do plano inclinado com relação à
horizontal é a. Determine a magnitude e a
direção da menor força necessária. O
coeficiente de atrito é p.

A25) Um cubo pequeno de massa m é


colocada sobre um plano inclinado como
indica-se na figura. O ângulo de inclinação do
plano é a e o coeficiente de atrito entre o plano
A22) Dois corpos de massas iguais a 4kg são e o cubo é p = 2.tan a. Determinar a mínima
colocados sobre um plano inclinado cujo força horizontal necessária para mover o cubo.
ângulo de-jnclinaçâo é 60°. O coeficiente de
atrito do corpo superior é p, = 0,1 e do corpo
inferior é p2 = 1- Determine o valor da força de
interação entre os corpos, quando ambos
deslizam juntos pelo plano inclinado.

IX a) 2.m.g.sen a b) V3.m.g.sen a
x2
c) m.g.sen a d) V?.m.g.sen a
e) 3.m.g.sen a
60"
A26) Por um plano inclinado (a = 45°) com a
horizontal lança-se para cima um corpo
A23) Determine o menor valor da aceleração pequeno. Determine o valor do coeficiente de
do corpo A em direção horizontal, para que os atrito, se o tempo de subida do corpo é 2
corpos 1 e 2 permaneçam imóveis com vezes menor que o de descida.
respeito ao citado corpo. As massas dos
corpos 1 e 2 são iguais e o coeficiente de atrito A27) Um bloco de massa m escorrega em uma
entre as superfícies dos corpos é p. A polia e calha cujos bordos formam ângulos de 90°. Se
os fios são ideais. o coeficiente de atrito cinético entre o bloco e 0'
material que constitui a calha é pc, determine a
1 aceleração do bloco.

A
2

A24) Por uma polia sujeita ao vértice superior


de um plano inclinado passa uma corda com
A28) No arranjo mostrado na figura, as
duas massas iguais a m presas nas
massas M e m são conhecidas. Existe atrito
extremidades. Determinar a força resultante
somente a cunha e o corpo de massa m, cujo
com que a corda pressiona o eixo da polia se o
coeficiente de atrito é igual a p. As massas da
coeficiente de atrito entre o plano inclinado e o
polia e dos fios podem ser desprezados.
corpo que descansa no plano é p e o ângulo
Determine a aceleração do corpo de massa m
de inclinação do plano é 0. O atrito no eixo da em relação à superfície horizontal na qual a
polia e a massa desta podem ser desprezadas. cunha desliza.

110
_____ Elementos da física - Mecânica I - Força de Atrito
a) o sentido da aceleração do carro. Justifique.
b) o valor desta aceleração em função de p e
da aceleração da gravidade g.
m
A31) (OBF-06) Dois blocos homogêneos e em
M 0 apoio fixo
forma de paralelepípedo, de massas mA = 3,0
kg e mB = 2,0 kg estão apoiados num piso e
formam um sistema conforme a figura. Por
meio de um cordão, deseja-se puxar e imprimir
g(i+-) gV2 um movimento retilíneo uniformemente
a)------- — b)
g(i+p)
c) acelerado ao sistema, inicialmente em repouso.
1+p 1 + p. + —
M
1. + — Considerando que o coeficiente de atrito
m m
cinético entre a superfície de B e a do piso vale
gV2 g(i + p)
d) 2 e) pB/P = 0,40; que entre as superfícies de A e de
M M B vale Pa/b = 0,50; que a medida “a" vale 18cm
p+—
m m e que o operador puxa o bloco B com uma
força F = 55N, calcule:
A29) (OBF-05) Determinar o valor da força de
atrito que atua sobre o bloco de 100 kg, A
considerando que o módulo da força F que
atua sobre o corpo, como indicado pelo
desenho, seja 100 N. O coeficiente de atrito
X ------ B 3
estático é 0,20 e o dinâmico 0,17.
H----- Ê----- H
a) a intensidade da aceleraçao do bloco A;
b) depois de quanto tempo o centro do bloco A
ficará alinhado verticalmente com a lateral do
bloco B?

A32) (OBF-07) Dois corpos com massas m, e


m2 estão conectados por uma corda
(inextensível e de massa desprezível) que
passa sobre uma roldana ideal fixa. No
instante t = 0 , quando o desnível entre os
A30) (OBF-08) Um paralelepípedo B está corpos é h, eles são abandonados, a partir do
sobre um plano horizontal. O coeficiente de repouso, de modo que o corpo de massa m,
atrito cinético entre eles vale p. Um fio desliza para baixo. Considere p o coeficiente
inextensível e sem peso é preso a ele e, de atrito cinético entre os corpos e as
passando por uma polia, é ligado a um outro superfícies.
corpo A que está pendurado. Sobre o bloco B
encontra-se um carro, como mostra a figura 5.
Este carro é acelerado de maneira que o corpo
A sobe com velocidade constante. h\
Considerando que as massas dos corpos A, B
e do carro são iguais, determine:
a) Determine a aceleração que os corpos
carro
adquirem.
b) Após um tempo At ambos os corpos estarão
B a uma mesma altura. Determine At.
O
A33) O bloco A possui uma massa de 40 kg e
o bloco B de 8 kg. Os coeficientes de atrito
entre todas as superfícies de contato são
A He = 0,20 e pc = 0,15. Dados: sen 25° = 0,42 e
cos 25° = 0,91
Determine
a) a aceleração do bloco B para P = 0;
b) a tensão na corda para P = 0;

111
Elementos da Física - Mecânica I - Força de atrito
c) a aceleração do bloco B para P = 40 N; a) a tensão na corda;
d) a tensão na corda para P = 40 N. b) a aceleração de cada bloco.

A c
i -

I

25°/

A34) Um homem que está parado dentro de A36) Na situação mostrada abaixo, o corpo de
um elevador, ao qual se move com uma dimensões desprezíveis e massa m está
aceleração constante, sustenta um bloco B de inicialmente em repouso sobre a cunha A e a
3 kg entre outros dois blocos de tal forma que uma distância d = 62,5 cm de seu topo,
o movimento de B em relação com A e C é medida ao longo do plano inclinado. A cunha A
iminente. Se os coeficientes de atrito entre está ligada através de um fio ideal
todas as superfícies são pe = 0,30 e pc = 0.25, (inextensível e de massa desprezível) de
determine comprimento D horizontal e outra cunha B de
mesma altura H que a cunha A. Em certo
instante, a cunha A, inicialmente em repouso,
a é submetida a uma aceleração constante a =
20 m/s2. Conhecemos o ângulo de inclinação e
o coeficiente de atrito entre o corpo e o plano
inclinado da cunha A, respectivamente, a = 37°
i (triângulo 3, 4, 5) e p = 0,25.

j ZnT H
a D
i. -

a

II I CUNHAA CfXHAB
a) Sabendo que o corpo, após projetado do
/ topo da cunha A, deve cair exatamente no topo
da cunha B, mantida a aceleração da cunha A

I
a) a aceleraçao de subida do elevador se cada
uma das forças exercidas pelo homem sobre
constante, para a esquerda, ache o
comprimento D do fio que une as cunhas, (g =
10 m/s2).
b) Para que o “pouso” do corpo no topo da
cunha B seja “suave”, isto é, não haja impacto
os blocos A e C tem uma componente com a superfície da cunha, ache o ângulo de
horizontal igual ao dobro do peso de B, inclinação p da cunha B.
b) as componentes horizontais das forças
exercidas pelo homem sobre os blocos A e C A37) No sistema da figura abaixo uma barra
se a aceleração do elevador é de 2,0 m/s2
de massa M e comprimento £ está presa à
para baixo.
extremidade de um fio que passa por uma
A35) Os coeficientes de atrito entre os blocos polia. No outro lado do fio encontra-se uma
A e C e as superfícies horizontais são pe = esfera de massa m < M, que pode deslizar ao
0,24 e pc = 0,20. Se mA = 5 kg, mB = 10 kg e longo do fio, através de um furo que
mc = 10 kg, determine: transpassa a esfera, com determinado atrito

112
Elementos da Física - Mecânica I - Força de atrito
constante. A massa da polia pode ser m.A
desprezada e considere g como a aceleração
da gravidade. No instante inicial a esfera está
localizada na mesma altura da extremidade
inferior da barra. h


,6 13
B C
O bloco chega ao ponto 8 com uma velocidade
M Í3
l- g h. Podemos afirmar que o
igual a J-

coeficiente de atrito cinético entre o bloco e o


plano é igual a:
a) tg 0 b) c)^ e) cotg 0
Quando o sistema é abandonado os dois ' 3
corpos se movimentam com aceleração
constante Determine a força de atrito entre a A41) Sobre um plano inclinado, com ângulo de
esfera e o fio se t segundos depois do início do inclinação 30°, coloca-se uma prancha plana
movimento a esfera está na mesma altura que
de massa 10 kg e sobre ela um corpo de
a extremidade superior da barra. massa 5 kg. O coeficiente de atrito entre o
corpo e a prancha é 0,15 e entre a prancha e o
A38) Quanto tempo t leva um corpo de massa
plano é 0,3. Determine a aceleração da
m para descer até a base de um plano
prancha.
inclinado de altura h e ângulo de inclinação p
se por outro plano inclinado com ângulo de A42) Um paralelepípedo B está sobre um
inclinação a esse mesmo corpo desliza plano plano horizontal. O coeficiente de atrito cinético
abaixo com movimento uniforme? Considere entre eles vale p. Um fio inextensivel e sem
que os coeficientes de atrito entre os dois peso é preso a ele e, passando por uma polia,
planos e o corpo são iguais. é ligado a um outro corpo A que está
pendurado. Sobre o bloco B encontra-se um
A39) O bloco A de 20 kg repousa sobre a carro, como mostra a figura. Este carro é
placa B de 60 kg na posição mostrada. acelerado de maneira que o corpo A sobe com
Desprezando a massa do cabo e da polia, e velocidade constante. Considerando que as
usando os coeficientes de atrito indicados, massas dos corpos A, B e do carro são iguais,
determine o tempo necessário para o bloco A determine o valor desta aceleração em função
escorregar 0,5 m sobre a placa quando o de p e da aceleração da gravidade g.
sistema é largado a partir do repouso.
carro

B o
0,5 m

c
A -- = °.2 A

30°
g(2n-l)
a)gn b)
2g + l
A40) Um bloco de massa m desce o plano c) g(p + l) d)g(2p+1)
inclinado a partir de A, com velocidade inicial d) s(1~g)
nula, como mostrado na figura a seguir. Vetor i+p
g: aceleração da gravidade
A43) Sobre um plano inclinado de um ângulo a
(sen a = 0,6) é colocada uma prancha de
massa m2 = 4 kg e sobre ela um corpo de
massa m, = 2 kg, conforme a figura. O

113
Elementos tia Física - Mecânica I - Força de Atrito
coeficiente de atrito entre o corpo e a prancha sua vez, encontra-se inicialmente em repouso
vale 0,1 e entre a prancha e o plano vale 0,2. sobre uma superfície horizontal sem atrito.
Sabendo que g = 10 m/s2, determine a Considere que em um dado instante uma força
diferença entre a aceleração do corpo 1 e a horizontal F passa a atuar sobre a
aceleração do corpo 2, ambas medidas em extremidade livre da corda, conforme indicado
relação ao referencial inercial. na figura. Para que não haja escorregamento
entre o bloco e a plataforma, o maior valor do
m,
módulo da força F aplicada, em newtons, é
fflj
Dado: g = 10 m/s2
a) 4/9 b) 15/9 c) 10
d) 20 e) 30
A) 0,6 m/s2 B) 1,2 m/s2
C) 1,6 m/s2 D) 2,4 m/s2 A46) (EN-17) Analise a figura abaixo.
E)0

A44) Dois blocos de massas m, e m2 são


colocados em um plano inclinado que forma
um ângulo a com a horizontal. Os coeficientes F
de atrito cinético entre o bloco de massa m, e
o plano e entre o bloco de massa m2 e o plano
valem, respectivamente, p-, e p2, com p, > p2.
Determine a força de interação entre os blocos,
supondo que o sistema entra em movimento
com aceleração não nula.

A acima mostra um bloco de massa 7,0 kg sob


uma superfície horizontal. Os coeficientes de
atrito estático e cinético entre o bloco e a
superfície são, respectivamente, 0,5 e 0,4. O
bloco está submetido a ação de duas forças de
a) (Pi - Fg )(mi _ m2 )g cos a mesmo módulo, F = 80 N, mutuamente
b) [sena-(p, -p2)cosa](m, -m2)g ortogonais. Se o ângulo 0 vale 60°, então,
c) (m1p1 -m2p2)gcosa pode-se afirmar que o bloco
Dado: g = 10 m/s2
(p, -p2)m1m2gcosa
a) descola-se da superfície, caindo
m, +m2 verticalmente.
[sena ~(m - p2)cosci]m1m2g b) desliza sob a superfície com aceleração
e)
m1 +m2 constante para a direita.
c) não se move em relação à superfície.
d) desliza sob a superfície com velocidade
A45) (EN-18) Analise a figura abaixo.
constante para a direita.
F e) desliza sob a superfície com aceleração
para a esquerda.

/ (pl■JSVg.-. iMKÃil.-?12 kg
•:WajlWWaWMMaf^^kg

A figura acima exibe um bloco de 12 kg que se


encontra na horizontal sobre uma plataforma
de 3,0 kg. O bloco está preso a uma corda de
massa desprezível que passa por uma roldana
de massa e atrito desprezíveis fixada na
própria plataforma. Os coeficientes de atrito
estático e cinético entre as superfícies de
contato (bloco e plataforma) são,
respectivamente, 0,3 e 0,2. A plataforma, por

114
FORÇAS NO MOVIMENTO CIRCULAR UNIFORME

Um corpo de massa m está executando um movimento circular, de centro O e raio R, com


velocidade de módulo constante v. O fato de não existir variação no módulo da velocidade implica
que a aceleração tangencial (ou linear) do corpo é nula. Porém, perceba que o vetor velocidade
varia com o tempo, uma vez que sua direção é sempre tangente à trajetória circular. Esta variação
do vetor velocidade provoca o aparecimento de uma aceleração, denominada “aceleração
centrípeta”. A aceleração tangencial é responsável pela variação do módulo do vetor velocidade,
enquanto que a aceleração centrípeta é responsável pela variação da direção do vetor velocidade.

A 2a figura abaixo mostra como se comporta a variação do vetor velocidade em um


pequeno trecho da trajetória do corpo, ou seja, em um arco de circunferência de ângulo central 0.

v. y2
e/2 e/2

O vetor Av = v2 -v, é determinado pela regra do paralelogramo, que na verdade é um


losango, desde que | v, |=| v21= v . Como em um losango as diagonais são perpendiculares,
conclui-se que os ângulos formados entre o segmento que liga as posições final e inicial e os
vetores v1 e v2 é igual a 0/2.

0 0 0
| Av |=| v11 sen —+1 v2 | sen - = 2vsen-

0 0
Como o ângulo 0/2 é pequeno pode-se usar a aproximação sen— = —. Assim:

0 0
| Av |= 2vsen-| = 2v^ ve

Além disso, a direção do vetor Av = v2 - v, é sempre radial, ou seja, ao longo de uma reta
que passa pelo centro da circunferência, com sentido para o centro da trajetória circular, como
indicado na figura.

Av
Quando o módulo de v é constante a aceleração centrípeta é dada ãcp =—. Como At é
sempre maior que zero segue que os vetores ãcp e Av possuem mesma direção e mesmo
sentido. Deste modo, a aceleração centrípeta possui direção radial, sempre direcionada do corpo
ao centro da trajetória circular.

115
Elementos da Física - Mecânica I - Força Centripeta

O módulo da aceleração centrípeta é dado por:

lã l-l^l v6 AO v => |ãcp|=^- ou |ãcp |= <b2R


V— = vw V—
At At At R r\

Como a aceleração tangencial é nula, a aceleração centrípeta é a aceleração resultante no


corpo em um movimento circular uniforme. O produto da massa do corpo pela aceleração
centrípeta resulta na força centrípeta:

- mv^ „
r-:p ~ macp => |Fcp|=m|ãcp| => |Fcp|=—ou |Fcp|=ma>2R
IX

Como a massa de um corpo é uma grandeza escalar estritamente positiva, os vetores


e Fcp possuem mesma direção e mesmo sentido. Como a aceleração centrípeta é a aceleração
resultante em um corpo em movimento circular uniforme, a força centrípeta é a força resultante
em um corpo em movimento circular uniforme. Deste modo, a força centrípeta é sempre um vetor
radial, direcionada do corpo ao centro da trajetória circular, conforme indicado na figura abaixo.

Observe que a força centrípeta é uma força fictícia, resultado da soma vetorial de todas as
forças atuantes no corpo, não existindo nenhum agente que execute a aplicação de Fcp no corpo.
Por exemplo, suponha que um avião está fazendo uma curva perfeitamente circular com
velocidade constante. A soma dos vetores peso e força de sustentação aerodinâmica é igual ao
vetor força centrípeta, que é a força resultante no movimento.

Note que, mesmo sendo constante o módulo da velocidade em um movimento circular


- mv2 -
uniforme e | F |=------ ■, o vetor força centrípeta não é constante, pois a direção de Fcp varia ao
R
longo da trajetória, se mantendo em direção radial. O que se mantém constante é apenas o
módulo da força centrípeta.

116
_ ________________________________________________ Elementos tfa Física - Mecânica I - Força Centripeta
FORÇAS NO MOVIMENTO CIRCULAR ACELERADO

Uma partícula de massa m está se movimentando ao redor de uma circunferência de raio


R. Considere que o movimento desta partícula é acelerado, ou seja, o módulo da velocidade
tangencial da partícula varia com o tempo. Além disso, devido à trajetória circular, a direção do
vetor velocidade varia com o tempo. Deste modo, a partícula apresenta aceleração tangencial e
aceleração centripeta, conforme indicado na figura. A aceleração tangencial (ou linear) ãt é
responsável pela variação do módulo do vetor velocidade, enquanto que a aceleração centripeta
ãcp é responsável pela variação da direção do vetor velocidade.
v
>t

O vetor aceleração tangencial é sempre tangente à trajetória da partícula e o vetor


aceleração centripeta está sempre direcionado da partícula em direção ao centro da trajetória
circular. A resultante destes dois vetores é o vetor aceleração resultante ãR. Na figura, ãt está no
mesmo sentido de v , ou seja, neste exemplo, o módulo da velocidade de aumenta com o tempo
(movimento acelerado). Porém, isto nem sempre ocorre. No movimento desacelerado o módulo
do velocidade diminui com o tempo e assim o sentido de ãt é contrário ao de v .

Para caracterizar o movimento é necessário escolher um sistema de eixos adequado. O


sistema de eixos tradicional, com o eixo x na horizontal e o eixo y na vertical, leva a expressões
muito complexas para as componentes das acelerações nestes eixos. O mais adequado é adotar
um sistema de eixos t x r, onde t é a direção tangencial era direção radial, com a origem
coincidindo com a posição da partícula. Perceba que este sistema de eixos rotaciona junto com a
partícula, mantendo o eixo t tangente à ãt e o eixo r sempre tangente à direção deãcp. Se t e f
são os versores dos eixos t e r, respectivamente, pode-se escrever que:

t v2 , dv
ãR =att+acpr aR = a.t + — r, onde a, = —
R ' R 1 dt

Multiplicando esta expressão pela massa m da partícula segue que:

v2 _
mãR = matt +m — r Fr = Ft t + Fcpr
R

O módulo da força resultante na partícula vale:

2 ( mv2 Y v4
FR=7(Ft)2+(Fcp)2 => Fr= (mat)2+ —- => FR=m.a2 + R2
V \ i' J

117
Elementos tia física - Mecânica I - Força Centripeta
Observações:

1) As expressões expostas neste item para ãR, FR e FR referem-se a valores instantâneos dessas
grandezas. Assim, na determinação de ãR, FR e FR deve-se calcular as componentes tangenciais
e radiais dessas grandezas para o mesmo instante t.

2) Perceba que toda a análise realizada na direção radial do movimento circular uniforme é válida
para a direção radial de um movimento circular acelerado. Isto ocorre pois a aceleração tangencial
é perpendicular à direção radial, não influenciando sobre as componentes radiais da aceleração e
da força resultante. Assim, pode-se afirmar que em qualquer movimento circular (uniforme ou
acelerado) a componente na direção radial da força resultante é igual à força centripeta.

3) Se a aceleração tangencial ãt apresentar módulo constante (movimento circular uniformemente


variado) tem-se v = v0 + a,.t. Supondo que o movimento inicia a partir do repouso segue que v =
at.t. Assim, a força resultante fica da forma:

v4 2 a4.t4 I a2.t4
Fr = m. a2 + R2 m. 'a, +-t—- => FR=maJV 1 + -àr-
' R2 K

a2t4
Neste caso, a aceleração resultante é dada por aR =a, J1 + .

MOVIMENTO PENDULAR

Um corpo de massa m é preso a um fio de comprimento t em um local onde a aceleração


da gravidade é g. A outra extremidade do fio é presa em um suporte fixado no teto. O fio é
inextensível. O corpo é colocado em uma posição em que o fio está tracionado e formando um
ângulo a (0 < a < 90°) com a direção vertical. O corpo é então abandonado com velocidade inicial
nula e passa a percorrer um arco de circunferência como trajetória. Desprezando a força de
resistência com o ar, durante toda a trajetória apenas duas forças atuam no corpo: a força peso P
e a tração f. Se a = 90° a tração nesta posição é igual a zero. Neste caso, atua no corpo apenas
a força peso. Para qualquer outro valor de a no intervalo ]0, 90°[ a tração não é nula na posição
inicial nem em qualquer outro ponto da trajetória.

Na figura estão representadas as forças que atuam no corpo e suas respectivas


decomposições nas direções tangencial e radial em três posições:

i) posição 1: ponto onde o corpo é abandonado com velocidade nula.

ii) posição 2: posição intermediária, quando o fio faz com a vertical um ângulo p tal que 0 < p < a.

iii) posição 3: quando o fio está na vertical.

118
Elementos da física - Mecânica I - Força Centrípeta

í(1-cosa)
t3

í(1-cosp)
—-
p

Em qualquer ponto da trajetória é possível decompor as forças que atuam no corpo em


duas direções perpendiculares: direção radial e direção tangencial. Sabe-se que a resultante na
direção radial sempre será a força centrípeta Fr = Fcp. Como a tração sempre possui direção radial,
a resultante tangencial Ft sempre será a componente da força peso na direção tangencial à
trajetória.
Como na posição 1 a velocidade do corpo é nula, a força centrípeta neste ponto também é
nula, implicando que a força resultante na direção radial é igual a zero:

Fr=Fcp O => IT, |=|P|.COSa => 1! = m.g.cos a

Na direção tangencial a força resultante é igual à componente do peso nesta direção:

| Ft11=| P | sen a => Fn = m.g.sen a

Na posição 2 a resultante radial é a força centrípeta, que neste caso não é nula:

mv2 T o
|Ft2 l=|FcP2 l=|f2|-|P|.cosp -i- = T2 - m.g. cos p

Mais uma vez, a resultante tangencial é a componente da força peso nesta direção:

| Ft21=| P | senp => Ft2 = m.g.senp


A posição 3 é a única da trajetória do corpo na qual a força peso está na direção radial.
Deste modo a componente tangencial da força resultante é nula. A resultante radial é a força
centrípeta, que na posição 3 é dada por:

mv3
|Ft3 I=|FCP3 l=|T3|-|P| T3 -m.g

A posição 3 é onde verifica-se a máxima velocidade do corpo, fazendo com que o módulo
da tração seja máxima na posição 3.

119
Elementos da Física - Mecânica I - Força Cenirípeta
FORÇAS NO MOVIMENTO NÃO RETILÍNEO

Até este momento foi apresentado como atuam as forças em um corpo que se movimenta
ao longo de uma reta ou de uma circunferência. Todavia, nem todo movimento é retilíneo ou
circular. Considere agora um corpo de massa m que está se movimentando no espaço em uma
trajetória não retilínea.

*2
Ci,

R1\

*1
C3
Rz,'

Cy Fcp3
c2: F.3
v3

Na figura, um corpo de massa m se desloca no espaço em uma trajetória qualquer, que


não é reta nem circular. Em cada ponto da trajetória o referido corpo está girando em torno de um
ponto do espaço. Como a trajetória não é circular, este ponto por onde o corpo está
instantaneamente girando em torno é variável. Na figura estão indicados três pontos (C1t C2 e C3),
que são centros instantâneos de rotação do corpo.
Em cada ponto de sua trajetória a força resultante no corpo pode ser decomposta em uma
componente tangencial e uma componente radial. A componente tangencial Ft é tal que
dv
Ft = m.ãt = m.— • Assim, o sentido de Ft será o mesmo de v se a velocidade estiver aumentando.

Por outro lado, o sentido de Ft será oposto ao de v se a velocidade estiver diminuindo. Na figura,
perceba que o módulo de v aumenta do ponto 1 ao ponto 2, fazendo com que o sentido de Ft
seja o mesmo de v nos pontos 1 e 2. Contudo, o módulo de v diminui entre os pontos 2 e 3.
Desta maneira, o sentido de Ft é contrário ao de v no ponto 3.
A componente radial está sempre direcionada de um ponto da trajetória ao centro de
rotação instantâneo. Como a componente radial não é responsável pela alteração do módulo da
velocidade, conclui-se que a componente radial atua somente para alterar a direção do vetor
velocidade. Desta forma, a componente radial da força resultante é centrípeta e portanto
= =
F'-F»“r-
mv2

OUTRAS EXPRESSÕES

É comum expressar a força centrípeta em função de outras grandezas:

mv2 mw2R2
1) Velocidade Angular: Fcp => Fcp m<»2R
R R~~

2
2?r 4n2mR
2) Período ou frequência: Fcp mco2R = m R => Fcp = => Fcd = 47t2f2mR
T CP

120
Elementos da Física - Mecânica I - Força Centrípeta
FORÇA CENTRÍFUGA

Suponha que uma pessoa esteja no interior de um carro que está se movimentando em
uma trajetória retilínea. Em determinado momento o carro passa a fazer uma curva. Para um
referencial inercial fora do carro surge uma força centrípeta direcionada da pessoa ao centro da
curva, que é a resultante radial de todas as forças que atuam no carro. Por outro lado, adotando o
carro como referencial não inercial, a pessoa sentirá a sensação de que outra força está atuando
sobre ela, jogando-a para o sentido oposto ao da força centrípeta. Essa força é a força centrífuga
- que atua, nesse caso, do centro para fora da curva.
Para o referencial inercial fora do carro a força centrífuga não existe. Este referencial
enxerga o carro acelerando para o centro da curva em virtude da força centrípeta (provocada pelo
atrito dos pneus com a pista). Dentro do carro, a pessoa faz a curva por causa das forças de
contato desta com o carro (banco, lateral do carro, cinto, volante, ...). Por esse motivo, a força
centrífuga é denominada força fictícia. Assim, conclui-se que as forças centrípeta e centrífuga são
diferentes. A força centrífuga só tem validade em um referencial ligado ao objeto que gira. A força
centrípeta é válida em qualquer referencial inercial fora do objeto que está girando.
Considere agora um disco que está girando com velocidade angular constante no sentido
anti horário. Sobre o disco está um bloco de massa m. Devido à presença de uma força de atrito o
corpo se mantém sempre sobre o mesmo ponto do disco. Solidariamente ao centro do disco está
um referencial inercial S. Este referencial observa três forças atuantes no corpo: Normal N, Força
de Atrito FA e Peso mg. A soma vetorial destas três forças é a força centrípeta FCr.

Fcp = N + Fa + mg

s
N
■Fa^
TRAJETÓRIA

o ->
mg

SENTIDO DE
ROTAÇÃO

Adote agora um referencial S* solidário ao corpo. Assim, S* está girando em torno do


centro O do disco, fazendo com que S* seja um referencial não inercial. Como as acelerações do
corpo e do referencial S* são iguais, em relação a S* o corpo está em equilíbrio. Como o
referencial S* é acelerado (possui aceleração centrípeta aCp), surge no corpo uma força de inércia,
mv2 F
denominada de força centrífuga FCf, dada por FCp = - maCp = ___t__Fcp.

S*
S
N

mg

SENTIDO
DE ROTAÇÃO

121
___________________________________________ Elementos ila Física - Mecânica I - força Centrípetai
As forças N, FA e mg ainda são medidas pelo referencial S* com o mesmo módulo que foi'
medido por S. Nota agora que:

FCP = N + FA + mg => 0 = — FCP + N + FA + mg => Fcf + N + FA + mg = 0

Esta última expressão é uma demonstração quantitativa de um fato já observado


qualitativamente, que é o equilíbrio do corpo em relação ao referencial S*.
Por mais que FCp = - FCf, NÃO pode-se afirmar que a força centrífuga é a reação da força
centrípeta, uma vez que estas duas forças não são medidas pelo mesmo referencial. A força
centripeta é medida por um referencial inercial em um corpo que esteja se movimentando em
qualquer trajetória que não seja a retilínea. A força centrífuga é medida por um referencial que
gira solidariamente ao corpo que está sendo analisado.

Exemplos:

1) Uma esfera de massa m = 1,0 kg está se movimentando em uma superfície retilínea horizontal.
Em determinado momento a esfera atinge uma região de forma circular de raio R = 20 m.
Sabendo que no topo da circunferência a velocidade da esfera é v = 10 m/s, determine a reação
normal neste ponto. Adote g = 10 m/s2.
Solução:

Analisando a decomposição de forças no ponto de altura máxima conclui-se que a resultante


radial do movimento da esfera é centrípeta. Portanto:
Fcp = P-N => N = P-Fcp = mg-^ = 1,0.10-^p-
= 10-5 = 5 N

2) Um motociclista está percorrendo um globo da morte, fazendo manobras em alta velocidade


dentro de uma superfície esférica de raio R = 2,5 m, em um local onde a aceleração da gravidade
é g = 10 m/s2. Determine a menor velocidade que a motocicleta deve possuir no ponto mais alto
do globo de modo a não perder contato com sua superfície interna.

Solução:

122
________________________ Elementos da Física - Mecânica I - Força Centrípeta
No diagrama ao lado estão indicadas as forças que atuam no
conjunto motociclista + motocicleta no ponto mais alto da trajetória.
\ Sabe-se que, independentemente da trajetória ser ou não circular, a
resultante radial é a força centrípeta: Fcp = N + P
N p A menor velocidade que a motocicleta deve possuir para não perder
contato com a superfície interna do globo ocorre quando a
motocicleta apenas tangencia o globo no ponto mais alto. Assim, a
|ãc normal neste ponto é nula:
FCP = P => = rfg => v2 = gR =>
K
V = VgR = y/10.2,5 = V25 = 5 m/s

3) Um rotor consiste em um cilindro de raio R que gira em torno de seu eixo de simetria vertical.
Um homem está no interior do rotor, com as costas apoiadas na superfície lateral interna do
cilindro. Sabe-se que o coeficiente de atrito entre o homem e a superfície do rotor é p. A
aceleração da gravidade local é g. Determine a mínima velocidade angular que deve girar o rotor
de modo que o homem possa levantar suas pernas do chão sem cair.
----- R— ■ ___

Solução:
-r. As forças que atuam no homem estão indicadas na figura ao lado.
Como o homem não cai quando levanta suas pernas, a força normal
com o chão é nula.
De modo a existir equilíbrio vertical tem-se que:
F,', Fat = P => N.p = mg => N=

A componente radial da força resultante é centrípeta:


R
Fcp = N meo2r-»
R =—mg
- CO =
Vmr
II

w
7
i

4) Um avião está voando com uma velocidade v constante. Em determinado momento o piloto
observa um obstáculo à sua frente e decide fazer uma curva, em um plano horizontal, para
desviar do obstáculo. Suponha que a força de sustentação do ar nas asas do avião é igual a F e
que P é o peso total do avião. Calcule o menor valor do raio R da curva que o piloto consegue
fazer no avião.
Solução:

123
______ Elementos da Física - Mecânica I - Força Cenuifleta
O esquema ao lado mostra a decomposição das
forças que atuam no avião. Apenas duas forças
atuam no avião, a força peso P e a força de
sustentação do ar nas asas F. Assim, pode-se
R
afirmar que FR = P + F . Sabe-se que esta força
resultante FR é centrípeta, uma vez que o avião
está fazendo um movimento circular uniforme.
Como a força peso é vertical e a força centrípeta
é horizontal, as forças FR , P e F formam um
triângulo retângulo, com hipotenusa em F. Deste modo:
mv2
F2 = P2+FR2 => fr=fcp=Vf2-p2 => — = Vf2-p2 => R=
R a/f2 -P2

5) Um corpo de massa m está peso a um fio de comprimento L, com a outra extremidade do fio
fixada em um teto horizontal, em um local onde a aceleração da gravidade é g. O corpo é posto a
oscilar em uma trajetória circular de raio R, contida em um plano horizontal.
a) Determine a tração no fio.
b) Determine a velocidade angular do corpo.
Solução:
a) Perceba que o movimento do fio gera uma superfície cônica,
motivo pelo qual este experimento é denominado de pêndulo
cônico. Pelo teorema de Pitágoras pode-se determinar a altura h
I
do cone: L2 = h2 + R2 => h = a/l2 - R2
L í
1 Sv O diagrama de forças está representado na figura ao lado. Como
! o corpo executa um movimento circular uniforme a força
--+ —+- ? resultante é centrípeta. Como o peso P é paralelo à altura h, a
Z tração T é paralela à direção do fio e a força centrípeta é paralela
í L—!#—
centro x
ao raio R, pode-se afirmar que o triângulo formado pelas forças
P , T e Fcp é semelhante ao triângulo formado por h, L e R.
T P T mgL
'■yP Deste modo: => T = .,"'at-
L h a/l2 - R2

b) Pela semelhança: =
p /rico2/ _ rfg r g
=> co =
h X aARTr2 a/l2 - R2

6) (UERJ-01) O globo da morte apresenta um motociclista percorrendo uma circunferência em alta


velocidade. Nesse circo, o raio da circunferência é igual a 4,0 m. Observe o esquema abaixo:
A

B
e = 6o°

O módulo da velocidade da moto no ponto B é 12 m/s e o sistema moto-piloto tem massa igual a
160 kg. Determine a componente radial da resultante das forças sobre o globo em B.
Solução:

124
___________________________ Elementos fla Física - Mecânica I - Força Centrípeta
O enunciado solicita o valor da força normal no globo no ponto B. Na figura
estão representadas as forças que atuam no sistema moto-piloto. A força N
representada é a reação normal que a superfície do globo faz nos pneus da
moto. A força normal que a moto faz no globo é a reação a esta força, ou
seja, possui o mesmo módulo de N, mesma direção e sentido contrário.
A direção radial a força resultante no sistema moto-piloto é a força
mv2
centrípeta: Fcp =N + P.cos60° —= N + mgcos60° =>

160(12)2
= N + 160.10.0,5 N = 4960 N
4

7) (UFPA-95) A figura abaixo representa um recipiente de forma hemisférica, girando com


velocidade angular constante <o = 10 rad/s em torno de um eixo vertical que passa pelo seu
centro de curvatura O. No interior do hemisfério, uma pequena esfera de aço, de massa m = 1,0
kg, gira, em equilíbrio dinâmico, acompanhando o movimento do hemisfério, sempre no mesmo
plano horizontal, de modo a manter constante o valor do ângulo 0. Nesta situação, é nula a força
de atrito que atua sobre a esfera num plano vertical a que contém o raio do hemisfério e o centro
da esfera. Sabendo que o raio do hemisfério vale 20 cm e supondo que a aceleração da gravidade
tem um valor g s 10 m/s2, pede-se:

a) Fazer um desenho, no seu formulário de respostas, representando todas as forças que atuam
na esfera, no plano a.
b) Calcular, em função de 0, o módulo da força que o recipiente exerce sobre a esfera.
c) Calcular o valor do ângulo 0.
Solução:
a) As forças estão indicadas na 1a figura
b) Na 2a figura está o cálculo da força
P resultante sobre a esfera. Como a esfera
está executando um MCU a resultante é a
força centrípeta:
„ P mg 1,0.10
cos 0 = — = —- N = —— N
N N N COS0
c) tg0 = ^- >/co2Rsen0

>erí^ _ o>2R>erTÍ COS0 = —f— 10


COS0 = = 0,5 => 0 = 60°
cos0 g co2R (10)20,2

125
____________________ ____________________________ Elementos da Física - Mecânica I - força Centrípeta
8) (ITA-12) Um funil que gira com velocidade angular uniforme em torno do seu eixo vertical de
simetria apresenta uma superfície cônica que forma um ângulo 9 com a horizontal, conforme a
figura. Sobre esta superfície, uma pequena esfera gira com a mesma velocidade angular
mantendo-se a uma distância d do eixo de rotação. Nestas condições, o período de rotação do
funil é dado por
a) 2n^d/ gsenG
b) 27t7d/gcos9
c) 2n,yd/gtan0
d) 2n^2d/gsen20
e) 2n^d/gcos0/gtan20

Solução: Alternativa C

Como o corpo executa um MCU a força resultante é a força centrípeta:


p
P Ag gT2
=> TT -2,pZ
= 2re
\gtgo
Ag

9) (EN-94) Uma bola é lançada para cima, com uma velocidade v, em uma direção que faz 60°
com a horizontal. Despreze a resistência do ar. O raio de curvatura de trajetória descrita pela bola
no ponto de altura máxima é:
v2 v2 v2 2v2 . 4v2
a)
' 4g
- b)^-
b) —
2g
c' —
c)
g
d) —
g e)v
Solução: Alternativa A
No ponto de altura máxima a velocidade possui apenas a
componente horizontal: vx = v.cos 60° = v/2
A única força atuante no ponto de altura máxima é o peso.
No ponto de altura máxima o peso é perpendicular à velocidade.
Assim, o peso é uma resultante centrípeta:
P = Fcp => mg = mvx2 /R => g = v2/4R R = yt/lg

10) (IME-89) Uma massa M = 20 kg é suspensa por um fio de comprimento t = 10 m, inextensível


e sem peso, conforme mostra a figura. A barra ABC gira em torno do seu eixo vertical com
velocidade angular constante de forma que o fio atinge a posição indicada. Determine:
a) a velocidade angular da barra;
c__ o
b) a tração no fio.
Dado; g = 10 m/s2 1= 10 m / ;

M? mq V
I 6M 1)5m J A

Solução:
a) Se 0 é o ângulo que o fio faz com a vertical: sen 0 = 0,6 => cos 0 = 0,8 => tg 0 = 3/4
t Fcp mw2R w2(d + ZsenO) 3 w2(1,5 + 10.0,6)
w = 1 rad/s
P mg g 4 10
mg 0,8 =^-10
b) cosO 0,8 = T = 250 N
T T

126
___________________________________________ Elementos da Física - Mecânica I - Força Centrípeta
11) (IME-96) Uma mesa giratória tem velocidade angular constante co, em torno do eixo y. Sobre
esta mesa encontram-se dois blocos, de massas m e M, ligados por uma corda inelástica que
passa por uma roldana fixa à mesa, conforme a figura abaixo. Considerando que não existe atrito
entre a massa e o bloco M, determine o coeficiente de atrito mínimo entre os dois blocos para que
não haja movimento relativo entre eles. Considere d a distância dos blocos ao eixo da rotação.
Despreze as massas da roldana e da corda.
yf

i
i
i
i
i
i
F d

Solução:
N2 1 I N, Analisando as forças verticais:
Fat<-=±
T<—|
1 Ni = mg e N2 = Nt + Mg = mg + Mg = (M + m)g
Analisando agora as forças horizontais que atuam em cada bloco
▼Mg
(adotando M > m) conclui-se que:
IT + Fat = Fcp1 ÍT = Mw2d-N1p T = Mw2d-mgp
[T = Fat + Fcp2 [t = mw2d + N,g T = mw2d + mgp
2mgp
Mv/d - mg|i = mv/d + mgp d
| M - m | w2

(ITA-94) Um fio tem presa uma massa M numa das extremidades e na outra, uma polia que
suporta duas massas; rr»! = 3,00 kg e m2 = 1,00 kg unidas por um outro fio como mostra a figura.
Os fios têm massas desprezíveis e as polias são ideais. Se CD = 0,80 mea massa M gira com
velocidade angular constante a> = 5,00 rad/s numa trajetória circular em torno do eixo vertical
passando por C, observa-se que o trecho ABC do fio permanece imóvel. Considerando a
aceleração da gravitacional g = 10,0 m/s2, a massa M deverá ser:
//////////////////////
a) 3,00 Kg
b) 4,00 kg c
0)
c) 0,75 kg
d) 1,50 kg / 0 M
e) 2,50 kg
mi D

Solução: Alternativa D
//////,
////////////////////// Sabe-se que a tração em uma máquina de Atwood vale
W T = 2m1m2g
. Como a polia esquerda não possui massa a força
' 1 m, + m2
resultante é nula: T = 2T, = 4mim2g = 4 10 = 30,0 N
m, + m2 3+1
Mg Se 0 é o ângulo formado pela tração e o peso Mg:
Fcp Jtfía>2L sen 0 >eríí a>2L_serítí
tg 0 =----- =-------------------------------------------- —-----------------
P ,Mg cos0 g
q 10
cos0 = —2— => cos cos00 = —z----- = 0,5
co2R (5)20,8
Tcos 0 = Mg 15 = M.10 => M = 1,5 kg

127
_____________ _______ ______________________ Elementos da Física - Mecânica I - Força Centrípeta
13) (ITA-94) Um motociclista trafega numa estrada reta e nivelada atrás de um caminhão de 4,00
m de largura, perpendicularmente à carroceria. Ambos estão trafegando à velocidade constante
de 72 km/h quando o caminhão se detém instantaneamente, devido a uma colisão. Se o tempo de
reação do motociclista for 0,50 s, a que distância mínima ele deverá estar trafegando para evitar o
choque apenas com mudança de trajetória? Considere o coeficiente de atrito entre o pneumático
e o solo p = 0,80, aceleração gravitacional g = 10,00 m/s2 e que a trajetória original o levaria a
colidir-se no meio da carroceria.
a) 19,6 m b) 79,3 m c) 69,3 m d) 24,0 m e)14,0m
Solução: Alternativa D
Apenas com mudança de trajetória a melhor estratégia para o motociclista é
inclinar a moto e fazer uma curva de menor raio possível.
mv2 v2 400
Fcp = Fa, => ----- = mgp => R = — =--------- => R = 50 m
R---------------------- gp 10.0,8
R-L/2 R2 = (R - L/2)2 + x2 => 502 = (50 - 2)2 + x2 => x2= 196 => x = 14,0 m
d = x + v.At = 14,0 + (20)(0,50) => d = 24,0 m

14) (ITA-04) A figura representa o percurso de um ciclista, num plano horizontal, composto de dois
trechos retilíneos (AB e EF), cada um com 6,0 m de comprimento, e de um trecho sinuoso
intermediário formado por arcos de circunferências de mesmo diâmetro, igual a 4,0 m, cujos
centros se encontram numerados de 1 a 7. Considere pontual o sistema ciclista-bicicleta e que o
percurso é completado no menor tempo, com velocidade escalar constante.
I 36 m _l

Se o coeficiente de atrito estático com o solo é p = 0,80, assinale a opção correta que indica,
respectivamente, a velocidade do ciclista, o tempo despendido no percurso e a frequência de
zigue-zague no trecho BE.
a) 6,0 m/s 6,0 s 0,17 s'1
b) 4,0 m/s 12 s 0,32 s1
c) 9,4 m/s 3,0 s 0,22 s1
d) 6,0 m/s 3,1 s 0,17 s1
e) 4,0 m/s 12 s 6,0 s'1
Solução: Alternativa B
I) para completar no menor tempo, o ciclista deve desenvolver Vmáx para realizar a curva e assim:
p.N = m.vmáx2/R_=>_p.mg = rn.vmáx2/R => 0,8.10 = vmâx2/2 => vmáx = 4m/s
b) Nos trechos ab ef o tempo de é dado por:
as = V. At => At = 6/4 = 1,5s., portanto: AtAB + aaef = 3,0s
Na parte sinuosa tem-se um deslocamento escalar total equivalente a 3C, onde C = 2nR
Logo: AtBE = 3C/V => AtBE = 6ttR/V => AtBE = (6n.2)/4 => AtBE =9,Os.
^total = ^ab + Atgp + AtBE = 12s
c) Cálculo da freqüência: v = 2nRf => 4 = 2.n.2.f => f=1/7t => f=0,32hz

15) (ITA-15) Na figura, o eixo vertical giratório z acima de O dada por imprime uma velocidade
angular cu = 10 rad/s ao sistema composto por quatro barras iguais, de comprimento L = 1 m e
massa desprezível, graças a uma dupla articulação na posição fixa X. Por sua vez, as barras de
baixo são articuladas na massa M de 2kg que, através de um furo central, pode deslizar sem atrito
ao longo do eixo e esticar uma mola de constante elástica k = 100 N/m, a partir da posição Oda
extremidade superior da mola em repouso, a dois metros abaixo de X. O sistema completa-se

128
__________________________________________________Elementos da Física - Mecânica I - Força Centrípeta
com duas massas iguais de m = 1kg cada uma, articuladas às barras. Sendo desprezíveis as
dimensões das massas, então, a mola distender-se de uma altura.
x

2Z,

L
a) 0,2 m b) 0,5 m c) 0,6 m d)0,7m e) 0,9 m

Solução: Alternativa B
Como as massas executam MCUs a força resultante em cada massa é a
■ r .x \ r• força centrípeta:
'e e ._ /T T,. n t ti m<O2LcOS0
mg xe
t\.t
0X mg
Fcp = (T + T ).cos 0 => T+T = cose - => T + T’ = mco2L (1)
T-Z
syr TJO
Analisando o equilíbrio vertical de cada massa m:
T’.sen 0 = mg + T.sen 0 => T'-T = (2)
Mg sen0
Subtraindo as equações (1) e (2):
mg
2T = ma>2L - => 2T.sen 0 = ma>2l_sen 0 - mg
sen0
O comprimento 2L pode ser calculado da seguinte forma: 2L = z + 2L.sen 0 => senO = 1-V-
sen0

Analisando as forças que atuam em M: 2T.sen 0 = Fe + Mg mw2Lsen 0 - mg = kz + Mg =


mco2L| 1- — -mg kz + Mg 1.100.1Í 1-—1-1.10 100.Z + 2.10
l 2L. l 2j
100-50z-10= 100Z + 20 => 150z = 70=> z = 0,466... m
A alternativa mais próxima é z = 0,5 m

16) (IME-93) Considere o veículo de massa M percorrendo uma curva inclinada, de ângulo 0, com
raio R constante, a uma velocidade V. Supondo que o coeficiente de atrito dos pneus com o solo
seja |i, calcule as velocidades mínima e máxima com que este veículo pode percorrer esta curva,
sem deslizamento.
R
3''

v~
Solução:
N Na iminência de subir:
R N =--------------------------
S1' Fcp =Fat.cos0 + N.senO R(sen0 + pcos0)
Fa, P + Fat. sen 0 = N. cos 0
N =-------- -------------
cos0-jisen0
Ve
gR(pcosQ + sen0)
Vmax
cos0-psen0
Na iminência de descer basta inverter o sentido da força de atrito:

129
Elementos da Física - Mecânica I - Força Centrípeta

N =--------- --------------
Fcp + Fal. cos 0 = N. sen 0 R(sen0 + pcos0) gR(sen9-pcos0)
=> => vmin =
P = Fal. sen 0 + N. cos 0 psen0 + cos0
N =--------------------
cos 0 - p sen 0

17) (OBF-09) O coeficiente de atrito estático entre a roupa de uma pessoa e a parede cilíndrica de
uma centrífuga de parque de diversões é p. Considere que a centrífuga possui raio R, sempre gira
em torno de um eixo vertical e encontra-se em um local de gravidade g. Seja uio a velocidade
angular mínima da centrífuga para que a pessoa permaneça colada à parede acima do piso.
Imagine que a centrífuga atinge uma velocidade angular de 2lü0 quando começa a frear.
Neste momento, determine, em função de p, g e R
a) a máxima aceleração angular possível para que a pessoa não deslize lateralmente;
b) o módulo da aceleração máxima resultante sobre a pessoa, considerando-a como um pequeno
bloco.
Solução:
a) Na figura ao lado estão indicadas as forças que atuam na pessoa.
Considere a situação na velocidade angular coo, mínima para que a
I FaC pessoa fique colada na parede da centrífuga. Neste caso pode-se
considerar Fatx = 0, uma vez que não existe tendência de deslizamento
lateral, apenas tendência de deslizamento vertical. Neste caso:
FatxJ
Fat = P = mg = Np = Fcp.p => mg = mco02Rp => co.,o2-_g_
_ o
Rp
I N t
J___ íb Considere agora o período em que a velocidade angular varia de coo até
P 2coo- De acordo com a decomposição de forças pode-se afirmar que:
Faty = P N = Fcp Fatx = ma
Sabe-se que a força de atrito resultante é dada por:
Fat = ^Fatx2 + Faty2 = Np =>
= Np => m m22a
a22 +
+mm22g =m
g22 = co44R
m22a> p22 => a2R2 + g2 = w4R2p2
R22p
Como g, R e p são constantes, o maior valor de a corresponde ao maior valor de co, que vale 2<i)0:
:max2R2 + g2 = 16co04R2p2 =16-yyR2p2 =16g2 => amax2R2=15g2 => amax=^^
a, => C^max
R p R

b) A aceleração tangencial máxima é dada por at max = amax.R = VÍ5g


A aceleração centrípeta máxima ocorre quando a velocidade angular é máxima:
O
”cp max = o>;>2 = 4c0qR =—
F
. \2 _ L c _2 16g2 Lc 16
Logo: amax max )2 +0,cp max ) *1'^9 2 => a,'max=9j15+-2
V M V H

18) (lpho-75) Uma barra gira com velocidade angular constante co em torno de um eixo vertical. 0
ângulo da barra com a horizontal é a. Na barra existe um anel que pode deslizar sobre sua
superfície. O coeficiente de atrito entre o anel e a superfície da barra é p. Determine sob que
condições o anel fica a uma distância L do ponto mais baixo da barra.

(4- R
------- 1___ -y
i /
i //
i /// L

130
Elementos da Física - Mecânica I - Força Centrípeta

Solução:
Considere um referencial que gira solidariamente ao anel
____ t-N\ a I com velocidade angular <0. Neste referencial surge uma
! Lcos a a força centrífuga de módulo F^ = mco2Lcos a, com direção
r----------
a > Fcg radial e sentido para fora da curva.
----- !— a Na figura ao lado o anel está na iminência de descer pela
i
i P barra. Para tanto a barra deve girar com um valor mínimo de
i / velocidade angular.
\// a Na direção horizontal: F^ + Fatcos a = Nsen a =>
mco2Lcosa
mca2Lcos a + Npcos a = Nsen a N=
sena-pcosa

Na vertical: Fatsen a + Ncos a = P => Npsen a + Ncos a = mg n=——


cosa + psena
mco2l cosa mg g sena-pcosa
Desta forma: => “min =
sen a - pcosa cosa + psena L cos a cos a + p sen a
Considere agora a situação em que o anel está na iminência
I de subir pela barra. Neste caso a força de atrito possui
! Lcos a 25a sentido contrário ao do caso anterior.
I----------------
a F cg Na direção horizontal: Fcg = Fatcos a + Nsen a =>
Fa^ a
i mrn2Lcos a = Npcos a + Nsen a => N= m— Lcosa
N=
i P sena + pcosa
i
i Na vertical: Falsen a + P = Ncos a
a
Npsen a + mg = Ncos a, N=—rg—
cosa - psena
mco2L cosa mg 1 g sena + pcosa
Desta forma: “max
sen a + pcosa cosa - psena L cos a cos a - p sen a
Assim, o anel fica a uma distância L do ponto mais baixo da barra se:
I g sen a - p cos a g sen a + pcosa
< to <
\ Lcos a cosa + psena L cos a cos a - p sen a

19) Uma superfície hemisférica é posta a rotacionar, com velocidade angular constante co, em
torno de um eixo vertical que passa por uma de suas extremidades. Em seu interior existem duas
esferas de massas M = 5 kg e m = 3 kg, ligadas por uma haste de massa desprezível. Sabe-se
que o raio R do hemisfério vale 1 m e que g = 10 m/s2. Determine o valor de co de modo que a
haste que liga as esferas se mantenha na horizontal e que o ângulo do segmento que liga cada
esfera ao centro do hemisfério forme 37° com a haste. Dado: sen 37° = 0,6 e cos 37° = 0,8.
Elementos da Física - Mecânica I - Força Centripeta
Solução:
Sendo F a força que a haste exerce em cada esfera, note que
existe equilíbrio na vertical:
Ni.sen37° = Mg => N1.0,6 = 5.10 => Nt = 250/3 N
N2.sen37° = mg => N2.0,6 = 3.10 => N2 = 50 N
Em cada esfera a força resultante é a centripeta:
N, Mw2ri = N^cos 37° - F (1)
mca2r2 = F - N2.cos 37° (2)
Somando as equações (1) e (2):
F p ; F co2[M(R + Rcos 37°) + m(R - Rcos 37°)] = (N, - N2)cos 37° =>
Mgl mg' rTI co2[5(1 + 0,8) + 3(1 - 0,8)] = (250/3 - 50)0,8 =>
k -►j
w2.9,6 = 80/3 => tá2 = 25/9 => <o = 5/3 rad/s
ri

20) Um balde é preso numa corda de comprimento L e gira numa circunferência horizontal. Gotas
de água caem do balde e atingem o solo ao longo do perimetro do círculo de raio a. Determine o
raio a quando o fio faz com a vertical um ângulo 0, em função de L e 0).

“i-------------
L

e
2L

1
a

Solução:
Na 1a figura estão representadas as forças que
_ Fcp
atuam no balde. Neste caso: tge
a p
/ dl
jrív2
Lsen 0 / tg0 = ^=LsêHÕ
sen0 v2_gLsen20
cosO
COS0 jaig
/ cosO
É com esta velocidade horizontal v, tangente à
trajetória do balde, que cada gota de água é
lançada quando abandona o balde. Na 2a figura tem-se uma vista superior do experimento.
Perceba que d é o alcance horizontal do movimento de cada gota. Logo, pode-se afirmar que:
d = vt => d^vY => d2 = gLsen20t2 t2=.d2cos0
COS0 gLsen 0

A projeção vertical do movimento de cada gota é uma queda livre: 2L-Lcos0 gü


2
{2 2L(2-cos0) d2cos0 _2L(2-cos0) ^2 2L2(2-cos0)sen2 0
g sen2 0 $ COS0

=> a2 = 2L2(2-cos0)sen20+L2sen2e
Pelo teorema de Pitágoras: a2 = d2 + L2sen2 0
COS0
4 -2cos0 4-cos0
a2 = L2 sen2 0 +1 a = Lsen0.
COS0 COS0

132
Elementos da Física - Mecânica I -Força Centríneta

Exercícios de Embasamento aceleração centripeta sobre os carrinhos na


situação considerada valem, respectivamente,
E1) (Puc/SP-03) Um avião descreve, em seu
movimento, uma trajetória circular, no plano
vertical (/oop), de raio R = 40 m, apresentando
no ponto mais baixo de sua trajetória uma
velocidade de 144 km/h.

&

r ■.

Sabendo-se que o piloto do avião tem massa


ÉaiOi&d
a) ^mgr e mr b) Jrg e mg
de 70 kg, a força de reação normal, aplicada , IF mr
pelo banco sobre o piloto, no ponto mais baixo, c) , - e — d) y/rg e nula
tem intensidade \g 9
a) 36 988 N b) 36 288 N c) 3 500 N e) Trg eg
d) 2 800 N e) 700 N
E4) (Unicamp-99) Uma atração muito popular
E2) (PUC/SP-07) A figura representa em plano nos circos é o “Globo da Morte", que consiste
vertical um trecho dos trilhos de uma numa gaiola de forma esférica no interior da qual
montanha russa na qual um carrinho está se movimenta uma pessoa pilotando uma
prestes a realizar uma curva. Despreze atritos, motocicleta. Considere um globo de raio
considere a massa total dos ocupantes e do R = 3,6 m.
carrinho igual a 500kg e a máxima velocidade a) Faça um diagrama das forças que atuam
com que o carrinho consegue realizar a curva sobre a motocicleta nos pontos A, B, C e D
sem perder contato com os trilhos igual a indicados na figura abaixo, sem incluir as forças
36km/h. O raio da curva, considerada circular, de atrito. Para efeitos práticos, considere o
é, em metros, igual a conjunto piloto + motocicleta como sendo um
ponto material.
b) Qual a velocidade mínima que a motocicleta
deve ter no ponto C para não perder o contato
com o interior do globo?
c

a) 3,6 b) 18 c) 1,0
d) 6,0 e) 10 D B

E3) (PUC/SP-12) Considere que, numa


montanha-russa de um parque de diversões,
os carrinhos do brinquedo, de massa total m,
passem pelo ponto mais alto do loop, de tal
forma que a intensidade da reação normal A

nesse instante seja nula. Adotando r como o


raio do loop e g a aceleração da gravidade E5) (Mackenzie-02) Um veículo necessita
local, podemos afirmar que a velocidade e a deslocar-se num trecho circunferencial de um

133
Hementos tia Física - Mecânica I -Força Centrípeta
autódromo, com velocidade escalar constante
de 180 km/h. O raio de curvatura da trajetória é
820 m. Para que esse movimento seja possível,
independentemente do atrito entre os pneus e
a pista, a estrada deverá apresentar uma 0.40m\
sobrelevação, em relação à horizontal,
correspondente a um ângulo a mínimo,
aproximadamente igual a:
... 130
2°__ 7°___ 17° 20°
sen 0,035 0,122 0,225 0,292 0,342 a) Represente graficamente, na folha de
cos 0,999 0,992 0,974 0,956 0,940 respostas, as forças que atuam sobre a esfera,
tan 0,035 0,123 0,231 0,306 0,364 nomeando-as. Determine o módulo da
resultante dessas forças.
a) 2° b) 7° c) 13° d) 17° e) 20° b) Determine o módulo da velocidade linear da
esfera e a freqüência do movimento circular
E6) (Mackenzie-12) No trecho de estrada
por ela descrito.
ilustrado, a curva pontilhada é um arco circular
e o raio da circunferência que o contém mede
E8) (UFSC-02) Um piloto executa um “looping"
500m.
com seu avião - manobra acrobática em que a
aeronave descreve um arco de circunferência
no plano vertical - que atinge, no ponto mais
baixo da trajetória, ao completar a manobra, a
velocidade máxima de 540 km/h. O raio da
trajetória é igual a 450 mea massa do piloto é
70 kg. Nessas manobras acrobáticas deve-se
considerar que a maior aceleração que o
organismo humano pode suportar é 9g (g =
aceleração da gravidade).

A placa sinaliza que a velocidade máxima


permitida, ao longo dessa linha, é 90km/h.
Considerando a segurança da estrada e Com base nos dados fornecidos, assinale a(s)
admitindo-se que essa velocidade máxima proposição(ões) CORRETA(S).
possa ocorrer independentemente do atrito 01. Se o raio de trajetória fosse menor do que
entre os pneus do automóvel e a 250 m,
pavimentação plana da pista, o ângulo de o piloto seria submetido a uma aceleração
inclinação mínimo, entre o plano da pista e a centrípeta máxima maior do que 9g (nove
horizontal, indicado na figura, deve medir, vezes a aceleração da gravidade).
aproximadamente, 02. A força centrípeta sobre o piloto, na parte
5,25° 6.10° 7.15° 8,20° 9,10° mais baixa da trajetória, é cinco vezes maior
sen 0,0992 0,106 0,129 0,143 0,158 do que o seu peso.
COS 0,996 0,999 0,992 0,990 0,987 04. O piloto é submetido a uma aceleração
tan 0.092 0,107 0,125 0,144 0,160 centrípeta máxima igual a 5g (cinco vezes a
|g| = 10mk* aceleração da gravidade).
a) 5,25° b)6,10° c)7,15° 08. A velocidade mínima para que o avião
d) 8,20° e) 9,10° complete a volta, no topo da trajetória, é igual
a 270 km/h.
E7) (Ufscar-01) A figura a seguir representa 16. A força que o avião faz sobre o piloto, na
um pêndulo cônico, composto por uma parte mais baixa da trajetória, é igual a 4200 N.
pequena esfera de massa 0,1 Okg que gira 32. A força que o piloto faz sobre o avião é
presa por um fio muito leve e inextensível, igual ao seu peso, em toda a trajetória.
descrevendo círculos de 0,12m de raio num 64. O piloto é submetido a uma aceleração
plano horizontal, localizado a 0,40m do ponto centrípeta máxima no topo da trajetória,
de suspensão. (Adote g = 10 m/s2.) quando a força de sustentação do avião é
mínima.

134
Elementos da Física - Mecânica I -Força Centrípeta

E9) (Mackenzie-04) Um avião efetua uma E13) (Udesc-2009) Um carro de massa m =


curva em um plano horizontal, de forma que o 1000 kg com velocidade escalar constante de
ângulo entre esse plano e a força de 72 km/h trafega por uma pista horizontal
sustentação (F) é a. Sendo P o peso do avião, quando passa por uma grande ondulação,
R o raio da curva ego módulo da aceleração conforme figura abaixo e mantém a mesma
velocidade escalar. Considerando que essa
da gravidade no local, a relação , entre a ondulação tenha o formato de uma
circunferência de raio R = 50 m. Calcule, no
intensidade da força de sustentação do avião e ponto mais alto da pista:
a intensidade de seu peso, é: 7

o a. R=IOm
1
I
p !
R n
a) lAsec a/Rg b) x^.tg a/Rg
a) A força centrípeta no carro.
c) Rg.cossec a/v2 d) v.sen,a/R2g
b) A força normal.
e) v2cos a/g (Dado: g = 10m/s2)

E10) (UFBA-87) Uma rodovia tem, numa de


E14) (UNB PAS-13)
suas curvas, o lado externo da pista em maior
cota que o lado interno. Sabendo que o raio de
curvatura e a inclinação da pista são,
respectivamente, de 142 m e 30°, e que o
coeficiente de atrito entre os pneus e a pista é
0,7, calcule, em metros por segundo, a
cm
velocidade máxima com a qual os carros
podem trafegar nessa curva, sem derrapar.
Considere g = 10 m/s2 e tg 30° = 0,6.

E11) (UESC-06) Considere-se um motociclista


no globo da morte, passando pelo topo com
velocidade de módulo igual a 36,0 km/h.
Sabendo-se que o raio do globo é de 2,5 m, a
massa do motorista e da motocicleta é de
200,0 kg e o módulo da aceleração da
gravidade local é de 10 m/s2, pode-se afirmar Na figura acima, um diagrama representa as
que o módulo da força de reação do topo do forças atuantes no centro de massa (cm) e no
globo sobre a moto é igual, em 103N, a solo referentes a um sistema motoqueiro-moto-
01)1,0 03)5,0 05)8,0 solo. A moto realiza uma curva de raio r e está
02) 2,0 04) 6,0 à velocidade constante v. Na figura, as forças
peso, normal e de atrito correspondem,
E12) (UESC-10) Um corpo de massa igual a respectivamente, a P, N e Fa. A partir dessas
2,0kg oscila amarrado à extremidade de uma informações, julgue os itens seguintes.
corda ideal de 1,6m de comprimento. (1) Na situação descrita, a força de atrito
Desprezando-se as forças dissipativas e desempenha o papel de força centrípeta.
sabendo-se que o módulo da aceleração da (2) O referido raio da curva (r) pode ser
gravidade local é igual a 10,0m/s2 e que o v2
determinado pela expressão r = -—, em que v
corpo passa pela posição de equilíbrio com pg
velocidade de módulo igual a 4,0 m/s, é correto
é a velocidade da moto, g é a aceleração da
afirmar que a intensidade da tensão na corda,
gravidade e p é o coeficiente de atrito entre
nesse ponto, é igual, em N, a
piso e pneus.
01)20 02)25 03)30
Resp: CC
04) 35 05) 40

135
Elementos da Física - Mecânica I -Força Centrípeta

E15) (UFRJ-06) Uma caixa é pendurada no


teto de um ônibus por meio de fios ideais
presos a um dinamômetro de massa
desprezível. A figura mostra esses objetos em
equilíbrio em relação ao ônibus, enquanto ele
está percorrendo um trecho circular de uma
estrada horizontal, com velocidade de 72 km/h.
Nessa situação, o dinamômetro mostra que a
tensão no fio é 65 N.

a) Calcule a velocidade tangencial do


astronauta representado na figura.
b) Determine a força de reação que o chão da
espaçonave aplica no astronauta que tem
massa m = 80 kg.
A 65 N
E17) (Fuvest-14) Uma estação espacial foi
projetada com formato cilíndrico, de raio R
igual a 100 m, como ilustra a figura ao lado.
Para simular o efeito gravitacional e permitir
direção
vertical 6,0 kg que as pessoas caminhem na parte interna da
casca cilíndrica, a estação gira em torno de
seu eixo, com velocidade angular constante w.
Sabendo que a massa da caixa é 6,0 kg,
As pessoas terão sensação de peso, como se
calcule o raio da curva da estrada.
estivessem na Terra, se a velocidade oj for de,
aproximadamente,
E16) (Unicamp-01) Algo muito comum nos
Note e adote:
filmes de ficção científica é o fato dos
A aceleração gravitacional na superfície da
personagens não flutuarem no interior das
Terra é g = 10 m/s2.
naves espaciais. Mesmo estando no espaço
sideral, na ausência de campos gravitacionais
externos, eles se movem como se existisse
uma força que os prendesse ao chão das
espaçonaves. Um filme que se preocupa com
esta questão é “2001, uma Odisséia no
Espaço”, de Stanley Kubrick. Nesse filme a
gravidade é simulada pela rotação da estação
espacial, que cria um peso efetivo agindo
sobre o astronauta. A estação espacial, em
forma de cilindro oco, mostrada abaixo, gira
com velocidade angular constante de 0,2 rad/s
em torno de um eixo horizontal E
perpendicular à página. O raio R da
espaçonave é 40 m.

a) 0,1 rad/s b) 0,3 rad/s c) 1 rad/s


d) 3 rad/s e) 10 rad/s

E18) (Fuvest-12) Nina e José estão sentados


em cadeiras, diametralmente opostas, de uma
roda-gigante que gira com velocidade angular
constante. Num certo momento, Nina se
encontra no ponto mais alto do percurso e
José, no mais baixo; após 15s, antes de a roda
completar uma volta, suas posições estão

136
Elementos da Física - Mecânica I -Força Centrípeta
invertidas. A roda-gigante tem raio R = 20m e a) 9Vã. b) 90(Vã)’1. c)90Vã. d) 900Vã.
as massas de Nina e José são,
respectivamente, MN = 60kg e Mj = 70kg.
Calcule E22) (AFA-04) Um carro de 1500 kg faz uma
a) o módulo v da velocidade linear das curva sem superelevação, com um raio de 75
cadeiras da roda-gigante; m, à velocidade de 54 km/h. O coeficiente de
b) o módulo aR da aceleração radial de Nina e atrito mínimo que deve haver entre o
de José; pavimento da estrada e os pneus, a fim de
c) os módulos Nn e Nj das forças normais que impedir a derrapagem do carro, é
as cadeiras exercem, respectivamente, sobre a) 0,1 *b) 0,3 c) 0,5 d) 0,6
Nina e sobre José no instante em que Nina se
encontra no ponto mais alto do percurso e E23) (AFA-05) O pêndulo da figura abaixo gira
José, no mais baixo. apresentando um ângulo 0 de abertura em
NOTE E ADOTE relação à vertical. Afirma-se que
7t = 3
Aceleração da gravidade g = 10m/s2
0 :
E19) (Fuvest-13) Um DJ, ao preparar seu
equipamento, esquece uma caixa de fósforos
sobre o disco de vinil, em um toca-discos
desligado. A caixa se encontra a 10cm do 1.
centro do disco. Quando o toca-discos é ligado,
no instante t = 0, ele passa a girar com I - a força centrípeta é a força resultante.
aceleração angular constante a = 1,1 rad/s2, II - variando a velocidade o período
até que o disco atinja a frequência final f = 33 permanece inalterado.
rpm que permanece constante. O coeficiente III - a tensão no fio diminui com o aumento de
de atrito estático entre a caixa de fósforos e o 0.
disco é pe = 0,09. Determine Estão corretas as afirmativas
a) a velocidade angular final do disco, u)f, em a) I e II apenas. c) II e III apenas.
rad/s; b) I e III apenas. d) I, II e III.
b) o instante tf em que o disco atinge a
velocidade angular cüf; E24) (AFA-08) Em uma apresentação da
c) a velocidade angular wc do disco no instante Esquadrilha da Fumaça, uma das acrobacias é
tc em que a caixa de fósforos passa a se o "loop", representado pela trajetória circular
deslocar em relação ao mesmo; da figura. Ao passar pelo ponto mais baixo da
d) o ângulo total A0 percorrido pela caixa de trajetória, a força que o assento do avião
fósforos desde o instante t = 0 até o instante t exerce sobre o piloto é
= tc-
Note e adote:
Aceleração da gravidade local g = 10m/s2.
n=3
E20) (AFA-88) Um carro deve fazer uma curva
de 250m de raio, sem derrubar, numa
velocidade escalar máxima de 36km/h. O
coeficiente de atrito entre os pneus e a estrada
é: a) maior que o peso do piloto.
OBS: o piso da estrada é sempre horizontal; g b) igual ao peso do piloto.
= 10 m/s2. c) menor que o peso do piloto.
a) 0,04b) 0,2 c) 0,5 d) 25 d) nula.

E21) (AFA-99) Um automóvel entra em uma E25) (AFA-07) Durante um show de patinação,
curva de 30° de inclinação, com velocidade 30 o patinador, representado na figura abaixo,
m/s. O raio da curva, em metros, para que não descreve uma evolução circular, com
haja escorregamento, é velocidade escalar constante, de raio igual a
(considerar g = 10 m/s2) 10,8 m. Considerando desprezíveis quaisquer

137
Elementos fla Física - Mecânica I -Força Centripeta
resistências, a velocidade do patinador, ao
fazer a referida evolução, é igual a.

f
53a
pista do golo

Dados: sen 53° = 0,80


cos 53° = 0,60 CO
a) 12 m/s c) 8 m/s
b) 7 m/s d) 9 m/s

E26) (IME-94) Uma pequena esfera está Nestas condições, determine:


suspensa por um fio ideal que está preso ao a) o valor da força de tração T a que fica
teto de um vagão. O trem faz uma curva plana submetido um dos fios;
horizontal de raio r, com velocidade v b) o valor da velocidade angular lü.
constante. Determine o ângulo 6 que o fio
forma com a direção vertical.

E27) (IME-97) Um disco rotativo paralelo ao


Exercícios de Fixação
solo é mostrado na figura. Um inseto de massa
m = 1,0 g está pousado no disco a 12,5 cm do F1) (Unesp-14) Em um show de patinação no
eixo de rotação. Sabendo-se que o coeficiente gelo, duas garotas de massas iguais giram em
de atrito estático do inseto com a superfície do movimento circular uniforme em torno de uma
disco é p, = 0,8 , determine qual o valor haste vertical fixa, perpendicular ao plano
mínimo da velocidade angular, em rpm horizontal. Duas fitas, F-, e F2, inextensíveis, de
(rotações por minuto), necessário para massas desprezíveis e mantidas na horizontal,
ligam uma garota à outra, e uma delas à haste.
arremessar o inseto para fora do disco.
Dado: g = 10 m/s2 Enquanto as garotas patinam, as fitas, a haste
e os centros de massa das garotas mantêm-se
num mesmo plano perpendicular ao piso plano
e horizontal.

haste
vertical
E28) (OBF-05) Um garoto de massa m está
sentado sobre um disco horizontal que gira
com velocidade constante em torno do eixo.
Sabendo que o garoto encontra-se a 3,0 m do
centro do disco e que o coeficiente de atrito
entre ele e o disco é 0,3, determine a maior
velocidade angular do disco capaz de manter o Considerando as informações indicadas na
garoto sentado nessa mesma posição. figura, que o módulo da força de tração na fita
Fi é igual a 120 N e desprezando o atrito e a
E29) (OBF-06) O dispositivo representado resistência do ar, é correto afirmar que o
consta de um eixo que gira com uma módulo da força de tração, em newtons, na fita
velocidade angular constante o> que tem preso, F2 é igual a
por meio de fios iguais, flexíveis e A) 120. B) 240. C) 60.
inextensíveis, duas bolas com massa de 1 kg D) 210. E) 180.
cada. Para uma determinada velocidade, o
ângulo a é igual a 37° e o raio da trajetória F2) (UFC-02) Considere uma partícula de
circular é de 0,2 m. massa m, submetida à ação de uma força
central atrativa do tipo F = k/r, onde r é a

138
Elementos da Fisica - Mecânica I -força Centrípeta
distância entre a partícula e o centro de forças força peso; a é o ângulo de inclinação das
fixo no ponto O, e k é uma constante. asas em relação ao plano horizontal; R é o raio
a) Mostre que se a partícula estiver de trajetória. São conhecidos os valores: a =
descrevendo uma órbita circular sob a ação de 45°; R = 1000 metros; massa do avião = 10000
tal força, sua velocidade independe do raio da kg.
órbita.
b) Mostre que o período de rotação da F
partícula, em torno do ponto O, é proporcional
a r.
R
F3) (UERJ-02) O cesto da máquina de lavar a J,
roupas da família mede 50 cm de diâmetro.
Durante o ciclo de centrifugação, o coeficiente X ▼ P
de atrito da roupa com a parede do cesto da
máquina é constante e igual a 0,5 e a Assinale a(s) proposição(ões) CORRETA(S),
aceleração angular do cesto é igual a 2 rad/s2. considerando, para efeito de cálculos, apenas
Calcule, em relação a esse ciclo de as forças indicadas na figura.
centrifugação: 01. Se o avião descreve uma trajetória
a) a velocidade de rotação mínima para que a curvilínea, a resultante das forças externas que
roupa fique grudada à parede do cesto; atuam sobre ele é, necessariamente, diferente
b) o número de rotações feitas pelo cesto, a de zero.
partir do repouso até atingir a velocidade de 3 02. Se o avião realiza movimento circular
rotações por segundo. uniforme, a resultante das forças que atuam
sobre ele é nula.
F4) (UFG-06) O chapéu mexicano, 04. A força centrípeta é, em cada ponto da
representado na figura, gira com velocidade trajetória, a resultante das forças externas que
angular constante. Cada assento é preso por atuam no avião, na direção do raio da
quatro correntes, que formam com a vertical trajetória.
um ângulo de 30°. As correntes estão presas à 08. A força centrípeta sobre o avião tem
borda do circulo superior, cujo diâmetro é de intensidade igual a 100000 N.
6,24 m, enquanto o comprimento das correntes 16. A velocidade do avião tem valor igual a 360
é de 6 m. A massa de cada criança é de 34 kg, km/h.
sendo desprezíveis as massas dos assentos e 32. A força resultante que atua sobre o avião
das correntes. não depende do ângulo de inclinação das asas
em relação ao plano horizontal.

Vn F6) (Unifesp-04) Uma estação espacial,


construída em forma cilíndrica, foi projetada
para contornar a ausência de gravidade no
espaço. A figura mostra, de maneira
simplificada, a secção reta dessa estação, que
possui dois andares. Para simular a gravidade,
a estação deve girar em torno do seu eixo com
líSfc- uma certa velocidade angular. Se o raio
■mm® externo da estação é R,

Dados: g = 10 m/s2, -73=1,7


Calcule:
a) a velocidade delas ao longo da trajetória
circular;
b) a tensão em cada corrente.

F5) (UFSC-00) Um avião descreve uma curva


em trajetória circular com velocidade escalar
constante, num plano horizontal, conforme
I
está representado na figura, onde é a força 2R
de sustentação, perpendicular às asas; P é a

139
Elementos da Física - Mecânica I -força Centrípeta
a) deduza a velocidade angular w com que a extremidade da mola está presa a um pino em
estação deve girar para que um astronauta, O, segundo a figura abaixo.
em repouso no primeiro andar e a uma a) Determine o valor da força que a mola
distância R do eixo da estação, fique sujeito a aplica na bola para que esta realize o
uma aceleração igual a g. movimento descrito.
b) Suponha que o astronauta vá para o b) Qual era o comprimento original da mola
segundo andar, a uma distância h do piso do antes de ter sido esticada?
andar anterior. Calcule o peso do astronauta
nessa posição e compare com o seu peso
quando estava no primeiro andar. O peso
aumenta, diminui ou permanece inalterado ?

F7) (Fuvest-09) Um acrobata, de massa MA =


60kg, quer realizar uma apresentação em que,
segurando uma corda suspensa em um ponto F9) (Unicamp-01) As máquinas a vapor, que
Q fixo, pretende descrever um círculo de raio R foram importantíssimas na Revolução
= 4,9m, de tal forma que a corda mantenha um Industrial, costumavam ter um engenhoso
ângulo de 45° com a vertical. Visando garantir regulador da sua velocidade de rotação, como
sua total segurança, há uma recomendação é mostrado esquematicamente na figura
pela qual essa corda deva ser capaz de abaixo. As duas massas afastavam-se do eixo
suportar uma tensão de, no mínimo, três vezes devido ao movimento angular e acionavam um
o valor da tensão a que é submetida durante a dispositivo regulador da entrada de vapor,
apresentação. Para testar a corda, com ela controlando assim a velocidade de rotação,
parada e na vertical, é pendurado em sua sempre que o ângulo 0 atingia 30°. Considere
extremidade um bloco de massa Mo, calculada hastes de massa desprezível e comprimento
de tal forma que a tensão na corda atenda às L = 0,2m, com massas m = 0,18 kg em suas
condições mínimas estabelecidas pela pontas, d = 0,1 m e aproxime 73 = 1,8.
recomendação de segurança. Nessa situação:
] Q
Q articulação
,g |o
d

Mo

Situação de teste

NOTE E ADOTE:
Força centrípeta Fc = mv2ÍR
Adote n s3 a) Faça um diagrama indicando as forças que
a) Represente, no esquema da folha de atuam sobre uma das massas m.
respostas, a direção e o sentido das forças que b) Calcule a velocidade angular íí para a qual
agem sobre o acrobata, durante sua e = 30°.
apresentação, identificando-as, por meio de
um desenho em escala. F10) (AFA-08) Um corpo de massa m, preso à
b) Estime o tempo tA, em segundos, que o extremidade de um fio, constituindo um
acrobata leva para dar uma volta completa em pêndulo cônico, gira num círculo horizontal de
sua órbita circular. raio R, como mostra a figura.
c) Estime o valor da massa Mo, em kg, que
deve ser utilizada para realizar o teste de
segurança.

F8) (Unicamp-93) Uma bola de massa 1,0 kg,


presa à extremidade livre de uma mola
esticada de constante elástica k = 2000 N/m,
descreve um movimento circular e uniforme de
raio r = 0,50 m velocidade v = 10 m/s sobre
uma mesa horizontal e sem atrito. A outra

140
Elementos da Física - Mecânica I -Força Centrípeta
Sendo g a aceleração da gravidade local e 9 o estrada plana e horizontal. Em um
ângulo do fio com a vertical, a velocidade do determinado instante, o caminhão entra em
corpo pode ser calculada por uma curva circular de raio igual a 51,2 m,
a) VRg b) Tw c) VRgsenG d) ^RgígO mantendo a mesma velocidade escalar.
Sabendo-se que os coeficientes de atrito
cinético e estático entre a caixa e o assoalho
F11) (AFA-08) A figura abaixo representa dois
horizontal são, respectivamente, 0,4 e 0,5 e
corpos idênticos girando horizontalmente em
considerando que as dimensões do caminhão,
MCU com velocidades lineares v-, e v2 A razão
em relação ao raio da curva, são desprezíveis
4 entre as intensidades das trações nos fios
e que a caixa esteja apoiada apenas no
*2 assoalho da carroceria, pode-se afirmar que a
ideais 1 e 2 é máxima velocidade, em m/s, que o caminhão
R R poderá desenvolver, sem que a caixa
escorregue é
a) 14,3 c) 18,0
b) 16,0 d) 21,5

F14) (EN-09) Uma pequena esfera (partícula)


fio 1 fio 2 de massa M desliza, a partir do repouso
2v?+Vz (posição A), por uma trajetória (no plano
a) b)
vr + vl vertical), passando pela posição B, da
A circunferência de raio R, com velocidade de
V?-Vz módulo V, como indica a figura abaixo.
c)
vi
d) 4
V1

F12) (AFA-13) Em um local onde a aceleração


da gravidade vale g, uma partícula move-se
sem atrito sobre uma pista circular que, por
sua vez, possui uma inclinação 0. Essa
partícula está presa a um poste central, por
meio de um fio ideal de comprimento t que,
através de uma articulação, pode girar
livremente em torno do poste. O fio é mantido
paralelo à superfície da pista, conforme figura Sabe-se que o coeficiente de atrito cinético
abaixo.
entre a partícula e a trajetória vale pc. O
módulo da força de atrito que atua na esfera,
no instante em que passa pela posição B, é
.1__ ■ igual a
a) pcMg b) gcMg sen 9
pcM(V2 +Rgcos9)
c) pcMg cos 9 d)
R
Ao girar com uma determinada velocidade
constante, a partícula fica "flutuando” sobre a pcV2gsen0
e)
superfície inclinada da pista, ou seja, a R
partícula fica na iminência de perder o contato
com a pista e, além disso, descreve uma F15) (ITA-73) Um flutuador em colchão de ar,
trajetória circular com centro em C, também de massa m, desloca-se num círculo horizontal,
indicado na figura. Nessas condições, a sobre uma mesa e preso à extremidade de um
velocidade linear da partícula deve ser igual a fio inextensível, de comprimento igual a 0,8 m,
com velocidade angular mostrada no gráfico (a
a)^p b)^7 0)7^7 d) 72^7 propulsão é dada pelos gases expelidos pelo
aparelho). Suponha a massa do aparelho
constante. Calcule as acelerações angular (a),
F13) (AFA-15) Uma determinada caixa é
tangencial(a) e centrípeta(ac) e assinale a
transportada em um caminhão que percorre,
resposta correta abaixo.
com velocidade escalar constante, uma

141
Elementos da física - Mecânica I -Força Centrípeta
u rd /s que: a máxima força de atrito estático f, e a
tangente trigonométrica do ângulo de
inclinação 0, da moto em relação à vertical,
7
serão dados respectivamente por:
f. (N) tg 0
5 a) 500 0,5
b) 600 0,5
3 c) 500 0,6
d) 600 0,6
1 e) 500 0,3

o 5 10 15 20 25 (s) F19) (IME-96) De acordo com a figura abaixo,


a(rd/s2) a(m/s2) ac(m/s2) o veículo 1, de massa total M, descreve uma
) A. 0,25 0,20 0,8 + 0,32t + O.sát2 trajetória circular de raio R, com uma
(
( ) B. 0,20 0,16 0,8 + 0,4t + 0,05t2 velocidade tangencial e constante v.
( ) C. 0,25 0,20 0,8 + 0,4t + 0,05t2
( ) D. 0,20 0,16 0,8 + 0,32t + 0,032^
( ) E. 0,25 0,16 0,8 + 0,32t + 0,032^ z1

F16) (ITA-73) Na questão anterior, se a massa


do aparelhe é 2,0 kg e se a tensão máxima
que o fio permite é 40 N, após quantos
Estabeleça a possibilidade do veículo 1 ser
segundos este se rompe ? considerado como um referencial inercial para
a) 5,0 s b) 12 s c) 15 s d) 20 s e) Nda o movimento do veículo 2 no seu interior.
F17) (ITA-81) A figura ao lado representa uma F20) (IME-10)
mesa horizontal muito lisa que gira em torno
de um eixo vertical com velocidade angular co
constante. Um objeto de massa m apoiado
sobre a mesa gira com a mesma velocidade
angular, graças apenas à ação de uma mola
de constante elástica k, de massa desprezível, 6 _
e cujo comprimento é t, quando não
solicitada. Podemos afirmar que : • pedra

Uma pedra está presa a um fio e oscila da


maneira mostrada na figura acima. Chamando
T a tração no fio e 0 o ângulo entre o fio e a
a) co é certamente maior que (k/m)1/2.
vertical, considere as seguintes afirmativas:
b) se t for desprezível e co = (k/m)1/2, o objeto I) O módulo da força resultante que atua na
pode estar localizado em qualquer ponto da pedra é igual a T sen0.
mesa. II) O módulo da componente, na direção do
c) a elongação da mola é x = k.A(m.co)-1. movimento, da força resultante que atua na
pedra é máximo quando a pedra atinge a
d) a elongação da mola é proporcional a co. altura máxima.
e) a aceleração tangencial do objeto é igual a III) A componente, na direção do fio, da força
k.Am-1. resultante que atua na pedra é nula no ponto
em que a pedra atinge a altura máxima.
F18) (ITA-88) Um motoqueiro efetua uma Está(ão) correta(s) a(s) afirmativa(s):
curva de raio de curvatura de 80 m a 20 m/s A) I e II, apenas B) I e III, apenas
num plano horizontal. A massa total C) II e III, apenas D) I, II e III
(motoqueiro + moto) é de 100 kg. Se o E) II, apenas
coeficiente de atrito estático entre o pavimento
e o pneu da moto vale 0,6, podemos afirmar F21) (OBF-02) Você já deve ter observado em
corridas de circuitos ovais que as curvas são

142
Elementos da Física - Mecânica I -Força Centrípeta
inclinadas. Esta inclinação das curvas, também
observadas em rodovias, é para aumentar a
segurança e diminuir o risco de derrapagens.
Considere que um automóvel realiza uma
curva de raio R e ângulo de inclinação 0.
Considere também que a pista está muito
escorregadia e o coeficiente de atrito estático
entre os pneus do carro e a pista é desprezível,
podendo o carro derrapar ao realizar a curva.

Determine:
a) A velocidade angular co do corpo em função
da aceleração da gravidade g, do comprimento
€ e do ângulo 0 de inclinação da corda.
b) O tempo para o corpo dar uma volta
a) Represente, num diagrama de forças, as completa no círculo.
forças que atuam no carro durante a curva,
identificando-as. F24) É dada uma mola de comprimento l0
b) Calcule o módulo da velocidade máxima (quando não submetida a forças) e constante
com a qual o carro pode realizar a curva sem elástica k, de massa desprezível. Uma das
risco de derrapar, em função do raio R e do extremidades da mola está presa a um ponto
ângulo de inclinação 0. fixo O de um plano horizontal perfeitamente
liso, à outra extremidade está preso um corpo
F22) (OBF-03) Um garoto gira três bolas de massa m e apoiado no mesmo plano.
amarradas entre si por cordas de 1 m de Imprime-se ao corpo uma velocidade
comprimento, num plano horizontal, conforme tangencial v, com a qual ele descreve
indicado na figura abaixo. Todas as bolas são movimento circular e uniforme em torno de O.
iguais e têm uma massa de 0,10 kg. Determinar o raio R da trajetória do corpo.

Exercícios de Aprofundamento
1
^-'1 m
1 m
A1) (UFG-11) Os povos da Antiguidade
usavam uma arma chamada funda, constituída
de uma corda dobrada ao meio, onde se
coloca o objeto a ser lançado. Para o
lançamento de um projétil de massa m, a
funda é girada no plano vertical, descrevendo
Plano horizontal uma circunferência de raio L, com o projétil
Responda às seguintes questões: passando rente ao solo. Após o lançamento, o
a) Quando a bola 3, da extremidade, estiver se projétil atinge o solo a uma distância D.
movendo com uma velocidade de 6,0 m/s, Considerando a corda inextensível e
quais serão as trações nas três cordas? desprezando a massa da corda e a resistência
b) Girando as bolas mais rápido, que corda do ar, calcule a força com que o lançador deve
romperá primeiro, supondo que todas as segurar a funda no instante imediatamente
cordas são iguais? Justifique sua resposta. anterior ao lançamento do projétil, quando a
corda forma um ângulo 0 com a vertical.
F23) (OBF-07) Em um pêndulo cônico temos
uma corda de comprimento £ e na sua
extremidade um corpo de massa m, que
realiza um movimento circular no plano. Como
conseqüência deste movimento, a corda
descreve a figura de um cone, razão pela qual l /
o pêndulo adquire esse nome. - D

143
flementos da Física - Mecânica I -Força Centrípeta
A2) (UFRJ-99) A figura representa uma roda- a) O módulo da força de tensão F, que
gigante que gira com velocidade angular permanece constante ao longo de todo o fio,
constante em torno do eixo horizontal fixo que em função de M e g.
passa por seu centro C. b) A razão K = sen a/sen 0, entre os senos
dos ângulos que o fio faz com a horizontal.
c) O número N de voltas por segundo que o
conjunto realiza quando o raio Ri da trajetória
descrita pela bolinha B for igual a 0,10 m.

A4) (ITA-85) O cachorrinho da figura tem


massa 10 kg e move-se num terreno horizontal
numa trajetória de raio de curvatura 1,0 m.
Num dado instante, sua velocidade é de 0,36
km/h e ele exerce contra o solo forças de 0,10
N (dirigida de A para o centro da curvatura C)
e de 0,050 N (tangencial). Sabendo que a mão
Numa das cadeiras há um passageiro, de 60kg do dono está na vertical erguida do centro da
de massa, sentado sobre uma balança de curvatura, podemos afirmar que a tensão na
mola (dinamõmetro), cuja indicação varia de guia e a aceleração tangencial do cachorrinho
acordo com a posição do passageiro. No ponto valem respectivamente :
mais alto da trajetória o dinamõmetro indica
234N e no ponto mais baixo indica 954N.
Considere a variação do comprimento da mola
desprezível quando comparada ao raio da roda.
Calcule o valor da aceleração local da
gravidade.
c
A3) (Fuvest-04) Um brinquedo consiste em
duas pequenas bolas A e B, de mesma massa A) zero e 5,0.10 ’3 ms -2
M, e um fio flexível: a bola B está presa na B) 0,23 e 5,0.10’3 ms 2
extremidade do fio e a bola A possui um C) 196 N e 5,0.10’3 ms 2
-2
orifício pelo qual o fio passa livremente. Para o D) 0,11 Ne 0,01 ms
jogo, um operador (com treino!) deve segurar o E) 0,23 e 0,01 ms "
fio e girá-lo, de tal forma que as bolas
descrevam trajetórias circulares, com o mesmo A5) (ITA-87) Considere a Terra como um
período T e raios diferentes. Nessa situação, corpo homogêneo, isotrópico e esférico de raio
como indicado na figura 1, as bolas R, girando em trono do seu eixo com
permanecem em lados opostos em relação ao freqüência v> (número de voltas por unidade de
eixo vertical fixo que passa pelo ponto O. A tempo), sendo g a aceleração da gravidade
figura 2 representa o plano que contém as medida no equador. Seja o’ a freqüência com
bolas e que gira em torno do eixo vertical, que a Terra deveria girar para que o peso dos
indicando os raios e os ângulos que o fio faz corpos no equador fosse nulo. Podemos
com a horizontal. Não há atrito entre as bolas e afirmar que:
o fio e considere sen 0 = 0,4, cos 0 = 0,9 e n = a) u’ = 4u
3. b) u’ = oo
c) Não existe u’ que satisfaça às condições do
problema.
1/2
d) u'= o2 + g
4ti2rJ
s1/2
e) u' = u2 - g
4n2R

A6) Um trem atravessa uma curva de raio de


Figura 1 Figura 2
curvatura igual a 100 m a 30 km/h. A distância
Assim, determine: entre os trilhos é de 1 m. De que altura é

144
Elementos da Física - Mecânica I -Força Centrípeta
preciso levantar o trilho para amenizar a horizontal, executa um movimento circular
pressão que o trem exerce sobre ele ao passar uniforme. O comprimento da mola não
pela curva? distendida vale Lo e a tração na mola aumenta
na razão direta do seu elongamento, sendo k a
A7) A figura abaixo representa um regulador tensão por unidade de elongamento. Seja f a
centrífugo. Cada esfera tem massa 2m e estão frequência (número de rotações por unidade
presas às duas barras, de comprimento / e de tempo). Determine: a) o raio R do
massas desprezíveis, através de uma junção movimento circular uniforme; b) a tração T na
móvel. A bucha B, de massa m, pode deslizar mola.
sobre o eixo com atrito desprezível, entretanto
A é fixa. O sistema gira em torno do eixo A11) Numa estrada asfaltada há um trecho em
vertical com velocidade angular w e a curva defeituoso, onde o lado externo ficou
aceleração da gravidade é g. Determinar o mais baixo do que o lado interno. Para prevenir
ângulo 0 que cada barra forma com o eixo os motoristas do perigo, quer-se sinalizar o
vertical. trecho indicado em uma tabuleta a máxima
velocidade admissível. A aceleração local da
gravidade é g. O coeficiente de atrito entre os
pneus e o asfalto é p. A inclinação da pista é 0
e seu raio de curvatura é R. Determinar a
máxima velocidade admissível.
< R;

a.
>e
A8) No sistema da figura, a bolinha de massa
m está amarrada por fios de massa A12) Um cubo muito pequeno, de massa m, é
desprezível ao eixo vertical AB e gira com colocado no interior de um funil que gira em
velocidade angular co em torno desse eixo. A torno de um eixo vertical com frequência f
distância AB vale /. Calcule as tensões nos fios rev/s. A parede do funil forma um ângulo 0
superior e inferior. Para que valor de s> o fio com a horizontal. O coeficiente de atrito
inferior fica frouxo? estático entre o cubo e o funil vale /ieo centro
do cubo está situado a uma distância r do eixo
de rotação. Determine o valor a) máximo e o
X- co
60° valor b) mínimo de f para que o cubo
m1 permaneça em repouso em relação ao funil.

A13) Sobre a superfície interna de uma esfera


30° oca de raio R, que gira em torno de seu eixo
vertical com velocidade angular constante w,
B se encontra um pequeno corpo A. Supondo
conhecido o ângulo a, encontrar o coeficiente
A9) Um disco que gira horizontalmente está de atrito mínimo no qual o corpo não caia
sujeito a um fio que possui um corpo preso na esfera abaixo.
sua extremidade, formando com a vertical um
ângulo a. A distância entre o ponto de
suspensão do fio até o eixo de rotação é igual
a d e a longitude do fio é L. Determinar a
velocidade angular de rotação do disco.

a
A14) A barra OA gira ao redor do eixo vertical
OB com velocidade angular w. O ângulo entre
A10) Um corpo que está preso a uma mola, o eixo e a barra é a. Pela barra desliza sem
cuja outra extremidade está presa ao solo atrito um corpo de massa m, unido ao ponto O

145
Elementos da Física - Mecânica I -Força Centrípeta
através de uma mola. Determinar a posição do conectada ao mesmo por uma mola idêntica á
corpo devido à rotação. A longitude da mola anterior. Determine o comprimento dv
sem deformação é l0 e sua constante elástica
é k.

EL, A
m
rrnTTK
d, H I
dj
C

A18) (UFPA-2006) Como em nossa região a


chuva é companheira habitual, também é
A15) Uma barra de peso desprezível AOO', comum que pessoas, após
dobrada como mostra a figura, gira com terem se protegido com um guarda-chuva,
velocidade angular w relativamente ao eixo inadvertidamente o ponham a girar, em torno’
OO'. Na barra foi colocada uma conta de do seu eixo, o cabo, acabando por provocar
massa m. Determinar a que distância / do outra chuva fora de hora, capaz de molhar os
ponto O, a conta ficará em equilíbrio, se o circunstantes. Se você perceber uma pessoa
coeficiente de atrito entre a conta e a barra é girar, com velocidade angular w, o guarda-
igual a /z. chuva molhado de raio R, com a aba a uma
altura h do solo, como a figura ilustra, e
considerando o campo gravitacional g, você
não será atingido pelos pingos se estiver a
uma distância daquela pessoa, superiora
<0

1
R

h
A16) Uma barra de peso desprezível, dobrada
como mostra a figura abaixo, gira com
velocidade angular a> relativamente ao eixo
OO’. No extremo da barra fixou-se um peso de
)
massa m. Determinar a força com que a barra
atua sobre a massa m.
0
a) rJi + 2^1 b) R +
w2Rh
V g g

d) hJ2+^ . w2R I Rh
e)------,-------
g Vh + R
V g
C5co

O’ A19) (OBF-04) Um pêndulo cônico de


comprimento L e massa m realiza um
movimento circular uniforme no interior de uma
A17) Uma barra horizontal, sem peso, possui superfície cônica, que não apresenta atrito
duas bolas de mesma massa m que podem quando tocada pela massa, como
mover-se sobre uma superfície sem atrito. Esta representado na figura.
barra está girando com uma velocidade
angular constante <o sobre um eixo vertical. As
bolas sâo conectadas por uma mola de rigidez
k, cujo comprimento não deformado é £0. A
bola que está mais próxima do eixo vertical é

146
Elementos da Física - Mecânica I -Força Centrípeta
A21) Uma conta com massa M, desliza ao
longo de um arame semicircular de raio R,
girando em torno de um eixo vertical a uma
O» taxa de duas voltas por segundo, conforme
mostrado na figura. O valor do ângulo 6 em
função das grandezas mencionadas, para o
L 6 qual a conta fica estacionária em relação ao
arame gigante é:
Dados: gravidade g

m*-
A geratriz do cone da superfície forma um
ângulo 9o com a vertical.
a) Qual a menor velocidade da massa do
pêndulo que a faz tocar a superfície?
b) Calcule a força que a superfície exerce
sobre a massa do pêndulo se a sua velocidade
for maior que aquela mínima. lOOg

A20) (OBF-06) O caminhão representado na


figura transporta uma bobina de aço. Os
coeficientes de atrito estático pE e cinemático
Pc, entre a bobina e a carroceria são
respectivamente iguais a 0,18 e 0,15. g T2 '
a) 0 = cos’ • d) 0 = sen' •
Considere que o caminhão esteja se movendo 4 ■ n2 • R _4nRj
com uma velocidade escalar igual a 20m/s em ' g ■ T2 '
uma estrada em duas situações distintas: a b) 0 = cos"1 • e) 0 = tg"1 •
4rt2 R
4n R
primeira, num trecho horizontal da estrada que
apresenta uma curva circular com a pista 'g'T:'
c) 0 sen
inclinada lateralmente (fig.1), e a segunda 4it R
(fig.2), em um trecho reto e horizontal da
estrada. A22) Um bloco, admitido como uma partícula
v de peso igual a 10N, repousa sustentado por
uma corda, em um plano liso e que pode girar
em torno do eixo dos y, conforme a figura.
500crn
1200cm Sendo t = 2m o comprimento da corda,
fig I
determinar a tensão na corda quando a
200cm velocidade de rotação do conjunto constituído
pelo plano inclinado e pelo bloco for igual a 10
, ri_ rpm.

© fig 2
a) Calcule, no primeiro caso, o menor valor do
raio de curvatura da pista ocupada pelo
caminhão que possibilite que ele complete a
curva sem que a sua carga deslize na 30* s
carroceria.
b) Calcule, no segundo caso, a velocidade com
que a bobina de aço colide contra a cabina do
veículo quando ele é obrigado a frear com uma A23) Em uma barra estão presos dois pesos
desaceleração constante e parar em exatos 10 de massas m e M, em um local onde a
s. aceleração da gravidade é g. A barra,
mediante uma articulação O, está unida ao
eixo OO’. O eixo OO’ gira com velocidade

147
Elementos tia Física - Mecânica I -Força Centrípeta
angular co. Suponha que a barra está em uma a) 8,1 m/s b) 6,7 m/s c) 4,8 m/s
situação de imponderabilidade, ou seja, o d) 2,7 m/s e) 1,5 m/s
torque resultante na articulação O é nulo.
A26) Um motociclista foi desafiado a entrar em
um globo da morte inclinado. Determine a
4 Z velocidade com que o motociclista deve passar
/P pelo ponto A para que seja feito o looping com
TXT segurança. Sabe-se que o raio do circulo vale
R e o coeficiente de atrito estático vale g.
Considere sen 37° = 0,6.
a

'M a1
Determinar o ângulo <p formado pela barra e a
vertical.

A24) Uma mola de constante elástica k, massa


m e comprimento natural í0 tem suas
extremidades unidas, formando uma
circunferência. A mola é posicionada em um
solo horizontal. Em determinado momento a
IgR gR 3+-J
V 55 k 4+-l
aNv
mola passa a rotacionar, em torno de seu b)
hJ 5
centro, com frequência f.
c) ./gRÍl+^-1
V k 3tv
d>
e) ./gRÍi+^-l

A27) Montamos o sistema a seguir, em que as


massas de A e B são respectivamente iguais a
750 g e 150 g. O bloco C, de largura 2 m, gira
Qual o raio final da circunferência formada pela em torno do eixo E com velocidade angular
mola? constante de 2 rad/s. Sabendo que g = 10 m/s2,
que tg 0 vale 4/3, que o comprimento do fio
A25) O cursor S de 2 kg de ajusta na haste que liga A e B mede 3 m, determine o
inclinada para a qual o coeficiente de atrito coeficiente de atrito entre as superfícies A e C
estático é pe = 0,2. Se o cursor está a 0,25 m para que estejam na iminência do
de A, determine a velocidade constante escorregamento.
máxima que ele deve ter para que não E
escorregue para cima. Considere g = 10 m/s2. CO
z A
0
3 B
c

0,25 m
d 2m
a) 0,3 b) 0,4 c) 0,5
A d) 0,6 e) 0,7

A28) Na figura, uma mola de comprimento


natural 20 cm e constante elástica 140 N/m
está amarrada a uma massa de 800 g. 0
coeficiente de atrito estático entre a massa e o

148
Elementos da Física - Mecânica I -Força Centrípeta
cone é 0,5 e o sistema gira com velocidade
angular constante de 5 rad/s. Sabendo que a
massa está na iminência de descer ao longo
da geratriz do cone, determine a deformação
da mola nessa configuração. (Dados: g = 10 h
m/s2 e cos 6 = 0,8)
Definição: geratriz é qualquer segmento que
tenha uma extremidade no vértice do cone e a
outra na curva que envolve a base.

a)1 b) 5/2 c) 3/2 d) 2 e) 5/3

A31) Um tubo de vidro de comprimento ( = 16


cm é preso por suas extremidades a um eixo
vertical zz’ mediante hastes horizontais de
comprimentos a = 9cmeb = 11 cm. No
a) 2,5 cm b) 5,0 cm c) 7,5 cm
interior do tubo coloca-se uma esfera de
d) 10 cm e)15cm
massa m = 20 g e faz-se o aparelho girar em
torno do eixo vertical. Calcule a freqüência de
A29) O instrumento da figura compõe-se da
revolução para que a esfera se mantenha no
barra em L, disposta em um plano horizontal e meio do tubo. Considere g = 9,81 m/s2.
de um corpo A de massa m, unido a uma mola
z’
fixa no ponto B. A constante de mola é k e seu b B
comprimento natural é Lo. O sistema gira ao
redor de um eixo vertical que passa pelo ponto
O, a uma velocidade angular constante co.
Determine a deformação da mola em função
de Lo, k, m e<o.

O
B;

<•> A
z
A
a) 4,43 Hz b) 26,5 Hz c) 58,1 Hz
d) 69,8 Hz e) 82,4 Hz

A30) As extremidades de dois fios de mesmo A32) Uma pessoa resolveu determinar o valor
comprimento são amarradas em dois pontos da constante elástica de uma mola de
fixos que estão alinhados verticalmente e comprimento natural 100 cm. Para tanto,
separados de uma distância h. Nas outras amarrou a ela um corpo de massa 1 kg,
extremidades dos fios existe preso um corpo conforme a figura 1, e deixou o sistema ficar
de massa m. O corpo é posto a oscilar em um em equilíbrio. A seguir, colocou a massa para
plano horizontal. Quando a velocidade angular girar num movimento circular uniforme com
velocidade angular de 5 rad/s, conforme a
do corpo vale <o = 2^^ (g é a aceleração da
figura 2. Percebeu, então, que a massa subiu
70 cm em relação à situação da figura 1.
gravidade) a tração no fio superior é T, e a Sabendo que g = 10 m/s2, determine o valor da
tração no fio inferior é T2. Determine o valor de
constante elástica.
A
L'

149
Elementos da Física - Mecânica I -Força Centrípeta
Figura 1 Figura 2
//////////////// Hll (IH motor
o
o
o
o
o
<o
o
o
~pOcm

O
o

A) 25 N/m B) 50 N/m C)1OO N/m


D) 125 N/m E) 200 N/m

A33) Na figura, uma plataforma gira com


velocidade angular constante e igual a 1 rad/s
em torno de um eixo vertical fixo ao seu centro. A figura acima mostra um dispositivo composto
Um bloco A de massa 10 kg repousa sobre a por um motor elétrico, cujo eixo se encontra
superfície da plataforma e está ligado, por ligado a uma polia ideal de raio R, solidária a
meio de um fio ideal, a um outro bloco B de uma segunda polia de raio r , sem
massa 5 kg, que pende verticalmente e é deslizamento. Solidário ao segundo eixo há um
impedido de sair da plataforma. Chamando de disco rígido metálico de raio r . Em duas
Lmin e Lmax respectivamente os valores mínimo extremidades opostas deste disco, foram
e máximo de L para os quais os blocos fixados dois pêndulos compostos idênticos,
permanecerão estacionários em relação à com fios ideais e esferas homogêneas, de
plataforma, encontramos Lmax = 3Lmin. massa m. Existe um fio extensível ligando as
Sabendo que g = 10 m/s2 e que o coeficiente
esferas inferiores, provendo uma força elástica
de atrito entre todas as superfícies vale 0,2, Feiástnca que as mantém na configuração
determine o valor de D. mostrada na figura. Determine, em função de g,
D L
m, r e R:
a) a velocidade angular cu do motor elétrico;
A b) a força elástica Feiástnca do fio extensível.
Dado:
• aceleração da gravidade: g .
■■I

1 rad/s

A34) Um automóvel arranca e, aumentando


uniformemente sua velocidade, percorre uma
curva em forma de arco de circunferência com
ângulo central de 30° e raio R. Sendo p o
coeficiente de atrito entre o asfalto e os pneus,
qual a velocidade máxima v que pode atingir o
automóvel ao final da curva?
1 pgR b) I
a) b)
1 + 3n V Vn2 +9
i TtpgR I rrpgR
c) d)
Vi+n2 Vl + 9n2
e) nda

A35) (IME-18)

150
ENERGIA

O conceito de energia é um dos temas mais importantes na ciência. Na vida cotidiana é


muito comum associar energia em termos de combustível para automóveis, eletricidade para
instrumentos residenciais e alimentos para o consumo. No entanto, estas idéias não definem a
energia de forma geral, são apenas aplicações cotidianas do uso de energia.

A energia está presente no Universo em várias formas. Todo processo físico que ocorra no
Universo envolve energia e transferências ou transformações de energia. Apesar de sua extrema
importância, a energia não é fácil de definir. Grandezas como velocidade, aceleração e força
fazem parte da experiência cotidiana, facilitando seu entendimento. Por mais que a energia
também esteja associada a situações cotidianas, como por exemplo a eletricidade dos
equipamentos eletrônicos, a noção de energia é bem mais abstrata.

O conceito de energia se aplica a sistemas mecânicos sem recorrer às leis de Newton. O


conceito de energia permite compreender fenômenos térmicos e elétricos para os quais as leis de
Newton não são úteis.

TRABALHO DE UMA FORÇA CONSTANTE

Alguns termos utilizados na física como velocidade, aceleração e força possuem um


significado similar na física como na vida diária. Porém, agora nos deparamos com um termo cujo
significado na física é particularmente diferente de seu significado cotidiano: trabalho. Para
compreender o que significa trabalho na física, considere o experimento de empurrar uma
geladeira sobre um solo áspero. Suponha que a mínima força necessária para mover a geladeira,
com velocidade constante, seja F. Sabe-se que o valor de F independe do deslocamento a ser
realizado pela geladeira. Porém, na prática, todos sabem que despende-se mais esforço muscular
para deslocar a geladeira, por exemplo, de 10 m do que de apenas 1 m. Este esforço realizado
por um força está associado diretamente ao conceito de trabalho de uma força. Para entender
melhor este conceito observe a situação que se ilustra na figura seguinte.

iY /•’cos ti

A?

Dentre todas as forças que atuam em corpo de massa destaquemos a força F, suposta
constante. Assuma que o deslocamento do corpo é Ar. Como o vetor F é constante tem-se que
o ângulo 0 formado por F e por Ar em cada instante do movimento do corpo se mantém
constante. Neste caso, o trabalho realizado pela força constante F é definido pelo produto escalar
da força F pelo deslocamento Ar:

WF = F.Af

Da definição de produto escalar segue que:

WF =|F|.| Ar |.cos0

151
Mementos da Física-Mecânica!-Trabalho e Potência

Nesta definição fica claro que quanto maior o valor do módulo do deslocamento Ar tem-se
um valor maior do trabalho da força F. Além disso, analisando o ângulo 6 formado pelos vetores
F e Ar conclui-se que:

1. Se 0 = 0 o trabalho é calculado diretamente pelo produto do módulo da força pelo módulo do


deslocamento. Neste caso o trabalho realizado pela força F é sempre não negativo;
2. Se 0 < 0 < 90° tem-se cos 0 > 0 e assim o trabalho da força F é sempre não negativo;
3. Se 0 = 90° tem-se cos 0 = 0 e deste modo o trabalho da força F é nulo;
4. Se 90° < 0 < 180° tem-se cos 0 < 0 e neste caso o trabalho da força F é sempre não positivo.

Os termos “não negativo” e “não positivo” foram usados nos lugares de positivo e negativo,
respectivamente, uma vez que o valor do trabalho da força F, independentemente do ângulo 0,
pode ser nulo caso o módulo de F ou o módulo de Ar seja igual a 0.

Na figura abaixo um bloco é empurrado sobre uma mesa lisa devido à ação de uma força
constante F que faz um ângulo 0 com o vetor deslocamento Ar. Suponha que | F | ,sen0 < m|g|,
ou seja, o bloco não perde contato com o solo durante seu movimento.

n XF

>

AF

V '"g

Na decomposição das forças atuantes no bloco pode-se identificar a força F , a força peso
mg e a força de reação normal n. O trabalho da força F é calculado por WF =|F|.| Ar|.cos9.
Como 0 < 0 < 90° segue que o trabalho da força F , neste exemplo, é positivo.

Note que tanto a força peso quanto a força normal são perpendiculares, durante todo o
movimento do bloco, ao vetor deslocamento Ar. Deste modo, segue que os trabalhos da força
peso e da força de reação normal são iguais a zero.

A unidade de trabalho é igual ao produto da força pelo comprimento. Em conseqüência, a


unidade no SI de trabalho é Newton.metro (N.m = kg.m2/s2). Esta combinação de unidades é
usada com tanta frequência que se recebe um nome próprio: Joule (J).

Una consideração importante é que o trabalho é uma transferência de energia. Se W é o


trabalho de um sistema e W é positivo, afirma-se que o sistema realizou trabalho e a energia se
transfere de sua vizinhança para o sistema. Por outro lado, se W é negativo afirma-se que foi
realizado trabalho sobre o sistema e a energia se transfere desde o sistema para sua vizinhança.

152
Elementos da física-Mecânica !- Trabalho e Potência
TRABALHO DE FORÇAS VARIÁVEIS

Suponha que um corpo de massa m se desloca ao longo do eixo x, desde a posição inicial
Xj até a posição xf. Seja F uma força variável que está atuando sobre o corpo. Neste caso não é
possível aplicar diretamente a expressão W = F.Ar.cos 0 para calcular o trabalho da força F uma
vez que a força F não é constante. Para calcular o trabalho realizado por F tomemos um
pequeno deslocamento Ax da trajetória do corpo. Como o tempo decorrido para o corpo percorrer
Ax é muito pequeno, pode-se considerar que F não varia neste intervalo de tempo. Seja Fx a
componente da força F sobre eixo x enquanto o corpo percorre a distância infinitesimal Ax. Deste
modo, o realizado trabalho pela força F no deslocamento infinitesimal Ax é igual W = Fx.Ax.

Perceba que a trajetória do corpo, desde Xi até xf, pode ser dividida em uma quantidade
muito grande de pequenos deslocamentos Ax. Para cada um destes pequenos deslocamentos Ax
o trabalho realizado é igual a Fx.Ax, onde o valor de Fx varia em cada intervalo. Fazendo um
gráfico da variação de Fx por x pode-se analisar como a componente de F na direção x varia com
o deslocamento do corpo. Neste gráfico o valor de Fx.Ax é numericamente igual a área AA do
retângulo destacado. Tomando valores de Ax cada vez menores e somando todos os valores
encontrados para Fx.Ax conclui-se que o trabalho total da força variável F, desde x, até x(, é
numericamente igual a área compreendida entre a linha do gráfico e o eixo x.

Xf
W = Área = ^Fx.Ax
Xf

Área = AA = 7\Ax
Fx
Fx
|T
r-

r
r- Fx
Trabalho

-1----X
► -
xf -1—x
Ax •v, V
Em um gráfico onde o comportamento da variável y (eixo vertical) em função da variável x
(eixo horizontal) é representado por uma linha contínua a área compreendida entre a linha do
gráfico e o eixo horizontal é igual à integral da variável y pelo infinitesimal da variável x:

x'
Área = J ydx

Assim, o trabalho de uma força F, quando um corpo é deslocado de uma posição inicial Xj
até uma posição final xf, é igual a:

Xf

w jFxdx
Xf

153
flementos da física-Mecânica!-Trabalhos Potência
TRABALHO TOTAL

Suponha um corpo de massa m que se desloca no espaço deste um posição inicial r; até
uma posição final rf. Suponha que n forças F,,F2t...Fn estão atuando no corpo. Desta forma, a
força resultante no corpo é igual a:

Fr = F1 + F2 + - + Fn

Multiplicando todos os termos desta expressão pelo diferencial do vetor posição dr:

FR.df = Fvdr + F2.dr +... + Fn.dr

Integrando todos os termos:

Q rf rf _ 0
|FR.dr =jFvdr +jF2.dr+... + jFn.dr
ri 1 «i «i

Cada integral da equação anterior é o trabalho realizado pela respectiva força no trajeto de
i; até rf. Deste modo, pode-se afirmar que:

WF'R WF) + WF2 +... + WFn

Desta maneira, conclui-se que o trabalho realizado pela força resultante (também
conhecido como “trabalho total”) é igual à soma dos trabalhos realizados por todas as forças
atuantes no corpo.

A figura abaixo exemplifica uma situação em que um corpo é arrastado por uma força F,
que faz um ângulo 0 com o horizontal, sobre um solo áspero.

F ^7
ti
■F
— ■Ax

Na decomposição de forças fk é a força de atrito, n a força de reação normal com o solo e


mg o peso do corpo. Logo, o trabalho da força resultante é igual a:

WF WF+Wn+Wmg + wfTk

Porém, neste caso, tanto força normal quanto peso são perpendiculares ao vetor posição,
implicando que os trabalhos da reação normal e da força peso são iguais a zero. Além disso,
pode-se observar que o ângulo formado pelo vetor força de atrito, suposta constante, e pelo vetor
posição é igual a 180°. Assim, para a situação proposta, tem-se que o trabalho da força resultante
é igual a:

WFr =|F|.| Ar |.cos0+| fk |.| Ar |.cos180° WFr =|F1.1 Ar |.cose-| fk |.| Af |

154
Elementos da física-Mecânica !-Trabalho e Potência
TRABALHO DA FORÇA PESO

A força peso, para pontos próximos à superfície da Terra, poder ser considerada uma força
constante. Para calcular o trabalho da força peso vamos separar a análise em três casos: corpo
subindo verticalmente, corpo descendo verticalmente e corpo em movimento não-vertical.

Ar Ar
F

37 y,

y. yr

1) Corpo Subindo Verticalmente: Quando o corpo está subindo na vertical, pela ação de uma
força externa F, o ângulo formado pelo vetor deslocamento Ar e pela força peso mg é igual a
180°. Suponhamos que neste deslocamento o corpo suba uma altura h. Portanto:

Wp =|mg||Ar|.cos180° => WP = -mgh

2) Corpo Descendo Verticalmente: Quando o corpo está descendo na vertical (não é necessário
que nenhuma força externa atua no corpo para tanto) o ângulo formado pelo vetor deslocamento
Ar e pela força peso mg é igual a 0. Suponhamos que neste deslocamento o corpo desça uma
altura h. Portanto:

Wp =| mg || Ar | .cosO => WP = mgh

3) Corpo em Movimento Não Vertical:

Ar
h

mg, ,
£

Suponha uma situação prática, onde uma geladeira deve ser empurrada sobre uma rampa,
conforme indicado na figura acima. Suponha que a massa da geladeira seja m, o comprimento da
rampa seja L, o ângulo de inclinação da rampa com a horizontal seja 0 e a altura, em relação ao
solo, do ponto mais alto da rampa seja h. Note que o ângulo formado pelo vetor deslocamento Ar
e pela força peso mg é igual a 90° + 0. Neste caso, o trabalho da força peso vale:

WP =|mg|| Ar |.cos(9O°+0) = -mgLsen0 => WP = -mgh

Caso o corpo estivesse descendo a rampa o valor encontrado para o trabalho da força
peso seria WP = mgh.

155
___________ _ ____ _ ___________________ [tememos da Física-Mecânica !-Trabalhos Potência
Do exposto anteriormente pode-se concluir que o trabalho da força peso independe da
trajetória, dependendo apenas da massa m do corpo e da distância vertical entre o ponto final e o
ponto inicial da trajetória. Deste modo, em um movimento em um plano, se o ponto inicial da
trajetória é P0(x0, yo) e o ponto final é o ponto Pf(xf, yf) o valor do trabalho da força peso é:

Wp = mg(y0 - yf).

O módulo de y0 - yf é a variação da altura do movimento: h = |y0 - yf|. Assim, se ao final de


sua trajetória o corpo subiu verticalmente o valor de y0 - yf é negativo e o trabalho da força peso
vale - mgh. Por outro lado, se ao final de sua trajetória o corpo desceu verticalmente o valor de
y0 - y< é positivo e o trabalho do peso vale + mgh.

Existem duas situações em que o trabalho da força peso é nulo em um corpo de massa m
não desprezível. A primeira situação ocorre quando os pontos iniciais e finais da trajetória do
corpo são iguais, ou seja, quando h = 0. A segundo situação ocorre quando a aceleração
gravitacional local pode ser considerada nula, como por exemplo em um ponto do espaço sideral,
distante o suficiente de qualquer astro, de modo que a atração gravitacional neste ponto possa ser
considerada nula.

Exemplos:

1) (ITA-89) Um objeto de massa m = 1,0 kg é lançado de baixo para cima, na vertical, com
velocidade vo. Ao passar por uma posição y! ele está com velocidade vi = 4,0 m/s e numa
posição y2 sua velocidade é v2 = 2,0 m/s . Desprezada a resistência do ar, o trabalho realizado
pela força da gravidade (Wg) entre yi e y2 e o deslocamento (y2 - y-i) são respectivamente: (Adota
g = 9,8 m/s2)
Wg(J) Y2 - Y, (m)
A) 6,1 6,0
B) -6,0 5.9.10’1
C) 1,0 6.1.10’1
D) -1,0 1,0.10’1
E) -6,0 6,1.10’1
Solução:
Como a aceleração é constante:
v22 = v,2-2g(y2-yi) => 16 = 4-2.9,8.(y2-y,) => y2 - y, = 6,1.10~1 m
O trabalho realizado pela força peso vale:
Wp = mg(y1-y2) = (1,0)(9,8)(-6,1.10-1) => WP = -6,0J

2) (ITA-88) Dois baldes cilíndricos idênticos, com as suas bases apoiadas na mesma superfície
plana, contém água até as alturas h, e h2, respectivamente. A área de cada base é A. Faz-se a
conexão entre as bases dos dois baldes com o auxílio de um fina mangueira. Denotando a
aceleração da gravidade por g e a massa específica da água por p, o trabalho realizado pela
gravidade no processo de equalização dos níveis será :

Tffl
h, [||T
llllllllllllll
XxvXvsVsVsVs

a) pAg(h, - h2)2/4 b) pAg(h, - h2)2/2 c) nulo d) pAg(h, + h2)2/4 e) pAg(h, + h2)2/2


Solução:

156
[iementos da Física-Mecânica i-Trabalho e Potência
O trabalho da gravidade no processo de equalização do
nível da água nos dois baldes é igual ao trabalho de levar,
h,-h; lentamente, uma coluna de água de altura Ah = -1 - do
2
h,-h;
balde esquerdo para o balde direito, conforme indica a
h<-h;
2
figura ao lado. Deste modo:
2
~^2 pgA^-h-,)2
W = mgAh = pgAAh2 = pgA
2 4
5Í ' '57

3) (Olimpíada de Física ldeal-2014) Em uma superfície horizontal repousam 5 blocos de ferro, cuja
densidade é igual a p, de mesma área de seção transversal S (figura 1). As alturas destes blocos
são iguais a h, 2h, 3h, 4h e 5h, respectivamente. Se a aceleração da gravidade local é g:

5h

4h

3h

5h
4h 2h
h:: -

figura 1 figura 2
a) determine o trabalho total da operação de empilhamento que consiste em colocar os blocos uns
sobre os outros conforme a figura 2;
b) demonstre que o valor total de colocar os 5 blocos empilhados independe da sequência dos
blocos na pilha.
Solução:
a) De modo a realizar o menor trabalho possível os blocos devem ser empilhados bem
lentamente, ou seja, as forças usadas para içar cada bloco devem possuir mesmo módulo que a
força peso. O trabalho total é igual a soma dos trabalhos de empilhamento de cada bloco.
WT = W2 + W3 + W4 + W5 = m2gh + m3g(h + 2h) + m4g(h + 2h + 3h) + m5g(h + 2h + 3h + 4h) =>
WT = gh[PS(2h) + 3pS(3h) + 6pS(4h) + 10pS(5h)] = pgSh(2 + 9 + 24 + 50) = 85pgSh
b) Suponha agora uma sequência aleatória de empilhamento, com o bloco b sendo colocado
sobre o bloco a, o bloco c sendo colocado sobre b, o bloco d sendo colocado sobre ceo bloco e
sendo colocado sobre d. Neste caso, a, b, c, d, e é uma permutação de 1,2, 3, 4, 5.
WT = Wb + Wc + Wd + We = mbgha + mcg(ha + hb) + mdg(ha + hb + hc) + meg(ha + hb + hc + hd) =>
WT = pgShbha + pgShc(ha + hb) + pgShd(ha + hb + hc) + pgShe(ha + hb + hc + hd) =>
WT = pgS(hahb + hahc + hahd + hahe + hbhc + hbhd + hbhe + hchd + hche + hdhe)
Como a expressão acima é simétrica em relação às alturas, o valor de WT independe da
sequência com que os blocos são empilhados.

157
fíementos da física-Mecânica!-Trabalho e Potência
TRABALHO DA FORÇA ELÁSTICA

Um bloco está sobre uma superfície horizontal sem atrito e é conectado a uma mola. Para
uma mola classificada como “mola ideal”, se a mola está esticada ou comprimida de uma pequena
distância comparada a seu comprimento, surge na mola uma força que pode ser representada
matematicamente como

Fe = - k.x

onde x é a posição do bloco em relação a sua posição de equilíbrio (x > 0) e k é uma constante
positiva chamada constante elástica da mola.
i I
i I

-F
i
i
i
t
=wwwwwwwi/w^,,arl I
i
i I

I— I
O sinal negativo na fórmula significa que a força Fe é uma força de restauração ou
restituição, em outras palavras, uma força que sempre tem a tendência de restaurar o sistema a
posição de equilíbrio. Nas duas figuras acima a linha pontilhada indica a posição em que a mola
está relaxada, ou seja, a posição em que a força na mola é nula. Contando o valor de x a partir
desta linha pontilhada é possível verificar que o sentido da força na mola é sempre contrário ao
sentido de x. Em outras palavras, a força que é requerida para esticar ou comprimir uma mola é
proporcional ao valor do estiramento ou compressão x, sempre em sentido contrário ao usado
para medir x a partir da posição de mola relaxada. Desta forma, uma vez colocado o corpo em
uma posição diferente da posição de equilíbrio, o corpo passa a oscilar em torno desta posição de
equilíbrio. Este sistema é denominado “sistema massa-mola”.

Esta lei para molas é conhecida como lei de Hooke. O valor de k é uma medida da rigidez
da mola. As molas mais rígidas possuem grandes valores de k enquanto que as molas mais
suaves possuem pequenos valores de k. Como se pode ver da equação, a unidade de k no SI é
N/m. A forma vetorial da equação é:

Fe = Fe i = -k.x.T

Para calcular o valor do trabalho da força elástica vamos agora adotar um eixo
unidimensional x cuja origem coincide com o ponto de equilíbrio e está orientado da esquerda
para a direita. Suponha que o bloco está se deslocando desde uma posição x, até uma posição xf,
se aproximando da posição de equilíbrio por valores negativos de x. O gráfico da força Fm por x
está representado abaixo:

-kx,

- kx(

* Xf
>
X

Fe = - kx

158
________ _ ____________________________________ Elementos da Física-Mecânica!-Trabalhos Potência
Sabe-se que o trabalho de uma força é numericamente igual a área do gráfico F por x.
Assim, o trabalho da força elástica é igual a área destaca no gráfico Fe por x:

kx2 kx2
wFe = (-kXj-kXt) Wp = —
2 2 2

No gráfico pode-se observar que |X|| > |x(| fazendo com que > 0 . Este resultado já era
esperado para o exemplo dado, uma vez que foi assumido que o corpo estava se deslocando da
esquerda para a direita, mesmo sentido da força elástica, ou seja, o ângulo formado pela força
elástica Fe e por dx é igual a zero. Caso o bloco estivesse se deslocando por valores negativos
de x, porém agora se afastando do ponto de equilíbrio (ou seja, da direita para a esquerda), o
trabalho da força elástica seria negativo, pois neste caso tem-se |Xj| < |Xf|. Este resultado também
concorda com a análise do ângulo entre os vetores Fe e dx, que neste caso é igual a 180°.
Generalizando, toda vez que |X|| > |xf| o trabalho da força elástica é positivo e toda vez que |X|| <
|xf| o trabalho da força elástica é negativo.

Da equação conclui-se que o trabalho realizado pela força elástica é nulo para qualquer
movimento que termine onde começou (x, = x(). Deste modo, o trabalho da força elástica
independe da trajetória do corpo, dependendo apenas das posições iniciais e finais do de sua
trajetória.

Exemplo:

1) Uma mola está presa em um teto, conforme indica a figura, em um local onde a aceleração da
gravidade é g = 9,8 m/s2. Um objeto de massa m = 0,55 kg é unido ao seu extremo inferior. O
objeto é lentamente levado até uma posição de equilíbrio, percorrendo uma distância d = 2,0 cm
desde a posição em que é unido à mola.
A) Qual é a constante elástica da mola?
B) Qual o trabalho realizado pela força elástica?

fi
f
Solução:
a) Na posição de equilíbrio o módulo do peso é igual ao módulo da força elástica:
Fs = mg => kd = mg => k = ^p = ^y^|^- = 2,7.102N/m

b) O trabalho da força elástica é dado por:


k/° kx 2 _ kd2_ (2,7.102)(0,02)2
kx2
=-5,4.1 CF2 J
f’~/2
2 2“------- "

159
Elementos da Física-Mecânica!-Trabalho e Potência
TRABALHO DA FORÇA DE ATRITO CINÉTICO

A força de atrito cinético sempre se opõe ao movimento do corpo. Logo, dado um


diferencial dr do movimento de um corpo, o vetor Fat cin possui mesma direção e sentido oposto
ao do vetor dr, ou seja, o ângulo formado pelos vetores Fatcjn e dr, em qualquer ponto da
trajetória, é 180°. Por este motivo o trabalho da força de atrito cinético, em um referencial inercial,
é sempre negativo.

Suponha agora a situação de arrastar um livro de massa m sobre uma mesa áspera, com
coeficiente de atrito cinético pc, de um ponto A para o ponto B. O livro é arrastado, pela ação de
uma força F, por duas trajetórias: (1) trajetória em linha reta e (2) semicírculo de diâmetro AB.
Adote a distância AB igual a 2R, ou seja, na trajetória (2) o raio do semicírculo é R. Perceba que
nos dois casos os módulos das forças de atrito são iguais. No caso (1) o vetor Fat é constante,
porém no caso (2), apesar do módulo constante, o vetor Fat varia sua direção, se mantendo
sempre tangente à trajetória semicircular. Nos dois casos o módulo da força normal é igual ao
módulo da força peso.

®,í.
F (2)
IT

1J Fat

(D
A
< ~2R

No trajeto (1) a força de atrito cinética é constante e portanto seu trabalho vale:

WF.. = Fat.AB =|Fat |.|2R|.cos180° N.pc.2.R = -2.m.g.p0.R

No trajeto (2) o trabalho da força de atrito cinética vale:

WFK2 = jFat.dr = J|Fat |. | dr |.cos180° = -1 Fat | j| dr |= -N.pc.AB = -N.pc.2.7t.R = -2.n.m.g.pic.R

Note que WFati *WF^2. Dos cálculos apresentados pode-se concluir que o trabalho da
força de atrito cinético depende da trajetória do corpo. No capítulo sobre conservação da energia
mecânica será apresentado o conceito de força conservativa e será possível identificar as forças
cujos trabalhos dependem ou não da trajetória.

Os últimos três resultados apresentados neste capítulo nos permite concluir que os
trabalhos da força peso e da força elástica independem da trajetória do corpo, dependendo
apenas dos pontos inicial e final da trajetória, no entanto, o trabalho da força de atrito depende da
trajetória do corpo.

No exemplo apresentado o corpo estava apoiado em uma mesa horizontal. Porém, nem
sempre um corpo está apoiado em uma superfície perfeitamente horizontal, podendo existir uma
inclinação na superfície de apoio do corpo. Suponha agora que um corpo de massa m é
deslocado da base até o topo de um plano inclinado cujo ângulo de inclinação é 0 e cujo
coeficiente de atrito cinético é pc. Neste caso, o trabalho da força de atrito é dado por:

160
Elementos da Física-Mecânica !-Trabalho e Poténda

-m.g.pc.cos6.—t—
Wf., = Fat.Ar =| Fat |. | Ar | ,cos180°= -N.pc.~ -m.g.pc.L
cos 6

Note que o valor encontrado para o trabalho da força de atrito cinético ao longo de um
plano inclinado é igual ao trabalho encontrado quando o corpo é deslocado em um plano
horizontal cujo comprimento da trajetória é igual à projeção horizontal da superfície do plano
inclinado. Este resultado permite generalizar o cálculo do trabalho da força de atrito para
superfícies inclinadas que não sejam retas. A única restrição é que o coeficiente de atrito cinético
se mantenha o mesmo em todos os pontos da trajetória.

—JXL<
«----------------------- L----------------------►

Na figura acima um corpo de massa m sobe uma superfície de inclinação variável, desde a
base até o seu topo, em um local onde a aceleração da gravidade é g. Suponha que o coeficiente
de atrito cinético entre o corpo e a superfície seja constante e igual a pc. Na figura está destacado
um pequeno deslocamento horizontal x do corpo, tomado ao longo de sua trajetória. Do que foi
exposto anteriormente conclui-se que o trabalho da força de atrito cinético para deslocar
horizontalmente o corpo de uma distância x vale WFat = - mg^x. Assim, o trabalho total da força
de atrito cinético para levar o corpo da base ao todo da superfície é igual a:

F =E-mgPex = -mgticXx = -m9PcL


W'rat

Assim, pode-se afirmar que o trabalho da força de atrito cinético ao longo de uma
superfície qualquer, cujo coeficiente de atrito cinético é constante, é igual ao trabalho encontrado
quando o corpo é deslocado em um plano horizontal cujo comprimento da trajetória é igual à
projeção horizontal da superfície.

161
fíementos da física-Mecânica i-Trabalho e Potência
Exemplos:

1) (ITA-78) Na situação ilustrada na figura ao lado, deseja-se levar um corpo do ponto "A" ao
ponto “B”, sob a ação da gravidade. Os caminhos a seguir serão ACB ou ADB. Sendo o
coeficiente de atrito o mesmo para os dois caminhos e WAcb e WAdb os trabalhos ao longo das
respectivas trajetórias, pode-se concluir que:
A

1 B

a) Wacb > Wadbí


b) Wacb < Wadbí
c) o trabalho total ao longo de qualquer trajetória é nulo, visto que há equilíbrio entre a força
gravitacional e a força de atrito;
d) Wacb = WAdb. pois o trabalho realizado somente depende de “h” e “d”.
e) Não é possível fazer nenhuma afirmação.
Solução: Alternativa D
Independentemente da trajetória apenas duas forças que atuam no corpo: Peso e Força de Atrito.
Assim, Fr = P + Fat => WFr = WP + WFai
O trabalho da força peso, independentemente do caminho, é igual a + mgh.
No item sobre trabalho da força de atrito cinético foi demonstrado que o trabalho da força de atrito
cinético em uma superfície de inclinação variável, que é o caso nos dois caminhos, é igual ao
trabalho da força de atrito cinético em uma superfície horizontal cujo comprimento da trajetória é
igual a projeção horizontal da trajetória inclinada. Como os trajetos ACB e ADB possuem a mesma
projeção horizontal d segue que WFa| = -mgpL.
Logo, Wacb = WAdb - mgh - mgpL, ou seja, o trabalho depende apenas de h e d.

2) (ITA-88) Um fio de comprimento L = 1,0 m tem, fixo em uma das extremidades, um corpo de
massa m = 2,0 kg, enquanto que a outra extremidade acha-se presa no ponto O de um plano
inclinado. O plano inclinado forma um ângulo 0 = 30° com o plano horizontal. O coeficiente de
atrito entre o corpo e a superfície do plano inclinado é p. = 0,25. Inicialmente o corpo é colocado na
posição A, em que o fio está completamente esticado e paralelo ao plano horizontal. Em seguida
abandona-se o corpo com velocidade inicial nula. Calcular a energia dissipada por atrito,
correspondente ao arco AB, sendo B a posição mais baixa que o corpo pode atingir. Dado g = 10
m/s2.

a) 6,8 J b) 4,3 J c) 3,1 J d) 10,0 J e) 16,8 J


Solução:

162
_________________________________________ Mementos da Física-Mecânica !-Trabalho e Potência
A energia dissipada pelo atrito é igual ao módulo do trabalho da força de atrito. Perceba que
durante toda a trajetória a força de atrito possui módulo constante, apesar da direção de seu vetor
variar, sendo tangente à trajetória em cada ponto de um quarto de arco de circunferência. Como o
diferencial do vetor deslocamento df também é tangente à trajetória o ângulo formado por Fat e
df é igual a 180°, em cada ponto do trajetória do corpo. Assim:
WFal = f Fat dr = j| Fat |. | dr |.cos 180° = -1 Fat | J| dr |= -m.g.cose.pc.ÃB =

„ 2nR nm.g.COS6.pr.R
= -m.g. cos 0.pc —— =-------
4 2
Substituindo os valores numéricos:
(3,14)(2,0)(10)(0,866)(0,25)(1,0)

A energia dissipada pelo atrito é igual ao módulo do trabalho da força de atrito: Ed = 6,8 J

3) (Fuvest-95) Um corpo de massa m está em movimento circular sobre um plano horizontal,


preso por uma haste rígida de massa desprezível e comprimento R. A outra extremidade da haste
está presa a um ponto fixo P, como mostra a figura (em perspectiva). O coeficiente de atrito entre
o corpo e o plano é constante. Num dado instante, o corpo tem velocidade de módulo v e
direção paralela ao plano a perpendicular à haste.

p m
R

a) Qual deve ser o valor de v para que o corpo pare após 2 (duas) voltas completas?
b) Qual o tempo gasto pelo corpo para percorrer a última volta antes de parar?
c) Qual o trabalho realizado pela força de atrito durante a última volta?
Solução:
a) A força resultante na direção tangencial do movimento é a força de atrito. Assim, a aceleração
tangencial pode ser calculada da seguinte forma:
Ft = Fat => mat = mgp => at = gp
Observe que, apesar do sinal positivo da aceleração, o sentido desta aceleração tangencial é
contrário ao sentido da velocidade, sempre diminuindo o valor de seu módulo.
Para o corpo parar depois de duas voltas o comprimento da trajetória é igual a d = 2.2rrR = 4nR
Pela equação de Torricelli: vf2 = v02-2ad => 0 = v2-2gp4nR => v = 272ngpR
b) Analisando o movimento de trás pra frente, tem-se uma velocidade inicial nula e uma
aceleração tangencial com sinal invertido ao calculado. Neste movimento:
Ax = vot + — => 2rrR = 0 + ^- => t = 2^
0 2 2 Vgp
c) Como foi exposto nos exemplos anteriores tem-se que o trabalho da força de atrito cinético cujo
módulo é constante é igual a - Fat.d, onde d é o comprimento da trajetória. Deste modo, o
trabalho da força de atrito na última volta vale:
WFal = - Fat.d = - mgp27tR = - 27tpgmR

163
Elementos da Física-Mecânica !-Trabalho e Potência
TRABALHO E ENERGIA CINÉTICA

Uma das possibilidades quando um sistema realiza trabalho é uma alteração da velocidade
dos corpos que compõe o sistema. Suponha um sistema formado por apenas um corpo de massa
m que se desloca de uma distância Ax sob a ação de uma força resultante FR = ^F- Neste
deslocamento, seja v, a velocidade inicial e vf a velocidade final do corpo.

A.x

v,

Para facilitar o entendimento, a demonstração do teorema trabalho-energia será feita para


dois casos: FR constante e FR variável. Ao final das demonstrações dos dois casos será possível
concluir que os resultados encontrados são rigorosamente iguais.

1. Força Resultante Constante

Se Fr é uma força constante então a aceleração ã de m é constante. Assim, pelo


equação de Torricelli segue que:

v2 = v2 + 2a.Ax a.Ax
2 2

Como a força FR é constante e possui direção paralela ao vetor deslocamento x, o


trabalho realizado pela força resultante vale:

mv2 mv2
WFr =| Fr || Ax |= m.a.Ax = m
2 2 ~2~ 2

2. Força Resultante Variável

dx
= J pR-dx = j mã.dx = í m—.dx = J m^.dv = jmv.dv = ^_22XL
X, Xj x, dt X| dt J 2 2

Note que independentemente se a força resultante é constante ou variável encontra-se o


mesmo resultado para o trabalho da força resultante:

mv2 mv2
Wfr
2 2

Diz-se que o trabalho realizado pela força resultante em uma partícula de massa m é igual
mv2 mv2
à diferença entre os valores final e inicial de uma quantidade ——. A quantidade —— representa

a energia associada com a velocidade instantânea da partícula. Esta quantidade é tão importante
mv2
que recebe um nome especial “energia cinética”: ——. Energia cinética é uma grandeza escalar e

164
____ __________________________________________ flementos tia física-Mecânica!-Trabalhos Potência
tem as mesmas unidades que o trabalho, ou seja, é medido em Joules no SI. A equação
demonstrada anteriormente afirma que o trabalho realizado em uma partícula pela força resultante
nesta é igual à variação da energia cinética da partícula. Com frequência é conveniente escrever a
equação do teorema trabalho-energia na forma:

Wfr=Ec,-Ec.

0 teorema trabalho-energia indica que a velocidade do sistema aumenta se o trabalho da


força resultante sobre o sistema é positivo e a velocidade diminui se o trabalho da força resultante
é negativo.

Exemplos:

1) (ITA-72) Um bloco de massa m = 3,0 kg desce uma rampa, a partir do ponto P onde estava em
repouso. De P até Q o atrito é nulo, mas de Q a R a superfície oferece um coeficiente de atrito
cinético igual a 0,25. No trajeto de Q a R o bloco encontra uma mola horizontal de constante
elástica k = 1,5 x 105 N/m. Nestas condições, os trabalhos realizados sobre o bloco pelas forças
de gravidade, de atrito e da mola (Tg, Ta, TM), até que o corpo chegue ao repouso comprimindo a
mola, serão aproximadamente:
Z\ni
P

2 , Om

L_ M----------
I
!
k

R
'*--------- 4,0m *■

Tg Ta TM joules
A) 60 30 30
B) -60 28 41
C) 60 -30 26
D) 60 30 30
E) 60 -30 - 30
Solução: Alternativa E
A força peso executa trabalho não nulo no trajeto PQ, cujo valor é:
Tg = + mgh = (3,0)(10)(2,0) = 60 N
Como a força de atrito entre Q e R é constante:
Ta = - Fat.dQR = - mgpdQR = - (3,0)(10)(0,25)(4,0) = - 30 J
No trajeto PR as forças que atuam no corpo são Normal, Peso, Atrito e Força Elástica. Deste
modo, o trabalho da força resultante é igual a:
WFr = WN + WFat + WP + WFe = - 30 + 60 + WFe = 30 + WFe
Pelo princípio do trabalho-energia sabe-se que o trabalho da força resultante é igual à variação da
energia cinética. Porém em P e em R a energia cinética é nula, uma que vez o corpo está parado
nestas posições. Logo o trabalho da força resultante entre os pontos P e R é zero:
WFr=30 + WFe = 0 => WFe = - 30 J

2) Um corpo sem velocidade inicial desliza sobre paredes lisas que passam por um fundo
horizontal. A longitude do fundo é l = 3,0 m. O coeficiente de atrito do corpo com o fundo é p =
0,25. A altura das paredes é H = 5 m. A que distância do centro do sistema o corpo irá parar?

T
H

l~ ♦

165
[iementos da física-Mecânica!-Trabalho e Potência
Solução:
Durante todo o movimento do corpo atuam apenas 3 forças: Peso, Normal e Atrito. Como o ângulo
da normal com o vetor deslocamento é sempre 90° o trabalho da normal é zero. Assim, o trabalho
da força resultante vale:
WFr = WN + WP + WFal = 0 + mgH - Npx = mgH - mgpx, onde x é a distância total percorrida pelo
corpo até parar.
Pelo teorema do trabalho-energia o trabalho da força resultante é igual à variação da energia
cinética. Como o corpo inicia e termina seu movimento em repouso tem-se que a variação da
energia cinética é zero:
WFr = mgH - mgpx = 0 => mgpx = mgH => x = — => x = —— = 20m
P 0,25
Como o comprimento do fundo é t = 3,0 m e 20 = 3.6 + 2 o corpo percorrerá completamente o
fundo por seis vezes e na sétima vez percorrerá mais 2,0 metros até parar. Como o início da
sétima vez que passa fundo é pela parede esquerda o corpo, percorrendo mais dois metros, irá
parar a um metro do centro do sistema, à sua direita.

3) (ITA-83) Um bloco de massa m = 2,0 kg desliza sobre uma superfície horizontal sem atrito, com
velocidade v0 = 10 m/s, penetrando assim numa região onde existe atrito de coeficiente p = 0,50.
Pergunta-se:
a) Qual é o trabalho (W) realizado pela força de atrito após ter o bloco percorrido 5,0 m com atrito?
b) Qual a velocidade do bloco ao final desses 5,0 m? (g = 10 m/s2).
W(J) v (m/s)
(A) + 50 7,1
(B) - 50 6,9
(C) + 100 0
(D) - 50 7,1
(E) 0 10
Solução: Alternativa D
a) Como a força de atrito é constante seu trabalho vale:
WFal = - Fat.d = - mgpd = - (2,0)(10)(0,50)(5,0) = - 50 J
b) Apenas três forças atuam no bloco: Normal, Peso e Atrito.
Como as forças Peso e Normal são perpendiculares ao vetor deslocamento tem-se que seus
trabalhos são nulos. Deste modo:
WFr = WN + WP + WFat = Erf - Ec0 = mv 2/2 - mv02/2 => 0 + 0 - 50 = 2v 2/2 -2.100/2 =>
v2 = 50 => v( = 7,1m/s

4) (UFC-05) Um corpo de massa m desloca-se da posição A para a posição B, seguindo a


trajetória semicircular mostrada na figura abaixo.

vo
* V
------
:1 d--------- *
Em outro instante, o mesmo corpo desloca-se da posição A para a posição B, seguindo a
trajetória retilínea, de comprimento d, indicada na mesma figura. Essas trajetórias localizam-se
sobre uma mesa (considere a mesa plana e horizontal). O módulo da velocidade inicial em ambos
os casos é v0 e a velocidade final no trajeto semicircular é zero. O coeficiente de atrito cinético
entre o corpo e a mesa, em ambos os casos, é p. Determine o módulo da velocidade final, v, em
função de v0, quando a partícula segue a trajetória retilínea.
Solução:

166
______ ________________________________________ Uementos da Física-Mecânica !-Trabalho e Potência
Como já foi explanado anteriormente o trabalho da força de atrito cinético, quando o módulo do
Fat constante, é igual - Fat.d, onde d é o comprimento da trajetória do corpo. Nos dois trajetos as
forças que atuam no corpo são Peso, Normal e Fat. Os trabalhos do Peso e Normal são nulos
uma vez que seus vetores são perpendiculares ao vetor deslocamento. Deste modo, pelo teorema
do trabalho energia no trajeto semicircular:
WFres = WP + WN + WFat = Ecj- Ec0 => 0 + 0-mgp7t|-
O-Eco => Ec0=™^

nXÍgpd
No trajeto reto: WFres = WP + WN + WFat = ECf - Eco => 0 + 0-^rígpd = ^y--
2
v2 = ngpd-2gpd => v = 7(n-2)gpd
nmgpd mv§ nmgpd
Ec0 = ---------- => —— = => gpd
2--------- 2 2 7t

Assim: v = 7(n-2)gpd v = 'l--.v0


71

5) (ITA-92) Um bloco de massa igual a 5,0 kg é puxado para cima por uma força F = 50 N sobre o
plano inclinado da figura, partindo do repouso. Use g = 10 m/s2. O coeficiente de atrito cinético
plano-bloco é p = 0,25.
a) Calcule a energia cinética com que o bloco chega ao topo do plano.
b) Calcule a aceleração do bloco em função do tempo.
c) Escreva a velocidade do bloco em função do tempo.

EC(J) a(m/s2) v(m/s)


a) 20 1,0 0,5 t2.
b) 25 1,2 0,6 t2.
c) 50 2,4 1,2 t. h = 3,0m
d) 25 1,2 1,2 t.
e) 15 1,0 0,4 t.
Solução: Alternativa D
a) As forças que atuam no bloco são: Peso, Normal, Atrito e F. Assim:
WFres = WP + WN + WFat + WF = ECf- Eco => - mgh + 0 - mgpd + Fí = ECf-0 =>
-5.10.3-5.10.0,25.3+50.372 = Ec, => Ec,= 24,63 J
b) Aplicando a 2a Lei de Newton no sistema:

F-Mgsen 45o- Mgpcos 45° = Ma => a = — -g(sen45°+pcos45°) =>


M
a=3jjí-ioí
50 —+ 0,25— a = 1,2 m/s2
5 2 2
c) v = at => v = 1,2 t

6) (EN-09) Um bloco de massa igual a 2,00 kg é solto de uma altura H = 3,00 m em relação a uma
mola de constante elástica igual a 40,0 N/m. Considere a força de atrito cinético entre a superfície em
contato constante e de módulo igual a 5,00 N. Desprezando a força de atrito estático quando em
repouso, isto é, desprezando as perdas de energia nas várias situações de repouso, a distância total
percorrida pelo bloco até parar, em metros, é
■J.iFi'.?,- ?,• -y.

a) 10,0 b)12,0 c) 12,5 d) 12,8 e) 13,0

167
Elementos da Física-Mecânica!-Trabalho ePotência
Solução:
Como a força de atrito deve ser desconsiderada nas situações de repouso, na
posição de equilíbrio final do bloco tem-se:
r- i * * m.g 2.10 A_
Fe = mg => k.Ax = m.g => Ax = = 0,5 m
H

i
Durante todo a trajetória do corpo atuam três forças no corpo:
AX Peso, Força de Atrito e Força Elástica.
Assim: WFres = WP + WFal + WFe
Observe que o corpo inicia e termina sem movimento com velocidade nula.
Deste modo segue que WFres = ECf - Ec0 = 0:
k A-a kA 40.0,52
0 = Wp + WFat + WFe = mg(h + Ax) - Fat.d + --------- = 2.10(3 + 0,5) - 5.d - =>
2 2
0 = 70-5d-5 = 0 => 5d = 65 => d = 13m

7) (ITA-94) Na figura, o objeto de massa m quando lançado horizontalmente do ponto A com


velocidade Va atinge o ponto B após percorrer quaisquer dos três caminhos contidos num plano
vertical (ACEB, ACDEB, ACGFEB). Sendo g a aceleração gravitacional e p o coeficiente de atrito
em qualquer trecho; T1t T2, T3 e Vb1, Vb2, Vb3 os trabalhos realizados pela força de atrito e as
velocidades no ponto B, correspondentes aos caminhos 1, 2, e 3 respectivamente podemos
afirmar que:
D
2
|m | C 1 E
A B
3
g f
a) T-i < T2 < T3 e Vbi > Vb2 -* Vb3 b) Tj < T2 <T3 e Vb1 = Vb2 = Vb3
c) Ti = T2 = T3 e Vbi > Vb2 > Vb3 d) T, > T2 >T3 e Vb1 < Vb2 < Vb3
e)Ti = T2 = T3 eVb1 — Vb2 — Vb3
Solução: Alternativa E
Vamos calcular o trabalho da força de atrito em duas situações, uma horizontal e outra inclinada:

L
I) WFal1 = Fal.L = m.g.p.L
II) WFat2 = Fat.(L7cos 0) = m.g.p.cos 0.(L/cos 0) = m.g.p.L
Como WFat1 = WFat2, para um mesmo comprimento horizontal L, não importa qual reta de
trajetória, o trabalho da força de atrito é o mesmo.
Como os desenhos propostos no enunciado são retas, então todos os trabalhos das forças de
atrito são iguais: Tj = T2 = T3
Como os o início e o final de cada trajetória estão na mesmo horizontal, a variação da energia
cinética é igual ao trabalho da força de atrito. Como os trabalhos são iguais então as energias
cinéticas finais são iguais, implicando que vb1 = vb2 = vb3.

8) (EN-88) Na posição mostrada na figura, a velocidade do bloco de 2 kg é 5 m/s, a mola é ideal,


tem constante elástica 20 N/m e está comprimida de 1 metro. O plano inclinado faz com o plano
horizontal um ângulo 0 cujo seno é 0,6. Sabe-se que a aceleração da gravidade no local é 10 m/s2
e que a energia cinética do bloco será 5 joules quando a mola estiver comprimida de 2 metros. 0
valor absoluto do trabalho realizado sobre o bloco, pela força de atrito entre o mesmo e o plano
inclinado, no deslocamento, da posição inicial (mostrada na figura) até à posição em que a mola
está comprimida de 2 metros, em joule, é:

168
Elementos da Física-Mecânica l-Trabalhoe Potência

a)0 b) 1 c)2 d) 3 e) 4
Solução: Alternativa C
Durante todo o movimento atuam sobre o corpos as forças Peso,
t
Normal, Força Elástica e Força de Atrito. Logo:
WFres = WP + WN + WFe + WFat - ECf - Eco =>
kx2 kx2
-+
mgdsen 0 + 0 *w
+ W,Fat = ECf-2^. =>
2
20.12 20.22 2.25 —P
2.10.1.0,6 + + W>Fat
ot — b-------- “
2 2
12 + 10-40 +WFat = 5-25 => WFat = -2J => |WFat| = 2 J

9) Um bloco de massa m desce o plano inclinado a partir de A, com velocidade inicial nula, como
mostrado na figura a seguir. Vetor g : aceleração da gravidade

B C
Í3
O bloco chega ao ponto B com uma velocidade igual a Podemos afirmar que o coeficiente

de atrito cinético entre o bloco e o plano é igual a:


a) tge b)^® c)^® d)^® e)cotg9
2 3 4
Solução: Alternativa D
Durante a descida do bloco atuam no corpos as forças Peso, Normal e Fat. O trabalho da força
normal é nulo uma vez que o ângulo entre a normal e o vetor deslocamento é sempre 90°.
WFre, = WP + WN + WFat = ECf- Eco => Xgh + 0-XÍgMBC = ^--0 =>

2p 2 tgo
* tge 2 tgo 2

10) (IME-87) Uma partícula de massa igual a 4,0 kg move-se no eixo “x” segundo a equação x =
212 - 3t, onde “x” é medido em metros e “t” em segundos. No tempo t = 3 s a partícula choca-se
contra uma mola de massa desprezível e coeficiente de mola k = 400 N/cm, conforme figura
abaixo. Determine a coordenada máxima, xmax, atingida pela partícula.
x—x k „
m

Solução:

169
Elementos lia Física-Mecânica!-Trabalho e Potência

x(t) = -3t + 2t2 = x0 + vot + — => x0 = 0 m, Vo = - 3 m/s e a = 4 m/s2


Equação horária da velocidade: v(t) = v0 + at = - 3 +4t
No instante do choque com a mola: x(3) = - 3.3 + 2.9 = 9 m e v(3) = - 3 + 4.3 = 9 m/s
Se o corpo, antes do choque, estava com movimento acelerado é porque existia uma força F que
atuava sobre ele no sentido positivo de x. Supondo que esta força continue a atuar no corpo após
o choque com a mola, pode-se aplicar o teorema do trabalho - energia:
myf mv! „ kAx2
FR=P + N + Fe+F => W^=Wp+WN + WFe+WF 0 + 0---------+ m.a.Ax =>
/2 2~ 2
4.81 40000.Ax.2
— + 4.4. Ax => - 162AX = - 20000AX2 + 8Ax =>
2 2 ........ . .................... ................
10000AX2 - 4Ax - 81 = 0 => Ax = 0,09m
Assim, a coordenada máxima da partícula éx = x3 + Ax = 9 + 0,09 = 9,09 m

11) (ITA-02) Um pequeno camundongo de massa M corre num plano vertical no interior de um cilindro
de massa m e eixo horizontal. Suponha-se que o ratinho alcance a posição indicada na figura
imediatamente no início de sua comida, nela permanecendo devido ao movimento giratório de reação
do cilindro, suposto ocorrer sem resistência de qualquer natureza. A energia despendida pelo ratinho
durante um intervalo de tempo T para se manter na mesma posição enquanto corre é:
.cilindro

*1
'camundcngo

2
a)E = ^g2T2 b)E = Mg2T2 c)E = ^-g2T2 d) E = Mg2T2 e) N.D.A.
2m M
Solução: Alternativa A
Suponha que as patas do camundongo façam uma força F sobre a
F superfície interna do cilindro. Como reação a superfície do cilindro faz
sobre o camundongo uma força de mesmo módulo F, mesma direção
□ e sentido contrário. Como o camundongo se mantém na mesma
posição indicada na figura, pode-se afirmar que F = Mg.
Mg .......................
Note que F é a força _ ______ ____sobre
resultante _ o cilindro, ou seja, F = mat.
Deste modo, a velocidade tangencial da superfície do cilindro vai
aumentando continuamente devido à aceleração at. Supondo que v é
a velocidade tangencial adquirida pela superfície do cilindro depois de um tempo T segue que:
r- v ,, MTg
F = mat=m—= Mg => v =—-
T m
Como cada diferencial de massa dm do cilindro possui uma velocidade tangencial v a energia
mv2
cinética do cilindro, neste instante, vale EC( = ——.
Antes do início do movimento do camundongo o cilindro encontrava-se em equilíbrio, implicando
que existe uma força (não indicada na figura) que sustenta o cilindro em seu centro. Logo, a força
F é a resultante no cilindro e assim o trabalho de F é igual à variação da energia cinética do
... , ... Ár_ mv2 m M2T2g2 M2 2t2
C =------ =--------- A = —Jr
cilindro: WF = AEC
2 2 m2 2m a
Esta energia é repassada do camundongo para o cilindro pelo movimento das patas do
camundongo. Assim, a energia despendida pelo camundongo vale — g2T2.
2m

170
________________________________________ Elementos ba Física-Mecânica!-Trabalho e Potência
12) Um tapete de massa 20 kg deve ser arrastado de uma sala para outra como mostra a figura. A
largura do corredor e do tapete é a mesma, 2 m. O coeficiente de atrito na primeira sala é 0,2 e na
segunda sala também é 0,2, enquanto que no corredor o coeficiente de atrito vale 0,1. Que
trabalho deve ser realizado para levar o tapete da primeira para a segunda sala?
corredor

Mi = 0,2 Mc = 0,1 Ml = 0,2

Solução:
x 2 4 6 Como existem diferentes valores do coeficiente de atrito
> nas salas e no corredor a força que deve ser aplicada ao
x (m)
tapete para levá-lo lentamente de uma sala para outra é
x variável, dependendo da área do tapete que está sobre
<—►
uma sala e sobre o corredor.
Considere o eixo unidimensional x indicado na figura.
Neste eixo, seja x a posição da extremidade esquerda do
tapete. Ao longo de todo o processo x varia de 0 a 4 m.
Para a posição do tapete indicada na figura ao lado, onde a
; M1 = O,2 Mc = 0,1 M2 = 0,2 extremidade esquerda do tapete está na posição x (0 < x <
2 m), a força necessária que empurrar o tapete com
velocidade constante vale:
F'= Fati’+ Fat2’ = m^^gp1 + m-gp2 = 20^^10.0,2 + 20-10.0,1 =>

F’ = Fati’ + Fat2’ = 40 - 20x + 10x = 40 - 10x, para 0 < x < 2 m.


No início, para x = 0, quando o tapete está totalmente na 1a sala, a mínima força para movimentá-
lo vale F, = 40 N, enquanto que na situação do tapete exatamente no meio do corredor, para x =
2 m, a força para movimentá-lo lentamente é igual a F2 = 20 N.
0 cálculo da força necessária para movimentar o tapete da posição no meio do corredor até a 2a
sala (2 m < x < 4 m) é análogo ao cálculo já realizado, para 0 < x < 2 m, encontrando uma
expressão para F que varia linearmente com a posição x.
A força final, quando o tapete está totalmente na 2a sala, é igual à força inicial, quando o tapete
estava totalmente na 1a sala: F3 = F, = 40 N. Como a força varia de maneira linear com a posição
x, pode-se traçar o gráfico de F por x ligando por retas as posições inicial (x = 0), mediana (x = 2
m) e a posição final (x = 4 m):
F(N) A área deste gráfico é numericamente igual ao trabalho
40 da força F. Perceba que a figura formada pelo gráfico é
a união de dois trapézios congruentes. Desta forma:
2 (40 + 20) 2 = 12QJ
20
F 2

x (m)
>
2 4

171
____________ __ ___________________________ Elementos áa Física-Mecânica i-Trabalho e Potência
13) Sobre uma tábua que encontra em um plano horizontal liso, existe um bloco de massa m,
unido ao teto através de uma mola ideal, inicialmente relaxada na vertical, de comprimento natural
í0. O coeficiente de atrito entre a tábua e o bloco valeu p. A tábua é lentamente afastada para a
direita até a posição em que o bloco fica na iminência de se movimentar sobre ela, situação em
que o fio forma com a vertical um ângulo a. Determinar o trabalho realizado pela força de atrito
nesse episódio, em joules, no referencial da Terra.

O
§
O

Solução:
Se í é o comprimento final da mola tem-se í.cos 0 = í0.
e"N
F? ^0
Logo: Aí = (. - í0 ^o — ^o —---- 1^]
COS0 COS0 )
Fat
I I . Na direção vertical tem-se:
Fe.cos 0 + N = mg => N = mg- kAí.cos 0
,, mg No momento em que o bloco se encontra na iminência de deslizar
sobre a tábua:
Fe.sen 0 = Fat => k.Aí.sen 0 = N.p => k.Aí.sen 0 = (mg - kAí.cos 0).p =>
k=mgp
k.Aí.sen 0 + kAí.cos 0.p = mgp =>
Aí(sen0 + pcos0)
Como o corpo é lentamente deslocado pode-se assumir que a variação da energia cinética é zero,
ou seja, o trabalho da força resultante é nulo. As únicas forças que realizam trabalho não nulo
sobre m são a força elástica e a força de atrito. Assim:
k(Aí)2 = mgp(Aí)2
WFr = WFat + WFe = 0 => WFat = -WFe
2 2(AÍ)(sen0 + pcos0)
mgpío(1-cos0)
WFat =
2 cos 0(sen 6 + p cos 0)

14) Uma corrente de comprimento L descansa, em parte, sobre uma mesa horizontal com
coeficiente de atrito p, mantendo o máximo comprimento possível para equilíbrio suspenso a partir
da borda. É feita uma pequena perturbação e a corrente começa a deslizar devido a ação da força
de gravidade sobre a parte da corrente que ficou pendurada fora da mesa. Que velocidade terá a
corrente quando seu extremo superior atingir a borda da mesa? Considere g como a aceleração
da gravidade.

Solução:
Seja d o máximo comprimento suspenso da corrente. De modo que haja equilíbrio, na iminência
de movimento, o peso da parte suspensa da corrente deve ser igual à força de atrito sobre a parte
da corrente que está na mesa:

172
Fiementos ha Física - Mecânica i-Trabalho e Potência
d L-d
P = Fal => m—g=m——gp d = Lp-dp => d = -----
1+p
L-x Para um comprimento suspenso x da corrente maior que d a corrente
entra em movimento. Com a corrente em movimento, a força
Fat resultante na corrente vale:
x L—x = IQ9 [Lp + (i-p)x], para d < x < L
fr =p-Fat =m —g-m—j— gp

Note que para x = d -LL-. tem-se FR = 0 (corrente em equilíbrio),


P 1 + |i
enquanto que para x = L tem-se FR = mg (corrente em queda livre).
Estes dois valores concordam com a análise conceituai feita sobre o movimento da corrente.
Como a força resultante varia linearmente com a posição x, pode-se traçar um gráfico da
dependência de FR por x:
Sabe-se que em um gráfico de F por x o trabalho da força
Fr
resultante é igual à área compreendida entre a linha do gráfico
mg e o eixo x.
Como a figura formada pelo gráfico é um triângulo retângulo:
Fr = í L —— mg F ... MgL
Fr
l 1+M 2 R 2(1 + p)
Pelo teorema do trabalho-energia, o trabalho da força resultante
é igual à variação da energia cinética:
Lh L X
MgL = Mv2 v= I gL
1+H /(1 + p) / \1 + M

15) Um quadrado homogêneo repousa sobre um plano horizontal. Deve-se rotacionar o quadrado
em 360° em torno de um de seus vértices, como indica a figura 1. Suponha que o coeficiente de
atrito entre o chão e o quadrado é constante em toda a superfície do quadrado. Realizando o
mesmo trabalho, determine quantas rotações completas podem ser feitas no mesmo quadrado em
tomo de seu centro?

a)1 b)2 c) 4 d) 6 e) 8
Solução:
Seja W o trabalho mínimo realizado por uma força F para girar um quadrado de
massa m e lado L em torno de um de seus vértices. Quatro forças atuam sobre o
quadrado: Peso, Normal, F e Força de Atrito. Como os trabalhos do peso e da
t- normal são nulos, o trabalho da força resultante no quadrado é igual a:
WFr = WF + WFat = AEC
Como o movimento de rotação é realizado bem lentamente, a variação da
energia cinética é zero: WF = -WFal
Deste modo, o trabalho da força F é diretamente proporcional à massa do quadrado, ao
coeficiente de atrito e ao comprimento da trajetória do centro de massa do quadrado.
Para analisar a 2a experiência, divida o quadrado original em 4 quadrados menores (cada um de
massa m/4 e lado L/2), ligando os pontos médios dos lados opostos. Note que girar o quadrado
original pelo centro equivale a girar, em torno do vértice central, cada um dos quatro quadrados
menores. Como os trabalhos devem ser iguais:

W, = W2
_ LV2
Lx/2
m.g.p.27t
.fm LV2
= 4 — .g.p.n —— n =2
2

173
Fiementos da Física-Mecânica!-Trabalho e Potência
POTÊNCIA

Potência Média

Como já foi detalhado nos itens anteriores deste capítulo, para se calcular o trabalho de
uma força não é necessário conhecer o tempo decorrido na realização desse trabalho. Na vida
prática, porém, o conhecimento desse tempo pode ser importante pois, de maneira geral, existe o
interesse em que um determinado trabalho seja realizado no menor tempo possível. Entre duas
máquinas que realizem o mesmo trabalho, com a mesma perfeição, a preferência é sempre pela
mais rápida. Para se medir a taxa temporal na qual se realiza um certo trabalho, define-se uma
grandeza denominada potência. Se uma força realiza um trabalho W durante um intervalo de
tempo At, a potência média Pot, dessa força é definida como sendo:

W
P°tmédia
At

Pode-se observar, pela definição dada, que quanto menor for o tempo empregado por uma
W
máquina para realizar um certo trabalho, maior será a sua potência. A relação Pot = — nos
mostra que a unidade de potência no S.l. será 1 J/s. Esta unidade é denominada de 1 Watt, em
homenagem a James Watt, inventor da máquina a vapor. Assim, a potência de 1 Watt
corresponde ao trabalho de 1 J realizado em 1 s, isto é, 1 J/s = 1 Watt = 1 W.

Outras duas unidades utilizadas para potência são Cavalo-Vapor (CV) e Horse Power
(PH), principalmente no meio automobilístico. Um cavalo-vapor representa o equivalente a 75
kgms ~1, em que 1 kg.m corresponde ao trabalho gasto para erguer uma massa de 1 kg a uma
altura de 1 metro. Um Horse Power se define como a potência necessária para elevar
verticalmente a uma velocidade de 1 pé/min uma massa de 33.000 libras. Convertendo CV e HP
para Watts, tem-se que:

1 CV = 735,5 W
1 HP = 745,7 W

Assim, 1CV = 0,9863 HP, e 1 HP = 1,0139 CV.

Outra unidade relacionada com potência é Watt-hora (Wh), que na verdade é uma unidade
de trabalho ou energia. O Watt-hora (Wh) é a medida de energia usualmente utilizada em eletro
técnica. Um Watt-hora é a quantidade de energia utilizada para alimentar uma carga com potência
de um Watt pelo período de uma hora. O valor de 1 Wh é equivalente a:

1 Wh = 3,6x103 J = 3,6 kJ = 3600 J.

Potência Instantânea

A potência instantânea é o limite para onde tende a potência média quando o intervalo de
tempo considerado At tende a zero. Neste caso, a expressão da potência instantânea fica em
função da derivada do trabalho pelo tempo:

m . dW
Potinst--^-

Substituindo o valor dW = F.dr:

174
Mementos da física-Mecânica !-Trabalhos Potência
n * dW F.dr e dr
Po,--dT potinst F.v Potinst = F.v. cos 0
dt dt

Quando uma força constante atua sobre um corpo que se move com velocidade constante
tem-se que a potência instantânea apresenta o mesmo valor em qualquer instante do movimento.
Neste caso as potências média e instantâneas são iguais:

Potmédia = Potinst = F.v.cosO, quando F e v são constantes

Gráfico da Potência Instantânea

dW
Da definição da potência instantânea Potjnst — segue que:

W = J Potj,instai
to

Sabe-se que toda expressão física definida por uma integral possui uma área associada.
Neste caso, fazendo o gráfico da dependência da potência instantânea pelo tempo a área
compreendida entre o gráfico e o eixo do tempo é numericamente igual ao trabalho realizado no
intervalo de tempo considerado.
'‘Pot

Area = W
>
to tr t

Rendimento

Não existe máquina ideal, ou seja, aquela cujo todo o trabalho produzido pela máquina
seja completamente aproveitado para realizar alguma atividade mecânica. Para as máquinas reais
o trabalho não aproveitado (também conhecido como trabalho dissipado) deve ser incorporado
como parcela do trabalho total; a outra parcela será trabalho útil. Para tais máquinas tem-se,
portanto:

W,' total — Wútil + WdissipadQ

Nessas condições, define-se como rendimento da máquina a razão entre o trabalho útil e o
trabalho total:

WÚBI
n=
Wtotal

Como na realidade Wúti| < Wtolai e ambos maiores que 0, o rendimento sempre será uma
fração da unidade. Para aumentar o rendimento das máquinas é necessário diminuir os atritos, o
que se consegue, por exemplo, por meio de lubrificantes ou rolamentos de esferas de aço.

175
ílementos da física-Mecânica!-Trabalhos Potência
Exemplos:

1) Um corpo de massa m é acelerado uniformemente, partindo do repouso até atingir a velocidade v0,
no tempo to. Obtenha a potência instantânea fornecida ao corpo em um instante t qualquer contado
desde o inicio do movimento.
Solução:
Se o corpo, partindo do repouso, atinge uma velocidade v0 em um tempo to sua aceleração vale:
Av = vo-° = Vo
a
At to-O to
A potência instantânea é dada por:

P°tinst = F.v = m.a.a.t = m.a2.t =


to

2) (ITA-85) Uma queda d’água escoa 120 m3 de água por minuto e tem 10,0 m de altura. A massa
específica da água é de 1,00 g/cm3 e a aceleração da gravidade é de 9,81 m/s2. A potência
mecânica da queda d’água é:
A) 2,00 W B) 235 x 10 5W C) 196kW D) 3,13x 103W E) 1,96x 102W
Solução:
A potência da queda d’água pode ser escrita como a razão entre a energia que pode ser retirada
de uma coluna de água de altura h pelo tempo levado para a água percorrer esta altura.
Pot = = p V g h = — p.g.h = — (103 )(9,81 )(10,0) = 196 kW
At At At 60

3) (ITA-94) Um navio navegando à velocidade constante de 10,8 km/h consumiu 2,16 toneladas
de carvão em um dia. Sendo q = 0,10 o rendimento do motor e q = 3,00.107 J/kg o poder calorífico
de combustão do carvão, a força de resistência oferecida pela água e pelo ar ao movimento do
navio foi de:
a) 2,5.10 4 N b)2,3.10sN c) 5,0 . 10 4 N d) 2,2. 10 2 N e) 7,5 .10 4 N
Solução: Alternativa A
Inicialmente note que a unidade da grandeza poder calorífico é J/kg, ou seja, sabendo a massa m
do carvão pode-se calcular a energia gerada pela queima desta massa de carvão. Dividindo esta
energia pelo tempo tem-se a potência total gerada pela queima do carvão. Multiplicando esta
potência total pelo rendimento tem-se a potência útil. Como a velocidade do navio é constante a
potência útil também pode ser calculada pelo produto da força executada para hélice do navio
para movimentá-lo pelo velocidade.
Potúti, = 7].Pottola) => F.v = n(M/At).q => F(3,00) = (0,10)[(2,16.103)/(24.3600)](3,00.107) =>
F = 2,5.104N
Como a velocidade é constante a força total de resistência é igual, em módulo, à força executada
pela hélice do navio: FreSistència - 2,5.104 N

4) (ITA-98) Um bloco maciço requer uma potência P para ser empurrado, com uma velocidade
constante, para subir uma rampa inclinada de um ângulo 0 em relação à horizontal. O mesmo
bloco requer uma potência Q quando empurrado com a mesma velocidade em uma região plana
de mesmo coeficiente de atrito. Supondo que a única fonte de dissipação seja o atrito entre o
bloco e a superfície, conclui-se que o coeficiente de atrito entre o bloco e a superfície é:
. Q , . Q . QsenO Q . QsenO
a) — b)------ c)---------- d)------------- e)-------------
P ' P-Q P-Q P-QcosO P-QcosG

176
fiementosba física-Mecânica !-Trabalho e Potência
Solução: Alternativa E

N Suponha que a velocidade com que o corpo sobe o plano inclinado é v. A força
mínima a ser aplicada ao bloco é igual a:
F = P.sen 0 + Fat = m.g.sen 0 + m.g.p.cos 0
F,
Fat Deste modo, a potência desenvolvida por F vale P = (m.g.sen 0 + m.g.p.cos 0)v
Na superfície horizontal a mínima força F’ para deslocar o corpo com velocidade
constante vale F’ = Fat = mgp
Portanto, a potência desenvolvida por F’ vale Q = F’.v => Q = m.g.p.v
p sen0 + pcos0 _ _ . „ Qsen0
Assim. —■ =---------- ---------- => pP = Q.sen 0 + Q.p.cos 0 => u. =----------------
Q p P -Qcos9

5) (ITA-00) Deixa-se cair continuamente areia de um reservatório a uma taxa de 3,0 kg/s
diretamente sobre uma esteira que se move na direção horizontal com velocidade V. Considere
que a camada de areia depositada sobre a esteira se locomove com a mesma velocidade V,
devido ao atrito. Desprezando a existência de quaisquer outros atritos. Conclui-se que a potência
em watts, requerida para manter a esteira movendo-se a 4,0 m/s, é:
Reservatório
de areia

Esteira ;j: V
\ h*:::::::::::::
O O O" CJ O U
a) 0 b) 3 c) 12 d) 24 e) 48
Solução: Alternativa E
Assim que um grão de areia colide com a esteira sua velocidade horizontal é zero. O contato com
a esteira faz o grão acelerar da velocidade nula para a velocidade v da esteira. A força F que os
roletes devem fazer sobre a esteira de modo a acelerar os grãos de areia vale:

F = m.a => F = m—= —(v-0) = —v = 3.4 = 12N


At At At
Como a velocidade é constante: Pot = F.v = (12)(4) = 48 W

6) (ITA-07) Projetado para subir com velocidade média constante a uma altura de 32m em 40s,
um elevador consome a potência de 8,5 kW de seu motor. Considere seja de 370kg a massa do
elevador vazio e a aceleração da gravidade g = 10 m/s2. Nessas condições, o número máximo de
passageiros, de 70kg cada um, a ser transportado pelo elevador é
a) 7 b) 8 c) 9 d) 10 e)11
Solução: Alternativa C
Como a velocidade é constante a força que o cabo deve exercer no elevador deve possuir o
mesmo módulo do peso total do elevador (elevador + peso dos passageiros): T = (M + nm)g
W
Deste modo: Pot = — => Pot.At = T.h => Pot.At = (M + nm)gh =>

8,5.103.40 = (370 + n.70).10.32 => n = 9,89


Logo, o número máximo de passageiros é n = 9

7) (ITA-09) Em 1998, a hidrelétrica de Itaipu forneceu aproximadamente 87600 GWh de energia


elétrica. Imagine então um painel fotovoltaico gigante que possa converter em energia elétrica,
com rendimento de 20%, a energia solar incidente na superfície da Terra, aqui considerada com
valor médio diurno (24h) aproximado de 170 W/m2‘ Calcule:
a) a área horizontal (em km2) ocupada pelos coletores solares para que o painel possa gerar,
durante um ano, energia equivalente àquela de Itaipu, e,
b) o percentual médio com que a usina operou em 1998 em relação à sua potência instalada de
14000 MW.

177
Fiementos da Física-Mecânica i-Trabalho e Potência
Solução:
Convertendo de Wh para J:
E = 87600 GWh = 8,76.104.109.3,6.103 J = 31,536.1018J
31,536.10i16
a) l = => 0,20.170 = — => A = 2,94.10® m2 = 294 km2
A.365.24.3600
E
b) Pot= => 14000.106 = => E = 44.150.1016 J
At 31,536.10®
31.536.1016
1= = 0,7143 q = 71,43 % de sua capacidade
44,1504.1016

8) (ITA-12) Um corpo movimenta-se numa superfície horizontal sem atrito, a partir do repouso,
devido à ação contínua de um dispositivo que lhe fornece uma potência mecânica constante.
Sendo v sua velocidade após certo tempo t, pode-se afirmar que
a) a aceleração do corpo é constante.
b) a distância percorrida é proporcional a v2.
c) o quadrado da velocidade é proporcional a t.
d) a força que atua sobre o corpo é proporcional a Vt.
e) a taxa de variação temporal de energia cinética não é constante.
1a Solução: Alternativa C
Como a potência P é constante o gráfico da potência pelo tempo é dado abaixo:
■ ■ Pot Como a área do gráfico da potência pelo tempo é numericamente
igual ao trabalho: WF = Pot.t
P Supondo que F é a força resultante no corpo (informação que nâo
consta no enunciado porém necessária para resolver a questão)
segue que o trabalho de F é igual à variação da energia cinética:
Mv2 2 Px
*t P.t ------ => v =—t
2 M
Como P e M são constantes segue que v2 oc t

2a Solução: Alternativa C

A potência instantânea no instante t é dada por Pot = F.v


Como esta potência é constante:

dv . ^dt
F.v = k => m — .v = k v.dv jv.dv = —jdt -(t-0) =>
dt m 2 m
2 2k V2 oc t
v =—t
m

178
Mementos da física-Mecânicai-Trabalho e Potência
Exercícios de Embasamento i j P (watts)

80-------:----- 1----- í----- i------ i----- 4-—


E1) (Unesp-09) Suponha que os tratores 1 e 2 da
figura arrastem toras de mesma massa pelas 60
rampas correspondentes, elevando-as à mesma
40
altura h. Sabe-se que ambos se movimentam
com velocidades constantes e que o 20
comprimento da rampa 2 é o dobro do
comprimento da rampa 1. Chamando de W, e W2 o V
1,0 2,0 3,0 4,0
-L
5,0 6,0 t(s)
os trabalhos realizados pela força gravitacional
sobre essas toras, pode-se afirmar que:
■<- '2
E5) (Fuvest-11) Usando um sistema formado por
uma corda e uma roldana, um homem levanta
h uma caixa de massa m, aplicando na corda uma
força F que forma um ângulo 0 com a direção
vertical, como mostra a figura. O trabalho
A) W, = 2W2; W, >0eW2<0.
realizado pela resultante das forças que atuam
B) Wi=2W2; W,<0eW2>O.
na caixa — peso e força da corda —, quando o
C) W1=W2; W1<0eW2<0.
centro de massa da caixa é elevado, com
D) 2W, = W2; W,>0eW2>0.
velocidade constante v, desde a altura ya até a
E) 2W, = W2; W,<0eW2<O.
altura yb, é:
E2) (UFPB-13) Em um trecho do trajeto João y
Pessoa - São Paulo, um avião desloca-se
horizontalmente e com velocidade constante.
Admita que as únicas forças que atuam sobre o Yb- —
avião são: a força propulsora gerada pelas
turbinas, a força de resistência do ar (atrito) e a ya—-
força peso. Com relação aos trabalhos realizados
por essas forças, considere as seguintes X
identificações: a) nulo. b) F(yb - ya).
• Tt: trabalho realizado pela força propulsora das
c) mg(yb - ya). d) F.cos (0)(yb - ya).
turbinas.
e) mg(yb - ya) + mvfe
• Tr: trabalho realizado pela força de resistência
do ar.
• TP: trabalho realizado pela força peso do avião. E6) (FEI-96) Um corpo de massa 10 kg é puxado
Com base no exposto, é correto afirmar: por uma mola de constante elástica K = 100 N/m.
3)Tr = Tt, Tp = 0 b) Tr = —Tt, Tp = 0 O comprimento natural é U = 2 m. Qual é o
c)Tr = Tt, Tp > 0 d) Tr= —Tt, Tp > 0 trabalho realizado pela força elástica para
e) Tr = 0, Tp= -Tt deslocar o corpo da posição x = 10 m para a
posição x = 4 m?
E3) (Unicamp-15) Qual o trabalho executado pela
força de atrito entre o pneu e o solo para parar m
um carro de massa m = 1000 kg, inicialmente a v
= 72 km/h, sabendo que os pneus travam no «I o=2m»
instante da frenagem, deixando de girar, e o »----- 4 m-------- >
carro desliza durante todo o tempo de frenagem? *------------ 10 m
a)3,6x104J. b)2,0x105J.
c)4,0x105J. d)2,6x106J. a) 6000 J c) 3000 J
b) 250 J
d) 500 J e) 125 J
E4) (UFPE-99) O desempenho de um sistema
mecânico pode ser representado pelo gráfico E7) (UESPI-08) No instante t = 0 uma partícula
abaixo, que mostra a potência fornecida pelo de massa 2 kg que se move ao longo do eixo x
mesmo em uma certa operação. Calcule o encontra-se na origem e tem velocidade v0 = -10
trabalho total, em joules, efetuado por esse m/s (o sinal negativo denota que o vetor
sistema nos três primeiros segundos. velocidade, nesse instante, aponta no sentido
negativo do eixo x). O gráfico a seguir ilustra a
força resultante na direção x atuando sobre essa

179
Elementos da Física-Mecânica!-Trabalhoe Potência
partícula em função da sua posição. Quando a plana. Considere que as rodas do vagão estão
partícula atingir a posição x = - 8 m, a sua bem lubrificadas a ponto de poder-se desprezar o
energia cinética, em joules, será: atrito das rodas com os trilhos. Durante esse
último translado, o motor acoplado ao cabo de
Fr (N)a
aço executa um trabalho de 4.000 J. Nesse
-8 0 contexto, considerando que o vagão, no último
--- ► translado, partiu do repouso, é correto afirmar
x (m) que esse vagão chega ao final da região plana
com uma velocidade de:
a) 10 m/s b) 8 m/s c) 6 m/s
•-20 d) 4 m/s e) 2 m/s

A) 20 B) 80 C) 100 D)180 E)200 E12) (UFRGS-10) O resgate de trabalhadores


presos em uma mina subterrânea no norte do
E8) (UESPI-10) Um goleiro arremessa uma bola Chile foi realizado através de uma cápsula
de futebol de 400 g com uma velocidade inicial introduzida numa perfuração do solo até o local
de 20 m/s, a partir de uma altura de 1 m acima do em que se encontravam os mineiros, a uma
campo de futebol. A bola quica várias vezes no profundidade da ordem de 600 m. Um motor com
gramado até que para sobre este. Considerando potência total aproximadamente igual a 200,0 kW
a bola como uma partícula material e a puxava a cápsula de 250 kg contendo um mineiro
aceleração da gravidade de módulo 10 m/s2, qual de cada vez.
a energia dissipada da bola desde o lançamento
até o repouso final?
A) 84 J B) 88 J C)92 J
D) 96 J E) 98 J

E9) (UFPB-10) Um foguete de 1 tonelada de


massa viaja com uma velocidade de 360 km/h
em uma região do espaço onde as forças da
gravidade são desprezíveis. Em um determinado
momento, seus motores são acionados e, após a
queima de 200 kg de combustível, sua Fonte: <http://www.nytimes.com/interactive/2010/
velocidade passa a ser de 720 km/h. Com base 10/12/world/20101013-chile.html?ref=americas >.
no que foi exposto, é correto afirmar que o Considere que para o resgate de um mineiro de
trabalho realizado sobre o foguete pelo motor, 70 kg de massa a cápsula gastou 10 minutos
durante a queima do combustível, corresponde a: para completar o percurso e suponha que a
a) 4,7 x 10'J d)1,4x107J aceleração da gravidade local é 9,8 m/s2.
b) 1,1x107J e)1,9x107J Não se computando a potência necessária para
c) 1,5x107J compensar as perdas por atrito, a potência
efetivamente fornecida pelo motor para içar a
E10) (UFPB-11) Um corredor de 80 kg de massa cápsula foi de
gasta 2 s para percorrer os primeiros 10 m de a) 686 W. b) 2.450 W. c) 3.136 W.
uma corrida. Admitindo que, ao chegar aos 10 m, d) 18.816 W. e) 41.160 W.
a sua velocidade era de 10 m/s, conclui-se que a
potência média do corredor, nesse trecho da E13) (UFRGS-14) Um plano inclinado com 5 m
corrida, foi de: de comprimento é usado como rampa para
a) 100W d) 1.000 W arrastar uma caixa de 120 kg para dentro de um
b) 200 W e) 2.000 W caminhão, a uma altura de 1,5 m, como
c) 500 W representa a figura abaixo.

E11) (UFPB-12) Em uma mina de carvão, o


minério é transportado para fora da mina por
meio de um vagão gôndola. A massa do vagão
mais a carga de carvão totalizam duas toneladas.
A última etapa do translado do vagão ocorre em
uma região completamente plana e horizontal. Considerando que a força de atrito cinético entre
Um cabo de aço, com uma das extremidades a caixa e a rampa seja de 564 N, o trabalho
acoplada ao vagão e a outra a um motor, puxa o mínimo necessário para arrastar a caixa para
vagão do interior da mina até o final dessa região dentro do caminhão é
(A) 846 J. (B)1056J. (C)1764J.
180
Elementos da Física-Mecânica 1-Trabalhos Potência
(D) 2820 J. (E) 4584 J. 0,20 m/s, devido à ação conjunta da gravidade,
do empuxo e da resistência viscosa do líquido ao
E14) (UFPE-00) Uma usina hidroelétrica de movimento. Podemos afirmar que a quantidade
90 MW produz energia elétrica por meio de uma de energia transformada em calor, a cada
turbina acionada pela água que cai de uma segundo, no sistema “objeto-líquido” é de:
cachoeira cuja altura é 100 m. Supondo que não
há perdas, calcule o volume de água, em m3, que
passa pela turbina em cada segundo.

E15) (UFPE-12) Um projétil com massa m = 0,10


kg é lançado com velocidade inicial de módulo v,
= 20 m/s, como mostra a figura. Quando ele
alcança a altura máxima de h = 8,0 m, a sua
velocidade tem módulo v2 = 10 m/s. Calcule o a) O b)0,14J c)0,16J d)14J e)16J
módulo do trabalho, em Joules, realizado pela
força de atrito entre o projétil e o ar, desde o E20) (Unesp-09) Segundo informação da
momento do lançamento até quando ele alcança empresa fabricante, um trator florestal (Trator
a altura máxima. Florestal de Rodas 545C) é capaz de arrastar
toras por meio do seu cabo exercendo sobre elas
v< uma força de módulo 2,0.105 N, com velocidade
constante de módulo 2,0 m/s. Desprezando a
massa do cabo e supondo que a força por ele
exercida seja horizontal e paralela ao solo,
E16) (UFPE-13) Um objeto com massa igual a
determine a potência útil desenvolvida pelo trator.
1,0 kg é lançado para cima na direção vertical
com velocidade inicial v0 = 10 m/s. Quando ele
E21) (Fuvest-99) Um veículo para competição de
retoma ao ponto de partida, a sua velocidade tem
aceleração (drag racing) tem massa M = 1100
módulo v = 8,0 m/s. Calcule o módulo do trabalho kg, motor de potência máxima P = 2,64 x 106 W
realizado pela força de resistência do ar, em
(3.500 cavalos) e possui um aerofólio que lhe
joules, ao longo de todo o trajeto do objeto.
imprime uma força aerodinâmica vertical para
baixo, Fa, desprezível em baixas velocidades.
E17) (Fuvest-89) Um objeto de 20 kg desloca-se
Tanto em altas quanto em baixas velocidades, a
numa trajetória plana retilínea de acordo com a
força vertical que o veículo aplica à pista
equação: s = 10 + 3t + t2, onde s é medido em
horizontal está praticamente concentrada nas
metros e t em segundos.
rodas motoras traseiras, de 0,40 m de raio. Os
a) Qual a expressão da velocidade do objeto no
coeficientes de atrito estático e dinâmico, entre os
instante t?
pneus e a pista, são iguais e valem p = 0,50.
b) Calcule o trabalho realizado pela força
Determine:
resultante que atua sobra o corpo durante um
deslocamento de 20 m.

E18) (Fuvest-98) Uma esteira rolante transporta


15 caixas de bebida por minuto, de um depósito
no subsolo até o andar térreo. A esteira tem
comprimento de 12 m, inclinação de 30° com a a) A máxima aceleração do veiculo quando sua
horizontal e move-se com velocidade constante. velocidade é de 120 m/s, (432 km/h), supondo
As caixas a serem transportadas já são que não haja escorregamento entre as rodas
colocadas com a velocidade da esteira. Se cada traseiras e a pista. Despreze a força horizontal de
caixa pesa 200 N, o motor que aciona esse resistência do ar.
mecanismo deve fornecer a potência de: b) O mínimo valor da força vertical Fa, aplicada
a)20W b)40W c)300W ao veículo pelo aerofólio, nas condições da
d)600W e) 1800W questão anterior.
c) A potência desenvolvida pelo motor no
E19) (Fuvest-99) Um objeto de massa 8,0 kg e momento da largada, quando: a velocidade
volume 1,0 litro está imerso em um líquido, de angular das rodas traseiras é m = 600 rad/s, a
densidade igual é da água, contido num grande velocidade do veículo é desprezível e as rodas
recipiente, como mostra a figura. O objeto se estão escorregando (derrapando) sobre a pista.
move para baixo com velocidade constante v =

181
Mementos da Física-Mecânica !-Trabalho e Potência
E22) (Unicamp-92) Uma hidrelétrica gera 5,0.109 a) 1200 W b) 2600 W c) 3000 W
W de potência elétrica utilizando-se de uma d)4000 W e) 6000 W
queda d’água de 100 m. Suponha que o gerador
aproveita 100% da energia da queda d’água e E26) (Unifesp-14) Em uma bancada horizontal da
que a represa coleta 20% de toda a chuva que linha de produção de uma indústria, um
cai em uma região de 400.000 km2. Considere amortecedor fixo na bancada tem a função de
que 1 ano tem 32.106 segundos, g = 10 m/s2. reduzir a zero a velocidade de uma caixa, para
a) Qual a vazão de água (m3/s) necessária para que um trabalhador possa pegá-la. Esse
fornecer os 5,0.10® W? amortecedor contém uma mola horizontal de
b) Quantos mm de chuva devem cair por ano constante elástica K = 180 N/m e um pino
nesta região para manter a hidrelétrica operando acoplado a ela, tendo esse conjunto massa
nos 5,0.10® W? desprezível. A caixa tem massa m = 3 kg e
escorrega em linha reta sobre a bancada,
E23) (Unicamp-93) Um carro recentemente quando toca o pino do amortecedor com
lançado pelo indústria brasileira tem velocidade Vo.
aproximadamente 1.500 kg e pode acelerar, do ,20cm;

repouso até uma velocidade de 108 km/h, em 10


segundos K
r m v=o
(fonte: Revista Quatro Rodas, agosto/92).
Adote 1 CV = 750 W. Sabendo que o coeficiente de atrito entre as
a) Qual o trabalho realizado nesta aceleração? superfícies da caixa e da bancada é 0,4, que a
b) Qual a potência do carro em CV? compressão máxima sofrida pela mola quando a
caixa para é de 20 cm e adotando g = 10 m/s2,
E24) (Unicamp-99) Um carregador em um calcule:
depósito empurra uma caixa de 20 kg, que a) o trabalho, em joules, realizado pela força de
inicialmente estava em repouso. Para colocar a atrito que atua sobre a caixa desde o instante em
caixa em movimento, é necessária uma força que ela toca o amortecedor até o instante em que
horizontal de 30 N. Uma vez iniciado o ela para.
deslizamento, são necessários 20 N para manter b) o módulo da velocidade Vo da caixa, em m/s,
a caixa movendo-se com velocidade constante. no instante em que ela toca o amortecedor.
a) Determine os coeficientes de atrito estático e
cinético entre a caixa e o solo. E27) (Unesp-07) O teste Margaria de corrida em
b) Determine o trabalho realizado pelo escada é um meio rápido de medida de potência
carregador ao arrastar a caixa por 5 m. anaeróbica de uma pessoa. Consiste em fazê-la
c) Qual seria o trabalho realizado pelo carregador subir uma escada de dois em dois degraus, cada
se a força horizontal aplicada inicialmente fosse um com 18cm de altura, partindo com velocidade
de 20 N? Justifique sua resposta. máxima e constante de uma distância de alguns
metros da escada. Quando pisa no 8o degrau, a
E25) (Fuvest-07) Em um terminal de cargas, uma pessoa aciona um cronômetro, que se desliga
esteira rolante é utilizada para transportar caixas quando pisa no 12° degrau. Se o intervalo de
iguais, de massa M = 80 kg, com centros tempo registrado para uma pessoa de 70kg foi de
igualmente espaçados de 1 m. Quando a 2,8s e considerando a aceleração da gravidade
velocidade da esteira é 1,5 m/s, a potência dos igual a 10m/s2, a potência média avaliada por
motores para mantê-la em movimento é Po. Em este método foi de
um trecho de seu percurso, é necessário planejar A) 180W. B) 220W. C) 432W.
uma inclinação para que a esteira eleve a carga a D) 500W. E) 644W.
uma altura de 5 m, como indicado. Para
acrescentar essa rampa e manter a velocidade E28) (Unesp-08) Um carrinho move-se para a
da esteira, os motores devem passar a fornecer esquerda com velocidade v0, quando passa a ser
uma potência adicional aproximada de empurrado para a direita por um jato d'água que
im,
produz uma força proporcional ao módulo de sua
Xj ixi fei V - 1.5 m/s velocidade, F„ = C.v.

IH = !

182
fiementostia física-Mecânicai-Trabalho e Potência
«A Com relação às forças que agem no bloco,
podemos afirmar que
a) a força F realiza um trabalho negativo.
__ i b) a força peso realiza um trabalho positivo.
c) a força normal não realiza trabalho.
........ Q d) a força de atrito não realiza trabalho.
e) a força resultante não realiza trabalho.
Tomando C = 200 N.s/m e v0 = 20m/s, calcule o
E32) (EsPCEx-10) Um bloco, puxado por meio
trabalho da força F necessária a ser produzida
de uma corda inextensível e de massa
pelo motor do carrinho, a fim de manter sua
desprezível, desliza sobre uma superfície
velocidade constante durante 10 s.
horizontal com atrito, descrevendo um
movimento retilíneo e uniforme. A corda faz um
E29) (Unicamp-15) Por sua baixa eficiência ângulo de 53° com a horizontal e a tração que ela
energética, as lâmpadas incandescentes transmite ao bloco é de 80 N. Se o bloco sofrer
deixarão de ser comercializadas para uso um deslocamento de 20 m ao longo da
doméstico comum no Brasil. Nessas lâmpadas, superfície, o trabalho realizado pela tração no
apenas 5% da energia elétrica consumida é bloco será de:
convertida em luz visível, sendo o restante (Dados: sen 53° = 0,8 e cos 53° = 0,6)
transformado em calor. Considerando uma a) 480 J b) 640 J c) 960 J
lâmpada incandescente que consome 60 W de d)1280J e) 1600 J
potência elétrica, qual a energia perdida em
forma de calor em uma hora de operação? E33) (EsPCEx-11) Uma força F constante de
a) 10 800 J. b) 34 200 J. intensidade 25 N atua sobre um bloco e faz com
c) 205 200 J. d) 216 000 J. que ele sofra um deslocamento horizontal. A
direção da força forma um ângulo de 60° com a
E30) (AFA-10) A figura abaixo representa três direção do deslocamento. Desprezando todos os
formas distintas para um bloco entrar em atritos, a força faz o bloco percorrer uma
movimento. distância de 20 m em 5 s. A potência
f2 desenvolvida pela força é de:
Dados: sen 60° = 0,87 e cos 60° = 0,50
wwwww \\\\\w\\\ WWWWW F /
(1) (2) (3)
Sabe-se que as forças F,, F2 e F3 são 60°
constantes e de mesma intensidade. Bloco
Desprezando-se qualquer resistência, pode-se
afirmar que, depois de percorrida uma mesma a)87W b)50W c) 37 W d)13W e)10W
distância, a energia cinética, E-,, E2 e E3,
adquirida em cada situação, é tal que E34) (ITA-76) Um partícula é deslocada de um
a) E, = E2 = E3 c) E, < E2 < E3 ponto A até outro ponto B, sob a ação de várias
b) E, > E2 = E3 d) E, = E2 > E3 forças. O trabalho realizado pela força resultante,
F, nesse deslocamento, é igual à variação da
E31) (EsPCEx-08) Um bloco B sobe a rampa de energia cinética da partícula :
um plano inclinado, descrevendo um movimento
a) somente se F for constante
retilíneo uniformemente acelerado. Sobre ele,
b) somente se F for conservativa
age uma força F constante, conforme a figura
c) seja F conservativa ou não
abaixo. Há força de atrito entre as superfícies do
d) somente se a trajetória for retilínea
bloco e da rampa.
e) em nenhum caso.
F
E35) (ITA-87) Um motor a explosão tem potência
B
de 50 kW e recebe, por hora, através da
combustão da gasolina, 2,1.10® kJ. Seu
rendimento e a potência dissipada por ele são
rampa
respectivamente.

Figura Ilustrativa

183
Mementos da Física-Mecânica 1-TrabalhoePotêncla
Exercícios de Fixação
m
F1) (UFTM-13) O funcionário de um armazém, I Fl = mg
responsável pela reposição de produtos,
empurra, a partir do repouso e em movimento
retilíneo, um carrinho com massa total de 350 kg
sobre uma superfície plana e horizontal.
9
Y a) Supondo que não haja atrito entre o bloco e o
plano inclinado, calcule o módulo da aceleração
do bloco.
b) Calcule a razão entre o trabalho WF da força F
e o trabalho WP do peso do bloco, ambos em um
—►
© deslocamento no qual o bloco percorre uma
distância d ao longo da rampa.
Em um determinado trecho de 8 m de
comprimento, ele dá três empurrões F3) (UESPI-12) As figuras A e B a seguir
consecutivos no carrinho, exercendo uma força mostram dois instantes do movimento
horizontal para a direita, cuja intensidade é descendente de um bloco de massa 1 kg sobre
representada no gráfico 1, em função da posição um plano inclinado de 0 = 37° com a horizontal. A
do carrinho. Nesse mesmo trecho, atua sobre o mola indicada é ideal, com constante elástica de
carrinho uma força de atrito de intensidade 200 N/m. Na figura A, o bloco tem velocidade de
constante, representada no gráfico 2.
4 m/s, e a mola está comprimida de 5 cm. Na
F(N)“ figura B, o bloco tem velocidade de 2 m/s, e a
mola está comprimida de 15 cm. Existe atrito
gráfico 1 entre o bloco e o plano inclinado. Considerando
500 sen(37°) = 0,6 e cos(37°) = 0,8 e a aceleração da
gravidade 10 m/s2, qual é a energia dissipada
pelo atrito entre os instantes mostrados nas
figuras A e B?
o 2 4 6 I —►
x(m)
Figura A
Fa,(N)"
gráfico 2
g
2 4 6 8
o—
x(m)
-100 —
Calcule:
a) a intensidade máxima da força resultante que
atuou no carrinho nos primeiros 2 m de Figura B g
deslocamento.
b) a velocidade atingida pelo carrinho ao final dos
8 m. íf)
F2) (UFRJ-06) Um plano está inclinado, em
_____________
A)1,3J B)2,1 J C) 3,8 J D) 4,6 J E)5,2J
relação à horizontal, de um ângulo 0 cujo seno é
igual a 0,6 (o ângulo é menor do que 45°). Um
F4) (Unicamp-87) Um bloco de massa m = 0,5
bloco de massa m sobe nesse plano inclinado
kg desloca-se sobre um plano horizontal com
sob a ação de uma força horizontal F, de módulo
atrito (p = 0,4) e comprime uma mola de
exatamente igual ao módulo de seu peso, como
constante elástica k = 1,6 x 102 N/m. Sabendo-se
indica a figura a seguir.
que a máxima compressão da mola pela ação do
bloco é x = 0,1 m, calcule:
a) o trabalho da força de atrito durante a
compressão da mola.
b) A velocidade do bloco no instante em que
tocou a mola

184
Flementos da Física-Mecânica !-Trabalho e Potência

J
água
[pEK__ J
gerador resistor
*1
i
k
m
7777777777777777, m

F5) (Uerj-05) Um produto vendido no


O gerador converte a energia mecânica do corpo
supermercado é recebido em caixas de papelão
em elétrica e alimenta um resistor imerso em um
contendo 16 embalagens de volume igual a
recipiente com água. Suponha que, até que o
1312,5 cm3 cada. As massas de cada
corpo chegue ao solo, depois de abandonado a
embalagem, do seu conteúdo e da caixa de
partir do repouso, sejam transferidos para a água
papelão são, respectivamente, 10 g, 1.000 g e
24 J de energia térmica. Sabendo que esse valor
100 g. O produto é entregue por um caminhão,
corresponde a 80% da energia mecânica, de qual
cuja carroceria está a 1,5 m de altura em relação
altura em relação ao solo o corpo foi
ao chão, e descarregado com o auxílio de uma abandonado? Adote g = 10 m/s2.
empilhadeira.
F8) (Fuvest-00) Uma pista é formada por duas
rampas inclinadas, A e B, e por uma região
horizontal de comprimento L. Soltando-se, na
rampa A, de uma altura Ha, um bloco de massa
m, verifica-se que ale atinge a altura HB na rampa
a) Calcule a densidade do produto, sabendo que, B (conforme figura), em experimento realizado na
em cada embalagem, 62,5 cm3 estão vazios. Terra. O coeficiente de atrito cinético entre o
b) Considere o descarregamento de uma única bloco e a pista é nulo nas rampas e igual a p na
caixa que se encontra sobre o piso da carroceria. região horizontal. Suponha que esse mesmo
Determine o módulo do trabalho realizado pela experimento seja realizado em Marte, onde a
força que a base da empilhadeira faz sobre essa aceleração da gravidade é gM = g/3, e considere
caixa. que o bloco seja solto na mesma rampa A e da
mesma altura Ha. Determine:
F6) (UFTM-12) Um motor ideal é usado para a) a razão RA = VA t^/Va Marte, entre as
acionar uma bomba de rendimento igual a 40%, velocidade do bloco no final da rampa A (ponto
cuja função é elevar 300 litros de água por A), e em cada uma das experiências (Terra e
minuto a uma altura de 20 m. Esse motor Marte).
consome óleo combustível de poder calorífico b) a razão RB = WTeira/WMarte, entre as energias
igual a 4,0 x 107 J/kg. Considerando g = 10 m/s2 mecânicas dissipadas pela força de atrito na
e dágua = 1,0 kg/L, responda: região horizontal, em cada uma das experiências
a) Qual é a potência efetiva do motor utilizado (Terra e Marte).
nessa tarefa? c) a razão Rc = HB T«ra/HB Marte, entre as alturas
b) Qual foi o consumo de óleo, em kg, utilizado
que o bloco atinge na rampa B, em cada uma
pelo motor, em uma hora de trabalho?
das experiências (Terra e Marte).
F7) (Unesp-07) A relação entre calor e outras \ Rampa Rampa ±7
\ B A < a
formas de energia foi objeto de intensos estudos
durante a Revolução Industrial, e uma Hb Ha
experiência realizada, por James P. Joule foi
imortalizada. Com ela, ficou demonstrado que o
trabalho mecânico e o calor são duas formas * L ♦
diferentes de energia e que o trabalho mecânico
poderia ser convertido em energia térmica. A
F9) (Fuvest-05) Um sistema mecânico faz com
figura apresenta uma versão atualizada da
que um corpo de massa Mo, após um certo
máquina de Joule. Um corpo de massa 2 kg é
tempo em queda, atinja uma velocidade
suspenso por um fio cuidadosamente enrolado
descendente constante Vo, devido ao efeito do
em um carretei, ligado ao eixo de um gerador.
movimento de outra massa m, que age como
freio. A massa m é vinculada a uma haste H,
presa ao eixo E de um cilindro C, de raio Ro,

185
Tie/nentos Ha física-Mecânica i-Trabalho e Potência
conforme mostrado na figura. Quando a massa 80 rodas e que as forças entre cada uma delas e
Mo cai, desenrola-se um fio que movimenta o o trilho tenham a mesma intensidade.
cilindro e o eixo, fazendo com que a massa m c) A aceleração do trem quando, na velocidade
descreva um movimento circular de raio Ro. A de 288 km/h, as máquinas elétricas são
velocidade Vo é mantida constante, pela força de acionadas como geradores de 8 MW de
atrito, entre a massa mea parede A, devido ao potência, freando o movimento.
coeficiente de atrito p entre elas e à força
centrípeta que age sobre essa massa. F11) (AFA-99) A figura abaixo mostra um corpo
E . em um plano inclinado, submetido à força F e ao
peso P . O trabalho, em joules, realizado por F
para deslocar o corpo por um metro, com
c velocidade constante, ao longo do plano, é,
aproximadamente,
m
(Considerar g = 10 m/s2; massa do corpo m = 1,0
,<-U kg; e coeficiente de atrito p = 0,3)
Mc F 30°
a) 3,8.
b) 6,8.
c) 7,8.
Para tal situação, em função dos parâmetros m, d) 9,8.
Mo, Ro, Vo, p e g, determine P
a) o trabalho Tg, realizado pela força da 45°
gravidade, quando a massa Mo percorre uma
distância vertical
correspondente a uma volta completa do cilindro F12) (AFA-02) Uma partícula de massa 1 kg se
C. move ao longo do eixo Ox. O módulo da força,
b) o trabalho Ta, dissipado pela força de atrito, em newtons, que atua sobre a partícula é dado
quando a massa m realiza uma volta completa. por F(x) = 2x - 2. Se a partícula estava em
c) a velocidade Vo, em função das demais repouso na posição x = 0, a sua velocidade na
variáveis. posição x = 4 m é
a) 3,5 m/s. b) 4,0 m/s.
F10) (Fuvest-10) Trens de alta velocidade, c) 4,5 m/s. d) 5,0 m/s.
chamados trens-bala, deverão estar em
funcionamento no Brasil nos próximos anos. F13) (AFA-08) O volume de água necessário
Características típicas desses trens são: para acionar cada turbina de uma determinada
velocidade máxima de 300 km/h, massa total central hidrelétrica é cerca de 700 m3 por
(incluindo 500 passageiros) de 500 t e potência segundo, "guiado" através de um conduto
máxima dos motores elétricos igual a 8 MW. forçado de queda nominal igual a 112 m.
Nesses trens, as máquinas elétricas que atuam Considere a densidade da água igual a 1 kg/L.
como motores também podem ser usadas como Se cada turbina geradora assegura uma potência
geradores, freando o movimento (freios de 700 MW, a perda de energia nesse processo
regenerativos). Nas ferrovias, as curvas têm raio de transformação mecânica em elétrica é,
de curvatura de, no mínimo, 5 km. Considerando aproximadamente, igual a
um trem e uma ferrovia com essas a) 5% b)10% c)15% d) 20%
características, determine:
NOTE E ADOTE F14) (Ciaba-10) Na figura acima o bloco de
1 t = 1000 kg massa 30 kg, que é abandonado do ponto A com
Desconsidere o fato de que, ao partir, os motores demoram alguns velocidade zero, desliza sobre a pista AB.
segundos para atingir sua potência máxima.
Considere que ao longo do percurso a força de
a) O tempo necessário para o trem atingir a atrito entre o bloco e a pista dissipa 60 J de
velocidade de 288 km/h, a partir do repouso, energia. A velocidade do bloco no ponto B, em
supondo que os motores forneçam a potência m/s, é
máxima o tempo todo. Dado: g = 10 m/s2
b) A força máxima na direção horizontal, entre
cada roda e o trilho, numa curva horizontal
percorrida a 288 km/h, supondo que o trem tenha

186
Elementos da Física-Mecânica !-Trabalho e Potência

10 m/s

10 m

a) 6,0 b)7,0 c)8,0 d) 9,0 e) 10,0

F15) (Ciaba-15) Em uma residência, há um


èh - -
aparelho de ar condicionado de potência 1 KW a)1,0kW b)1,5kW c)2,0kW
que é ligado em metade dos dias do mês, por 8 d)2,5kW e)3,0kW
horas a cada dia. Nessa mesma casa, o chuveiro
é de potência 4 kW e é ligado por 1 hora, todos F18) (EN-00) Uma partícula de massa M
os dias. Considere o custo do kWh como sendo desloca-se ao longo de uma trajetória retilínea.
R$ 0,50. Ao fim de um mês de 30 dias, o valor a Sabe-se que no instante t = 0, quando a partícula
ser pago no mês pelo custo do consumo do ar possui uma velocidade v = 1 m/s e ocupa a
condicionado e do chuveiro juntos é posição x = 0, uma força que tem a mesma
a) R$ 40,00. b) R$ 60,00. c) R$ 80,00. direção e sentido do vetor velocidade atua sobre
d) R$120,00. e) R$240,00. a partícula. Sob ação desta força, após um
deslocamento de 2 metros, a partícula passa a
F16) (EN-83) Uma partícula de massa 1 kg é ter uma velocidade v1 = 3m/s. Com o auxílio do
liberada do repouso, em A, sobre uma guia cujo gráfico dessa força em função da posição, pode-
perfil é um quadrante de circunferência de 1 m de se afirmar que a massa M da partícula, em kg, é:
raio. 0 trabalho realizado pela força de atrito que F(N)
atua sobre a partícula no trecho AB é de -2 J. De
B em diante, a partícula percorre na horizontal
uma distância de 2 m até C, onde pára.
Considerar g = 10 m/s2 e desprezar os efeitos na
resistência do ar. A potência média desenvolvida
pela força de atrito no trecho BC é de:
a)-4W A m-*| p
b) -8W 2 S(m)
c) -6W a) 0,5 b) 1,0 c) 2,0 d) 4,0 e) 8,0
d) -10W
e) -12W F19) (EN-10) Em uma academia de ginástica
uma pessoa exerce sobre um aparelho, durante
B C dois segundos, uma força constante e 400 N. A
função temporal da velocidade da mão que
F17) (EN-85) Uma bomba de água aspira de provoca essa força é mostrada no gráfico abaixo.
uma cisterna descarregando 600 litros por A velocidade da mão tem a mesma direção e
minuto, a uma velocidade de 10 m/s e numa sentido da força durante todos o movimento.
altura de 10 m. A potência mínima da bomba Quais são, respectivamente, o trabalho realizado
(desprezando as perdas e considerando g = 10 pela força nesse intervalo de tempo, e a potência
m/s2) será, em quilo-Watt: máxima aplicada ao aparelho?
v(m/s)

1.0

187
fíementos da física-Mecânica!-TrabalhosPotência
a) 200 N.m e 200 W b) 400 N.m e 200 W um gerador. O rendimento do sistema é de 80%
c) 400 N.m e 400 W d) 800 N.m e 400 W e a potência elétrica que o gerador oferece em
e) 800 N.m e 800 W seus terminais é de 16 kW.
(Jwulwi

F20) (ITA-63) Um trem elétrico sobe uma rampa


com velocidade constante. Durante a subida a
potência gasta pela locomotiva é P watts. O
mesmo trem, quando uma reação plana e com a
mesma velocidade, gasta P’ watts. Pergunta-
se: Qual é o coeficiente de atrito?
VtXTA LATI MAL
VISTA FRONTAL
Sendo dadas a densidade da água (1,0 g/cm3) e
a aceleração da gravidade (10 m/s2), aponte a
e
alternativa que traz o valor correto da vazão da
água:
a) 0,50 m3/s b) 5,0 m3/s c) 50 m3/s
F21) (EN-13) Um bloco de massa 5,00 kg desce,
d) 5,0. 102 m3/s e) Outro valor
com atrito desprezível, a pista da figura, sendo
sua velocidade inicial v0 = 4,00 m/s e a altura h =
F25) Um corpo de peso P é arrastado para cima
4,00 m. Após a descida, o bloco percorre parte
de um plano inclinado de ângulo a mantendo
do trajeto horizontal AB, agora com atrito e,
uma determinada velocidade constante. Sobre
então, colide com uma mola de massa
um piso horizontal é possível arrastar o peso P
desprezível e constante k = 200 N/m. Se a
junto a um outro peso Pi com a mesma
compressão máxima da mola devido a essa
velocidade anterior. Determinar P1 sendo p o
colisão é Ax = 0,500 m, o trabalho da força de
atrito, em joules, vale (Dado: g = 10 m/s2) coeficiente de atrito do solo horizontal.

F26) Um automóvel, de 800 kg de massa,


percorre em três horas 120 km na horizontal e
h
400 m na vertical (desnível), com velocidade
constante. O coeficiente de atrito do piso é 1/50.
As resistências do mecanismo absorvem 30% da
|A/\/l potência. Calcular a potência do motor em CV,
A B
usando 1 W = 750 CV e a aceleração da
a)-72,0 b)-96,0 c)-140 gravidade igual a 10,0 m/s2.
d)-192 e)-215
F27) Uma escada rolante liga um andar de uma
F22) (IME-77) Um motor com rendimento de 90% loja com outro situado 7,5 m acima. 0
aciona um guincho de rendimento de 40%. comprimento da escada é de 12 m e ela se move
Sabendo que a potência fornecida ao motor é de a 0,60 m/s. a) Qual deve ser a potência mínima
5 kW, calcule a velocidade constante com a qual do motor para transportar até 100 pessoas por
um peso de 450 kgf será levantado pelo guincho. minuto, sendo a massa média 70 kg? b) Um
homem de 70 kg sobe a escada em 10 s. Que
F23) (IME-79) Um trem constituído de 20 vagões trabalho o motor realiza sobre ele? c) Se o
com peso de 200.000 N cada e uma locomotiva homem, chegando ao meio, põe-se a descer a
com peso de 500.000 N desloca-se sobre trilhos escada, de tal forma a permanecer sempre no
horizontais com velocidade constante de 60 meio dela, isto requer que o motor realize
km/h. A resistência ao movimento equivale a uma trabalho? Em caso afirmativo, com que potência?
força com módulo de 1% do peso. Calcule:
a) a força de tração exercida pela locomotiva. F28) Considere a figura abaixo. O bloco de
b) a potência desenvolvida pelo motor da massa M, que está preso a uma mola, possui
locomotiva. velocidade inicial v0 dirigida da esquerda para a
direita e sua posição é tal que a mola não exerce
F24) O esquema seguinte representa os nenhuma força sobre ele, isto é, a mola não está
principais elementos de um sistema rudimentar comprimida nem esticada. O bloco percorre uma
de geração de energia elétrica. A água que jorra
distância ( para a direita antes de parar na
do tubo faz girar a roda, que, por sua vez, aciona
posição pontilhada. A constante da mola é k e o

188
Mementos da física-Mecânica l-Trabalhoe Potência
coeficiente de atrito cinético entre a mesa e o
bloco vale p<;.
Exercícios de Aprofundamento
£ A1) (ITA-78) Uma corda de massa “M” e
1 x comprimento “L”, acha-se pendurada em um
• i i prego, conforme figura. Devido a uma pequena
perturbação, a corda começa a deslizar.
Determine para o deslocamento C da massa M: Desprezando-se os atritos, pode-se afirmar que a
a) o trabalho realizado pela força de atrito; velocidade v da corda, no instante em que a
b) o trabalho realizado pela força de atrito; mesma abandona o prego, é dada por:
c) o trabalho realizado pelo peso do bloco;
d) o trabalho realizado pela reação normal da
mesa sobre o bloco;
e) o trabalho total realizado sobre o bloco;
f) determine a distância l em função das
r
2
/Rh

grandezas pertinentes usando o teorema que


relaciona o trabalho com a energia cinética.
A2) (ITA-85) Três blocos B,, B2 e B3 de mármore,
F29) O trabalho necessário para empurrar uma de mesma massa específica p e mesma área de
pedra de 10 kg da base até o topo de um plano secção transversal A tem alturas
inclinado é 400 J. Para empurrar a mesma respectivamente iguais a hi, h2 e h3, sendo h, >
pedra do topo do plano inclinado até a base o h2 > h3. Eles estão inicialmente no solo
trabalho necessário é 250 J. Qual a altura do horizontal, repousando sobre suas bases. Em
plano inclinado? (Obs; Suponha g = 10 m/s2) seguida são empilhados, formando uma coluna
a) 3,25 m b)1,5m
b) 1,5 m c) 1,625 m de altura h = hi + h2 + h3. A aceleração da
d) 0,375 m e) 0,75 m gravidade é g. Quanto vale o trabalho realizado
na operação de empilhamento?
F30) Uma locomotiva de brinquedo de massa
m é acelerada, a partir do repouso e livre de A3) (IME-76) Uma bomba submersa no fundo de
quaisquer forças dissipativas, numa pista um rio de 4 m de profundidade descarrega água
horizontal. Sabe-se que o motorzinho da sobre a margem através de uma tubulação de 40
locomotiva fornece uma potência constante mm de diâmetro com uma velocidade de 60 m/s.
igual a P. A distância que esse brinquedo Achar a potência da bomba em CV, para os
percorre até o instante t é seguintes casos:

<r- Pt3'2
O ( P \3/2
c) - -
2kmJ
t1'2
a) Não há perdas;
b) o rendimento é de 65%.
Considere o peso especifico da água 1000
kgf/m3.

A4) Em cima de uma mesa estão espalhadas n


F31) Um bloco desliza sobre um plano moedas iguais, cada uma com massa m e
inclinado com atrito (ver figura). No ponto A, a espessura t. Com essas moedas todas forma-se
velocidade é vA = 2 m/s e no ponto B, distando uma pilha. Nesse processo, qual o trabalho da
1 m do ponto A ao longo do plano, vB = 3 m/s. gravidade, sendo g a sua aceleração?
Obtenha o valor do coeficiente de atrito
cinético entre o bloco e o plano. A5) De um poço de profundidade H = 20 m retira-
se água com um balde. O balde é cheio de água
até sua borda. Durante a elevação parte da água
derrama e volta a cair no poço. Supondo que o
balde se eleva em movimento uniforme e a
velocidade com que se derrama água é
constante, determinar o trabalho que deve ser
realizado para subir o balde, se até chegar em
cima ficam 2/3 da massa inicial de água. A
a) ^3 b)^ c)^+1 d)V3+l e)73-l massa do balde vazio é m = 2 kg e seu volume,
V=15/.

189
Elementos tia Física-Mecânica!-Trabalho e Potência
A6) Os pacotes mostrados na figura são A10) O motor de uma bomba hidráulica tem
lançados sobre um plano inclinado em A com potência igual a 500 W. Em quanto tempo,
uma velocidade de 1 m/s. Os pacotes deslizam aproximadamente, esta bomba enche um
ao largo da superfície ABC até uma esteira reservatório de 1000 litros colocando a uma
transportadora que se move com uma velocidade altura de 10 m de modo que a água seja injetada
de 2 m/s. Se é conhecido o coeficiente de atrito no reservatório com uma velocidade escalar de 2
cinético pc = 0.25 entre os pacotes e a superfície m/s?
ABC, determine a distância d se os pacotes
devem chegar a C com uma velocidade de 2 m/s. A11) Um cubo homogêneo com aresta 2a e
I in/s massa m apóia-se em uma superfície horizontal.
Em uma das arestas superiores aplica-se uma
força F, horizontal e normal à aresta. Determinar
o mínimo coeficiente de atrito para que o cubo
possa tombar e o trabalho que o operador
• \30°
despende até levar o cubo à posição instável,
• -3 sendo g a aceleração da gravidade.

A7) O elevador E tem uma massa de 3 000 kg


quando está completamente carregado e se
conecta como se mostra a um contrapeso W de
1 000 kg de massa.

wl.:
A12) Três caixas de 20 kg estão em repouso na
correia que passa sobre a polia e está presa ao
bloco de 40 kg. Sabendo que o coeficiente de
atrito entre a correia e a superfície horizontal a
também entre a correia e as caixas é de 0,50,
II
w p-
E \c
determine a velocidade da caixa B quando ela cai
da correia em E.
■<2 m 2 m

Bn cn Dn
2m
2 m -»|

E
rm o o
s:
a0 40 kg
Determine a potência em kW que entrega o
motor quando A13) Um helicóptero é usado para erguer do
a) o elevador se move para baixo com uma oceano um astronauta de massa igual a 70 kg
velocidade constante de 3 m/s, até uma altura de 17 m, numa direção vertical,
b) o elevador tem uma velocidade para cima de 3 por meio de um cabo. A aceleração do
m/s e uma desaceleração de 0.5 m/s2. astronauta vale g/8.
a) Calcule o trabalho realizado pelo helicóptero
A8) Que potência mínima deve ter uma bomba sobre o astronauta.
que eleva água por um tubo até uma altura h. A b) Qual é o trabalho realizado pela força
seção do tubo é S e o volume de água que gravitacional sobre o astronauta?
bombeia por unidade de tempo é Vt.
A14) O bloco de peso 10 N parte do repouso e
A9) Um esquiador, deslizando através de uma sobre a rampa indicada na figura mediante a
superfície horizontal de gelo a uma velocidade aplicação da força F de direção constante e cuja
constante v = 6,0 m/s, se dirige a uma superfície intensidade varia com a abscissa x de acordo
asfaltada. O comprimento do esqui é L = 2 m. O com o gráfico. O trabalho realizado de O até A
coeficiente de atrito entre o asfalto e o esqui é pelo atrito existente entre o bloco e a rampa é
igual a p = 1. Que espaço percorrerá o esquiador igual a 10 J, em valor absoluto. Adotar g = 10
até parar totalmente? m/s2. Nestas condições a velocidade do bloco, ao
atingir o ponto culminante A, é igual a:

190
Elementos tia Física-Mecânica !-Trabalho e Potência
F(N) figura abaixo. Durante toda a travessia a
velocidade do barco, V,, manteve-se constante e
dirigida sempre perpendicularmente à correnteza
do rio e o motor desenvolveu uma força
4 ni
constante igual a 1,47 x 103 N, na direção de V,.
Calcule a potência, P, desenvolvida pelo motor
1---- r- durante a travessia do rio.
1 2 3 4 5 x (m)

Figura 1 Figura 2
ZZZ ////I
A15) Um corpo de peso 20 N sobre um plano
inclinado sem atrito é puxado por uma força F
paralela a esse plano. O corpo parte do repouso
e após dois segundos e após dois segundos
atinge uma altura de dois metros acima do ponto
de partida. A potência desenvolvida pela força F 2 — -___ =
é dada pelo gráfico abaixo.
“ Pot(W)
50 __________

A18) Ergue-se um corpo de massa m, em um


local onde a aceleração da gravidade é g, até o
cume de uma montanha, uma vez, percorrendo o
^(s) caminho ADC e outra, o caminho ABC.
0 1 2
Determinar o trabalho realizado pela força F nos
dois primeiros segundos do movimento e a
velocidade do corpo no fim desse tempo. Adote g
= 10m/s2.

A16) Suponha que você dispõe de 7 estacas de h


madeira, todas idênticas (de massa m) e da
forma de um prisma de base quadrada, com
altura h e aresta de base a. Suponha também
que está disponível um vidro de cola para E< +■
madeira. Determine o trabalho necessário para, í
partindo das estacas deitadas no solo, formar a a) Demonstrar que se a subida é lenta , o
palavra ITA, da forma que está na figura abaixo, trabalho realizado será o mesmo, caso, o
podendo para isso mover as estacas e colar o coeficiente de atrito p em ambos os trajetos seja
que achar necessário. igual;
Obs: Considere que nas colagens o trabalho b) Determine o valor do trabalho realizado.
despendido é nulo e que a estaca horizontal
inferior do A está na metade da altura. A19) Uma corrente homogênea muito grande de
«i I" „ comprimento L possui velocidade constante v. De
repente encontra um plano inclinado liso, o qual
forma um ângulo a com a horizontal. Determine
que comprimento da corrente está sobre o plano
inclinado no instante em que a corrente fica em
repouso.

A17) (OBF-03) Um barco a motor, ao atravessar


um rio cuja velocidade da correnteza é V2 = 8,0
km/h e largura L = 720 m, chegou à margem
oposta em um ponto situado a uma distância d = a
T O O a g o
160 m abaixo do ponto de partida, como mostra a

191
Elementos da física-Mecânica !-Trabalho e Potência

a) v
2Lsena
b) v
1 L . It
c) v.sena —
d) O trabalho de F é igual à variação da energia
gsena cinética do corpo.
g N9 e) O corpo descreverá uma trajetória elíptica
sobre a mesa.
d)-^- e) nda
sena A22) Uma caixa parte do repouso de desliza
sobre um plano inclinado de 37° com o a
horizontal, indo de encontro a uma mola de
A20) (ITA-87) Um homem cuja massa é 70 kg
constante elástica igual a 160 N/m,
está sentado sobre um andaime pendurado num
comprimindo-a de 25cm. O coeficiente de atrito
sistema de roldanas. Ele se eleva puxando a
cinético entre a caixa e o plano inclinado vale
corda que passa pela roldana fixa. Considerando
g = 9,8 m s'2, desprezando os atritos, resistência p = 0,5 e caixa foi inicialmente abandonada a
e a massa do andaime e supondo que o homem uma distância de 75cm da mola relaxada.
se eleva muito lentamente, calcular Determinar a massa M dessa caixa.
Dado: g = 10 m/s2, sen 37° = 0,6
cos 37° = 0,8.

I6,

37° to Brito

a) 3,0 kg b) 2,5 kg c) 2,0 kg


d)1,5 kg e) 1,0 kg
a) a força que ele precisa exercer.
b) o acréscimo em sua energia total quando ele A23) Uma haste leve metálica de comprimento
se eleva de 50 cm. L = 1,0 m pode girar livremente em torno de
c) o deslocamento de ponto de aplicação de cada um ponto O de um plano inclinado. Em sua
uma das forças aplicadas ao sistema homem + outra extremidade existe um corpo de massa
andaime e, a partir daí, o trabalho de cada uma m = 1,0 kg, inicialmente em repouso em um
dessas forças. ponto A do plano inclinado. O coeficiente de
atrito cinético entre o corpo e o plano inclinado
A21) (ITA-02) Um corpo de massa M, mostrado
vale p = 0,5. A inclinação do plano inclinado é
na figura, é preso a um fio leve, inextensível, que
tal que sen 0 = 0,6 e cos 0 = 0,8. Determine o
passa através de um orifício central de uma
valor da menor velocidade v0 que deve ser
mesa lisa. Considere que inicialmente o corpo se
imposta ao corpo no ponto A de modo que o
move ao longo de uma circunferência, sem atrito.
mesmo atinja o ponto B (diametralmente
O fio é, então, puxado para baixo, aplicando-se
oposto ao ponto A), o mais alto de sua
uma força F , constante, a sua extremidade livre. trajetória. Suponha g = 10 m/s2.
Podemos afirmar que:

a) V4rt + 6 m/s b) 2V2n + 3 m/s


a) O corpo permanecerá ao longo da mesma
circunferência. c) + 6 m/s d) V87t + 6 m/s
b) A força Fnão realiza trabalho, pois é e) 2x/n + 3 m/s
perpendicular à trajetória.
c) A potência instantânea de F é nula.

192
flementos da física - Mecânica /- Trabalho e Potência
A24) Um livro está localizado no limite entre
duas mesas. O comprimento do livro é l e sua
massa é m. Os coeficientes de atrito cinético
entre o livro e as superfícies das mesas são
distintos e valem p, e p2, como indicado na
figura. A aceleração local é g. Determine o
mínimo trabalho necessário para arrastar o
livro da 1a mesa para a 2a mesa.
í

Mi______ s __ m a) - 2irpmv2 b) - 27tmgv2/p


I 22 I c) - npmgv2 d) - rtmg/(2pv2)
e) - npmv2
1
J A27) Uma locomotiva de massa m inicia seu
movimento tal que sua velocidade varia de
a) mgW2 b) mgZIm-pj | acordo com a lei v = kVx , onde k é uma
2(Pi + f2)
constante e x a distância percorrida. Determine o
mgl(pi+p2) trabalho da força resultante na locomotiva
c) d)
2____ durante um intervalo de tempo t contado desde o
e) + P2 início.

A28) (OBF-10) Um bloco de 3,0 kg, inicialmente


A25) O sistema da figura está em repouso e
em repouso, está posicionado numa superfície
na iminência do movimento por causa do atrito
plana e sem atrito. O bloco é movimentado por
existente entre a superfície horizontal e o
8,0 m por uma força constante de 12 N, em 3,0
corpo B. Retirando-se o corpo C, o sistema
segundos. Qual a potência média da força
começa a deslizar, sendo o coeficiente de
durante o movimento?
atrito estático igual ao dinâmico. Determine o | 3,0 kg | —>,F^12N /////////
trabalho realizado pelo atrito do instante em
que se inicia o movimento até quando o corpo •8.0 m
A atinge a velocidade de 6 m/s, sabendo que
as massas de A, B e C são respectivamente
iguais a 3 kg, 2 kg e 4 kg. Dado: g = 10 m/s2. A29) (OBF-10) Uma força F = 500 N é aplicada
em um bloco de massa 50 kg (perpendicular a
uma das superfícies) conforme o diagrama a
S seguir (0 = 30°). (desconsidere todos os efeitos
de quaisquer tipos de atrito neste sistema).

B
>50 k<
F
1§______
A Horizontal
a)-30 J b)-40J c)—45 J d)-50J e)-60 J
a) Sabendo que a distância entre A e B é de 1
A26) Mediante a ação de uma força F o bloco metro, determine o trabalho realizado pela força
de massa M traslada com uma velocidade F entre A e B.
constate sobre a superfície interna de um b) Determine a velocidade com que o corpo
cilindro de raio R. O coeficiente de atrito atingirá no ponto B. Considere que o corpo parte
do repouso no ponto A.
cinético entre o bloco e o cilindro é igual a p.
Suponha que a força F é, a todo momento, A30) (OBF-05) Um sólido de massa m = 100 kg
tangente à trajetória). Calcule o trabalho desliza sobre um plano horizontal sob a ação de
realizado pela força de atrito em uma volta. uma força constante paralela ao plano. O
coeficiente de atrito entre o móvel e o plano é
0,10. O corpo passa por um ponto A com

193
Elementos da física-Mecânica !-Trabalho e Potência
velocidade 2,0 m/s e, após o intervalo de 10 s, deslocado de um ângulo 0 (0 < 0 < rr/2) ao
passa por um ponto B com a velocidade de 22,0 longo da superfície semicircular.
m/s.
a) Qual o módulo da força?
b) Qual o trabalho realizado pela força durante o
deslocamento de A para B?

A31) Uma corrente uniforme de massa m e


comprimento L é originalmente colocada sobre
um plano inclinado liso de modo que o ponto
médio da corrente esteja no ponto mais alto do
plano (figura A). A seção transversal do plano
inclinado é um triângulo isósceles de lados
congruentes iguais a L e ângulos congruentes
iguais a 0. Devido a uma pequena ka2
a) ——+ mgsen0-mgacos0
perturbação, a corrente entra em movimento.
Determine a energia cinética da corrente k02a2
quando a mesma estiver totalmente sobre o b) - + mgasen0
2
lado esquerdo do plano inclinado (figura B).
c) 2kn2a2 - mga sen 0 + mga cos 0
Considere que a aceleração da gravidade é g.
d) 2k02a2 -mgasen0
L
£?■
k02a2
e) + mgasen0
e 9 9
2
(A> (B)
A34) Um corpo de massa 400 g está preso a
um fio de comprimento 1,5 m, girando num
mgLsen0 mgLsen0 mgL cos 0
a) b) c)
:2! 4 2 plano horizontal em MCU com w = y rad/s,
mgLcos0
d) e) mgL tg 0 constituindo um pêndulo cônico. Uma força
4 tangente à trajetória passa então a ser
aplicada sobre o corpo até que ele adquira a
A32) Um bloco possui uma energia cinética
inicial de 128 J no ponto A. O bloco sobe o
2 ZsÕ
velocidade angular de ------ rad/s, atingindo
plano inclinado com uma desaceleração 3
uniforme. Ao passar pelo ponto B, sua energia um novo MCU. Determine o trabalho desta
cinética variou em 80 J e o trabalho da força força neste intervalo de tempo, sabendo que g
de atrito de A para B é - 35 J. Calcule o = 10 m/s2.
trabalho da força de atrito no trajeto AC, onde a) 0,6 J b) 0,9 J c) 1,3 J
C é o ponto onde a velocidade do bloco é nula. d)1,5J e)1,9J

A35) Uma bala de massa 10 g é atirada


horizontalmente contra um bloco de madeira
de 100 g que está fixo, penetrando nele 10 cm
até parar. Depois, sem retirar a primeira bala, o
bloco é suspenso de tal forma que se possa
mover livremente e uma bala idêntica à
A33) Uma força variável F se mantém primeira é atirada contra ele. Considerando a
tangente a uma superfície semicircular lisa de força de atrito entre a bala e a madeira em
raio a. A força varia de modo que o bloco de ambos os casos como sendo a mesma,
massa m é deslocado bem lentamente desde a conclui-se que a segunda bala penetra no
posição 1 até a posição 2. Considere que na bloco a uma profundidade de
posição 1 a deformação da mola é nula. aproximadamente:
Suponha também que a mola possui massa a) 7,6 cm b) 8,4 cm c) 9,2 cm
desprezível e constante k. Supondo que g é a d) 9,6 cm e) 10 cm
aceleração da gravidade, calcule o trabalho
realizado pela força F quando o bloco é A37) Que trabalho um corpo tem de realizar
para subir um plano inclinado de altura h = 10

194
Mementos da Física-Mecânica !-Trabalhos Potência
m? Sabe-se que o ângulo de inclinação do desempenho com a moto Honda CB 1300,
plano é 45° e a massa do corpo é 30 kg. O modelo que havia acabado de ser lançado no
coeficiente de atrito cinético entre o corpo e o Brasil. O teste indicava, entre outros dados,
plano diminui linearmente ao longo do caminho que a retomada de velocidade de 50 km/h até
desde p = 0,5 na base até p = 0,1 no cume. 100 km/h demorava exatamente 5 s. Constava
A) 900 J B) 1.200 J C) 2.400 J também na matéria a informação que o teste
D) 3.000 J E) 3.900 J foi realizado sempre na potência máxima da
moto. Assinale a alternativa que indica a
A38) Um sistema mecânico é montado para relação, em unidades do SI, entre a velocidade
estudar as dissipações de energia. Há o bloco e o tempo neste teste de retomada de
A, de massa 4m, que é abandonado da velocidade.
posição vertical 2. Há o bloco B, de massa m, a) v = Vl92,9 + 115,7.t
que guarda coeficiente de atrito cinético p com b) v = 13,9 +2,8.t
o plano inclinado e está inicialmente na c) v = 50 + 10.t
posição 1. Determine, considerando a d) v = V2500 + 15OO.t
gravidade no local igual a g, a velocidade com
e) nda
que o bloco A chega ao solo.

2-Aí A41) Que trabalho é necessário realizar para


4m fazer uma tábua larga, que descansa sobre o
1g solo, gire no plano horizontal, ao redor de um
p a? O
eo

3 --

A39) Na figura,
m e comprime
escorregar. Apli
vertical para baixo
que a corda 1
imediatamente após
inteiramente para fora da i,
v é a velocidade adquirid;
qualquer ao cair do repouso,
de uma altura t, determine o trabalr..

I— 2(15 —I

(15
2£i
B

A
A) 18m9r 17mgí
B) C) —
25 25 25
D) 15m^ 4,5mg£
E)
25 25

A40) Em março de 2007 foi publicado em uma


revista de circulação nacional um teste de

195
ENERGIA POTENCIAL DE UNI SISTEMA

Considere um sistema que consiste de um livro e a Terra, que interagem através da força
gravitacional. É realizado trabalho sobre o sistema ao levantar o livro lentamente desde o repouso
através de um deslocamento vertical Ar, como ilustra a figura. Este trabalho realizado sobre o
sistema pode ser interpretado como uma maneira de transferir energia para o sistema,
aumentando sua energia.

y “

Ar

yr m

Note que o livro está em repouso no início e no final da trajetória, fazendo com que não
exista variação da energia cinética do sistema. Deste modo, a variação da energia do sistema não
ocorre na forma de energia cinética, devendo aparecer como alguma outra forma de
armazenamento de energia. Depois de levantar o livro, se o mesmo for liberado acabará voltando
à posição inicial pela ação da força da gravidade. Nesta situação, o livro agora possui energia
cinética, cuja origem está no trabalho que foi feito ao se levantar o livro. Logo, pode-se afirmar que
quando o livro está em um ponto de maior altura a energia do sistema possui potencial para
converter-se em energia cinética, porém isto só ocorre quando é permitido ao livro cair. Em
consequência, o mecanismo de armazenamento de energia antes do livro ser liberado se chama
energia potencial. Será demonstrado que a energia potencial de um sistema somente se associa
com tipos específicos de forças.

ENERGIA POTENCIAL GRAVITACIONAL

Suponha que uma força vertical é aplicada em um objeto de massa m, levantando-o de


uma altura inicial y, até uma altura final yf. Supõe-se que processo é realizado bem lentamente,
sem aceleração, de modo que o módulo da força aplicada seja igual ao módulo da força peso do
objeto. O objeto pode ser considerado como uma partícula em equilíbrio que se move com
velocidade constante. O trabalho realizado pela força F sobre o sistema é dado por:

WF = F.Ar =| F|.| Ar j.cosO = mg(yf - y,) mgyf -mg^

Nesta equação o trabalho representa uma transferência de energia ao sistema e a energia


do sistema aparece em uma forma que se denomina energia potencial. Em consequência, a
quantidade mgy pode ser identificada como energia potencial gravitacional: Ep = mgy

A unidade da energia potencial gravitacional no SI é Joule, a mesma unidade de trabalho e


energia cinética. A energia potencial, como o trabalho e a energia cinética, é uma grandeza
escalar. Note que a expressão Ep = mgy é válida somente para objetos próximos à superfície da
Terra, onde g é aproximadamente constante.

196
Elementos da física-Mecânica!-Energia Mecânica
O trabalho da força F pode agora escrito como:

WF = mgyf - mgy = Epf - EpÉ = AEp

Desta forma, o trabalho realizado sobre o sistema é igual à variação da energia potencial
gravitacional do sistema.

Note que no processo de subida atuam apenas duas forças: F e P . Como a variaçao da
energia cinética do corpo foi suposta igual a zero, pelo teorema do trabalho-energia:

WF + WP = AEC = 0 => WP = —WF = —AEP

A energia potencial gravitacional de corpo de massa m pode ser interpretada como a


modalidade de energia associada unicamente à sua posição (em particular à sua coordenada y) e
que pode ser convertida em algum momento em energia cinética.

Ao resolver situações problemas deve-se escolher um referencial para a determinação das


coordenadas y das posições iniciais e finais da trajetória do corpo. Porém, perceba que o valor de
AEp = mgAy independe da escolha deste referencial, uma vez que a quantidade Ay é constante
para qualquer eixo y vertical que se adote. Assim, a escolha do referencial é arbitrária, contudo, é
usual adotar um referencial unidimensional y cuja origem está na superfície da terra.

ENERGIA POTENCIAL ELÁSTICA

Considere um sistema que consta de um bloco e uma mola. A força que a mola exerce
sobre o bloco é dada por Fe = - kx. O trabalho realizado pela força elástica Fe é dado por:

_kx* kxf
Wf
F«~ 2 2

■viwwwv

Suponha que uma pessoa aplique uma força F sobre o bloco de modo a comprimi-lo de x
unidades de comprimento. Considere que este processo foi feito bem lentamente, de modo que o
bloco se movimentasse com velocidade constante. Neste caso o módulo da força F é igual ao
módulo da força elástica Fe. Como estas são as duas únicas forças que realizam trabalho sobre o
bloco e como a variação da energia cinética é zero:

kx* kx§
WFe + WF = 0 => WF = -WFe WFe
2 2

Nesta situação, as coordenadas inicial e final do bloco são medidas desde sua posição de
equilíbrio, x = 0. De novo (como no caso gravitacional) determina-se que o trabalho realizado
sobre o sistema é igual à subtração entre os valores final e inicial de uma expressão relacionada
unicamente com a configuração geométrica do sistema. A função de energia potencial elástica
associada com o sistema massa-mola se define mediante

197
fíementos da física-Mecânica!-Cnergia Mecânica

_kx2
^-Pelástica —

Deste modo, o trabalho da força elástica pode ser escrito em função da variação da
energia potencial acumulada na mola:

kxg kx,2 _ _
-y- = Ep0-Epf -AEp
2

A energia potencial elástica do sistema pode ser interpretada como a energia armazenada
na mola deformada. A energia potencial elástica armazenada em una mola é zero sempre que a
mola não está deformada (x = 0). Como a energia potencial elástica é proporcional a x2, conclui-se
que Ep elástica sempre é positiva em uma mola deformada.

FORÇAS CONSERVATIVAS E NÃO CONSERVATIVAS

Considere um corpo de massa m que será transladado de um ponto A até um ponto B.


Dentre todas as forças que atuam no corpo considere duas forças distintas F, e F2 .

Na figura estão indicados três possíveis caminhos para levar o corpo do ponto A ao ponto
B. Se o trabalho realizado pela força F, for independente do trajeto utilizado para levar o corpo do
ponto A ao ponto B afirma-se que F, é uma força conservativa. Neste caso verifique que o
trabalho realizado pela força F, para levar o corpo de A para B é o negativo do trabalho de levar o
corpo de B para A, independentemente das trajetórias de ida e volta:

W'^-.
t b =jFrdrAB = jF1.(-dfBA) = -jF1.drBA - WF, B—>A WFl a-»b + WFl B_(A = 0

Logo, pode-se afirmar que uma força é conservativa se e somente se for nulo o trabalho
que ela efetua sobre um corpo que descreve uma trajetória fechada e retoma à posição inicial.
São exemplos de forças conservativas: peso, força elástica e todas as forças cujo trabalho total é
nulo (força centrípeta, força normal em um deslizamento, força de tração em um pêndulo,...).

Suponha agora que o trabalho realizado pela força F2 apresenta valores diferentes para
diferentes trajetos de levar o corpo de A para B. Neste caso afirma-se que F2 é uma força não
conservativa ou força dissipativa. Como consequência, caso alguma força dissipativa atue em
um corpo, conclui-se que, em qualquer trajeto, não foi conservada a capacidade que o sistema
tinha de realizar trabalho. Neste caso, em uma trajetória fechada, com o corpo retornando à
posição inicial, o corpo apresenta uma energia cinética final inferior a energia cinética inicial. As
forças de atrito e a resistência ao movimento, no ar ou nos líquidos, são sempre forças resistentes
e não conservativas.

198
__________________________________________ Elementos da Física-Mecânica!-Energia Mecânica
FORÇAS CONSERVATIVAS E ENERGÍA POTENCIAL

Considere um corpo onde atuam apenas forças conservativas, ou seja, força peso, força
elástica e forças cujo trabalho seja igual a zero. Nestas condições afirma-se que a força resultante
no corpo é conservativa. Logo, o trabalho desta força resultante não depende da trajetória seguida
pelo corpo durante o movimento. O trabalho somente depende das coordenadas inicial e final. Em
consequência, pode-se definir uma função de energia potencial tal que o trabalho realizado pela
força resultante, que é conservativa, seja igual ao negativo da diminuição da energia potencial do
sistema.

Fr =Ê| +F2 +- + Fn w.F


r
WFF1 +WFF2 +... + WpFn = EPo -Epf =-AEp

Em um sistema de n partículas pode afirmar que:

WFRi=EPol-EPf1, WFr2 - FPo2 FPf2 ■ ^FRn = EpOn-Epfn

Somando estas n expressões obtém-se:

ÈWFRi = È(EPoi -EPr.) => ÈwFrí=-ÈaePí


k-1 k=1 k=1 k=1

Guarde bem este resultado, ele será muito importante na demonstração da conservação
da energia mecânica.

ENERGIA MECÂNICA

A energia mecânica de um corpo é definida como a soma de sua energia cinética de


translação com sua energia potencial:

Emecânica Epinética Epotencial

A energia cinética de translação de um corpo de massa m com velocidade v é sempre


mv2
dada por Ec = ——. Por outro lado, existem várias modalidades de energia potencial, como
energia potencial gravitacional, energia potencial elástica e energia potencial eletrostática. Porém,
neste capítulo, serão estudadas apenas a energia potencial gravitacional em pontos próximo à
superfície terrestre e a energia potencial elástica.

Se h é a altura de um corpo de massa m em relação a um sistema de referência, sabe-se


que a energia potencial gravitacional deste corpo, para pontos próximos à superfície da Terra, é
igual a Ep = mgh, onde g é a aceleração da gravidade local. Sabe-se também que a energia
potencial elástica é igual a — ■ , onde k é a constante elástica da mola e Ax = |£f- f0|. é a
variação do comprimento da mola. Desta forma, caso alguma força elástica esteja atuando, a
energia mecânica de um corpo de massa m é igual a:

mv2 u k(Ax)2
^mecânica ~ +
2

Caso nenhuma força elástica esteja atuando no corpo a energia mecânica será:

tr mv2 u
Emecânica =-y~+m9h

199
fíementos da física-Mecânica i-ínergia Mecânica

No caso de um sistema de partículas, a energia mecânica do sistema é determinada


somando as parcelas da energia cinética, energia potencial gravitacional e energia potencial
elástica de todos as partículas do sistema. Assim, considerando um sistema de partículas em que
façam parte n partículas de massas mi, m2 mn, cada um com velocidades Vi, v2, ..., vn e alturas
hi, h2 hn, respectivamente, em relação a um referencial; m molas, cada uma com constante
elástica ki, k2, .... km, deformadas em Ax1f Ax2 Axm, respectivamente, a energia mecânica deste
sistema de partículas é dada por:

n m n m ki(AXj)2
-
E mecanica
k-1 Z k=1 k=1 2
energia energia potencial
cinética do do sistema
sistema

Para um sistema em que não exista força elástica a energia mecânica é dada por:

Emeeâniea=È^+Èmighi

k=1 z k=1______ z
energia energia
cinética do potencial do
sistema sistema

Logo, para um sistema de n partículas tem-se:

^mecânica
ÊECi+ÈEPi
k=1 k=1

CONSERVAÇÃO DA ENERGIA MECÂNICA

Suponha um sistema de n partículas onde atuam apenas forças conservativas, ou seja,


apenas forças peso, forças elásticas ou forças cujo trabalho seja igual a zero. Pelos conceitos já
expostos neste capítulo tem-se que o somatório do trabalho das forças resultantes em todas as
partículas é igual ao negativo da variação da energia potencial:

-ÈAEPi
k=1 k=l

Por outro lado, pelo teorema trabalho-energia o somatório do trabalho das forças
resultantes em todas as partículas é igual à variação da energia cinética total do sistema.

Éw^Jaec,
k=1 k=1

Deste modo, segue que:

-f aePí=£aeCí £aeCí + £aePí = 0 =>


k=1 k=1 k=1 k=1
(Ecf1 - Ec01) + (Ec(2 — Ec02)+...(Ec(rl -EcOn) + (Ept1 - Ep01) + (Epf2 - Ep02) + ...(Ep,'fn - EPon ) “ 0
ECqi + Ec^ + - + EcOn + Ep01 + Ep02 +... + EpOn = Ecf1 + Ecf2 +... + Ecfn + Epf1 + Epf2 + ■ ■ ■ + Epfn =>
energia dnéticica energia potencial energia cinética energia potencial
inicial do sistema inicial do sistema final do sistema final do sistema
^mecânica inicial “ ^mecânica final

200
_________ __ _______________ ____________ Elementos da Física-Mecânica !-Energia Mecânica
Esta última expressão denomina-se "Conservação da Energia Mecânica” e é um dos mais
importantes teoremas da física. É válida apenas para sistema em que todas as forças atuantes
sejam conservativas ou cujo trabalho seja igual a zero.

ENERGIA MECÂNICA E FORÇAS NÃO CONSERVATIVAS

Considere um sistema de partículas onde atuem forças conservativas e forças não


conservativas. Sabe-se que a força resultante no sistema é igual à soma vetorial das forças
conservativas e das forças não conservativas. Do mesmo modo, o trabalho da força resultante no
sistema é igual à soma dos trabalhos das forças conservativas e do trabalho das forças não
conservativas:

WrhR, - w
* VF conservativas
4- W
1 * ¥F não conservativas

Como o trabalho da força resultante no sistema é igual à variação da energia cinética de


todas as partículas do sistema e o trabalho das forças conservativas é igual ao negativo da
variação da energia potencial do sistema segue que:

Afinal do sistema ~ ^^inicial do sistema ~ ~^Pfinal do sistema + ^Pinicial do sistema ^F não conservativas

^F não conservativas ~~ (^^final do sistema + ^Pfinal do sistema ) — (^^inicial do sistema ^Pinidal do sistema )
inidal do sistema
Wp não conservativas ~ ^mecânica final do sistema — ^mecânica inicial do sistema
'mecânica final do sistema
Wp não conservativas ~ AEmecânica do sistema

Logo, conclui-se que o trabalho das forças não conservativas é igual à variação da energia
mecânica do sistema.

Considere o experimento desenhado abaixo, onde um corpo de massa m, preso a uma


mola, desce um plano inclinado áspero.

N
Fat

Fe
p

Das forças que atuam no corpo, a força peso e a força elástica são forças conservativas, a
reação normal produz trabalho nulo ao longo da trajetória e a força de atrito cinético é não
conservativa. Desta forma, pode-se afirmar:

... mv? , k(Ax0)2


Wpat - AFmeC = E «I
L_mec final ^mec inicial WFat=-^- + mghf +
2
Considerando que o corpo desce uma distância d ao longo do plano inclinado e que o
coeficiente de atrito cinético é constante e igual a p segue que:

... mv? ., , . k(Axf )2 k(Ax0)2 mv2


-N.d = -^L- + mg(hf-h0) +
2 2 2~
-(mgpcos0)d = ^E--^^- + mgdsen0 + k(Axf)2 k(Ax0)2
2 2

201
Fiementos da Física-Mecânica!-Energia Mecânica
Exemplos:

1) (ITA-95) A figura ilustra um carrinho de massa m percorrendo um trecho de uma montanha


russa. Desprezando-se todos os atritos que agem sobre ele e supondo que o carrinho seja
abandonado em A, o menor valor de h para que o carrinho efetue a trajetória completa é:
A'C\m
a) 3R/2
b) 5R/2
c) 2R
h
d) V(5gR)/2 R
e) 3R

Solução:
B Pela conservação da energia mecânica, quanto menor o valor de h menor será
a velocidade do carrinho ao alcançar o ponto de altura máxima B da trajetória
circular. Esta velocidade no ponto B deverá ser suficiente para a normal do
carrinho com a montanha russa seja nula em B. Neste caso o carrinho tangencia
suavemente a trajetória circular em B, retomando o contato logo depois e
completando o loop. Como a normal é nula em B a força resultante é a força
peso, que é uma resultante centrípeta (voltada para o centro da trajetória):
n r- mVR 2 r-,
P = Fcp => mg = —-=> v|=gR
IX

Note que, durante toda a trajetória, as forças que atuam no carrinho são peso, que é conservativa
e normal, cujo trabalho é nulo. Assim, existe a conservação da energia mecânica. Adotando
energia potencial nula no solo::
0 + /h=-^ + 5R
-7“ + x(ghA = + >(2R) h
2 2

2) (AFA-03) Um homem de dois metros de altura, com peso igual a 900N, preso por dos pés a
uma corda elástica, pula de uma ponte de 100 m de altura sobre um rio. Sendo a constante
elástica da corda equivalente a 300 N/m e seu comprimento igual a 72 m, pode-se afirmar que a
menor a distância entre a cabeça do homem e a superfície da água foi, em metros.
a) 0 b) 4 c) 6 d) 2
Solução: Alternativa B (MAIS PRÓXIMA)
O enunciado não deixa claro em que situação o homem pula a
ponte, se em pé ou sentado. Se o homem estiver em pé existe
1 m uma distância, de aproximadamente 1 m, entre seu centro de
massa e o ponto onde a corda está amarrada. Se o homem
pula sentado pode-se considerar desprezível a distância do
ponto onde está amarrada a corda e o centro de massa do
homem. Adotando o centro de massa do homem em um ponto
que está aproximadamente a um metro dos seus pés, temos,
L + Ax
pela conservação da energia mecânica:
,. , k.(Ax)2
m.g.(L + Ax + 2) = — => 900(74 + Ax) = 150(Ax)2 =>
(Ax)2 - 6Ax - 444 = 0 =>
1 m Ax = -18,28 (não convém) ou Ax = 24,3 m.
Portanto, a menor distância da cabeça à água é:
1 m
d = H - (L + Ax + 2) = 100 - (72 + 24,3 + 1) => d = 1,7m
d Comentário: A resposta d = 2 m seria encontrada (sem
T~ aproximações) somente se fosse considerado que o homem
estava sentado no momento do salto.

202
______ _________________________________________ Elementos da Física-Mecânica!-Energia Mecânica
3) (ITA-90) Uma pequena esfera penetra com velocidade v em um tubo oco, recurvado, colocado
num plano vertical, como mostra a figura, num local onde a aceleração da gravidade é g. Supondo
que a esfera percorra a região interior ao tubo sem atrito e acabe saindo horizontalmente pela
extremidade, pergunta-se: que distância, x, horizontal, ela percorrerá até tocar o solo ?

3R
2

a)X’1|T<K*"2R> b)x'Jv c) X = V ^3R2


d) x= —(v2+gR)
g
e) Outro valor.

Solução: Alternativa D
Enquanto a esfera está no interior do tubo não atuam sobre ela forças dissipativas. O solo será
adotado como referencial para energia potencial nula.
Se Ví é a velocidade com que a esfera abandona o tubo:
I) Eml = Em2 => mv2^ + mg2R = mVí2/2 + mg3R/2 => v4 = -^v2 +gR
Após o abandonar o tubo a esfera sofre um lançamento horizontal:
II) x = Vít => t = X/Ví

III) h = gt2/2 => 3R/2 = gx2/2v,2 => x2 = (3R/g)v!2 => X= (v2+gR)


g

4) (ITA-91) Uma haste rígida de peso desprezível e comprimento £, carrega uma massa 2m em
sua extremidade. Outra haste, idêntica suporta uma massa m em seu ponto médio e outra massa
m em sua extremidade. As hastes podem girar ao redor do ponto fixo A, conforme a figura. Qual a
velocidade horizontal mínima que deve ser comunicada às suas extremidades para que cada
haste deflita até atingir a horizontal ?
a) vi = s[gt e v2 = ^o.Sgí A B

tu
b) Vi = J2gí e V2 = ^O.Sgf
t
c) v, = Tgí e v2 = Is Ò m

t/2
d) v, = fegc e v2 = 72,4gí 2m è- m
V, V2
e) Nenhuma das anteriores.
Solução: Alternativa D
Em ambos os casos não atuam nos corpos forças dissipativas, ou seja, existe a conservação da
energia mecânica. O ponto mais baixo da trajetória será adotado como referencial para energia
potencial nula: EmOi = Emfi 2mv12/2 = 2mgl => v1 =
No 2o caso as velocidades angulares das duas massas são iguais: v2/í = v27(Z72) => v2’ = v2/2
Pela conservação da energia mecânica:
Pm02 ~ Emfê mv22/2 + mv2,2/2 = mgí + mg/72 5v22/4 = 3gí => v2 — ->/2,4gí

203
_________ _ ____________________________________ Elementos da Física-Mecânica!-Energia Mecânica
5) (ITA-78) Na figura ao lado, a mola é ideal; a situação (a) é a de equilíbrio estável do sistema
massa-mola e a situação (b) é a da mola em repouso. Abandonando-se o bloco “M" como indica a
situação (b), pode-se afirmar que a máxima velocidade que o bloco "M” atingirá dada por:

a) 72gd Ml

b) Vg(h + d)
c) 72g(h + d) liãll
d) V^h
e) 7g(2h + d) II
(o) (b)

Solução: Alternativa E
A situação apresentada em (a) é a de equilíbrio estático, ou seja, o ponto onde a força elástica
possui módulo igual à força peso.
Fe = P => k.d = m.g => k=-!^
d
Na situação (b), a partir do momento em que o corpo passa ater contato com a mola a força
resultante neste vale FR = P - Fe. Assim, pode-se separar o movimento do corpo em (b) em três
momentos:
i) enquanto FR > 0 a velocidade do corpo aumenta;
ii) no instante em que FR = 0;
iii) enquanto FR < 0 a velocidade do corpo diminui.
Desta forma, a máxima velocidade do corpo ocorrerá no instante em que P = Fe, exatamente a
situação descrita na situação (a). Igualando a energia mecânica no ponto em que o corpo é
abandonado e no ponto em que P = Fe:
myjf ,, .. mv2 ^0 kd2
+mg(h + d) = — + mgií + — ^g(h + d) = ZÍYÍ + zW
’ 2 / 2
2g(h + d) = v2 + gd => v2 = g(2h + d) - v = 7g(2h + d)

6) (ITA-09) A partir do repouso, um carrinho de montanha russa desliza de uma altura H = 20 73 m


sobre uma rampa de 60° de inclinação e corre 20m num trecho horizontal antes de chegar em um
loop circular, de pista sem atrito. Sabendo que o coeficiente de atrito da rampa e do plano
horizontal é 1/2, assinale o valor do raio máximo que pode ter esse loop para que o carrinho faça
todo o percurso sem perder o contato com a sua pista.
a) R = 85/3 m.
b) R = 4(73 - 1) m.
c) R = 8(73 - 1) m. H 2R
d) R = 4(2^3 -1)m.
e) R = 40(73 -1)/3m. 60ü,
4
20 m
Solução: Alternativa C

Condição para fazer o loop: Fcp = P => v2 = gR


Como a energia é dissipada somente pelo atrito (adote o solo para energia potencial nula):
1 | rrí\/2
WFat=Emf-Emo => -_rrígucos60°------------- jrígiiL = píg2R + —------- jrígH =>
sen60° 2
-^20V3^-^20 = ^-20V3 => R = 8(73-1)m
-XmH. cot 60° —jgfpL = #2R + 4r- =>

204
____ _____________________________________ Elementos âa Física-Mecânica!-Energia Mecânica
7) (ITA-84) Uma mola de massa desprezível tem constante elástica k e comprimento Lo quando
não esticada. A mola é suspensa verticalmente por uma das extremidades e na outra extremidade
é preso um corpo de massa m. Inicialmente o corpo é mantido em repouso numa posição tal que
a força exercida pela mola seja nula. Em seguida, a massa m é abandonada com velocidade nula.
Desprezando as forças dissipativas, o comprimento máximo (L) da mola é dado por:

jL
B
a) L = Lo + mg/k b) L = mg/k c) L = Lo + 2mg/k
d) L = 2mg/k e) L = 0,5 + Lo + mg/k
Solução:
O experimento do enunciado será utilizado para uma
análise completa do movimento de um corpo preso a
uma mola em trajetória vertical.
As figuras ao lado mostram três momentos do
movimento do corpo. A 1a figura representa o
L momento em que o corpo é abandonado, sem
ponto onde
F. = 0 e v = 0
velocidade inicial, em uma posição em que não há
AXi força na mola. Na 2a figura o corpo está em
ponto onde Fe = movimento descendente, em uma posição em que a
P e v é máxima
AX2
força elástica possui módulo igual ao da força peso.
ponto onde Fe é Sabe-se que a força resultante no corpo, em qualquer
máxima e v = 0
instante, é dada por FR = P + Fe. No movimento
descendente do corpo a força peso e a força elástica possuem sentidos contrários. Enquanto o
módulo da força elástica for menor que o módulo do peso a força resultante estará no sentido de
cima para baixo. Por outro lado, enquanto o módulo da força elástica for maior que o módulo do
peso a força resultante estará no sentido de baixo para cima.
Assim, desde a posição do corpo na 1a figura até a posição na 2a figura a velocidade do corpo
somente aumenta. Logo, na 2a figura temos a situação de máxima velocidade do corpo. Desde a
situação da 2a figura até a situação da 3a figura a velocidade do corpo diminui. Na 3a figura está
ilustrada a situação em que o corpo instantaneamente fica em repouso, para em seguida iniciar
seu movimento de subida. Neste ponto, a força elástica é máxima.
Como na 2a figura a força elástica possui módulo igual ao da força peso:
P = Fe => mg = kAx-i => Ax., = —
k
Devido à não existência de forças dissipativas, desde a situação da 1a figura até a situação da 3a
figura ocorre conservação da energia mecânica:
k(Ax1 + Ax2)/
mgjAíCr-HzHfT) = KlAXl + ZXX2^ => 2mg = kAx, + kAx2 => 2mg = mg + kAx2 => Ax, =
2 k
Deste modo, conclui-se que Ax! = Ax2, ou seja, o movimento do corpo é simétrico com relação ao
instante mostrado na 2a figura.
Assim, o comprimento máximo da mola é L = Lo + Axi + Ax2 = Lo + 2mg/k

205
_______ ___________________________________ fíementos tia física-Mecânica i-fnergia Mecânica
8) (ITA-83) Um pêndulo de comprimento t é abandonado na posição indicada na figura e quando
passa pelo ponto mais baixo de sua trajetória tangencia a superfície de um líquido, perdendo em
cada uma dessas passagens 30% da energia cinética que possui. Após uma oscilação completa,
qual será, aproximadamente, o ângulo que o fio do pêndulo fará com a vertical?
I—z---- 1
r—r
I /
I /

I ..x

a) 75° b) 60° c) 55° d) 45° e) 30°


Solução:
Se o corpo perde 30% de sua energia cinética em cada
I tangências então mantém 70% do valor. Em uma oscilação
j-e'' ' completa o corpo tangencia duas vezes o líquido. Assim:
ÍCOS 0 I
Ec2 = 0,7.Ec, = 0,7(0,7.Ec0) = 0,49.Ec0
Sabe-se que existe a conservação da energia mecânica
desde a posição inicial até o instante imediatamente anterior
à 1a tangência do líquido. A energia mecânica também se
conserva no instante imediatamente posterior à 2a tangência
no líquido até a posição em que o fio para instantaneamente.
O nível do líquido será adotado como padrão para energia potencial nula.
Ec2 = 0,49.Eco => Ep2 = 0,49.Epo => mg(í - to>s 0) = 0,49.mgí => 1 - cos 0 = 0,49 =>
cos 0 = 0,51 => 0 = 60°

9) (ITA-11) Um pêndulo, composto de uma massa M fixada na extremidade de um fio inextensível


de comprimento L, é solto de uma posição horizontal. Em dado momento do movimento circular, o
fio é interceptado por uma barra metálica de diâmetro desprezível, que se encontra a uma
distância x na vertical abaixo do ponto O. Em conseqüência, a massa M passa a se movimentar
num círculo de raio L - x, conforme mostra a figura. Determine a faixa de valores de x para os
quais a massa do pêndulo alcance o ponto mais alto deste novo círculo

Solução:
Como não atuam forças dissipativas em M então existe a conservação da energia mecânica
(adote o ponto mais baixo da trajetória para energia potencial nula):
MgL = M.g.2(L - x) + ^— => 2gL= 4g(L - x) + v2 => v2= 2gL - 4gL+4gx v2 = 4gx - 2g L
II) Condição para o fio não afrouxar:
v2 - Mv2min
T + Mg= 0 + Mg = V 2 = (L-x)g
vmin
(L-x) • L-x
III) Condição do problema:
vmin2 => 4gx - 2gL > Lg - gx => 5gx S 3gL => x > 0,6L
V > Vmin => v2 S Vmin

206
____ _____________________________________Elementos da física-Mecânica !-Energia Mecânica
10) (ITA-13) No interior de uma caixa de massa M, apoiada num piso horizontal, encontra-se
fixada uma mola de constante elástica k presa a um corpo de massa m, em equilíbrio na vertical.
Conforme a figura, este corpo também se encontra preso a um fio tracionado, de massa
desprezível, fixado à caixa, de modo que resulte uma deformação b da mola. Considere que a
mola e o fio se encontram no eixo vertical de simetria da caixa. Após o rompimento do fio, a caixa
vai perder contato com o piso se
M
a) b > (M + m) g/k.
b) b > (M + 2m) g/k. A-
c) b > (M - m) g/k.
d) b > (2M - m) g/k. I m |
e) b > (M - 2m) g/k.

Solução: Alternativa B
Mg
Para a caixa perder contato é necessária uma força elástica tal que: kAx = Mg => Ax
k
Pela conservação da energia mecânica:

mgâx +
= mgAx —2rng(bmin + Ax) = k(bmin + Ax)(bmin - Ax) =>

=> b > (M + 2m)g/k

11) (ITA-10) Um pequeno bloco desliza sobre uma rampa e logo em seguida por um “loop” circular
de raio R, onde há um rasgo de comprimento de arco 2R<p como ilustrado na figura. Sendo g a
aceleração da gravidade e desconsiderando qualquer atrito, obtenha a expressão para a altura
inicial em que o bloco deve ser solto de forma a vencer o rasgo e continuar em contato com o
restante da pista.

Solução:

Após o bloco perder contato com o loop o mesmo fica submetido


a um lançamento oblíquo. Pela expressão do alcance de um
lançamento oblíquo:

A = v°sen2(? => v*=-^-


g g coscp
Durante toda a trajetória não atuam forças dissipativas. Adotando
o solo para referencial de energia potencial nula, igualando a
energia mecânica no ponto onde o corpo é abandonado e no
ponto onde perde contato com o loop:
_jrígh = ^V° + X>g(R + Rcostp) =>
2
+ ^R(1 + cos<p) => h=R 1 + cosq> +
1
p 2 cos <p 2cos<p

207
_______ _______________________________ Mementos da física-Mecânica i-fnergia Mecânica
12) (ITA-07) Um corpo de massa m e velocidade v0 a uma altura h desliza sem atrito sobre uma
pista que termina em forma de semi-circunferência de raio r, conforme indicado na figura.
Determine a razão entre as coordenadas x e y do ponto P na semi-circunferência, onde o corpo
perde o contato com a pista. Considere a aceleração da gravidade g.
m H
v0

h p
y

O r X

Solução:
Conservação da energia mecânica:
mgh + mv02/2 = mgy + mvp2/2 => vp:2 = 2g(h - y) + v02
mv2 2gh + Vp
Em P a normal é zero: Fcp = Pcos 0 => —- = mg y => 2gh - 2gy + v02 = gy => y=
3g
2
X 3rg
Por Pitágoras: x2 = r2 - y2 -1
y 2gh + v^

13) (ITA-10) No plano inclinado, o corpo de massa m é preso a uma mola de constante elástica k,
sendo barrado à frente por um anteparo. Com a mola no seu comprimento natural, o anteparo, de
alguma forma, inicia seu movimento de descida com uma aceleração constante a. Durante parte
dessa descida, o anteparo mantém contato com o corpo, dele se separando somente após um
certo, tempo.Desconsiderando quaisquer atritos, podemos afirmar que a variação máxima do
comprimento da mola é dada por

k
'rne anteparo

___
a) [ m g sen a + m >/a(2gsena + a)] / k b) [ m g sen a + m ^a (2g cos a + a)] / k
c) [ m g sen a + m ^a(2gsena-a)] / k d) m (g sen a - a ) / k
e) mg sen a / k

Solução: Alternativa C

No instante em que o corpo de massa m perde contato com o anteparo tem-se:


, , m(gsena-a)
ma = mgsena - kAx, => Ax1 = ——---------- -
k
Como o corpo de massa m percorreu com aceleração constante a uma distância Ax,:
v?=2aAx1 => V2=.2ma(gsena-a)
111 k
kAx2
Pela conservação da energia mecânica: - mgAx^ena = -mgAx2sena =>
2
2m2a(gsena-a) m2(gsena-a)2 2m2g(gsena-a)sena , , 2
---------- ------ — +--------- --------- ----------- ———— -------= kAx2 - 2mgAx2sena =>
k2Ax2 - (2mgksena)Ax2 = m2 (gsena - a)(2a + gsena - a - 2gsena) =>

208
Elementos tia física-Mecânica!-Energia Mecânica
k2Ax2 - (2mgksena)Ax2 + m2(gsena - a)2 =0

2mgksena ± 74m2g2k2sen2a - 4k2m2(g2sen2a - 2agsena + a2)


Ax2
2k2
2mgksena ± - 4kZm?g2Sén2a + 8k2m2agsena - 4k2m2a2)
Ax2
2l<r~ ~
2mgksena ± -J8k2m2agseina - 4k2m2a2) _ /mg/sena ± XXmV2a9sena a2
Ax2 =
2i? /k/

mgsena ± m^/aPgsena - a)
Ax2
k
Uma vez que Ax2 > 0, o único sinal admissível é +: Ax2 = m9sena + mVa(29sena a)
k

14) (IME-82) A esfera de um pêndulo tem uma massa de 0,2 kg e é liberada do repouso na
posição mostrada. Sabe-se que o cabo se rompe com uma tração de 5,0 N. Determine o valor de
h para o ponto onde ocorrerá a ruptura. Dados: r = 0,75 m e g = 9,81 m/s2.
-«-------- r----------►
o
h

r/___ L
Solução:
Seja 9 o ângulo formado pelo fio com a horizontal no momento da ruptura,
mv^
Pela conservação da energia mecânica: mgh =—— => v2 = 2gh

mv2 _T h m2gh _T h
No momento da ruptura: Fcp =T-mgsen8 => -mg— -mg—
r r r r
T 3mgh 3.0,2.9,81.h
i =-------- =>
=> = ---------------------
o5 = => h = 0,637 m
r 0,75

15) (IME-84) Um cursor de dimensões desprezíveis e de massa m = 0,250 kg está livre ligado a
uma mola cuja constante é k = 150 N/m e cujo comprimento livre vale 100 mm. Se o cursor é
liberado a partir do repouso em A e se desloca ao longo da guia, sem atrito, determinar a
velocidade com a qual ele atinge o ponto B. Considere a figura contida no plano vertical.

i;
i:
I
I GOOmm
I
I
I
I
,1

B
400mm

209
Elementos da física-Mecânica!-Energia Mecânica
Solução:
kAx^ mvs , kAx| x2
Pela conservação da energia mecânica: A +mghA - .
2
150(0,5)2 +2.0,25.10.0,6 = 0,25vB2+150(0,3)2 => 37,5 + 3 = 0,25vB2 + 13,5 =>
0,25vb2 = 27 => vB = 10,39 m/s2

16) (IME-06) Uma partícula parte do repouso no ponto A e percorre toda a extensão da rampa
ABC, mostrada na figura abaixo. A equação que descreve a rampa entre os pontos A, de
coordenadas (0,h) e B, de coordenadas (h,0),é
y =----- 2x + h
h
enquanto entre os pontos B e C, de coordenadas (h,2r), a rampa é descrita por uma circunferência
de raio r com centro no ponto de coordenadas (h,r). Sabe -se que a altura h é a mínima
necessária para que a partícula abandone a rampa no ponto C e venha a colidir com ela em um
ponto entre A e B.
y
A
c
h
b2 r

I
i
i

0 *----------- h x
Determine o ponto de colisão da partícula com a rampa no sistema de coordenadas da figura
como função apenas do comprimento r. Dado: aceleração da gravidade = g.
OBS: despreze as forças de atrito e a resistência do ar.
Solução:
Para que a partícula abandone a rampa no ponto C, no mínimo, a normal deve ser igual a zero,
como a resultante das forças é a centrípeta, então:
2
P + N = fcp => m.g + 0 = => vc = gr
Conservando a energia nos pontos A e C:
2 5.r
Ema ^Mc => m.g.hA = m.g.hc + —y- => g.h = g.2r + => h = —
2 2
2x2 5r
5r
Assim, a equação da parábola fica igual a y =------- 2x + — .
5r 22 ’
As equações horárias da partícula valem x = ^ - e y = 2r - -^-.
2
2
5r-2x 2r 25r2 - 20rx + 4x2
Eliminando t nestas equações: y = 2r - g_____ r 8r
2L 2,/gr _
Igualando com a equação da parábola:
2x2 o 5r „ 25r2-20rx + 4x2 15±4j5
36x2- 180x+ 145^ = 0 => x= r, onde o
5r 2 8r 6

sinal + não é possível pois x < 5r/2, logo x


15-4-75 r
6
2
2 15-4^5
r -2 15-4V5 f+ 2
5r 8r
Substituindo na equação da parábola: y = — y=—
5r 6 6 9

210
Mementos da física-Mecânica i-ínergia Mecânica
17) (IME-15)

k
,1
-1
h

Uma mola comprimida por uma deformação x está em contato com um corpo de massa m, que se
encontra inicialmente em repouso no Ponto A da rampa circular. O corpo é liberado e inicia um
movimento sem atrito na rampa. Ao atingir o ponto B sob um ângulo 9 indicado na figura, o corpo
abandona a superfície da rampa. No ponto mais alto da trajetória, entra em contato com uma
superfície plana horizontal com coeficiente de atrito cinético p. Após deslocar-se por uma distância
d nesta superfície horizontal, o corpo atinge o repouso. Determine, em função dos parâmetros
mencionados:
a) a altura final do corpo Hf em relação ao solo;
b) a distância d percorrida ao longo da superfície plana horizontal.
Dados:
• aceleração da gravidade: g;
• constante elástica da mola: k;
• raio da rampa circular: h;
Solução:
a) Pelo princípio da conservação da energia mecânica entre os pontos A e B e usando o nível de
kx2 mv2
referência o ponto A: EmA = EmB => EPg + EPe = Ecb => m.g.L cos0 + 2 ~~2

kx^
2mgh cos0 + kx2 = mv2 => v2 = 2gh cos0 +----- (Eq. I)
m
Aplicando-se a equação da altura máxima para o lançamento oblíquo, temos
v2.sen2 0
HF = (h - h cos0) + AH => HF = h (1 - cos0) +
2g
r kx^ sen2 0
Usando v2 da eq. I, temos: HF = h (1 - cos0) + 2ghcos 0 +----- .
I m I 2-g

b) Aplicando-se o teorema da energia cinética: - WFAT = Erf - Ed => p.m.g.d = . v2

1 í kx2 hcos30 +
d =---- . 2ghcos0 +----- .cos20 => d = ———. cos20
2pg m 2mp.g

18) (IME-81) Uma esfera de massa M e raio r, desliza sem atrito, a partir do repouso sobre uma
superfície esférica de raio R. A esfera está inicialmente no topo da superfície esférica. Determine
o ângulo 0 que o vetor posição do centro da esfera em relação ao centro da superfície esférica,
forma com a vertical, no momento em que esfera abandona a superfície de deslizamento.
Solução:
mv2 . , „ n
N = 0 => Fcp = m.g.cos 0 => —— = m.g.cos0 => \r = g.R.cos 0

mv2 _
Em0 = Emf => m.g.R —— + m.g.R. cos 0
2
2,g.R = g.R.cos 0 + 2.g.R.cos 0 => 3.cos 0 = 2 => cos0 = —

211
_______________ Elementos da física - Mecânica /- Energia Mecânica
19) (ITA-15) Uma massa puntiforme é abandonada com impulso inicial desprezível do topo de um
hemisfério maciço em repouso sobre uma superfície horizontal. Ao deslocar-se da superfície do
hemisfério, a massa terá percorrido um ângulo 0 em relação à vertical. Este experimento é
realizado nas três condições seguintes, I, II e III, quando medidos os respectivos ângulos 0|, 0u e
©ui-
I. O hemisfério é mantido preso à superfície horizontal e não há atrito entre a massa e 0
hemisfério.
II. O hemisfério é mantido preso à superfície horizontal, mas há atrito entre a massa e o
hemisfério.
III. O hemisfério e a massa podem deslisar livremente pelas respectivas superfícies.
Nesta condições, pode-se afirmar que.
a) 0n < 0i e 0111 < 01-
b) 0ii < 0| e 0111 > 01.
c) 0|| > 0|. e 0111 < 01.
d) 0ii > 0|. e 0IU > 0|.
e) 0i = 0ni.

Solução: Alternativa C
mv, Q
I. N = 0 => Fcp = m.g.cos 0| —= m.g.cos 0| V|2 = g.R.cos 0|

i»-T Em0 = Emf => m.g.R


mV|2 n
—^- +m.g.R. cos 0|
a

2
2.g.R = g.R.cos 0| + 2.g.R.cos 0| => 3.cos 0, = 2 => cos0,
3
mvU A 2
I II. N = 0 => Fcp = m.g.cos 0N => —1L = m.g. cos 0N => v,i = g.R.cos Ou
R
p' mv^
WFat = Em( - Em0 WFat = —+ m.g.R. cos 0N - m.g.R =>

2WFat = m.g.R.cos 0M + 2.m.g.R.cos 0n - 2.m.g.R =>


2.m.g.R + WFal = 2 + WFat
cos O,,
3.m.g.R 3.m.g.R 3
2
Como WFat < 0 tem-se que cos0N < —. Como a função f(x) = cos x é decrescente para 0 < x < 90°

segue que 0n > 0|


III. Trabalhando com um referencial com a mesma velocidade do hemisfério
F, surge na massa uma força de inércia F,.
; zep
N = 0 => Fcp = m.g.cos 0in - F|.sen 0in => mvni _ m.g.cos0|n-F|.sen0m =>
R
..V|||2 _- FpR. sen0IN
g.R.cos 0in -
m
524 + m.g.R.cose,,, -■ 2 s R - n R cos n;!!-F|Rsene«
Emo=Emf => m.g.R = —111 + m.g.R.cos0II( => 2.g.R = g.R.cos 0m - + 2.g.R.cos ©ui =>

coso => cos%>|


3.g.R.cos0U| = 2.g.R + => COS0||, COS0|||
m ó omg o
Como a função f(x) = cos x é decrescente para 0 < x < 90° segue que 0m < 0,.

20) (ITA-02) Um pequeno camundongo de massa M corre num plano vertical no interior de um
cilindro de massa m e eixo horizontal. Suponha-se que o ratinho alcance a posição indicada na
figura imediatamente no início de sua corrida, nela permanecendo devido ao movimento giratório
de reação do cilindro, suposto ocorrer sem resistência de qualquer natureza. A energia
despendida pelo ratinho durante um intervalo de tempo T para se manter na mesma posição
enquanto corre é:

212
Elementos da física-Mecânica !-Energia Mecânica
.cilindro

J camundonga

*a)E = ^g2T2 2
b) E = M g2 T2 C) F
2m
Solução:
Para que
que o cil
módulo ig
Pelo princ
e sentidos
cilindro, acl
Como não há forças dissipativas, toda eJ
cilindro. Como cada Am de massa do cilinu

(que é de rotação) do cilindro é igual a E =

M2g2
m—
mv2 ma2T2 m
Como v = aT => E
2 2 2

21) Uma haste sem peso de comprimento í com


girar em torno da outra extremidade A. Inicialmente esta
tocando o cubo de massa M. Uma pequena perturbação faz com
horário. Para que valor de M/m a haste forma um ângulo de 60° c^.
separação dos corpos?

M
t
a!

Solução:
A velocidade do cubo é igual à componente horizontal da velocidade da esfera e a aceleração do
cubo é igual à componente horizontal da velocidade da esfera. Seja ã a aceleração da esfera.
Podemos escrever que ã = ãt + ãcp, onde ãt é a componente tangencial da aceleração e ãcp é a
componente centrípeta da aceleração. Assim, a componente horizontal da aceleração da esfera é:
v2
ah =at.sena~ —cosa.

Como o cubo possui este mesmo de aceleração, a força normal de contato com a esfera vale:
y2
N = M.ah = M.at.sena-M. —cosa

v2
No momento de separação dos corpos N = 0: at. sen a = — cos a

A componente at no momento de separação dos corpos é devido somente à gravidade:


a, = g.cos a.

213
___________ ____ ___________________________ Mementos da Física-Mecânica 1-Fnergia Mecânica
Portanto, a velocidade da esfera no momento de separação vale v = -JgZ sen a e a velocidade do
cubo neste momento é igual a u = v.sena = sena^/gísena .
m v2 M.v2.sen2g
De acordo com a conservação da energia mecânica: m.g.í = m.g.í.sena + —— +
2
„ . t., . . . , M 2-3.sena
Substituindo obtemos: — = 4.
m sen3 a
22) Dado o pêndulo abaixo, de comprimento 3d, sabe-se que A, B e C são estacas rígidas,
perpendiculares ao plano do papel. Quando soltamos o pêndulo, a partir do repouso, o fio irá se
chocar com a estaca B, retendo um seu pedaço e continuando a oscilar em torno do ponto B. 0
mesmo raciocínio repete-se com o ponto C. Determinar a velocidade do pêndulo quando o fio se
afrouxar.
A?___
/ \
3d
O
/ \
à/ \d

c
___3v d B
Solução:

A? 3d
O

\d

......
d B\

\ 2d

A figura ilustra o movimento do corpo preso ao fio, desde o momento em que é abandonado até o
instante em que o fio afrouxa. A condição para o fio afrouxar é que a tração seja igual a zero:
mv2 7 o
Fcp=P.cos0 => ------ = mgcos0 => \r = gdcos 0 => \r = gh
d
O nível de energia potencial nula será a reta BC. Pela conservação da energia mecânica entre o
instante inicial e o instante em que o fio afrouxa:
3^gd^p- = ^y- + XÍgh (x2) => /dV3 =/h + 2^h 3v2
dV3 = 3h => dV3
g
gdj3
V=
3

214
_______ ________________________________________ Elementos da física-Mecânica!-Energia Mecânica
23) A figura representa um trilho sem atrito, cuja parte em forma de circunferência possui raio R.
Qual a altura H necessária para se abandonar um corpo de modo que, não possuindo velocidade
suficiente para percorrer toda a circunferência, se desprenda do trilho e passe pelo centro da
circunferência.

'i
-R-
H

Solução:

yA x No instante em que o corpo perde contato com o trilho a


força normal é igual a zero:
mv^
jé *x
H
Fcp=P.cos0 => ------ = mgcos0 => v2 = gRcos0 (1)
R
Considere um sistema de eixos coordenados com
origem no ponto onde o corpo perde contato com o
trilho, conforme indicado na figura.
gx2
Pela equação da trajetória: y = x. tg 0 -
2v2 cos2 0
Para que o corpo passe pelo centro da circunferência então o ponto de coordenadas x = R.sen 0 e
gR/sen2 0
y = - R.cos 0 deve pertencer à equação da trajetória: -Xeos0 = X-sen0.tg0-
2v2 cos2 0
gRsen2 0 sen20 „ sen2 0 +cos2 0 1 gRsen2 0
—= 5— =--------- ++cos
COS00=
=----------------------- v2 (2)
2v cos 0 COS0 COS0 COS0 2cos0
Igualando as expressões (1) e (2):
gtfsen2© , , , >/3 2 VãqR
—--------- =<Xcos0 => tg2 0 = 2 => sec2 0 = 3 => cos0 = — => v2 =----------
2cos0 X a 3 3
Como não há forças dissipativas, a energia mecânica se conserva desde o instante inicial até o
instante em que o corpo perde contato com o trilho. Adotando o solo como referencial para
energia potencial nula:
Xv2 + Xgh (x2) 2/^R + R^
XgH = — 2H = 2R + V3R
2
H = |1 + ^R
2

215
fíementos da física-Mecânica i-ínergia Mecânica
Exercícios de Embasamento E3) (UEL-13) Considere a figura a seguir.
Despreze qualquer tipo de atrito.
E1) (UEM-14) Do terraço de um prédio, uma I
M L c
pessoa arremessa uma bola verticalmente •s 30’

para baixo. No instante em que a bola deixa a


mão da pessoa, a energia cinética da bola é de
a) O móvel de massa M = 1200 kg é
10 J e, em relação ao solo, a energia potencial
uniformemente acelerado (com aceleração a) a
gravitacional da bola é de 16 J. No instante em
partir do repouso em t = 0 segundos, atingindo
que a bola toca o solo, sua energia cinética é
B, em t = 10 segundos, com a velocidade de
de 20 J. Com base nessas informações,
108 km/h. Calcule a força resultante que atua
assinale o que for correto.
01) A energia mecânica total da bola no no móvel de A até B.
b) No ponto B, a aceleração a do móvel deixa
instante em que ela toca o solo é de 30 J.
de existir. Calcule a distância BC percorrida
02) A variação da energia potencial
pelo móvel, sabendo-se que ele alcança C no
gravitacional varia linearmente com a altura.
instante t = 15 segundos. Considerando g = 10
04) A quantidade de energia mecânica
m/s2, determine a energia mecânica total do
convertida em outras formas de energia,
móvel em C. Apresente os cálculos realizados
durante o movimento de queda da bola, é de 6
J. na resolução deste item.
08) No movimento de queda da bola, só atuam
E4) (UFES-12) Um bloco de massa 0,10 kg é
forças conservativas.
abandonado, a partir do repouso, de uma
16) Durante a queda da bola, o trabalho
altura h de 1,2 m em relação a uma mola ideal
realizado pela força peso sobre a bola é
positivo. de constante elástica 0,10 N/cm. Como é
mostrado na figura rotulada como “Depois”, ao
E2) (Mackenzie-14) Dois garotos brincam em lado, o bloco adere à mola após o choque. No
uma rampa de “skate”, conforme ilustra a desenho, A é o ponto de abandono do bloco, B
é o ponto de equilíbrio da mola, e C é o ponto
figura 1. Um desses garotos sai do repouso, do
onde há maior compressão da mola. Despreze
ponto A, em um certo instante, e o outro, do
ponto B, também do repouso, após um perdas de energia por atrito.
determinado intervalo de tempo. .EZJa
h

V.v. rt B

I I
H1 C

r-'
Antes Depois
A) Identifique, em um diagrama, as forças que
h
atuam no corpo, quando a deformação da
mola é máxima.
Figura 2 B) Determine a velocidade do bloco
Sabe-se, no entanto, que ocorreu um encontro imediatamente antes de se chocar com a mola.
entre ambos, no ponto C e que os dois C) Determine o trabalho realizado sobre o
percorreram suas respectivas trajetórias em bloco pela força gravitacional entre os pontos
um mesmo plano vertical, conforme ilustra a Ae B.
figura 2. Todas as forças de resistência ao D) Determine a deformação máxima sofrida
movimento são desprezíveis. Sabendo-se que pela mola.
a altura h mede 3,60 m e considerando-se g =
10 m/s2, a velocidade relativa de um garoto, E5) (Unesp-10) O Skycoaster é uma atração
em relação ao outro, no instante do encontro, existente em grandes parques de diversão,
tem módulo representado nas figuras a seguir. Considere
a) 12,0 km/h b) 21,6 km/h c) 24,0 km/h que em um desses brinquedos, três
d) 43,2 km/h e) 48,0 km/h aventureiros são presos a cabos de aço e
içados a grande altura. Os jovens, que se

216
Elementos da física - Mecânlcal- Energia Mecânica
movem juntos no brinquedo, têm massas movimento horizontal do atleta (corrida) em
iguais a 50 kg cada um. Depois de solto um movimento vertical, sem perdas ou acréscimos
dos cabos, passam a oscilar tal como um de energia. Na análise de um desses saltos, foi
pêndulo simples, atingindo uma altura máxima obtida a seqüência de imagens reproduzida ao
de 60 metros e chegando a uma altura mínima lado. Nesse caso, é possível estimar que a
do chão de apenas 2 metros. Nessas velocidade máxima atingida pelo atleta, antes
condições e desprezando a ação de forças de do salto, foi de, aproximadamente,
resistências, qual é, aproximadamente, a Altura máxima —•.
máxima velocidade, em m/s, dos participantes do centro de massa
durante essa oscilação e qual o valor da maior
3.2 m
energia cinética, em kJ, a que eles ficam centro de massa
submetidos? do atleta

0,8 m

Desconsidere os efeitos do trabalho


muscular após o início do salto.
a) 4m/s b) 6m/s c) 7m/s d) 8m/s e) 9m/s

E8) (Fuvest-11) Um esqueitista treina em uma


pista cujo perfil está representado na figura ao
lado. O trecho horizontal AB está a uma altura
h = 2,4 m em relação ao trecho, também
horizontal, CD. O esqueitista percorre a pista
no sentido de A para D. No trecho AB, ele está
com velocidade constante, de módulo v = 4
m/s; em seguida, desce a rampa BC, percorre
o trecho CD, o mais baixo da pista, e sobe a
E6) (Unicamp-06) Um brinquedo que muito outra rampa até atingir uma altura máxima H,
agrada às crianças são os lançadores de em relação a CD. A velocidade do esqueitista
objetos em uma pista. Considere que a mola no trecho CD e a altura máxima H são,
da figura abaixo possui uma constante elástica respectivamente, iguais a
k = 8000 N/m e massa desprezível.
Inicialmente, a mola está comprimida de 2,0
cm e, ao ser liberada, empurra um carrinho de A B
massa igual a 0,20 kg. O carrinho abandona a
mola quando esta atinge o seu comprimento h \
relaxado, e percorre uma pista que termina em
uma rampa. Considere que não há perda de
energia mecânica por atrito no movimento do Ç D
carrinho. NOTE E ADOTE
moU comprimida g = 10 m/s2

/ Desconsiderar:
— Efeitos dissipativos.

|»Q-_______ . carrinho
— Movimentos do esqueitista em relação ao esqueite.

a) 5 m/s e 2,4 m. b) 7 m/s e 2,4 m.


a) Qual é a velocidade do carrinho quando ele
c) 7 m/s e 3,2 m. d) 8 m/s e 2,4 m.
abandona a mola?
e) 8 m/s e 3,2 m.
b) Na subida da rampa, a que altura o carrinho
tem velocidade de 2,0m/s?
E9) (Fuvest-14) Em uma competição de salto
E7) (Fuvest-08) No "salto com vara”, um atleta em distância, um atleta de 70 kg tem,
imediatamente antes do salto, uma velocidade
corre segurando uma vara e, com perícia e
treino, consegue projetar seu corpo por cima na direção horizontal de módulo 10 m/s. Ao
saltar, o atleta usa seus músculos para
de uma barra. Para uma estimativa da altura
alcançada nesses saltos, é possível considerar empurrar o chão na direção vertical,
que a vara sirva apenas para converter o produzindo uma energia de 500 J, sendo 70%

217
fíementos da física-Mecânicai-ínergia Mecânica
Exercícios de Embasamento E3) (UEL-13) Considere a figura a seguir.
Despreze qualquer tipo de atrito.
E1) (UEM-14) Do terraço de um prédio, uma l
r m c
pessoa arremessa uma boia verticalmente _30’
* *Á
para baixo. No instante em que a bola deixa a
mão da pessoa, a energia cinética da bola é de
a) O móvel de massa M = 1200 kg é
10 J e, em relação ao solo, a energia potencial
uniformemente acelerado (com aceleração a) a
gravitacional da bola é de 16 J. No instante em
partir do repouso em t = 0 segundos, atingindo
que a bola toca o solo, sua energia cinética é
B, em t = 10 segundos, com a velocidade de
de 20 J. Com base nessas informações,
108 km/h. Calcule a força resultante que atua
assinale o que for correto.
no móvel de A até B.
01) A energia mecânica total da bola no
b) No ponto B, a aceleração a do móvel deixa
instante em que ela toca o solo é de 30 J.
de existir. Calcule a distância BC percorrida
02) A variação da energia potencial
pelo móvel, sabendo-se que ele alcança C no
gravitacional varia linearmente com a altura.
instante t = 15 segundos. Considerando g = 10
04) A quantidade de energia mecânica
m/s2, determine a energia mecânica total do
convertida em outras formas de energia,
móvel em C. Apresente os cálculos realizados
durante o movimento de queda da bola, é de 6
na resolução deste item.
J.
08) No movimento de queda da bola, só atuam
E4) (UFES-12) Um bloco de massa 0,10 kg é
forças conservativas.
abandonado, a partir do repouso, de uma
16) Durante a queda da bola, o trabalho
altura h de 1,2 m em relação a uma mola ideal
realizado pela força peso sobre a bola é
de constante elástica 0,10 N/cm. Como é
positivo.
mostrado na figura rotulada como “Depois”, ao
lado, o bloco adere à mola após o choque. No
E2) (Mackenzie-14) Dois garotos brincam em
desenho, A é o ponto de abandono do bloco, B
uma rampa de “skate”, conforme ilustra a
é o ponto de equilíbrio da mola, e C é o ponto
figura 1. Um desses garotos sai do repouso, do
onde há maior compressão da mola. Despreze
ponto A, em um certo instante, e o outro, do
perdas de energia por atrito.
ponto B, também do repouso, após um
determinado intervalo de tempo. . JZZL
h

Figura 1 i
B Depois
Anrcs
C A) Identifique, em um diagrama, as forças que
h
h/2 atuam no corpo, quando a deformação da
mola é máxima.
Figura 2 B) Determine a velocidade do bloco
Sabe-se, no entanto, que ocorreu um encontro imediatamente antes de se chocar com a mola.
entre ambos, no ponto C e que os dois C) Determine o trabalho realizado sobre o
percorreram suas respectivas trajetórias em bloco pela força gravitacional entre os pontos
um mesmo plano vertical, conforme ilustra a Ae B.
figura 2. Todas as forças de resistência ao D) Determine a deformação máxima sofrida
movimento são desprezíveis. Sabendo-se que pela mola.
a altura h mede 3,60 m e considerando-se g =
10 m/s2, a velocidade relativa de um garoto, E5) (Unesp-10) O Skycoaster é uma atração
em relação ao outro, no instante do encontro, existente em grandes parques de diversão,
tem módulo representado nas figuras a seguir. Considere
a) 12,0 km/h b) 21,6 km/h c) 24,0 km/h que em um desses brinquedos, três
d) 43,2 km/h e) 48,0 km/h aventureiros são presos a cabos de aço e
içados a grande altura. Os jovens, que se

216
Elementos âa Física-Mecânica !-Energia Mecânica
movem juntos no brinquedo, têm massas movimento horizontal do atleta (corrida) em
iguais a 50 kg cada um. Depois de solto um movimento vertical, sem perdas ou acréscimos
dos cabos, passam a oscilar tal como um de energia. Na análise de um desses saltos, foi
pêndulo simples, atingindo uma altura máxima obtida a sequência de imagens reproduzida ao
de 60 metros e chegando a uma altura mínima lado. Nesse caso, é possível estimar que a
do chão de apenas 2 metros. Nessas velocidade máxima atingida pelo atleta, antes
condições e desprezando a ação de forças de do salto, foi de, aproximadamente,
resistências, qual é, aproximadamente, a Altura máxima —•.
máxima velocidade, em m/s, dos participantes do centro de massa
durante essa oscilação e qual o valor da maior li
3,2 m
energia cinética, em kJ, a que eles ficam centro de massa
submetidos? do atleta

0,8 m

Desconsidere os efeitos do trabalho


muscular após o início do salto.
a) 4m/s b) 6m/s c) 7m/s d) 8m/s e) 9m/s

E8) (Fuvest-11) Um esqueitista treina em uma


pista cujo perfil está representado na figura ao
lado. O trecho horizontal AB está a uma altura
h = 2,4 m em relação ao trecho, também
horizontal, CD. O esqueitista percorre a pista
no sentido de A para D. No trecho AB, ele está
com velocidade constante, de módulo v = 4
m/s; em seguida, desce a rampa BC, percorre
o trecho CD, o mais baixo da pista, e sobe a
E6) (Unicamp-06) Um brinquedo que muito outra rampa até atingir uma altura máxima H,
agrada às crianças são os lançadores de em relação a CD. A velocidade do esqueitista
objetos em uma pista. Considere que a mola no trecho CD e a altura máxima H são,
da figura abaixo possui uma constante elástica respectivamente, iguais a
k = 8000 N/m e massa desprezível.
Inicialmente, a mola está comprimida de 2,0
cm e, ao ser liberada, empurra um carrinho de A B
massa igual a 0,20 kg. O carrinho abandona a
mola quando esta atinge o seu comprimento h
relaxado, e percorre uma pista que termina em
uma rampa. Considere que não há perda de
energia mecânica por atrito no movimento do Ç D
carrinho. NOTE E ADOTE
moli compnmidi / g - 10 m/s2
/ / Desconsiderar:
/ / — Efeitos dissipativos.
— Movimentos do esqueitista em relação ao esqueite.

a) Qual é a velocidade do carrinho quando ele a) 5 m/s e 2,4 m. b) 7 m/s e 2,4 m.


abandona a mola? c) 7 m/s e 3,2 m. d) 8 m/s e 2,4 m.
b) Na subida da rampa, a que altura o carrinho e) 8 m/s e 3,2 m.
tem velocidade de 2,0m/s?
E9) (Fuvest-14) Em uma competição de salto
E7) (Fuvest-08) No "salto com vara", um atleta em distância, um atleta de 70 kg tem,
corre segurando uma vara e, com perícia e imediatamente antes do salto, uma velocidade
treino, consegue projetar seu corpo por cima na direção horizontal de módulo 10 m/s. Ao
de uma barra. Para uma estimativa da altura saltar, o atleta usa seus músculos para
alcançada nesses saltos, é possível considerar empurrar o chão na direção vertical,
que a vara sirva apenas para converter o produzindo uma energia de 500 J, sendo 70%

217
Elementos da Física-Mecânica i-Energia Mecânica
desse valor na forma de energia cinética.
Imediatamente após se separar do chão, o E12) (UFPE-99) Um bloco de 5,0 kg desliza
módulo da velocidade do atleta é mais próximo num plano horizontal sem atrito com
de velocidade v = 5,0 m/s e choca-se
a) 10,0 m/s b) 10,5 m/s c) 12,2 m/s elasticamente contra uma mola de constante
d) 13,2 m/s e) 13,8 m/s elástica k = 500 N/m, como indicado na figura.
Calcule a compressão máxima sofrida pela
E10) (Fuvest-14) No desenvolvimento do mola, em cm.
sistema amortecedor de queda de um elevador
de massa m, o engenheiro projetista impõe
que a mola deve se contrair de um valor
máximo d, quando o elevador cai, a partir do
repouso, de uma altura h, como ilustrado na
figura ao lado. Para que a exigência do
projetista seja satisfeita, a mola a ser
empregada deve ter constante elástica dada
por ______ E13) (AFA-09) Uma partícula é abandonada de
uma determinada altura e percorre o trilho
m esquematizado na figura abaixo, sem perder
contato com ele.
v
\
h I
2r) 3R ;
i /

Considere que não há atrito entre a partícula e


o trilho, que a resistência do ar seja
desprezível e que a aceleração da gravidade
seja g. Nessas condições, a menor velocidade
possível da partícula ao terminar de executar o
Note e adote: terceiro looping é
forças dissipativas devem ser ignoradas; a) TW b) Tw c) TÜRg d) VW
a aceleração local da gravidade é g.
E14) (EN-11) Uma pequena esfera rígida de
2mg(h + d) 2mg(h-d) 2mgh
a) b) c) massa m é liberada do repouso da posição 1,
& d2 localizada a uma distância vertical H acima da
d)^ e)^ borda de uma cavidade hemisférica de raio R
d d (ver figura). A esfera cai e toca, tangenciando,
a superfície rugosa desta cavidade (posição 2)
E11) (Fuvest-15) Uma criança de 30 kg está com o dobro da velocidade com a qual deixa a
em repouso no topo de um escorregador plano mesma (posição 3), parando
de 2,5 m de altura, inclinado 30° em relação ao momentaneamente na altura h acima do plano
chão horizontal. Num certo instante, ela da borda (posição 4). Despreze a resistência

•rr
começa a deslizar e percorre todo o do ar. A razão H/h é igual a
escorregador. Determine
a) a energia cinética E e o módulo Q da
quantidade de movimento da criança, na
metade do percurso;
b) o módulo F da força de contato entre a
criança e o escorregador;
■ 1 '□i
c) o módulo a da aceleração da criança.
Note e adote:
Forças dissipativas devem ser ignoradas.
A aceleração local da gravidade é 10 m/s2.
sen30° = cos60° = 0,5
sen60° = cos30° = 0,9 a) 4/3 b) 3/2 c) 2 d) 3 e) 4

218
Elementos da Física-Mecânica!-Energia Mecânica
o aro numa posição P em uma barra horizontal
E15) (ITA-75) A variação da energia cinética fixa ao longo da qual o aro pode deslizar sem
de uma partícula em movimento, num dado atrito. Soltando o aro do ponto P, qual deve ser
referencial inercial, entre dois pontos distintos sua velocidade, em m/s, ao alcançar o ponto T,
P e Q é sempre igual a 2m de distância?
I. à variação da energia potencial entre esses T 2m P
dois pontos, a menos de sinal, quando a força
resultante aplicada à partícula por
conservativa. 2m
II. ao trabalho da resultante das forças
aplicadas à partícula para deslocá-la entre O
esses dois pontos.
III. à variação de energia potencial entre esses a) 7w b) c) ,/23,4

dois pontos, a menos de sinal, quando a força d) y/êãfi e) 7^2


resultante aplicada à partícula for não-
conservativa. E18) (ITA-09) Num filme de ficção, um foguete
A - somente I é correta de massa m segue uma estação espacial, dela
B -1 e II são corretas aproximando-se com aceleração relativa a.
C - somente III é correta Para reduzir o impacto do acoplamento, na
D - II e III são corretas estação existe uma mola de comprimento L e
E - somente II é correta. constante K. Calcule a deformação máxima
sofrida pela mola durante o acoplamento
E16) (ITA-83) Um cone, de altura h e raio da sabendo-se que o foguete alcançou a mesma
base igual a R é circundado por um trilho em velocidade da estação quando dela se
forma de parafuso, conforme a figura. Uma aproximou de uma certa distância d > L, por
partícula é colocada sobre o trilho, na vértice hipótese em sua mesma órbita.
do cone, deslizando, sem atrito, até a base.
Com que velocidade angular, em relação ao E19) (ITA-01) Um bloco com massa de 0,20
eixo do cone, ela deixa o trilho, no plano da kg, inicialmente em repouso, é derrubado de
base? uma altura de h = 1,20 m sobre uma mola cuja
constante de força é k = 19,6 N/m.
Desprezando a massa da mola, a distância
máxima que a mola será comprimida é
a) 0,24 m b) 0,32 m c) 0,48 m
h d) 0,54 m e) 0,60 m

E20) (ITA-88) Uma foca de 30 kg sobre um


trenó de 5 kg, com uma velocidade inicial de
4,0 m/s inicia a descida de uma montanha de
P 60 m de comprimento e 12 m de altura,
atingindo a parte mais baixa da montanha com
a velocidade de 10,0 m/s. A energia
h = 0,82 m mecânica que é transformada em calor será:
R = 0,20 m (Considere g = 10 m/s2)
g = 9,8 m/s2
(A ) 2 n rad / s
( B ) 4,0 rad / s *0*
( C ) 20 n rad / s í
( D ) Depende do número de voltas que ela oi
dá em torno do eixo do cone.
(E ) 20 rad / s
a) 8.400 J b) 4.200 J
4
E17) (ITA-08) Um aro de 1kg de massa c) 2.730 J d) 1.470 J
encontra-se preso a uma mola de massa e) Impossível de se determinar sem o
desprezível, constante elástica k = 10N/m e conhecimento do coeficiente de atrito cinético
comprimento inicial Lo = 1m quando não entre o trenó e a superfície da montanha.
distendida, afixada no ponto O. A figura mostra

219
Elementos Ha Física-Mecânica!-Energia Mecânica
E21) (IME-14) Um bloco, que se movia à
velocidade constante v em uma superfície
horizontal sem atrito, sobe em um plano
inclinado até atingir uma altura h,
permanecendo em seguida em equilíbrio 3
estável. Se a aceleração da gravidade local é
g, pode-se afirmar que
a) v2 = 2gh. b) v2 > 2gh. c) v2 < 2gh.
d) v2 = gh/2 e) v2 = 4gh.

E22) (OBF-02) Um corpo de massa 1kg parte


do repouso de um ponto A, a uma altura de J
5m, sobre uma rampa curva e com atrito
desprezível. No final da rampa tem um trecho fl ------ 1----------------------
horizontal BC com 9m de comprimento. Neste 12 3 4 x(m)
trecho, o coeficiente de atrito cinético entre o Na posição x = 3 m, a energia cinética dessa
corpo e a superfície é 0,2. No final deste trecho partícula é 2 J.
está uma mola de constante elástica 400 N/m. a) Qual é a energia total dessa partícula?
O corpo choca-se com a mola comprimindo-a b) Determine os valores de x para os quais a
de uma distância x. Durante a compressão não energia cinética é nula.
existe atrito entre o bloco e a superfície.
A 25) (OBF-08) Nos parques aquáticos existe um
brinquedo denominado “toboáguas”. Nele as
5m pessoas escorregam de certa altura e caem
B-. ►:C em uma piscina. Em um destes brinquedos,
9/7? que possui 21 m de altura, um garoto, no
a) Calcule o módulo da velocidade com que o interior de uma bóia plástica, escorrega do
corpo chega em B. ponto mais alto até sua parte mais baixa.
b) Calcule o módulo da velocidade com que o Considerando que o atrito entre as superfícies
corpo atinge a mola em C. do toboáguas e da bóia plástica realiza um
c) Calcule a compressão x da mola. trabalho igual a 500 J, e a massa do conjunto
(garoto + bóia) é igual a 50 kg, qual será a
E23) (OBF-02) Um homem, de massa 60 kg, velocidade do garoto no final do movimento de
encontra-se sobre uma balança, dentro de descida?
uma caixa que escorrega a partir do repouso,
da altura H, ao longo da pista esboçada na E26) (OBF-12) A energia potencial de uma
figura abaixo. Desprezando possíveis forças partícula de 50 g é mostrada no gráfico ao lado
de resistência, determine o valor de H de modo em função da sua posição.
que a balança acuse um valor de 54 kg para a (a) Estime os pontos de retorno da partícula
massa do homem quando a caixa dentro da quando a energia mecânica for 1 J.
qual ele se encontra passar pelo ponto mais (b) Determine a velocidade máxima da
alto da trajetória circular de raio R = 2 m. partícula quando a energia mecânica for 5 J.
U(x)(J)
r
32-
1—
0—
-1—
-2—
-3—
E24) (OBF-07) Uma partícula move-se ao
-4--
longo do eixo OX sob a ação de uma força de
-5-
módulo F, sendo sua energia potencial dada
-6-
pela curva representada na figura: -2 3j o T 2
x(m)

220
[tememos da física-Mecânica i-fnergia Mecânica
Exercícios de Fixação d) 0,4m e) 0,5m

F4) (Unifesp-10) Um dos brinquedos prediletos


F1) (UESP1-11) Uma bola de peso 1 N é solta
de crianças no verão é o toboágua. A emoção
do repouso de uma altura de 1 m acima do
do brinquedo está associada à grande
solo. A cada choque com o solo, a bola perde
velocidade atingida durante a descida, uma
20% da sua energia mecânica, em relação à
vez que o atrito pode ser desprezado devido à
que ela possuía no instante imediatamente
presença da água em todo o percurso do
anterior à colisão. O movimento da bola é
brinquedo, bem como à existência das curvas
vertical. Desprezando a resistência do ar, qual
fechadas na horizontal, de forma que a criança
a altura máxima atingida pela bola após a
percorra esses trechos encostada na parede
segunda colisão com o solo?
lateral (vertical) do toboágua. Sabendo que a
A) 48 cm B) 64 cm C) 72 cm
criança de 36 kg parte do repouso, de uma
D) 86 cm E) 92 cm
altura de 6,0 m acima da base do toboágua,
colocado à beira de uma piscina, calcule:
F2) (Mackenzie-05) Próximo à borda de uma Dado: g = 10,0m/s2
piscina, existe um escorregador, conforme
•L t f
ilustra a figura abaixo.

'p
r ■ „ 4

30'
2,40 m

0,350 m
I ■75
& '■ s; •% tí. I•
A 7- ’
& £•

Uma criança de massa 40,0kg sai do repouso


T
2,0 m
no ponto P do escorregador e, depois de um
certo tempo, atinge a superfície livre da água,
a qual está 35,0cm abaixo do nível da borda.
(www.pt.wikipedia.org/wiki/Toboágua)
Sabe-se que, em todo o trecho do
escorregador, a criança perdeu 25% da a) A força normal, na horizontal, exercida
energia mecânica que possuía em P; por isso, sobre a criança pela parede lateral do
ela atingirá a superfície livre da água num toboágua, no ponto indicado na figura (curva
ponto situado a: do toboágua situada a 2,0 m da sua base)
a) 19,0cm de A. b) 52,2cm de A. onde o raio de curvatura é igual a 80 cm.
c) 60,6cm de A. d) 69,0cm de A. b) A força dissipativa média exercida pela água
e) 102,2cm de A. da piscina, necessária para fazer a criança
parar ao atingir 1,5 m de profundidade,
F3) (UFES-03) Um bloco de massa m = 1 kg considerando que a criança entra na água da
desliza sobre uma superfície horizontal plana, piscina com velocidade, na vertical,
com uma velocidade inicial v = 5 m/s, em aproximadamente igual a 10,9 m/s,
direção a um anteparo preso a uma mola ideal desprezando-se, neste cálculo, a perda de
de constante elástica k = 20 N/m. A superfície energia mecânica no impacto da criança com a
horizontal é perfeitamente lisa, exceto num água da piscina.
trecho rugoso, cuja extensão é d = 1 m, e o
F5) (Unifesp-15) Uma pista de esqui para
coeficiente de atrito é p = 1,09. Ao atingir o
treinamento de principiantes foi projetada de
anteparo, o bloco comprime a mola, pára, e é
lançado de volta. No instante em que o bloco modo que, durante o trajeto, os esquiadores
pára, a mola está comprimida de não ficassem sujeitos a grandes acelerações
nem perdessem contato com nenhum ponto da
v pista. A figura representa o perfil de um trecho
H. dessa pista, no qual o ponto C é o ponto mais
alto de um pequeno trecho circular de raio de
« ►
curvatura igual a 10 m.
d
a) 0,1m b) 0,2m c) 0,3m

221
Mementos da Física-Mecânica i-fnergia Mecânica
b) Suponha que o Homem-aranha, em queda
livre, lance verticalmente um cabo de fios de
teia de aranha para interromper a sua queda.
Como ilustra a figura (2a), no momento em que
o cabo se prende, a velocidade de queda do
fora de escala
hA
Homem-aranha tem módulo Vo. No ponto de
30 m
altura mínima mostrado em (2b), o cabo de
teia atinge uma deformação máxima de AL =
Os esquiadores partem do repouso no ponto A 2,0m e o Homem-aranha tem, nesse instante,
e percorrem a pista sem receber nenhum velocidade V = 0. Sendo a constante elástica
empurrão, nem usam os bastões para alterar do cabo de teia de aranha, neste caso, k =
sua velocidade. Adote g = 10 m/s2 e despreze 7700N/m, calcule Vo.
o atrito e a resistência do ar.
a) Se um esquiador passar pelo ponto B da F7) (Unicamp-13) Em agosto de 2012, a NASA
pista com velocidade 10%/2 m/s, com que anunciou o pouso da sonda Curiosity na
superfície de Marte. A sonda, de massa m =
velocidade ele passará pelo ponto C?
1000kg, entrou na atmosfera marciana a uma
b) Qual a maior altura hA do ponto A, indicada
velocidade v0 = 6000m/s .
na figura, para que um esquiador não perca
a) A sonda atingiu o repouso, na superfície de
contato com a pista em nenhum ponto de seu
Marte, 7 minutos após a sua entrada na
percurso?
atmosfera. Calcule o módulo da força
resultante média de desaceleração da sonda
F6) (Unicamp-08) Nas cenas dos filmes e nas
durante sua descida.
ilustrações gráficas do Homem-aranha, a
b) Considere que, após a entrada na atmosfera
espessura do cabo de teia de aranha que seria
a uma altitude h0 = 125 km, a força de atrito
necessário para sustentá-lo é normalmente
reduziu a velocidade da sonda para v = 4000
exagerada. De fato, os fios de seda da teia de
m/s quando a altitude atingiu h = 100 km. A
aranha são materiais extremamente
partir da variação da energia mecânica, calcule
resistentes e elásticos. Para deformações AL
o trabalho realizado pela força de atrito neste
relativamente pequenas, um cabo feito de teia
trecho. Considere a aceleração da gravidade
de aranha pode ser aproximado por uma mola
de Marte, neste trecho, constante e igual a
de constante elástica k dada pela fórmula g Marte = 4 m/s2.
k=^1010^N/m,
onde L é o comprimento
F8) (Fuvest-06) Uma pista de skate, para
inicial e A a área da seção transversal do cabo. esporte radical, é montada a partir de duas
Para os cálculos abaixo, considere a massa do rampas Ri e R2, separadas entre A e B por
Homem aranha M = 70kg. uma distância D, com as alturas e ângulos
indicados na figura. A pista foi projetada de tal
forma que um skatista, ao descer a rampa Ri,
salta no ar, atingindo sua altura máxima no
ponto médio entre A e B, antes de alcançar a
rampa R2.

91 < \ R2 B A
|T
Rl/iqj ÍH0 = 8.0m
i?h y Mi
o0 = qn°
30° 1 □ 10 = 30°
'j
a) Determine o módulo da velocidade VA, em
m/s, com que o skatista atinge a extremidade
A da rampa Ri.
X''; v = o b) Determine a altura máxima H, em metros, a
(D partir do solo, que o skatista atinge, no ar,
(2a) (2b) entre os pontos A e B.
a) Calcule a área A da seçao transversal do c) Calcule qual deve ser a distância D, em
cabo de teia de aranha que suportaria o peso metros, entre os pontos A e B, para que o
do Homem-aranha com uma deformação de skatista atinja a rampa R2 em B, com
1,0 % do comprimento inicial do cabo. segurança.

222
Elementos da Física-Mecânica!-Energia Mecânica
b
F9) (UFPE-00) A figura mostra um bloco de
0,10kg inicialmente forçado contra uma mola
de constante elástica k=480N/m,
comprimindo-a de 10cm. Ao se soltar, o bloco k r\
desliza sobre uma superfície horizontal lisa, •xAOOÕOO-z*' /\ —
exceto no trecho AB, de 50cm, onde o B
coeficiente de atrito cinético é igual a 0,25. Em
seguida o bloco sobe uma rampa sem atrito,
retornando posteriormente à superfície
horizontal podendo atingir a mola. Quantas k k k
a) d. b) d c) 2d
vezes o bloco passará pelo ponto A antes de M + tn M-m M+m
parar completamente? 2k . d I Mmk
d) d. e) —
I7 M-m M llM + m

F12) (AFA-89) Um pêndulo simples, de mesma


A B massa m e comprimento L, executa oscilações
de amplitude angular 0, num local onde a
F10) (Unicamp-01) Que altura é possível atingir aceleração da gravidade é g. Determine a
em um salto com vara? Essa pergunta retorna tração no fio no ponto mais baixo.
sempre que ocorre um grande evento esportivo
como os jogos olímpicos do ano passado em a) mg
L
Sydney. No salto com vara, um atleta converte b) mg sen0
sua energia cinética obtida na corrida em energia c) mg (1 - 2cos0)
e \
potencial elástica (flexão da vara), que por sua d) mg (3 - 2cos0)
vez se converte em energia potencial
gravitacional. Imagine um atleta com massa de
80 kg que atinge uma velocidade horizontal de F13) (AFA-11) Duas esferinhas A e B, de
10 m/s no instante em que a vara começa a ser massas 2m e m, respectivamente, são
flexionada para o salto. lançadas com a mesma energia cinética do
a) Qual é a máxima variação possível da altura ponto P e seguem as trajetórias indicadas na
do centro de massa do atleta, supondo que, ao figura abaixo.
transpor a barra, sua velocidade é praticamente
nula?
\ TRAJETÓRIA
b) Considerando que o atleta inicia o salto em pé \ DE A
e ultrapassa a barra com o corpo na horizontal,
devemos somar a altura do centro de massa do
atleta à altura obtida no item anterior para
obtermos o limite de altura de um salto. Faça
uma estimativa desse limite para um atleta de 2,0
II TRAJETÓRIA \
DE B \
X
X
m de altura. X
c) Um atleta com os mesmos 2,0 m de altura e X

massa de 60 kg podería saltar mais alto? ZZZZZZZZZZZZZZZZZZZZZ^ZZZZZZZ^V

Justifique sua resposta. Sendo a aceleração da gravidade local


constante e a resistência do ar desprezível, é
F11) (Ufes-04) A figura abaixo mostra uma
mola de constante elástica k, comprimida a correto afirmar que a razão — entre as
vb
uma distância d de sua posição de equilíbrio.
Na extremidade livre da mola, é fixado o bloco velocidades das esferinhas A e B
A, de massa M. À frente do bloco A, encontra- imediatamente antes de atingir o solo é
se o bloco S, de massa m. Os blocos A e B a) igual a 1 c) maior que 2
estão em contato, porém não ligados. Após a b) maior que 1 d) menor que 1
mola ser liberada, o bloco B é lançado sobre o
plano horizontal. Considere-se que o atrito com F14) (EN-11) Um pequeno bloco de massa m
o plano é desprezível. A velocidade final do = 2,0 kg é lançado da posição A com
bloco B é de velocidade de módulo igual a 4,0 m/s. O trecho
ABC do percurso, no plano vertical, possui

223
Mementos da Física-Mecânica i-Fnergia Mecânica
atrito desprezível e o trecho CD, de
comprimento igual a 1,0 m, possui atrito cujo F16) (EN-14) Um pêndulo, composto de um fio
coeficiente cinético é 0,20.73. Despreze a ideal de comprimento L = 2,00 m e uma massa
resistência doar e considere a energia M = 20,0 kg, executa um movimento vertical de
potencial gravitacional zero no nível BC. Após tal forma que a massa M atinge uma altura
passar pela posição D, a máxima energia máxima de 0,400 m em relação ao seu nível
potencial gravitacional (em joules) atingida mais baixo. A força máxima, em newtons, que
pelo bloco é agirá no fio durante o movimento será
Dado: |g| = 10,0 m/s2 Dado: |g| = 10,0 m / sz
A v - a) 280 b) 140 c) 120 d) 80,0 e) 60,0

Fr
0.80 m
D
F17) (ITA-72) Um bloco de massa m = 3,0 kg
desce uma rampa, a partir do ponto P onde
L B X X C
30°
estava em repouso. De P até Q o atrito é nulo,
mas de Q a R a superfície oferece um
a) 14,0 b) 13,0 c)12,0 d) 11,0 e) 10,0 coeficiente de atrito cinético igual a 0,25. No
trajeto de Q a R o bloco encontra uma mola
F15) (EN-11) Um bloco é solto de certa altura horizontal de constante elástica k = 1,5 x 10 5
sobre uma mola ideal vertical que possui N/m. Nestas condições, os trabalhos
constante elástica K, como mostra a figura 1. realizados sobre o bloco pelas forças de
O bloco passa a ficar preso à mola (despreze gravidade, de atrito e da mola (T g, T,, T M),
as perdas nesta colisão) comprimindo-a a te até que o corpo chegue ao repouso
parar momentaneamente. A figura 2 mostra o comprimindo a mola, serão aproximadamente:
gráfico da Energia Cinética (Ec) do sistema /\m
mola - bloco em função da deformação da P

u
mola (y). Sabe-se que Ec é medida em joules e
y em metros. Analisando o gráfico, conclui-se 2, Om
que o valor da constante elástica K, em N/m, é
1 I
I-
! R
---- 4, Om
T. Ta T M joules
A) 60 30 30
§ B) -60 28 41

i
K C) 60 -30 26
D) 60 30 30
E) 60 - 30 -30
//////////
Figura 1 F18) (ITA-72) Na questão anterior a mola sofre
800 uma compressão de aproximadamente :

tf i-4. a) 0,40 m b) 1,0 cm c) 0,20 m


d) 2,0 cm e) 4,0 cm

F19) (ITA-72) Ainda na anterior, quando a


500
mola lança o bloco de volta, este sobe a rampa
a uma altura h’ aproximadamente igual a:
A) zero B) 8 cm C) 1,0 cm
300 D) 2,0 m E) 0,33 m

F20) (ITA-72) Uma partícula de massa m está


100

oL
i.. 7 f presa a uma mola de constante elástica k,
girando um círculo horizontal de raio R, com
o 0.6 1.0 1.5 2.0 2.5 3.0 velocidade angular constante. Para diferentes
Figura 2 Y(m> velocidades angulares da partícula (em
movimento sempre angular) a energia cinética
a) 200 b) 300 c) 400 d) 450 e) 500 desta (Ec) pode ser expressa, em função do

224
Elementos da Física-Mecânica!-Energia Mecânica
raio do círculo, pelo gráfico (Ro é o F24) (ITA-82) Sobre um plano inclinado de um
comprimento da mola não deformada): ângulo a sobre o horizonte fixa-se um trilho
ABCDE composto das porções: AB = DE = L
(na direção do declive do plano inclinado) e da
semi-circunferência BCD de raio R, à qual AB
e ED são tangentes. A partir de A lança-se
uma bolinha ao longo de AB, por dentro do
trilho. Desprezando todos os atritos e
resistências, podemos afirmar que a mínima
Ie velocidade inicial que permite que a bolinha
descreva toda a semi-circunferência BCD é:

o o

■« 'c

F21) (ITA-73) Uma pedra de massa igual a 50


gramas desliza, a partir do repouso, sobre um
telhado inclinado de 30° com a horizontal. a) 7(3R + 2í)gsena
Percorrendo uma distância de 5,0 m com um
b) 72gísena •
coeficiente de atrito cinético igual a 0,2, ela
chega à borda do telhado e inicia uma queda c) Qualquer velocidade inicial é suficiente
livre. Qual será sua energia cinética após ter d) A/3gR + 2gí
caído 2,0 m ? (suponha g = 10 m/s2) e) Nenhuma. É impossível que a bolinha faça esse
a) 0,82 J b)1,82J c) 2,25 J percurso.
d) 2.250 J e) Nenhuma das respostas acima.
F25) (ITA-88) Um pêndulo simples é
F22) (ITA-73) Na questão anterior, a dois constituído de um fio de comprimento L, ao
metros de queda livre, qual é a distância qual se prende um corpo de massa m. Porém,
aproximada da pedra à parede ? (Suponha a o fio nâo é suficiente resistente, suportando, no
parede na mesma linha da borda do telhado). máximo, uma tensão igual a 1,4 mg, sendo g a
a)0,2m b) 1,82 m c) 2,82 m aceleração da gravidade local. O pêndulo é
d 2,0 m e) Nenhuma das respostas acima. abandonado de uma posição em que o fio
forma um ângulo a com a vertical. Quando o
F23) (ITA-76) Abandona-se, com velocidade pêndulo atinge a posição vertical, rompe-se o
inicial nula, uma partícula de mas a m, no fio. Pode-se mostrar que:
interior de uma casca hemisférica, na posição a) cos a =1,0 b) cos a = 0,4
definida pelo ângulo a (ver figura). Supondo c) sen a = 0,8 d) sen a = 0,4
que nâo haja atrito, a força F que a casca e) cos a = 0,8
exerce sobre a partícula quando esta se
encontra no ponto mais baixo de sua trajetória, F26) (ITA-97) Um pequeno bloco, solto com
é dada por: velocidade nula a uma altura h, move-se sob o
efeito da gravidade e sem atrito sobre um trilho
em forma de dois quartos de circulo de raio R
que se tangenciam, como mostra a figura. A
mínima altura inicial h que acarreta a saída do
bloco, do trilho, após o ponto A é:
a) F = mg(2cos a - 1) b) F = mg(3 -2cos a)
c) F = mg(1 - 2cos a) d) F = 2mg(1 - cos a)
e) F = mg

225
fiementos da física-Mecânica i-fnergia Mecânica
a) 4 R/3. a) 3,0 x 103 N/m b) 4,5 x 103 N/m
c)7,5x103N/m d) 1,2 x 104 N/m
b) 5 R/4. A e) 3,0x104 N/m

c) 3 R/2. 2R F30) (ITA-86) Uma haste rígida de


comprimento “L” e massa desprezível é
h R
d) 5 R/3. suspensa por uma das extremidades de tal
1 1 maneira que a mesma possa oscilar sem atrito.
e) 2 R. Na outra extremidade da haste acha-se fixado
um bloco de massa m = 4,0 kg. A haste é
F27) (ITA-98) Considere uma partícula maciça abandonada no repouso, quando a mesma faz
que desce uma superfície côncava e sem o ângulo 0 = 60 0 com a vertical. Nestas
atrito, sob a influência da gravidade, como condições, a tensão | f| sobre a haste, quando
mostra a figura. Na direção do movimento da o bloco passa pela posição mais baixa, vale :
partícula, ocorre que: Obs: adotar para g s 10,0 m/s 2
m.4,o kg
x
i
i

a) a velocidade e a aceleração crescem.


cr i
i
i
b) a velocidade cresce e a aceleração
decresce. L
c) a velocidade decresce e a aceleração
cresce.
d) a velocidade e a aceleração decrescem.
e) a velocidade e a aceleração permanecem
constantes.
a) 40 N b) 80 N c)160N d)190N e)210N
F28) (ITA-98) Um 'bungee jumper’ de 2 m de
F31) (IME-08) Um bloco de massa m = 4 kg
altura e 100 kg de massa pula de uma ponte
parte de um plano horizontal sem atrito e sobe
usando uma 'bungee cord', de 18 m de
um plano inclinado com velocidade inicial de
comprimento quando não alongada, constante
Quando o bloco atinge a altura de 1 m, sua
elástica de 200 N/m e massa desprezível,
velocidade se anula; em seguida, o bloco
amarrada aos seus pés. Na sua descida, a
escorrega de volta, passando pela posição
partir da superfície da ponte, a corda atinge a
inicial. Admitindo que a aceleração da
extensão máxima sem que ele toque nas
gravidade seja igual a 10 m/s2 e que o atrito do
rochas embaixo. Das opções abaixo, a menor
plano inclinado produza a mesma perda de
distância entre a superfície da ponte e as
rochas é: energia mecânica no movimento de volta, a
velocidade do bloco, ao passar pela posição
a) 26 m b) 31 m c) 36m d) 41 m e) 46m.
inicial, é
a) 1 m/s b) 2 m/s c) 3 m/s
F29) (ITA-06) Um anel de peso 30N está preso
d) 4 m/s e) 5 m/s
a uma mola e desliza sem atrito num fio
circular situado num plano vertical, conforme
F32) (IME-92) A figura mostra um tubo de
mostrado na figura. Considerando que a mola
comprimento t e seção reta quadrangular,
não se deforma quando o anel se encontra na
posição P e que a velocidade do anel seja a constituído de um material extremamente leve
mesma nas posições P e Q, a constante (massa desprezível). O tubo é suspenso por
elástica da mola deve ser de uma articulação que o deixa livre para girar
num plano vertical. No fundo do tubo é
p: colocado um bloco de massa m. Determine a
velocidade horizontal v a ser dada inicialmente
2 cmJp- à extremidade inferior do tubo, para que o
anel bloco comece a deslizar em seu interior
10 cm exatamente no instante em que o ângulo
descrito pelo tubo for de 120°. Despreze todo e

226
Elementos tia Física - Mecânica /- Energia Mecânica
qualquer atrito.

1“
A B

I F36) (OBF-06) Um jovem de massa 100kg


fixado pelos tornozelos a um cabo elástico,
solta-se do parapeito de uma ponte (A) para
praticar "bungee jump". A superfície do rio
encontra-se 70m abaixo do parapeito da ponte.
O cabo elástico tem um comprimento não
deformado igual a 40m e uma constante
F33) (IME-10) Um bloco de 4 kg e velocidade
elástica igual a 300N/m.
inicial de 2 m/s percorre 70 cm em uma
superfície horizontal rugosa até atingir uma
mola de constante elástica 200 N/m. A
aceleração da gravidade é 10 m/s2 e o bloco
comprime 10 cm da mola até que sua
velocidade se anule. Admitindo que durante o
processo de compressão da mola o bloco
desliza sem atrito, o valor do coeficiente de
atrito da superfície rugosa é:
A) 0,15 B) 0,20 C) 0,25 D) 0,30 E) 0,35

F34) (OBF-04) Um bloco de massa m é


abandonado sobre o trilho e desliza, a partir do
ponto A, como representado na figura abaixo.
A a) Calcule o maior comprimento atingido pelo
"TV” C c

wd
cabo elástico.
b) Se a máxima aceleração desejada pelos
responsáveis pelo brinquedo é igual a 30m/s2
(3g) verifique se este valor é ultrapassado
calculando o valor da máxima aceleração a
que o jovem fica submetido.

8 F37) (OBF-06) O tubo cilíndrico representado


0 coeficiente de atrito cinético entre o trilho e o permite o movimento do corpo cilíndrico G com
bloco no trajeto AB é p. A seção circular que massa m = 2,50 kg e contém, presa à base,
se inicia no ponto B, não tem atrito. uma mola de constante elástica k = 0,120
a) Qual a menor velocidade que o bloco deve kN/m. A situação A mostra o corpo cilíndrico
ter no ponto B para que consiga passar pelo no exato instante em que ele é simplesmente
ponto C? abandonado e a situação B o instante em que,
b) Qual a altura hA para que isso ocorra? por efeito de sua queda, a mola é mostrada
sob deformação máxima possível. Existe
F35) (OBF-05) Um corpo de massa M igual a durante toda a queda uma força de atrito
2kg é abandonado de uma certa altura de um constante entre o móvel e a sua guia cilíndrica
plano inclinado e atinge uma mola ideal de de valor 5,00 N. Como a medida “a” vale 2,00
constante elástica igual a 900 N/m, m calcule:
deformando-a de 10 cm. Entre os pontos A e
B, separados 0,50 m, existe atrito cujo
coeficiente de atrito vale 0,10. As outras
regiões não possuem atrito. A que distância de
A o corpo M irá parar?

227
Elementos Ha Fisica-Mecânica!-Energia Mecânica
m

-G

a a h

b -G
Z1
a) Qual é a força que a superfície exerce sobre
o bloco na base (ponto A)? E no ponto B, onde
A B acaba a superfície?
a) a medida “b” em metros; b) A que distância do ponto A o bloco atinge a
b) a máxima velocidade alcançada pelo corpo superfície horizontal?
cilíndrico G durante a queda (deixe indicada a
raiz quadrada). F40) (QBF-08) Um bloco de massa m é
liberado do repouso sobre um plano inclinado
F38) (OBF-07) Um anel de massa m = 40 g de uma altura H. O bloco desliza sobre o plano
está preso a uma mola e desliza sem atrito ao com atrito desprezível até sua base, quando
longo de um fio circular de raio R = 10 cm, então desliza sobre uma superfície rugosa com
situado num plano vertical. A outra coeficiente de atrito cinético p, chocando-se
extremidade da mola é presa ao ponto P que com uma mola de constante elástica k,
se encontra a 2 cm do centro O da comprimindo-a de x e parando
circunferência. Calcule a constante elástica da momentaneamente; a mola em seguida se
mola para que a velocidade do anel seja a distende, arremessando o corpo de volta ao
mesma nos pontos B e D, sabendo que ela plano inclinado e esse sobe a uma altura h. A
não está deformada quando o anel estiver na distância percorrida pelo corpo sobre a
posição B. superfície rugosa até o momento do repouso
momentâneo é igual a d. Qual a expressão
que determina a altura h que o corpo sobe?
m

H
k

F41) (OBF-13) Um pequeno corpo de massa m


pode deslizar ao longo de uma superfície
horizontal de comprimento 3R (de A a B na
figura) e então ao longo de uma trajetória
circular de raio R. O coeficiente de atrito
cinético é 0,5 entre os pontos A e B e nulo ao
longo da circunferência. O bloco sai do
F39) (OBF-07) Um bloco de massa m = 0,1 kg repouso no ponto A com a mola comprimida.
desliza para baixo sobre uma superfície sem Qual deve ser a menor compressão da mola
atrito como mostra a figura. O bloco parte do para que o bloco percorra todo o círculo sem
repouso de uma altura h = 2,5 R acima da perda de contato?
base do loop circular, onde R = 40 cm é o raio
do loop. Considere 0 = 60°.
/E

A B

228
Elementos da Física-Mecânica i-Fnergia Mecânica
F42) Um bloco de massa m = 5 kg, deslizando vertical de raio r. Mostre que a diferença entre
sobre uma mesa horizontal, com coeficiente de o valor máximo da tensão Tmáx na corda e seu
atrito cinético e estático 0,5 e 0,6, valor mínimo Tmm é independente da
respectivamente, colide com uma mola de velocidade v0 da bola quando medido no ponto
massa desprezível, de constante de mola k = mais baixo da circunferência e determine Tmâx
250 N/m, inicialmente na posição relaxada. O Tmin.
bloco atinge a mola com velocidade de 1 m/s.
a) Qual a deformação máxima da mola? b) F47) Um corpo de massa m avança pela
Que acontece depois que a mola atinge sua superfície de uma cúpula hemisférica de raio
deformação máxima? c) Que fração da energia R. Considerando que a velocidade inicial do
inicial é dissipada pelo atrito nesse processo? corpo é v0 e este se encontrava à altura h0 da
base da cúpula no instante inicial, determinar a
F43) Um corpo de massa m = 300 g, enfiado força normal exercida pelo corpo sobre a
num aro circular de raio R = 1 m situado num cúpula quando este se encontra a uma altura h

a.
plano vertical, está preso por uma mola de da base da mesma.
constante elástica k = 200 N/m ao ponto C, no
topo do aro. Na posição relaxada da mola, o
corpo está em B, no ponto mais baixo do aro.

Se soltarmos o corpo em repouso a partir do
ponto A indicado na figura, com que F \l h
velocidade ele chegará a B?
Õ\

F48) O corpo de 10 N pode deslizar sem atrito


ao longo da barra horizontal. Sabendo que a
mola tem constante de 600 N/m e está
B' indeformada na posição ilustrada, determine a
velocidade v0 necessária para o corpo alcançar
F44) Um pequeno bloco é liberado em A com o ponto C.
velocidade nula e move-se ao longo da guia,
sem atrito, em direção ao ponto B, onde deixa
a guia com velocidade horizontal. Sabendo 0.125 m
que h = 2,40 m e b = 0,9 m, determine a) a
velocidade do bloco ao atingir o solo em C; b)
a distância correspondente c.

T~
b z"
I -'C
JI T
Exercícios de Aprofundamento
A1) (UESPI-12) As figuras A e B a seguir
mostram dois instantes do movimento
descendente de um bloco de massa 1 kg sobre
•<------------- c----------------- ►
um plano inclinado de 0 = 37° com a
horizontal. A mola indicada é ideal, com
F45) Com relação ao problema anterior,
constante elástica de 200 N/m. Na figura A, o
supondo que seja dada uma altura h, mostre
bloco tem velocidade de 4 m/s, e a mola está
que a velocidade em C é independente da comprimida de 5 cm. Na figura B, o bloco tem
altura b e determine a altura b para que a
velocidade de 2 m/s, e a mola está comprimida
distância c seja máxima (calcule também o
de 15 cm. Existe atrito entre o bloco e o plano
correspondente valor de c.)
inclinado. Considerando sen(37°) = 0,6 e
cos(37°) = 0,8 e a aceleração da gravidade 10
F46) Uma bola de massa m, presa a uma m/s*, qual é a energia dissipada pelo atrito
corda inextensível, gira numa circunferência

229
fíementos da física-Mecânica i-ínergia Mecânica
entre os instantes mostrados nas figuras A e vertical. Nesse ponto determine: (Considere g
B? = 10m/s2).
Figura A a) a velocidade da esfera;
b) a componente do peso da esfera na direção
g do fio (direção radial);
c) a força centrípeta sobre a esfera; e
d) a tração no fio.
1
L/2i n*Í60° L = 80 cm
X
Figura B
g
< I
60°'1 % A

C
X

A) 1,3 J B) 2,1 J C) 3,8 J B

D) 4,6 J E) 5,2 J
A4) (UFRJ-04) Uma bolinha de gude de
A2) (UFG-07) Uma bolinha de massa m é dimensões desprezíveis é abandonada, a
lançada, por uma mola horizontal de constante partir do repouso, na borda de um hemisfério
elástica k , em uma rampa lisa de ângulo de oco e passa a deslizar, sem atrito, em seu
inclinação 0 com a horizontal que possui no interior.
topo uma curva de raio R, conforme figura
abaixo. posição onde foi
abandonada a
bolinha

/ ••'r

<Z D

Q Calcule o ângulo 0 entre o vetor-posição da


Considere: bolinha em relação ao centro Cea vertical
Aceleração gra/oacíonal » £
para o qual a força resultante f sobre a
A bolinha move-se rente a uma parede lisa
bolinha é horizontal.
perpendicular à rampa e, ao fazer a curva,
passa por P, que se encontra a uma altura H
A5) (UFSC-99) Na figura abaixo, dois blocos
da base do plano, atingindo o ponto Q a uma
iguais de massa m trafegam, ambos, com
distância D da vertical que passa por P. velocidade V constante, num piso, onde os
Nessas condições, calcule:
atritos são pequenos e podem ser
a) A deformação da mola.
desprezados. A distância entre eles no nível
b) A força que a parede exerce sobre a bolinha inferior é d. Ao atingir o nível superior, a
no ponto mais alto da trajetória.
distância entre eles passa a ser d' e a
velocidade V'. Sabendo-se que o desnível
A3) (UMC-01) Uma esfera de 0,5 kg está
entre os pisos é h, pode-se afirmar que:
presa na extremidade de um fio de
comprimento 0,8 m e massa desprezível, e a i ------- 1 —

outra extremidade está presa ao teto de uma


sala. A esfera é abandonada no ponto A (veja
o desenho anexo) e ao passar por B, um pino h
P, preso perpendicularmente à parede e
situado a uma distância do teto igual à metade
do comprimento do fio, atua sobre este de 1. o valor de d1 não depende de h.
modo a mudar o raio da trajetória da esfera
conforme mostra o diagrama. Ao atingir o 2. V' = -Jv2 - 2gh
ponto C o fio faz um ângulo de 60° com a 3. V' = V - gh

230
Mementos da Física-Mecânica l-ínergla Mecânica

4.d' = L2-^!.
\ V2 .m
2,0 m
2ghd2 E
5. d' 'd2 -
V2
6. d' = d
7. d' = d - YÍ a) o módulo da velocidade ao passar pelo
ponto C;
2g b) a distância vertical, correspondente ao
desnível h;
A6) (AFA-12) Uma pequena esfera de massa c) a distância DE, sabendo-se que o bloco
m é mantida comprimindo uma mola ideal de atinge o final da rampa com velocidade nula no
constante elástica k de tal forma que a sua ponto E.
deformação vale x. Ao ser disparada, essa
esfera percorre a superfície horizontal até A8) (EN-12) O bloco uniforme de massa m =
passar pelo ponto A subindo por um plano 0,20 kg e altura H = 20 cm oscila comprimindo,
inclinado de 45° e, ao final dele, no ponto B, é alternadamente, duas molas dispostas
lançada, atingindo uma altura máxima H e verticalmente (ver a figura abaixo). Despreze
caindo no ponto C distante 3h do ponto A, os atritos. As molas, de constantes ki = 1,0.103
conforme figura abaixo. N/m e k2 = 2,0.103 N/m, possuem massas
desprezíveis e, quando não deformadas, tem
suas extremidades separadas pela distância d.
/B H Sabe-se que as molas sofrem a mesma
h compressão máxima h = 10 cm. No instante
(ijWMJ! A45° D em que o centro de massa C do bloco estiver
I----- 3/7—------------H
HCC eqüidistante das molas, a sua energia cinética,
Considerando a aceleração da gravidade igual em joules.é
age desprezando quaisquer formas de atrito,
pode-se afirmar que a deformação x é dada
por
a) |3m9h | b) 2^ h5
lõ k J mg
í5 mgHf2
dl
d
12 k J d) 3-----
l mg J

h
A7) (EN-04) Num plano inclinado com atrito
desprezível e inclinação de 60°, encontramos
um bloco de massa 2,0 kg apoiado sobre uma
mola ideal cuja constante elástica é igual a k =
200 N/m, comprimida de 40 cm a partir de sua Dado: |g| = 10m / s2
posição de relaxamento, e a seguir liberada. O a) 4,8 b) 5,0 c) 5,2 d) 7,3 e)7,5
bloco sobe o trecho da rampa inclinada BC
cuja extensão é de 60 cm atingindo a rampa A9) (ITA-56) Num plano vertical está colocado
DE, chegando ao ponto D com vetor um trilho com a forma da figura, sendo que, de
velocidade tangente à rampa. Sabendo-se que A até B tem-se uma semi-circunferência de
a distância horizontal CD vale 2,0 m, o raio r = 1,0 m. A partir da altura h = 1,2 m
coeficiente de atrito cinético entre o bloco e a abandona-se um pequeno corpo que desliza
rampa DE é igual a 0,8 e que g = 10,0 m/s2. sem atrito sobre o trilho, devido à ação da
Calcule: gravidade.
a) Quais as coordenadas do ponto P no qual a
massa abandona o trilho caindo depois
livremente?
b) Nesta queda livre, em que posição a massa
corta o eixo dos y?

231
Elementos da Física-Mecânica!-Energia Mecânica
* y A12) (ITA-05) Um objeto pontual de massa m
Bi desliza com velocidade inicial v , horizontal, do
topo de uma esfera em repouso, de raio R. Ao
h \p escorregar pela superfície, o objeto sofre uma
força de atrito de módulo constante dado por f
J. = 7 mg / 4n. Para que o objeto se desprenda
O.\ x da superfície esférica após percorrer um arco
de 60° (veja figura), sua velocidade inicial deve
iA^ ter o módulo de
a) 72gR/3 m V

b) 73gR/2
R 60”
A10) (ITA-99) Um pêndulo é constituído por c) /egR/2
uma partícula de massa m suspensa por um d) 37gR/2
fio de massa desprezível, flexível e
inextensível, de comprimento L. O pêndulo é e) 3^R
solto a partir do repouso, na posição A, e
desliza sem atrito ao longo de um plano de
inclinação a, como mostra a figura. Considere A13) (IME-10)
que o corpo abandona suavemente o plano no Siniacào n
ponto B, após percorrer uma distância d sobre
ele. A tração no fio, no instante em que o corpo h
deixa o plano, é:

m /
Qa
L
h/2

h/3 ~ ■
B

0
Na Situação I da figura, em equilíbrio estático,
a) mg (~) cos a b) mg cos a a Massa M, presa a molas idênticas, está a
uma altura h/3. Na Situação II, a mola inferior é
c) 3 mg (^) sen a d) mg (£) sen a subitamente retirada. As molas, em repouso,
têm comprimento h/2. O módulo da velocidade
e) 3 mg. da Massa M na iminência de tocar o solo na
Situação II é:
A11) (ITA-02) Uma massa é liberal a partir do Observação: g: Aceleração da Gravidade
repouso de uma altura h acima do nível do a) 4gh/[2x/2] b)3gh/[2>/2]
solo e desliza sem atrito em uma pista que
termina em um “loop” de raio r, conforme c) 2gh/[2>/2] d) gh / [2^2]
indicado na figura. Determine o ângulo 0 e)0
relativo à vertical e ao ponto em que a massa
perde o contato com a pista. Expresse sua A14) (IME-13)
resposta como função da altura h, do raio r e
da aceleração da gravidade g.
i

R
h>r

Um objeto puntiforme de massa m é lançado


do ponto A descrevendo inicialmente uma
trajetória circular de raio R, como mostrado na
figura acima. Ao passar pelo ponto P o módulo

232
Elementos da Física-Mecânica!-Energia Mecânica
da força resultante sobre o objeto é Jl7rng, suspensão do pêndulo. Abandona-se do
repouso o corpo na posição horizontal.
sendo g a aceleração da gravidade. A altura
Determine o maior valor de d para que o corpo
máxima hmax que o objeto atinge na rampa é:
descreva um círculo completo tendo o prego
a)3R b) (a/17 -1)R c)(VT7+1)R
como centro.
d) (VÍ7 + 2)R e) 18R

A15) Um corpo sem velocidade inicial desliza


sobre paredes lisas que passam por um fundo
3-L
xi I >
horizontal. A longitude do fundo é / = 2,0 m. O
coeficiente de atrito do corpo com o fundo é p. i
i y
= 0,3. A profundidade das paredes é H = 5 m. — T-"
A que distância do centro do sistema o corpo
irá parar?
A19) Quatro caixas pesando 250 N cada são

CO h—/—*i
colocadas, como ilustrado, na correia
transportadora que está desacoplada de seu
motor. A caixa 1 está mais à direita da parte
horizontal da correia. Se o sistema é liberado
do repouso, determine a velocidade da caixa 1
A16) Uma mola, de rigidez k e longitude /, está quando cai da correia no ponto A. Suponha
em posição vertical sobre uma mesa. Desde que os pesos da correia e dos cilindros são
uma altura H em relação ao nível da mesa pequenos em comparação com os pesos das
deixa-se cair uma bola de massa M. Qual a caixas.
velocidade máxima que terá a bola durante
seu movimento descendente. Despreza-se o
atrito.
.1,95 m.

4
.1,95 m.

n3 n2
.1.95 m ----------.1,95 m.

1 B
1
O "T
1.50 m

t/ A

l
VZZZZZZZZZZ; A20) Dois pontos materiais de massas M e m
estão unidos entre si por meio de um fio que
A17) Um arame é dobrado em forma de arco passa em A por uma polia fixa. A massa m
com raio R. No arame foi colocada uma conta, pende verticalmente; a maior M descansa
que pode movimentar-se ao longo do arame sobre um plano inclinado liso que forma um
sem atrito. No momento inicial a conta ângulo a com a vertical. M inicia seu
encontrava-se no ponto O. Qual velocidade movimento, deslizando ao longo do plano, sem
horizontal é necessário transmitir à conta, a fim velocidade inicial, partindo do ponto Bo situado
de que a mesma, percorrendo parte do trajeto na vertical de A. Demonstrar que o ponto M
no ar, caísse novamente no ponto B no efetua oscilações de amplitude
arame? 2m(M - m)h cos a
x = B0B , sendo h = B0A e
m2 - M2 cos2 a
cumprindo-se a condição: M.cos a< m < M.

h 2>
A18) O fio indicado na figura abaixo possui
comprimento / tem preso em uma extremidade a ;
um corpo de massa m. Um prego está
colocado a uma distância d abaixo do ponto de

233
Elementos da Física-Mecânica!-Energia Mecânica

A21) Um corpo é abandonado do repouso ■ oV.


sobre trilhos acoplados a uma circunferência, a
uma altura h = 2R do solo. Sendo R o raio da a c
h
circunferência, determine a máxima altura
alcançada pelo corpo, em relação ao solo,
depois que abandona os trilhos. P
m

R • m

A24) (OBF-01) Um corpo de massa m e


dimensões desprezíveis é solto, a partir do
repouso, do ponto A indicado na figura a
seguir. O corpo está amarrado a um fio ideal,
A22) Ao saltar de um trampolim, um esquiador inextensível e de massa desprezível, com
avança por uma rampa AB (coberta de neve) comprimento L, que encontra-se preso a um
que forma um ângulo a - 30° com o horizonte. pino. Sabe-se que o fio suporta sem partir uma
Antes do salto, ele supera uma pequena área tensão máxima igual a 2mg, onde g denota a
horizontal BC cujo comprimento não aceleração da gravidade. Despreze os efeitos
consideramos. No instante do salto, o de atrito e resistência do ar.
esportista faz um impulso com suas pernas
para adquirir uma componente vertical de F-*---------- --------
velocidade vy = 1 m/s. A altura da rampa é h =
9 m, o coeficiente de atrito entre os esquis e a
neve é p = 0,08 e a linha de aterrissagem CD
h
\ x
---- -CTb L
lg
forma um ângulo p = 45° com o horizonte.
Determine a distância t entre o ponto C - em
que o esportista se separa do trampolim - e o
ponto da linha CD em que este toca o solo. a) Calcule o desnível h entre os pontos A e B,
sabendo que o fio se parte no ponto B.
b) Determine as componentes horizontal e
vertical da velocidade do corpo no instante
em que ele atinge o solo.
B C
A25) (OBF-12) Uma caixa escorrega por uma
pista sem atrito. Quando chega no ponto A,
início do trecho curvo na forma de um arco de
P, D circunferência de raio R = 1,75 m, a sua
velocidade é 3 m/s. Determine a velocidade da
caixa quando estiver no ponto B. Dado cos 37°
A23) Um corpo de massa m foi pendurado no = 0,8 e sen 37° = 0,6.
ponto fixo O através de um fio de comprimento
£. No instante inicial, o fio forma um ângulo a
com a vertical e a velocidade do corpo é igual
53° / R..
a zero. Largando o corpo, o fio encontra um 1 53°
prego P de sentido perpendicular ao plano de
avanço do corpo, que está a uma distância h 37°
do teto e cujo ângulo que OP forma com a
vertical é p. a) Determinar o ângulo mínimo a
que faz com que o fio, após o encontro com o
prego, se enrosque neste, b) Determine c
também a mudança no valor da tração no fio
quando este bate no prego. A26) (OBF-01) Uma pequena esfera perfurada
de massa m pode deslizar, sem atrito, ao longo
de um aro de raio R mantido fixo no plano
vertical (ver figura). Suponha que a energia

234
Elementos da Física-Mecânica!-Energia Mecânica

A.
mecânica da esfera, medida em relação ao
ponto mais baixo do aro, vale 1,5mgR, onde g
é o módulo da aceleração da gravidade.
r
Despreze a resistência do ar.

a)_r_ b) C)-^_
' M+m 5M 2M + 3m
m
d)' —
M+m
e)'
2M + m
a) Calcule o módulo da aceleração tangencial
da esfera quando ela atinge o ponto mais alto
de sua trajetória. A29) Um pequeno corpo está localizado na
b) Calcule a força normal exercida pelo aro parte superior de uma esfera lisa, que está fixa
sobre a esfera quando ela atinge o ponto mais no solo. Devido a uma pequena perturbação, o
alto de sua trajetória (indique claramente o corpo desliza pela superfície da esfera até
módulo, a direção e o sentido). determinado ponto, para depois percorrer uma
trajetória parabólica no ar até colidir com o
A27) (OBF-04) Considere um trilho sem atrito solo.
na forma do arco 2tc - 28 de uma
circunferência de raio R, como representado
na figura.

R Sabendo-se que R = 1,5 m, determine o


alcance L do movimento do corpo. Suponha g
= 10 m/s2.
A a) 1,98 m b)1,63m c) 2,53 m
d) 2,19 m e) 2,87 m
Qual velocidade vo deve ter uma partícula que
pode deslizar livremente pelo trilho, a partir do A30) Uma haste rígida de comprimento 3L e
seu ponto mais baixo, para que ela salte de massa desprezível pode girar, no plano
uma extremidade à outra, entrando assim em vertical, em torno de uma rótula que está
movimento periódico? localizada a uma distância L do extremo
esquerdo da haste. Nos extremos da haste
A28) Um balanço infantil de massa M está estão localizados dois objetos de massas m e
apoiado em um solo horizontal. Uma criança é 2m, conforme a figura. A aceleração da
colocada na cadeirinha do balanço e o sistema gravidade é g. Em determinado momento o
é deslocado até a posição em que a corda do sistema é abandonado a partir da posição
balanço esteja horizontal e então abandonado horizontal.
com velocidade inicial nula. Suponha que a L 2L
massa do conjunto criança + cadeirinha seja O- -o—
rótula
o
2m
igual a m. Determine o valor mínimo do
coeficiente de atrito estático entre o solo e a
base do balanço de modo que não ocorra a) Determine as velocidades dos objetos
movimento relativo entre ambos durante todo o quando a haste se encontra na posição
movimento da criança. vertical.
b) Neste momento, qual é a força exercida
pela haste na rótula?
c) Qual a aceleração dos objetos no instante
imediatamente posterior ao dos objetos serem
abandonados na posição horizontal?

235
Elementos üa Física - Mecânica i-Energia Mecânica
relação ao elevador e obtemos o valor de
A31) Um corpo de massa m está ligado a uma 0,6 m/s2.
barra de massa desprezível e comprimento R, II. Colocamos o plano inclinado fora do
fixa no ponto O. O sistema é solto a partir do elevador (sobre uma mesa fixa) e deixamos
repouso como na figura. Determine a os corpos partirem do repouso e se
velocidade do corpo quando sua aceleração moverem durante 1 s.
tiver direção horizontal. Determine a variação de energia mecânica do
sistema na experiência II.
Dados: g = 10 m/s2 e cos 0 = 0,6.

0
O)<

d) 2,/Rg e) V2Ri

A32) Construiu-se um “estilingue” de molas A34) Uma corrente homogênea de 4 m de


conforme ilustra a figura. Na direção ABC, as comprimento é abandonada no interior de um
molas se encontram sem deformação, nos tubo liso que está dobrado em 60°, como
seus respectivos comprimentos naturais de 6,0 mostra a figura abaixo. O sistema está em uma
cm para cada mola. Uma pedra de massa m = região onde a aceleração da gravidade é g =
100 g é puxada ao longo de uma distância de 10 m/s2. Determine a velocidade da corrente
8,0 cm, sendo mantida na posição D do quando a mesma estiver totalmente na vertical.
espaço pela ação de uma força F, cuja direção
é perpendicular a ABC. As molas possuem
constantes elásticas de k = 4 N/cm. Se um I8
menino posiciona o “estilingue” de modo que a
pedra seja lançada formando um ângulo de
30° com a horizontal, qual a altura máxima, em
relação á posição D, atingida pela pedra, após
retirarmos a força F?
, =6C»,.
C

A35) Uma mola ideal é usada para fornecer


energia a um bloco de massa m, inicialmente
em repouso, o qual pode mover-se sem atrito
m = 100
gramas em toda superfície, exceto entre os pontos A e
B. Ao liberar o sistema massa-mola, o bloco
passa pelo ponto P com energia cinética de
1/20 da energia potencial gravitacional.
y

a) 7 cm b)11cm c)15cm d) 19 cm
e) impossível determinar H
A J
d
A33) Na figura, temos um plano inclinado de 0
em relação à horizontal. Os fios e polias são Considerando que o bloco pare no ponto B e
ideais e as massas de A e B são iguais a m. O que h = 0,15H e d = 3H, calcule a porcentagem
coeficiente de atrito entre os corpos e o plano da energia total dissipada pela força de atrito.
é o mesmo para A e B. Com o plano inclinado
são feitas duas experiências: A36) Na figura, um corpo de massa m, preso a
I. O plano é colocado em um elevador que tem um fio de comprimento L, é solto a partir do
aceleração para cima de 5 m/s2 em relação
repouso, da posição A, em um plano vertical.
à Terra. É medida a aceleração de B em
Há um prego P situado a uma distância 0,8 L

236
Elementos da Física - Mecânica !- Energia Mecânica
abaixo do ponto onde o fio é preso. O módulo 15 m/s. O cabo atinge o dobro de seu
da aceleração resultante sobre o corpo é o comprimento normal quando a pessoa atinge o
mesmo no ponto A e na posição mais baixa, ponto mais baixo de sua trajetória. Determine a
imediatamente antes do fio encostar no prego. constante elástica do cabo. Despreze a
resistência do ar e adote g = 10 m/s2.
A)256 N/m B)288 N/m
C)64 N/m D)72 N/m
E)128 N/m
0,8L
A39) Um bloco de massa m está se
deslocando sobre um plano horizontal com
velocidade Vo (ver figura). O trecho ABDC tem
J_ Pi raio de curvatura constante e centro de
curvatura em O. Desprezando dissipações de
0 corpo continua seu movimento circular para
energia, podemos afirmar que a força que o
a esquerda após o fio encostar no prego.
bloco aplica sobre o apoio, em D, vale:
Determine o valor de cos a, em que a é o (g é a gravidade local)
ângulo formado entre o fio e a vertical, na c
subida, no instante em que a direção da
aceleração do corpo é horizontal.
d ok^-i B
i )
A37) Um bloco muito fino, de massa m e de i /
comprimento igual a L, é colocado sobre um i m i---- >vo i
ZZZ/////Z A
plano inclinado de a com a horizontal,
conforme a figura. A superfície do plano é A)^+| ’v02 3
perfeitamente lisa entre os pontos Q e P. A B) m —+—-g
d 2 a
partir daí, a superfície é rugosa. O bloco é
homogêneo, de seção constante, mas o
polimento na parte AB é diferente do polimento D)
na parte BC, de modo que o coeficiente de
atrito entre o plano e a parte AB vale pi-i e entre
o plano e a parte BC vale p2. Após passar pelo
ponto P, o corpo percorre uma distância d até
MS-ll
encostar na mola, de constante elástica k. A40) (ITA-18) Na figura, presa a um fio de
Determine a velocidade máxima atingida pelo comprimento de 1,0 m, uma massa de 1,0 kg
bloco. gira com uma certa velocidade angular num
Dados: plano vertical sob a ação da gravidade, com
AB = BC = L/2, L = 1 m, d = 2 m, f = 2 m, pi = eixo de rotação a h = 6,0m do piso. Determine
0,6, ji2 = 0,4, sen a = 0,6, m = 10 kg, g = 10 a velocidade angular mínima dessa massa
m/s2 e k = 100 N/m. para a ruptura do fio que suporta no máximo a
tração de 46N, bem como a distância ao ponto
P do ponto em que, nesse caso, a massa
tocará o solo.

h
A38) Bungee jumping é um esporte radical,
muito conhecido hoje em dia, em que uma IHIIIIlíTllllllIlrtllllilllllllilHIilI
pessoa salta de uma grande altura, presa a um
cabo elástico. Considere o salto de uma
pessoa de 72 kg. A velocidade máxima
atingida pela pessoa durante a queda é de

237
IMPULSO

Considere uma partícula de massa m sobre a qual atua uma força resultante F. A 2a Lei de
— dp
Newton pode ser expressa da seguinte forma F= , onde p = mv. Desenvolvendo esta
dt '
expressão tem-se:

«2 *2 _
p= dp jFdt = m j dv j Fdt = mv2 -mv,
Fdt = d(mv) =>
dt
‘1 v.

*2 _
Sabe-se que mv é a quantidade de movimento da partícula. A integral jFdt é
ti

conhecida como o impulso da força F, calculada no intervalo de tempo que vai de t, até t2, sendo
•2 _
representada por T = j Fdt. Caso a força F seja constante pode-se escrever esta integral na
<1

forma T = F.At. Nas unidades do SI, a magnitude do impulso de uma força é expressa em N.s.
Pela definição de Newton, tem-se que N.s = (kg.m/s2).s = kg.m/s
Perceba que o impulso é uma grandeza vetorial, que possui mesma direção e sentido da
força F. Decompondo F em coordenadas retangulares:

*2 *2 l2 l2
T= j(Fxi+Fyj+Fzk)dt => T = TjFxdt+ j J Fy dt + k J Fzdt => T = lxT + ly j + lzk
*1 t, t, t.

Sabe-se que toda definição física representada por um integral possui uma interpretação
gráfica. Deste modo, em um gráfico F x t, o módulo do impulso, entre os instantes t, e t2 em cada
uma das direções x, y e z, é igual a área compreendida entre a linha do gráfico e o eixo do tempo.
A figura abaixo mostra a variação temporal das componentes Fx, Fy e Fz de uma força F. Do
exposto anteriormente segue que A1 = |lx|, A2 = |ly| e A3 = |lz|.

‘■'J P-.

A,
A2
O 'l l2 t O '1 l2 t o 'l '2 t

A equação mv1 +T = mv2 demonstra que quando sobre uma partícula atua uma força F
durante um intervalo dado, a quantidade de movimento final mv2 da partícula pode ser calculada
pela soma vetorial de sua quantidade de movimento inicial mv1 e o impulso da força F durante 0
intervalo de tempo considerado. Como a quantidade de movimento e o impulso são grandezas
vetoriais, determinam-se três equações, cada uma em um eixo:

mvx1+lx mvx2 mvy1 + ly =mvy2 mvz1 + lz = mvz2

238
________ _________________________ Mementos da Física-Mecânica !-Impulso e Quantidade de Movimento
Os valores de lx, ly ou l2 podem ser negativos. Para tanto basta que a respectiva
componente da força esteja no sentido dos valores negativos do eixo. Por exemplo, suponha uma
força constante F de módulo 20 N está atuando que um corpo de massa m = 10 kg, que se move
para a esquerda em uma superfície horizontal.

F
10 kg
+ —►
X

Adotando um eixo unidimensional x paralelo à força F e com sentido oposto ao desta


força, tem-se que a força F medida nesta referencial x vale F = -20i, ou seja, Fx = - 20 N, Fy = 0
e Fz = 0. Desta forma, o impulso exercido pela força F durante 3 segundo vale:

I = F.At = (-207).3 = -607 N.s => lx = - 60 N.s

O gráfico de Fx pelo tempo fica da seguinte forma:


“Fx(N)
t (s)
*

-20

FORÇA MÉDIA

Suponha que o módulo de uma força F, que atua sobre um corpo de massa m, varia com
o tempo de acordo com o gráfico da figura 1 abaixo.

F“ F4

Fm
Ai
A2

> *
t t
Figura 1 Figura 2

O módulo do impulso da força F é igual à área compreendida entre a linha do gráfico e o


eixo do tempo, ou seja, na figura 1 tem-se | T, |= A1. Considere agora o mesmo corpo de massa m
sendo submetido a uma força constante de módulo Fm durante o mesmo intervalo de tempo At,
provocando um impulso 72 conforme indicado na figura 2. Sabe-se que | |= A2. A força Fm será
considerada a força média do experimento 1 quando A, = A2, ou seja, quando os módulos dos
impulsos das duas forças forem iguais no mesmo intervalo de tempo.

239
fiementos da física-Mecânica !-Impulso e Quantidade de Movimento
CENTRO DE MASSA

Sejam m-i e m2 duas partículas separadas por uma distância d, conforme ilustrado na figura
seguinte. Adota-se um sistema unidimensional x com origem coincidindo com m, e que passa por
m2. Seja Xj a posição da partícula i, para i = 1 e 2. Neste sistema tem-se Xt = 0 e x2 = d.
XCM

-►
CM ^m2 x
d ----- >

O centro de massa CM do sistema formado pelas partículas de massas m, e m2 é definido


como o ponto do eixo Ox de coordenada xCM = XllTl1 + X2ÍTl2 . Substituindo x, = 0ex2 = d segue
m, +m2
m2
que xCM - d. Analisando a expressão, pode-se concluir que se m-, > m2 o CM estará mais
m, +m2
próximo da partícula 1, enquanto que se m, < m2 o CM estará mais próximo da partícula 2. Por
outro lado, perceba que se as massas forem iguais tem-se xCM , fazendo com que o centro de
massa do sistema coincida com o ponto médio do segmento que une as partículas.

Suponha agora que n partículas m,, m2, ..., mn estão distribuídas no espaço. Estas
partículas são consideradas pontos materiais, implicando que suas dimensões são desprezíveis
em comparação com outras distâncias relativas ao experimento como, por exemplo, as distâncias
entre as partículas. Um sistema de referência tridimensional inercial Oxyz é adotado para medir a
posição das n partículas. Deste modo, o vetor posição r, mede a posição da partícula de massa
mi, para 1 < i < n, neste sistema.

rri2
f2
y
-►

O vetor posição que define o centro de massa (CM) do sistema formado pelas n partículas
é dado por rCM = —m2[2 + ■ ■ ■ + mnrn Assjmi as coordenadas do centro de massa, no sistema
m, +m2 + ... + mn
Oxyz, são dadas por:

+m2x2 +... + mnxn m1y1+m2y2+... + mnyn m,z1 +m2z2 +...-t-mnzri


XCM - VCM ZCM “
m, +m2 + ... + mn m, +m2 + ... + mn m, +m2 + ... + mn

Utilizando a linguagem de vetores pode-se escrever que rCM = xCMi + yCM] + zCMk .

240
_______ _____________________ Elementos da física - Mecânica /- Impulso e Quantidade de Movimento
Como 0 vetor velocidade é obtido derivando no tempo o vetor posição, a velocidade do
centro de massa é dada por vCM = . Deste modo, segue que:
dt

m1v1+m2v2+... + mnvn
Vcm
m, +m2 + ... + mn

Derivando novamente no tempo obtém-se a aceleração do centro de massa:

m^ + m2ã2 +... + mnãn


ãCM - m, +m2 + ... + mn

Note que a determinação do CM depende do sistema de eixos cartesianos adotado.


Quando 0 sistema de eixos não é fornecido, é necessário escolher um e essa escolha deve ser
feita de modo a simplificar ao máximo as contas. Uma estratégia bastante usual é colocar a
origem do sistema de eixos sobre um das partículas. Por exemplo, considere dois corpos no
espaço, de massas mi e m2, a uma distância L um dos outro. Colocando a origem do sistema
sobre 0 corpo de massa m,, pode-se trabalhar com apenas um eixo.

mi ITI2 X
—►
0 L

m,x, + m2x2 m,.0 + m2.L m; L


XCM
m, + m2 m, + m2 m, +m2

Perceba que se m-, = m2 o CM coincide com o ponto médio do segmento que liga os dois
corpos, resultado já esperado.
Suponha agora que os corpos A, B, C e D estão localizados em um plano conforme
indicado na figura. As massas de cada um também estão indicadas na figura. Determine o centro
de massa do sistema.

. k y (cm)

4
, 2m ]

3 •--?*>m
1

1
| 2m
A,
—►
3m 1 2 3 4 x (cm)

x mAxA + mBxB + mcxc + mDxD 3m.O + 2m. 1 + 2m.3 + m.4 12^


= 1,5 cm
mA + mB + mc + mD 3m + 2m + 2m + m 8_ní
_mAyA+mByB+mcyc+moyD 3m.0 + 2m.4 + 2m.1 + m.3 13ja<
jcm — — —-------------------------------------------------- —-------- — i, uzo cm
mA + mB + mc + mD 3m + 2m + 2m + m 8>n

241
F/ementos da Física-Mecânica!-Impulso e Quantidade de Movimento
CORPO RÍGIDO

Todo corpo rígido pode ser interpretado como um conjunto muito grande de partículas que
+ m2r2 + ... + mnf„
estão extremamente próximas. Assim, teoricamente, a expressão fCM =
m, +m2 + ... + mn
ainda é válida para o cálculo do vetor posição do centro de massa de um corpo rígido, porém,
devido ao número muito grande partículas, sua aplicação é muito complicada. Como as partículas
estão muito próximas, pode-se admitir uma distribuição contínua de massa. As partículas tornam-
se infinitesimais de massa dm e as somas que resultam nas coordenadas do CM se tornam
integrais, a saber:

xCM=^Jxdm, yCM=lfydm, zCM=ljzdm

Caso o corpo seja homogêneo, a massa é distribuída de forma uniforme ao longo de todo
o volume. Assim, pode-se afirmar que a densidade p do corpo é dada por p = —= —. Assim, as
V dv
expressões das coordenadas do CM de um corpo rígido ficam da forma:

xCM=-JjxdV, yCM=4fydV’ zcM=^fzdV

Quando o corpo possuir uma das dimensões muito menor que as outras duas e for plano,
ou seja, possuir formato de uma chapa, as expressões do centro de massa ficam da forma:

XCM ifxdS. Ycm ijyds


Como aplicação prática, vamos determinar o CM de uma placa em formato de triângulo
retângulo. Como deseja-se determinar coordenadas, é necessário designar um sistema de eixos,
que será adotado como ilustrado na figura, com o eixo x sobre um dos catetos e a origem
coincidindo com um dos vértices.
y “ Considere um ponto P(x, y) ao longo da
hipotenusa do triângulo. Por semelhança segue que
b ■
— = —, ou seja, y = x—. Além disso, dS = y.dx:
x a a
y ***Tí 2 íax2dx =
íi
::
x“-ifxdS-êíxydx‘sWdx a2 Jo
a
*X 2 ax3 2 ( a3 2a
dx -—0 = —
a2 o 3 a2 3 3
y “

b ■ Observando o eixo y, tem-se que dS = (a — x)dy


2 r ( a V
dy*: y“4íyds-ê jy(a-x)dy

=-r
b
a 2 L 2 ay2 ay3
ay--y dy
* abJo b ) ab o ~2 3b~
X a X

2 f ab2 ab2 2 ab2 b


ab 2 3 ab 6 ~3
Na verdade, pode-se demonstrar que o CM de todo triângulo homogêneo (não apenas do
triângulo retângulo) se encontra sobre seu baricentro.

242
_____________________________ Elementos da Física - Mecânica !-impulso e Quantidade de Movimento
Em determinados corpos rígidos não é necessário fazer contas para identificar o centro de
massa. Se um corpo rígido apresenta um eixo de simetria, o CM desse corpo está contido sobre
esse eixo de simetria. Além disso, se um corpo apresenta dois eixos de simetria, o centro de
massa do mesmo encontra-se na interseção desses eixos. Desta forma, caso o corpo possua uma
geometria simétrica, seu centro de massa coincide com seu centro geométrico. Por exemplo, o
centro de massa de uma chapa retangular está localizado sobre o encontro das diagonais. Do
mesmo modo, o centro de massa de uma tetraedro regular coincide com o centróide da pirâmide.
Nas figuras abaixo estão indicados os centros de massas de alguns corpos simétricos.

» /
• CM
* / '

Circulo Triângulo Equilátero Paralelogramo Octógono Regular


CM = centro do CM = baricentro = CM = encontro das diagonais CM = encontro das
circulo incentro = ortocentro diagonais maiores

""íAcm.......

Esfera Paralelepípedo Tetraedro Regular Octaedro Regular


CM = centro da CM = encontro das diagonais CM = encontro das alturas, CM = ponto médio da altura
esfera maiores fica h/4 de cada face

Em corpos rígidos não simétricos é possível determinar o CM dividindo o corpo original em


corpos menores, cujos centros de massa sejam conhecidos. Por exemplo, pode-se calcular o CM
de um trapézio dividindo-o em um retângulo e dois triângulos.

ÁLy

0 a b c x

Os vértices e as coordenadas dos centros de massa de cada parte em que ficou dividido o
trapézio original são:
0 + a + a 2a 0+0+h h
Vértices do triângulo 1: (0, 0), (a, 0) e (a, h) => xCM1 - o e y cmi -
3 o 3 3
_ Q a+b h
Vértices do retângulo R: (a, 0), (b, 0), (b, h) e (a, h) => xCMR = a + —— 6 VcMR ~ 2
2
b + c + b 2b+ c 0+0+h h
Vértices do triângulo 2: (b, 0), (c, 0), (b, h) => xCM2 = e VcM2
3 3 3

243
_____________________________ Flementos da Física-Mecânica!-Impulso e Quantidade de Movimento
Como a massa de cada região é proporcional à sua área:
gX2a+(b_a)XÍʱÈ) + (^-b)X (2b±c_)
$1XCM1 + SqXçm r + ^2XCM2 . X 3 M X / 3
XCM ~
S, + Sr + S2 (-a + b + c)^
X *
2a2 + 3b2 - 3a2 + c2 - 2b2 + bc -a2 + b2 + c2 + bc
3(-a + b + c) 3(-a + b + c)

ah X .. x
,, (c-b)hX
yy + (b-a)h^ +
_ SlYcMI + SqYcm r + S2ycM2 ' X X 3 h(a + 3b-3a + c-b)
ycM= Si + Sr+S2 (-a + b + c)X 3(-a + b + c)
X
(-2a + 2b + c)h
3(-a + b + c)

Corpos com Orifício

No cálculo do centro de massa de um objeto que inclui um orifício pode-se proceder do


seguinte modo:
1) Efetua-se o cálculo do centro de massa do objeto inteiro (como se não existisse o orifício)
2) Efetua-se o cálculo do centro de massa do orifício (como se fosse só ele)
3) Subtrai-se o produto do centro de massa vezes a massa do orifício do produto do centro de
massa do objeto inteiro vezes a massa do objeto.

Dito de outro modo: É como se o orifício fosse feito de um material que tem uma massa
negativa, que é subtraída da massa do objeto. Vamos considerar, a título de exemplo, uma chapa
triangular, homogênea, que possui um orifício circular de diâmetro d.

y
■‘cmA

IcmO

v ^cmO X

rnorCMQ
*CM
mA-m0

onde, mA representa a massa da chapa completa (ou seja, sem se considerar o orifício), m0 é a
massa correspondente ao orifício se ele fosse constituído pelo mesmo material homogêneo que o
resto da chapa, rCMi é o vetor posição do centro de massa de mA (ponto de encontro das
medianas) e rCM0 é o vetor posição do centro de massa de m0 (corresponde ao centro da
circunferência). A massa real da chapa (m = mA - m0) é fácil de determinar, o problema está em
determinar as massas mAe m0 separadamente.

244
fiementosda física-Mecânica i-Impulso e Quantidade de Movimento
SISTEMA DE PARTÍCULAS

É conhecido que a quantidade de movimento de uma partícula é igual ao produto de sua


massa pelo vetor velocidade instantânea: P = mv. Este conceito pode ser estendido para um
sistema de n (n > 2) partículas. A quantidade de movimento de um sistema de n partículas é igual
à soma das quantidades de movimento de todas as partículas:

Psist = m1v1+m2v2+... + mnvn

Substituindo a equação da velocidade do centro de massa segue que:

Psist = (m1+m2+... + mn)vCM Psist = Mvcm , onde M = m, + m2 + ... + mn

d Psist d(MvCM) dVçM


Pela 2a Lei de Newton: FR => Fr = Fr=M Fr = MãCM
dt dt dt

Sabe-se que a força resultante em um sistema de partículas é igual à soma vetorial das
forças que atuam em todas as partículas. Se F; é a força resultante na partícula i, 1 < i < n, pode-
se afirmar que FR = F, + F2 +... + Fn. Pode-se dividir as forças que atuam em uma partícula em
duas categorias: forças internas e forças externas. Seja a força interna sobre a partícula i
devida a sua interação com a partícula j. Suponha que Fiext a resultante das forças externas que
atuam sobre a partícula j.

Analisando isoladamente cada partícula, pode-se escrever que:

miã1=Rn2,+Rn3t- -+f;?+Fr
m2ã2=F£+F£ --+^+Fr

mnãn +...+F^.l+Fr

Somando membro a membro:

miãt + m2ã2 +... + mnãn = (F'n2‘ + F**) + (F™ + F*') +... + (F™-1 + Ri-,tn) + F1eX,+F2eXt ...+Fr
Pelo princípio da Ação e Reação (3a Lei de Newton) sabe-se que F- + Fí"* = 0, onde i, j =
1,2, , n, i * j. Assim:

d Psist
MãCM APsist jÈF-dt
dt i=1

Desta última expressão, conclui-se que apenas as forças externas fazem com que exista
variação da quantidade de movimento de um sistema. As forças internas não provocam variação
da quantidade de movimento de um sistema.

245
________________________ _______ fiementos da física-Mecânica!-Impulso e Quantidade de Movimento
O MOVIMENTO DO CENTRO DE MASSA

Suponha que n partículas, de massas m,, m2, ...mn, estão se movimentando no espaço,
com velocidades iguais a vv v2, .... vn, respectivamente. Do item anterior, sabe-se que
„ - n n

Fr = MãCM, onde M = m, + m2 + ... + mn, e FR = ^Fiext , ou seja, ^F®** = MãCM. Deste modo,
i=i i=i
pode-se enunciar que quando forças externas atuam sobre um corpo ou um conjunto de
partículas, o centro de massa se move como se toda a massa estivesse concentrada neste ponto
e sobre ele atuasse uma força igual à soma das forças externas que atuam sobre o sistema. Note
que não é necessário que exista alguma massa na posição definida por rCM, apenas do ponto de
vista teórico, para análise do movimento do sistema, é como se toda a massa do sistema
estivesse concentrada neste ponto.

Este raciocínio ocorre tanto para um sistema de partículas quanto para um corpo rígido.
Por exemplo, quando um martelo é arremessado obliquamente, tem-se a impressão que seu
movimento é irregular, principalmente quando se observa a trajetória da extremidade do cabo do
martelo. Porém, o centro de massa do martelo, que está próximo à parte metálica do martelo,
executa uma trajetória parabólica, como ilustrado na figura abaixo.

Isso ocorre pois, quando lançado, a única força atuante no martelo é seu peso, que é uma
força externa. Assim, o centro de massa do martelo executa a mesma trajetória de uma partícula
em lançamento oblíquo no campo gravitacional terrestre, ou seja, uma trajetória parabólica.

Outra aplicação prática do movimento do centro de massa é o lançamento de uma


granada. Em determinado momento a granada explode, dividindo-se em vários pedaços menores.
A explosão ocorre devido a forças internas, fazendo com que a única força externa atuante na
granada ou seus fragmentos, em qualquer instante, seja seu peso. No caso do martelo é mais
intuitivo entender o movimento de seu centro de massa, uma vez que o mesmo encontra-se
interno ao seu volume. No caso da granada, após a explosão, nenhum pedaço da granada vai
acompanhar seu CM. Mesmo assim, o CM da granada vai continuar executando uma parábola, a
mesma que seria descrita caso a granada não tivesse explodido, independentemente de quantos
fragmentos surjam devido a explosão.

246
Fiementos da Física-Mecânica i-Impulso e Quantidade de Movimento

trajetória do CM
dos fragmentos
da granada

Pelo fato de quantidade de movimento ser uma grandeza vetorial, é possível que o
movimento do CM se comporte de uma maneira em um eixo e de maneira diferente em outro eixo.
Por exemplo, considere o experimento de colocar uma barra na posição vertical sobre um plano
horizontal liso. Devido a qualquer pequena perturbação, a barra inicia seu movimento de queda.

CM

CM

CM

Perceba que as duas forças atuantes sobre a barra, normal e peso, possuem direção
vertical. Desta maneira, o somatório das forças externas que atuam na barra possui componente
horizontal nula. Como FR = MãCM, então a componente horizontal da aceleração do centro de
massa é nula. Como o centro de massa inicia seu movimento parado, em um referencial preso ao
solo, e aCMx = 0, segue que a coordenada x do centro de massa da barra não muda de posição
neste referencial. Assim, a barra cai sobre o chão mantendo sempre o centro de massa sobre
uma mesma reta vertical.

CONSERVAÇÃO DA QUANTIDADE DE MOVIMENTO

No item anterior foi demonstrada a seguinte expressão: Ap^, jÈF^dt


i=1

Analisando esta expressão pode-se concluir que a variação da quantidade de movimento


de um sistema é nula em apenas duas situações:

1) A soma das forças externas atuantes no sistema é nula;

2) 0 tempo em que os corpos dos sistema interagem tende a zero.

Neste capítulo analisaremos apenas a situação em que a soma das forças externas é nula.
No capitulo sobre colisões mecânicas será analisada a situação em que o tempo de interação dos
corpos do sistema tende a zero.

247
___________________________________ Flemenlos da Física-Mecânica i-Impulso e Quantidade de Movimento
Do que foi exposto anteriormente, conclui-se que quando a soma das forças externas em
um sistema é nula ocorre a conservação da quantidade de movimento:

n
pext
=0 Ap = 0 Po=Pf
i=1
miV10 + m2v20 + ... + m„vn0 irqv-K +m2v’2f
: +... + mnvnf

Quando a soma das forças externas em um sistema é nula afirma-se que o sistema é
mecanicamente isolado. Como força é uma grandeza vetorial, é possível que um sistema seja
mecanicamente isolado em uma direção e não em outra. Neste caso, pode-se afirmar que o
sistema é mecanicamente isolado apenas na direção x ou y ou z. Acompanhe os exemplos
seguintes para entender melhor a aplicação deste princípio.

1. Homem andando sobre uma prancha na água

n2|
N,
4 —
-Fat
Far>* F2
Pi
Considere que um homem, de massa m,, anda ao longo de uma prancha, de massa m2, de
sua extremidade esquerda para a direita. A prancha está sobre a água e o atrito entre eles pode
ser considerado desprezível. O sistema é formado apenas pelo homem e pela prancha, cujas
decomposições de forças estão indicadas nas figuras acima. Perceba que a única força interna ao
sistema é a força de atrito trocada entre o homem e a prancha. Note que esta força é de atrito
estática, uma vez que não existe movimento de arrasto entre os pés do homem e a prancha.
Como não existe deslocamento vertical nos corpos, pode-se afirmar que P, + N, = 0 e P2 + N2 = 0.
Desta maneira, a soma das forças externas ao sistema vale:

^Fext = P, + N, + P2 + N2 0+0=0

Logo, pode-se afirmar que este sistema é mecanicamente isolado, ocorrendo a


conservação da quantidade de movimento.

 v2

Note que, para um referencial solidário à água, a quantidade de movimento inicial é igual a
zero, uma vez que tanto o homem quanto a prancha estão parados no início neste referencial.
Admita que, em determinado momento após entrar em movimento, a velocidade do homem é v,.
enquanto que a velocidade da prancha é v2. Logo, pela conservação da quantidade de
movimento:

p0=pf => 0 = mlv1+m2v2

Como v4 e v2 são vetores paralelos e horizontais, adotando um referencial horizontal


orientado da esquerda para a direita segue que:

0 m1v1 + m2v2 0 = m1v1+m2(-v2) => m-iv-i = m2V2

248
_________ ________________________ Elementos da Física-Mecânica i-Impulso e Quantidade de Movimento
Analisando o centro de massa, pode-se chegar ao mesmo resultado. Como a soma das
forças externas é nula, a velocidade do centro de massa não é alterada pelo deslocamento do
homem ao longo da prancha. Como no início tanto o homem quanto a prancha estão parados, em
relação a um referencial solidário a água, tem-se que vCm = 0. Após o homem entrar em
movimento a velocidade do centro de massa continua nula:

m1v1 + m2v2
Vcm - =0 m^, + m2v2 0 => 0 = m1v1 +m2(-v2) 01,71 = m2v2
m., + m2

2. Dois corpos presos a uma mola em um plano horizontal


Suponha que dois blocos estão unidos por uma mola ideal
m< x.OOOO.r m2 e o sistema encontram-se em um plano horizontal liso. Os blocos
então são aproximados, de forma a comprimir a mola, e depois
abandonados, passando a executar um movimento oscilatório em
A aN2 torno do centro de massa do sistema.
-Fb Fe Analisando as forças atuantes no sistema formado pelos
dois blocos mais a mola, observa-se que a força elástica é uma
força interna (motivo pelo qual surge como uma par de forças
Pi *p2 ação e reação Fe e -Fe) e as normais e pesos são forças
externas. Como não existe deslocamento vertical nos corpos,
pode-se afirmar que P, +N, =0 e P2 +N2 = 0. Desta maneira, a
Vi v2 soma das forças externas ao sistema vale:

£ Fext = P, + N, + P2 + N2 = 0 + 0 0

Desta maneira, em qualquer instante, ocorre a conservação da quantidade de movimento


do sistema. Como os dois blocos são abandonados a partir do repouso, a quantidade de
movimento inicial é nula. Se, em determinado momento, as velocidades dos blocos são v, e v2,
com os sentidos indicados na figura, pode-se afirmar que:

Po ~ Pr => 0 = n^v, + m2v2

Como v, e v2 são vetores paralelos, adotando um referencial horizontal orientado da


esquerda para a direita segue que:

0 = m1v1 +m2v2 0 = m1(-v1) + m2v2 m,Vi = m2v2

Analogamente ao exemplo do homem sobre a prancha, o resultado acima também poderia


ser obtido através do centro de massa, que continua parado, independentemente do
deslocamento de cada bloco preso à mola.

^m^+m^ p
^CM m,!/, + m2v2 = 0 => 0 = m1v1+m2(-v2) => rmv, = m2v2
m, + m2

Neste experimento ocorre algo bem interessante. O centro de massa do sistema formado
pelos blocos encontra-se sobre a mola (supondo que a mola foi fixada aos corpos simetricamente
aos mesmos). Durante todo o movimento oscilatório, a mola e os corpos se movimentam. Porém,
independentemente se a mola está comprimida ou esticada, o único ponto da mola que está
parado em relação ao solo, é o centro de massa do sistema formado pelos blocos.

249
________ ___________________ Fiementos da Física-Mecânica !-Impulso e Quantidade de Movimento
3. Corpo descendo um plano inclinado
Um pequeno bloco, de massa m,, é
colocado sobre a superfície lisa de um plano
inclinado de massa m2, que pode se deslocar
sobre um plano horizontal sem atrito.
A decomposição das forças atuantes em
cada corpo está representada ao lado. Perceba
que a força N,, devido ao contato dos corpos, é
\a uma força interna ao sistema. Além disso, o plano
inclinado não possui movimento vertical:

-N1y + N2 + P2 = 0 n2 + P2 = N1y

O somatório das forças externas, que


atuam no sistema formado pelo bloco mais o plano
p.
inclinado, vale:

XFex,=Pl+P2+N2=Pl+N1y
n2
Note que P, + N1y é a resultante vertical no
bloco de massa m^ que é não nulo, uma vez que
o bloco desce aceleradamente. Assim, pode-se
afirmar que o sistema não é mecanicamente
;v2
isolado. Entretanto, quantidade de movimento é
uma grandeza vetorial, sendo possível que ocorra
sua conservação para uma determinada direção e
não para as outras direções. É esta exatamente a
situação do experimento em questão.

Perceba que o somatório das forças externas possui componente horizontal nula, uma vez
que R, e Nly são forças verticais. Assim, pode-se afirmar que ocorre a conservação da quantidade
de movimento apenas para a direção horizontal, para o sistema formado pelo bloco e pelo plano
inclinado:

Pox Pt, => 0 = m^- v1x) + m2v2 => noviços <p = m2v2

onde <p é o ângulo que o vetor vd faz com a horizontal.

Observações:

1) Mesmo que houvesse atrito entre o bloco e o plano inclinado, haveria conservação da
quantidade de movimento na direção horizontal, uma vez que esta força de atrito seria uma força
interna ao sistema. Contudo, caso existisse atrito entre o plano inclinado e o plano horizontal, não
existiría mais a conservação da quantidade de movimento na direção horizontal, já que esta força
de atrito seria uma força externa ao sistema formado pelo bloco mais plano inclinado.

2) O bloco desce aceleradamente nas direções horizontal e vertical, percorrendo uma trajetória
retilínea, que faz um ângulo <p a horizontal. O ângulo <p é maior que o ângulo a do plano inclinado.

250
flementos da física-Mecânica l-Impulso e Quantidade de Movimento
SISTEMAS DE MASSA VARIÁVEL

Até este momento analisamos corpos ou sistemas de partículas de massa constante.


Contudo, isto nem sempre ocorre. O exemplo mais clássico de sistema de massa variável é o
lançamento de um foguete. Tradicionalmente, um foguete é constituído de uma fuselagem
metálica e de um propelente sólido, altamente combustível. A queima do propelente produz gases
que são ejetados em alta velocidade, projetando o foguete. A transformação do propelente sólido
em gás faz com que a massa do sistema formado pela fuselagem mais propelente seja variável.
Inicialmente será adotado um referencial inercial fixado no solo. Ignore a ação da
gravidade e a resistência do ar. Esta consideração pode parecer absurda em alturas próximas ao
solo, mas é bem razoável quando o foguete já se encontra bem acima da atmosfera terrestre.
Suponha agora que, em determinado momento, o foguete está viajando com velocidade v, sendo
M sua massa total neste instante. Considere que, após um intervalo de tempo infinitesimal dt, a
massa do foguete seja M + dM, a velocidade dos gases expelidos é U e a velocidade do foguete é
v + dv. Note que a grandeza dM é negativa, uma vez que o foguete perdeu massa pela queima do
combustível sólido.

dM M + dM
v + dv
------------- ►

X
—►

Como o sistema é mecanicamente isolado:

p0 = pf => M.v = - dm.U + (M + dm)(v + dv) =>


M.v = - dm.U + M.v + M.dv + v.dm + dm.dv => (U - v)dm = M.dv + dm.dv

O termo dm.dv é muito pequeno comparado com os demais da expressão e pode ser
desprezado. Assim, a expressão fica da forma:

(U - v)dm = M.dv

É possível simplificar esta expressão, trabalhando com a velocidade u' dos gases relativa à
velocidade do foguete. Experimentalmente verifica-se que, enquanto o propelente sólido é
queimado, u’ se mantém constante. Devido à orientação do eixo x e impondo que u’ seja positivo:

u’ = |U - (v + dv)|

Entretanto, como os gases são expelidos para trás em relação ao foguete e a orientação
do eixo x é da esquerda para a direita, segue que v + dv > U, ou seja:

u’ = v + dv-U => U - v = dv - u'

Portanto:

(U-v)dm = M.dv => (dv - u')dm = M.dv => dm.dv - u’.dm = M.dv

Mais uma vez o termo dm.dv pode ser desprezado frente aos demais da equaçao:

- u’.dm = M.dv

Dividindo os dois membros da equação por dt:

251
Mementos da Física-Mecânica I-Impulso e Quantidade de Movimento
. dm ,, dv
-u .— = M.—
dt dt

Note que -R = -^ é o consumo de combustível do foguete e a=— é a aceleração do


dt dt
foguete. Portanto, segue que:

R.u’ = M.a

Esta última expressão é conhecida como primeira equação do foguete, que relaciona o
consumo de combustível R, a velocidade dos gases em relação do foguete u’, a massa do foguete
e a aceleração do foguete a. Perceba que, pela segunda lei de Newton, o termo M.a é a força
resultante sobre o foguete. Voltando a equação diferencial original, pode-se determinar uma
relação entre a velocidade do foguete e a variação de sua massa (lembre-se que u’ é constante):

U' fVf , fl
-u’.dm = M.dv => dv -----dm => dv = -u "M< —dm
M Jvo 'Mo M
Mn
v( — v0 = — u’(ln M( — In Mo) => vf-v0=u’.ln —

Esta equação recebe o nome de segunda equação do foguete.

Pela definição de logaritmo, tem-se que

Vf-VQ
Mo
e u'
Mf

Assim, para obter um incremento de velocidade igual à velocidade de escape dos gases,
Vf- v0 = u’, a relação de massas deve ser igual a e1 = 2,72, ou seja, Mf deve ser aproximadamente
um terço de Mo. Para vf - v0 = 2u’, a relação Mo/Mf vale e2 = 7,4 e para vf - v0 = 3u’ vale e3 s 20, ou
seja, neste último caso apenas 1/20 da massa original do foguete representa carga útil. Na
prática, o sistema é projetado para maximizar, em conjunto, a velocidade atingida e a carga útil do
projeto. Isso justifica a utilização de foguetes de vários estágios, onde a massa total é reduzida
ejetando os estágios quando o combustível de cada um se esgota. Note que a carcaça de um
estágio é um peso morto considerável, composto de partes metálicas de tanques de combustível e
motores.
Por exemplo, para atingir a Lua com um foguete de apenas um estágio é necessário lançar
o foguete com uma velocidade próxima à velocidade de escape, valor impossível de alcançar
utilizando os combustíveis atuais.

252
Elementos da Física-Mecânica!-Impulso e Quantidade de Movimento
Exemplos:

1) (ITA-15) Uma chapa metálica homogênea quadrada de 100 cm2 de área, situada no plano xy
de um sistema de referência, com um dos lados no eixo x tem o vértice inferior esquerdo na
origem. Dela, retira-se uma porção circular de 5,00 cm de diâmetro com o centro posicionado em
x = 2,50 cm e y = 5,00 cm. Determine as coordenadas do centro de massa da chapa restante.
a) (Xc,yc) = (6,51, 5,00) cm b) (xc, yc) = (5,61,5,00) cm
c)(Xc, yc) = (5,00, 5,61) cm d) (xc, yc) = (5,00, 6,51) cm
e) (xc, yc) = (5,00, 5,00) cm
Solução: Alternativa B
,.y

10 - x2S2 5.10.10 - 2,5.3,14.(2,5)2


XCM ~ XCM “
s,-s2 100-3,14.(2,5)2
x-! = 5,61 cm
5 A coordenada y do centro de massa não sofre alteração pois tanto a
chapa quanto o furo circular possuem mesma coordenada y do centro
X de massa:
0 10* yCM = 5,00 cm
2,5

2) Calcule o centro de massa de um corpo homogêneo em forma de tronco de cone, com raio de
base maior r,, raio de base menor r2 e altura h.
Solução:
oy Seja H a altura do cilindro original que gerou o tronco
de cone. Por semelhança de triângulos segue que:
/ \ H H-h H-y
/ \ r, ” r2 " r
/ \
/ \
/ \ h l =-*
/ \ i) Hr2 = Hr, - hr.
/ \ r,-r2
H / \
____ \ (H-y)r,
TTTrs ii) r
H
h-y Por simetria, percebe-se que XcM = 0
dy|
h Para um diferencial de altura dy, tem-se dV = nr^dy

y Ycm = VfydV = Johynr2 dy = G dy =


Fl
H2
*x . < £h(H2y-2Hy2 + y3)dy
VH2
2
_ "í |"ÍH2y2 2Hy3 nr2 ( H2h2 2Hh3 h4 7tr,2h2
VH2^ 2 3 d------- 6-8 —+ sf—h
4 ~ VH2 2 3 4 12V H H

r2h
____________ e 8(r, - r2) 3(r, - r2 )2 h(6r2 - 8r2 + 8r,r2 + 3r2 - 6r,r2 + 3r22) h(r2 + 2r,r2 + 3r2)
4(r,2 + r,r2 + r2) [_ ri r' 4(r2 + r,r2 + r2) ” 4(r,2 + r,r2 + r2)
Note que fazendo r2 = 0 tem-se o yCM de um cone, que vale h/4.

3) (Escola Naval-01) Uma partícula de massa m, inicialmente em repouso, está sob a ação de
força resultante F cujo módulo varia com o tempo, de acordo com o gráfico abaixo. No instante
tj, a velocidade da partícula vale:

253
Elementos da física-Mecânica !-Impulso e Quantidade de Movimento

Fl

■>

f2.....
x (F\ F)tl + F2t2 (^1 + ■F'; )/[
d)
m 2m 2m
e) nda
' 2m
Solução:
Sabe-se que, no gráfico Fxt, a área compreendida entre a linha do gráfico e o eixo do tempo é
numericamente igual ao impulso da força. Além disso, as partes do gráfico abaixo do eixo x são
contadas como impulso negativo. Note que, por estar abaixo da origem no eixo y, F2 assume valor
negativo. Assim: , = ~ U . ^t, + F2t2 - F2t, = (F, + F2 )t, - F2t2
2 2 2 2
Como o impulso é igual à variação da quantidade de movimento:
■ . n (F + F2 )t. - F2t2 (F, + F2 )t, - F2t2
I = Ap = mv - 0 => —----- —----- — = mv => v = —----- —----- —
2 2m

4) (1TA-00) Uma lâmina de material muito leve de massa m está em repouso sobre uma
superfície sem atrito. A extremidade esquerda da lâmina está a 1 cm de uma parede. Uma formiga

considerada como um ponto, de massa ~, está inicialmente em repouso sobre essa

extremidade, como mostra a figura. A seguir, a formiga caminha para frente muito lentamente,
sobre a lâmina. A que distância d da parede estará a formiga no momento em que a lâmina tocar
a parede?

n
1cm
parede parede

lâmina
titnn /////// tuuu uum
(K) 2 cm (B) 3 cm (C) 4 cm (D) 5 cm (E) 6 cm
Solução: Alternativa E
Como o movimento da formiga sobre a lâmina envolve apenas forças internas, existe a
conservação da quantidade de movimento:
Csv.,
Po = Pf => 0 = rrifVf + m,(- v() => m,
mf —j- => (m/5)(d - 1) = m(1) => 5 = d-1 =>
& m'7
d = 6 cm

5) (Escola Naval-13) Uma granada, que estava inicialmente com velocidade nula, explode,
partindo-se em três pedaços. O primeiro pedaço, de massa M-, = 0,20 kg, é projetado com uma
velocidade de módulo igual a 10 m/s. O segundo pedaço, de massa M2 = 0,10 kg, é projetado em
uma direção perpendicular à direção do primeiro pedaço, com uma velocidade de módulo igual a
15 m/s. Sabendo-se que o módulo da velocidade do terceiro pedaço é igual a 5,0 m/s, massa da
granada, em kg, vale
a) 0,30 b) 0,60 c) 0,80 d) 1,0 e) 1,2
Solução: Alternativa C

254
____________ Elementos da Física-Mecânica!-Impulso e Quantidade de Movimento
Como uma explosão envolve apenas forças internas, ocorre a
M conservação da quantidade de movimento:
Po=Pf => P,+P2+P3=0 => p,+p2=-p3
Como p, e p2 são perpendiculares, é possível formar um triângulo
P2
retângulo de equilíbrio com os vetores apresentados, sendo que p3 ocupa
a posição da hipotenusa. Assim:
Pa Q
PÍ + P2 = Pa => + m2v2 = m3Va => 4 + — = 25m3 m3 = 0,50 kg
4 3
mT = m1 + m2 + m3 = 0,20 + 0,10 + 0,50 = 0,80 kg

6) (AFA-14) Considere duas rampas A e B, respectivamente de massas 1 kg e 2 kg, em forma de


quadrantes de circunferência de raios iguais a 10 m, apoiadas em um plano horizontal e sem
atrito. Duas esferas 1 e 2 se encontram, respectivamente, no topo das rampas A e B e são
abandonadas, do repouso, em um dado instante, conforme figura abaixo.
c< P

B
z//Zz z///?/////////////////////
Quando as esferas perdem contato com as rampas, estas se movimentam conforme os gráficos
de suas posições x, em metros, em função do tempo t, em segundos, abaixo representados.
x(m)x
10

* (S)
0
3
2
Desprezando qualquer tipo de atrito, a razão mi/m2 das massas m3 e m2 das esferas 1 e 2,
respectivamente, é
a) 1/2 b) 1 c)2 d) 3/2
Solução: Alternativa A
Pelo gráfico:
. I Ax. I 10 , .... . I AxB I 10 ,
|VaI= At = -y=m/s;
V3m/S' ii) | vbB l=
H) |V |= At V3 m/S
-N,
Como as rampas não possuem movimento vertical:
-N1y+NA+PA=0 => NA+PA=N1y
Na |Pa O somatório das forças externas, que atuam no sistema formado
por uma das rampas mais sua esfera , vale:
XFex,=Pl+PA+NA=Pl+N1y
Assim, F':extx = 0, implicando na conservação da quantidade de movimento na direção horizontal:
Qox = Qfx => 0 = mAvA-m1v1 => v.
m,
Além disso, devido à ausência de forças dissipativas, ocorre também a conservação da energia
mecânica:
m, m3 v3 , mAVl _ mAVA mA d
Em0 = Em( => m,gR jn,v? mAA VAl => m,gR “T —~+1

2 2 T m3 2 2

255
Elementos da Física-Mecânica/-Impulso e Quantidade de Movimento
>0tím, =-^^í—+ 1 => 6m,2 = 1 + m, => 6mi2-m,-1=0 =>
2 3
(Sm, + 1)(2m, -1) = 0 m, — kg
2 a
mBVB mR
Analogamente, pode-se demonstrar que m2gR = —5- + 1
m2
/ >ec> f 2 11
>0tím2= j— m2 => 3m22 = 2 + m2 => 3m22 - m2 - 2 = 0 =>

(m2 - 1 )(3m2 + 2) = 0 => m2 = 1 kg


. m, 1/2 1
Logo: —- =----- = —
m2 1 2

7) (ITA-00) Uma sonda espacial de 1000 kg, vista de um sistema de referência inercial, encontra-
se em repouso no espaço. Num determinado instante, seu propulsor é ligado e, durante o
intervalo de tempo de 5 segundos, os gases são ejetados a uma velocidade constante, em relação
à sonda, de 5000 m/s. No final desse processo, com a sonda movendo-se a 20 m/s, a massa
aproximada de gases ejetados é:

1000 kg

<20 m/s 5000 rn/s*


(A) 0,8 kg (B) 4 kg (C) 5 kg (D) 20 kg (E) 25 kg
Solução: Alternativa B
Como as únicas forças que atuam no sistema são internas, pode-se aplicar a conservação da
quantidade de movimento:
Psonda + Pgases = 0 => 1 000.20 + m(- 5000) = 0 => m = 4 kg

8) (ITA-94) Uma granada de massa m é lançada a partir de um ponto do gramado de um campo


de futebol com velocidade inicial Vo =30 m/s que forma com a horizontal um ângulo a = 45°.
Segundo o relato de um observador: “No ponto mais alto de sua trajetória a granada explodiu em
dois fragmentos iguais, cada um de massa m/2, um dos quais (o primeiro), aí sofreu uma 'parada'
e caiu verticalmente sobre o campo. O segundo fragmento também caiu sobre o campo." Nestas
condições. Desprezando-se a resistência do ar pode-se afirmar que o segundo fragmento atingiu
o campo a uma distância do ponto de lançamento igual a :
a) 45, Om b) 67,5 m c)135m d) 90,0
e) O relato do observador contraria a lei da conservação da quantidade de movimento.
Solução: Alternativa C
No ponto mais alto da trajetória a coordenada x é metade do alcance do lançamento original:
x, = A/2 = v^sen 20/2g = (900)(sen 90°)/(20) = 45 m
A explosão da granada envolve apenas forças internas, ou seja, ocorre a conservação da
quantidade de movimento: p0 = pf => mv0 = (m/2)(0) + (m/2)v’ v’ = 2v0
Como x, = v0.At e x2 = v’.At => x2/x, = v’/v0 => x2 = 2x, = 90 m
Assim: AT = x,+x2 => AT=135m

9) (ITA-91) Um pêndulo simples de comprimento r e massa m é posto a oscilar. Cada vez que o
pêndulo passa pela posição de equilíbrio atua sobre ele, durante um pequeno intervalo de tempo
t, uma força F. Esta força é constantemente ajustada para, a cada passagem, ter mesma direção
e sentido que a velocidade de m. Quantas oscilações completas são necessárias para que o
pêndulo forme um ângulo reto com a direção vertical de equilíbrio ?
a)n=ni^£; h) n = ; c) n = ;
2Ft 2Ft 2Ft
d) n = ÍSÍ + 1; e) Nenhuma das anteriores.

256
Momentos da físico - MoçAnlca / - Impulsa o Quantidade de Movimento
Solução: Alternativa C
Em cada atuação da força F tem-se:
I) I = Ap => F.At = m.Av => Av = F.t/m
II) Evidentemente, após o último impulso, tem-se a conservação da energia mecânica:
mv2/2 = mgí => v =j2gt, onde v é velocidade da massa ímediatamente depois do último
impulso da força F
III) Como em cada oscilação ocorrem duas passagens pela posição de equilíbrio, então:
2n.Av =72gí => 2nFt/m=72gC => n = m^gí/2Ft

10) (ITA-05) Um vagão-caçamba de massa M se desprende da locomotiva e corre sobre trilhos


horizontais com velocidade constante v = 72,0 km/h (portanto, sem resistência de qualquer
espécie ao movimento). Em dado instante, a caçamba é preenchida com uma carga de grãos de
massa igual a 4/Vf, despejada verticalmente a partir do repouso de uma altura de 6,00 m (veja
figura). Supondo que toda a energia liberada no processo seja integralmente convertida em calor
para o aquecimento exclusivo dos grãos, então, a quantidade de calor por unidade de massa
recebido pelos grãos é
a) 15 J/kg. grãos 4M
b) 80 J / kg.
c) 100 J/kg. l l l l l V
d) 463 J / kg. s M
e) 578 J / kg. o o
Solução: Alternativa C
Aplicando conservação do momento linear: p0 = pF => M.20 = 5M.V’ => V = 4m/s
- c c c M.400 5M.16
Q- Eo - EF => Q- E■co co + Epo Ecf => Q— ——- - + 4M.g.h -

Q = 400M — =100J/kg
4M

11) (ITA-10) Uma massa m, com velocidade inicial Vo colide com um sistema massa-mola m2 e
constante elástica K, inicialmente em repouso sobre uma superfície sem atrito, conforme ilustra a
figura. Determine o máximo comprimento de compressão da mola, considerando desprezível a
sua massa.
Z7?2

Solução Ideal:

Observe que durante todo o intervalo de tempo que a massa m, estiver em contato com a mola
sua velocidade diminui, enquanto que a velocidade de m2 aumenta. Desta forma, Ax vai ser
máximo no instante em que as velocidades dos dois corpos forem iguais. Desde que o sistema é
mecanicamente isolado, vale a conservação da quantidade de movimento:
, x m.Vn
m^o = (rrh + m2)v => v =---- z—sí—
m, +m2
Pela conservação da energia mecânica:
m,Vo (m1+m2)v2 kAx2 __ 2 lim-+'rríí)m?vr
mivo + kAx2 kAx2 mivo ~ jéyL
~T~=— /“'T"
"T miV°= (ml+m2)Z m, +m2
AX2=^ 1 m, 01^2
Ax v0
k m1 +m2 kím-j +m2)

257
_______________________________ Fiementos da Física-Mecânica!-Impulso e Quantidade de Movimento
12) (ITA-88) Nas extremidades de uma haste homogênea, de massa desprezível e comprimento
L, acham-se presas as massas m, e m2. Num dado instante, as velocidade dessas massas são,
respectivamente, v, e v2, ortogonais à haste (ver figura). Seja vCM a velocidade do centro da
massa, em relação ao laboratório e seja co o módulo da velocidade angular com que a haste se
acha girando em torno de um eixo que passa pelo centro de massa. Pode-se mostrar que:

f X
m.
L
/wa

Vcm CO

m,ym2v2 |v,-v21
a)
m, +m2 L
b) m2v2-m1v1 I V2 ~ V1 I
m, +m2 L
m,v, +m2v2 I v,-v21
c)
m, +m2 L
m.v, + m,v, (v2 +v,)
d) ——!----- —-
m, +m2 L
m,v, -m2v2 (V2+Vi)
e)
m, +m2 L
Solução: Alternativa D
m,v, +m2v2
Por definição, sabe-se que vCM . Considere agora que v, =| v11 e v2 =| v21.
m, +m2
Trabalhando com um eixo vertical, parado em relação ao local onde está sendo realizado o
it^v, - m2v2
experimento, orientado de baixo para cima, tem-se, para esse eixo, que vCM
m, +m2
VcmA Para facilitar o cálculo da velocidade angular com que cada
f V1 - Vcm corpo gira em trono do CM, é necessário fazer com que o CM
esteja parado. Para tanto, será adotado um outro eixo vertical y'
d, d2
O V’cM = 0
■Q que se movimenta com vCM. Este eixo mede velocidade v-, - vCm
para o corpo 1, velocidade v2 + vCM para o corpo 2 e velocidade 0
para o CM. Pela definição de centro de massa: d, = m-2-—
V2 + Vcm, , rn1 + m2
m,v, -m2v2
v,-
m, +m2 = JPí< + m.v, - + m2v2 _ p<(v, +v2) v.+v2
Assim: co = ——-----
d. m2L m2L L
m, +m2

13) (ITA-16) Dois garotos com patins de rodinhas idênticos encontram-se numa superfície
horizontal com atrito e, graças a uma interação, conseguem obter a razão entre seus respectivos
pesos valendo-se apenas de uma fita métrica. Como é resolvida essa questão e quais os
conceitos físicos envolvidos?
Solução:
Aplicando-se conservação do momento linear para os instantes antes e depois da interação,
m v
temos: panl=p d„ => 0 = pa-pb => pa = pb => ma.va = mb.vb => —t = -í- (I)
Pdep
mb va
Usando-se o teorema de energia:

258
Mementos da física - Mecânica /- Impulso e Quantidade de Movimento
mvo . vo
W = AEC => W(at = -Eco => -m.g.p.d => M-gd = -^
p.g.d -7?-

v2
Pro patinador A: pa.g.da = -£ (II)

v2
Pro patinador B: pb.g.db =-y (III)

Dividindo-se II por III: V2


Mb-db

Como as rodinhas são idênticas, segue que pa = pb: Xi= lí


vb M
P.
Usando a equação 1: —
ma Pa

14) (ITA-78) Considera-se um bloco de massa m sobre outro de massa M. Inicialmente m desliza
sobre M sem atrito, com uma velocidade v0. A partir do ponto p o coeficiente de atrito entre as
duas superfícies em contato é não-nulo (jj * 0). Se o bloco M puder deslizar sobre o plano
horizontal sem qualquer atrito, pode-se afirmar que a distância x percorrida por m sobre M,
contada a partir do ponto p, será dada por:
p*0
v0 I H*0
m x
| P I
MVp mvp MVp
A) X = B) X = C) X =
2p(m + M)2g ’ 2p(m + M)g' 2p(m + M)g ’
D) x = 0 (distância nula); E) x = Nenhum dos valores acima;
Solução: Alternativa C
A decomposição de forças abaixo é feita em algum ponto a partir do ponto p. Antes do ponto p
não existe interação horizontal entre os corpos, com m se deslocando e M parado. A partir do
ponto p, devido ao atrito, m passa a desacelerar e M passa a se deslocar para a direita. No
momento em que m parar em relação a M, os dois corpos passam e de deslocar com a mesma
velocidade, em relação a um referencial solidário ao piso.
n Inicialmente perceba que os corpos não se movimentam na
direção vertical: i)Nl+P1=0; ii) N2+P2-N, = 0;
Deste modo, a soma das forças externas vale (Fat e N, são
F.,
-N, internas): £ Fext = Pt + (P2 + N2 ) = -N, + N, = 0
PJ N2f -F.,
—►
Como a soma das forças externas é igual a zero, então ocorre a
conservação da quantidade de movimento.
p,|
, +. M)v =>
mvo,
mv0 = (m
v =------
m+M
onde v é a velocidade que os dois corpos possuem, em relação a piso, no instante em que m para
em relação a M. Pelo teorema do trabalho energia:
(m + M)v2 mvj (m + M) m2v2 mvp
W- Ecf -Ec0 -mggx -mgpx =
2 Y 2 (m + M)2 2
Mv2
—-—1 x =---------- 2------
m+M 2p(m + M)g

259
___________________________________ [/ementes da Física-Mecânica!-Impulso e Quantidade de Movimento
15) (ITA-16) Um bloco de massa m encontra-se inicialmente em repouso sobre uma plataforma
apoiada por uma moda, como visto na figura. Em seguida, uma pessoa de massa M sobe na
plataforma e ergue o bloco até um altura h da plataforma, sendo que esta se desloca para baixo
até uma distância d. Quando o bloco é solto das mãos o sistema (plataforma+pessoa+mola)
começa a oscilar e, ao fim da primeira oscilação completa, o bloco colide com a superfície da
plataforma num choque totalmente inelástico. A razão entre a amplitude da primeira oscilação e a
da que se segue após o choque é igual a

□ O

a) ^(m + M) / x/2tiM.
J__ d h

iiiiiiimiiiiiiiiiiiiiiiiiiiiiiiiiiiiiiiiiiiiiiii
b) 7(M-m)h/V2dM
C) ^(M + m)h/V2dM d) ,/(M-m)d/V2hM
e) ^(M + m)d/VhM

Solução: Não Há Alternativa Correta


Soltar o bloco é equivalente a retirar uma força igual a mg sobre a mola. Assim, a amplitude inicial
Al 1 A A mg
A! vale: mg = kA-, => A, =

Colo o choque do bloco com a plataforma se dá após uma oscilação completa da plataforma, a
posição deste choque é igual à posição da plataforma no instante em que o bloco é solto, ou seja,
até a colisão o bloco percorreu uma altura h.
Pela conservação da quantidade de movimento:
s2
m '
m72gh = (M + m)v => v2 = 2gh
M+m
a • kA2 (M + m)v2 2gh
Assim: —- = A2 m
2 2 (M + m)k
Logo: -yL = J-(M + m)d
a2 v 2Mh

16) (IME-84) Um astronauta de massa m move-se no espaço interplanetário com velocidade


uniforme v. Ele segura um pequeno objeto de massa Am. Num dado momento o referido
astronauta atira o objeto com velocidade v0, em relação ao seu movimento inicial. Determinar a
distância da posição real do astronauta àquela que este ocuparia se não tivesse lançado o objeto,
decorrido um tempo t após o lançamento.
Solução:
Pela conservação da quantidade de movimento:

(m + Am)v = Am(v + v0) + mv’ => mv + Amv = Amv + Amv0 + mv’ v- = v_ Am Vo


m

A variação da velocidade do astronauta vale: Av = v - v’ Vq


m

Assim, variação da distância percorrida pelo astronauta é igual a: Ad = Av.t =—vot


m

260
__________________________________ Mementos da Física-Mecânica !-Impulso e Quantidade de Movimento
17) (IME-86) Um automóvel de massa igual a 800 kg desloca-se com uma velocidade de 10 m/s.
Em um dado momento, dá-se uma explosão interna e o carro parte-se em dois pedaços de 400 kg
cada um. Devido à explosão, uma energia de translação de 1600 Joules é comunicada ao sistema
constituído pelas duas partes do carro. Ambos os pedaços continuaram a se mover na mesma
linha do movimento inicial. Determine o módulo e o sentido das velocidades de cada um dos
fragmentos após a explosão.
Solução:
Como não agem forças externas no automóvel (a explosão envolve apenas forças internas) pode-
se aplicar a conservação da quantidade de movimento:
i) po = Pf => mv0 = (m/2)v, + (m/2)v2 => Vt + v2 = 20
mv^ x4 _> 2.800(10)2 + 4.1600 = 800.V,2 + 800.v22 =>
ü) (DYo+E=!2XÍ. + niY2
2 4 4
v,2 + v22 = 208 => v,2 + (20 - v,)2 = 208 => v,2 + 400 - 40v, + v,2 = 208 =>
v,2 - 20v, + 96 = 0 => (v, - 8)(v, - 12) = 0 => v, - 8 m/s ou v, = 12 m/s =>
v2 = 12 m/s ou v2 = 8 m/s => as velocidades finais são 8 m/s e 12 m/s

18) (IME-95) Em uma fábrica de bombons, tabletes de balas caem continuamente sobre o prato
de uma balança, que originariamente indicava leitura nula. Eles caem de uma altura de 1,8 m à
razão de 6 por segundo. Determine a leitura da escala da balança, ao fim de 10s, sabendo que
cada tablete tem massa de 10g e as colisões são completamente inelásticas.
NOTA: Despreze a resistência do ar.
Considere g = 10 m/s2
Solução:
Se as balas caem à razão de 6 por segundo conclui-se que a cada At = 1/6 s uma bala é
abandonada. Este é o mesmo intervalo de tempo entre dois choques consecutivos na balança. Se
Fo é a força provocada pelo choque de uma bala na balança, pelo teorema do impulso pode
4
afirmar que: I = Ap => F0.At = mAv = m^2gh => Fo.—= 0,010.72.10.1,8 => Fo = 0,36 N
6
T 10
Ao final de 10 s o número de balas em cima da balança é: n = — 60 balas

Deste modo, a indicação da balança no instante 10 s é devido à massa das 60 balas que estão
sobre a balança mais a força do choque da 60a bala:
N = Fo+ n.mg = 0,36 + 60.0,010.10 = 6,36 N
N
Como a balança indica massa e não força a indicação correta é — 0,636 kg = 636 g
9

19) (IME-06) Um corpo de 500g de massa está inicialmente ligado a uma mola. O seu movimento
é registrado pelo gráfico abaixo, que mostra a aceleração em função da posição, a partir do ponto
em que a mola se encontra com a compressão máxima. A abscissa x = 0 corresponde á posição
em que a deformação da mola é nula. Nesta posição, o corpo foi completamente liberado da mola
e ficou submetido à aceleração registrada no gráfico. Determine:
a) a variação da quantidade de movimento no 2s após o corpo ser liberado da mola;
b) o trabalho total realizado desde o começo do registro em x = - 0,5m até x = 3m.
a(in/s2) .,

1’ -0.5 2.0 7o -+-


6,0 S.O x(m)

Solução:
Se multiplicarmos os valores de aceleração pela massa do corpo (em kg) obteremos o gráfico da
força resultante no corpo em função da posição. Assim, podemos calcular a velocidade do corpo
quando perde contato com a mola:

261
__________ Elementos da Física-Mecânica!-Impulso e Quantidade de Movimento
=> (°..5)(50)(0.5)_(0^ = VQ.50m/s

Assim, o tempo decorrido para o corpo percorrer as posições de 0 a 2,0 m pode ser calculado por:
at2 I—4tt2
Ax = vot,+^- => 2,0 = (5,0)1,
(5,0)t,++ ^|^- => 2t2-5t,+2 = 0 => (t, - 2)(2t, - 1) = 0 =>
t, = 0,5 s
Observe que 2 s corresponde ao segundo instante em que o corpo passaria pela posição 2,0 m,
em seu retorno, se continuasse com uma aceleração de -4 m/s2..
A velocidade na posição x = 2,0 m é: v3 = v§+2.a.Ax => v2 =25 + 2(-4)(2) => v2 = 3,0 m/s
Como após a posição x = 2,0 m o corpo mantém a velocidade constante igual a 3,0 m/s, então se
passarão exatamente 1,5 s no movimento do corpo desde x = 2,0 m e x = 8,0 m. Assim, podemos
concluir que após 2 s do corpo perder contato com a mola ele está em x = 8,0 m e com uma
velocidade de 3,0 m/s. Portanto, a variação da quantidade de movimento nos 2 segundo de
movimento após x = 0 é dada por:
Ap = p8- Po = m(v8 - Vo) = (0,5)(3,0 - 5,0) => Ap = - 1,0 kg.m/s
b) A partir do gráfico da força resultante pela posição temos:
Ww6 a 3) = A(-0.5 0) - A(0a 3) = (°'5)(52°)(0.5) _ (0 5)(4)(2) = 2 25 j
(-o,s aa 0)

20) (IME-12)
bola

mola

camnl1° liillif 5 m/s

///////////
A figura apresenta um carrinho que se desloca a uma velocidade constante de 5 m/s para a direita
em relação a um observador que está no solo. Sobre o carrinho encontra-se um conjunto formado
por um plano inclinado de 300, uma mola comprimida inicialmente de 10 cm e uma pequena bola
apoiada em sua extremidade. A bola é liberada e se desprende do conjunto na posição em que a
mola deixa de ser comprimida. Considerando que a mola permaneça não comprimida após a
liberação da bola, devido a um dispositivo mecânico, determine:
a) o vetor momento linear da bola em relação ao solo no momento em que se desprende do
conjunto;
b) a distância entre a bola e a extremidade da mola quando a bola atinge a altura máxima.
Dados:
• Constante elástica da mola: k= 100 N.m'1
• Massa da bola: m = 200 g
• Aceleração da gravidade: g - 10 m.s’2
Observação:
A massa do carrinho é muito maior que a massa da bola.

Solução:

a) Usando o referencial no carrinho:


k(Ax)2 mv'2 100(0,10): 0,200.v'2
2 -mg + 2 => 2 — = 0,200.10.0,10 + => v’ = 2 m/s
2
Mudando o referencial para o solo:
i) vx = vc-v'.cos30°=5->/3 m/s
ii) vy = v'.sen30°=1m/s

262
Mementos da Física-Mecânica !-Impulso e Quantidade de Movimento
5-73 í
Assim, o vetor momento linear vale: p = (mvx,mvy) p= kg.m/s
5 ’5
b) vy = gt => 1 = 10t => t = 0,1 s
Axc = vct = 5(0,1) = 0,5 m
Axb = vxt = 0,5 - 0,1 Vã m
lvÉ.»í.0,5.
7Í3
Logo: d = ^(Axc -Axb)2 +Ay2 = 7(0,1.73 )2 + (0,05)2 = 70,0325 =----- m
20

21)(IME-13)
y
Sentido de rotação
do corpo
força

/ .30 ; ]

/ força

solo
Um corpo de 4 kg está preso a um fio e descreve um movimento circular solo. Na posição
indicada na figura, ele sofre a ação de uma força, no plano xy, perpendicular ao seu movimento
que o libera do fio, sendo o impulso nesta direção igual a 4oVã kg m/s. Determine:
a) a variação do vetor momento linear entre o instante em que o corpo é liberado do fio e o
instante que atinge o solo;
b) a coordenada x do ponto onde o corpo atinge o solo.
Dados:
• raio do movimento circular: 6,4 m
• velocidade do corpo preso no fio no ponto mais alto
• aceleração da gravidade: 10 m/s2.
Solução:
a) Aplicando-se a conservação da energia mecânica do ponto mais alto da trajetória (A) até o
instante em que o corpo se desprende (B) do fio, temos:
D
Eca = Eca + EpgA => Vb2 = Va2 + 2.g.h => Vb2 = Va2 + 2.g. — => Vb=10m/s
Logo o módulo do momento linear neste instante é: Q = 40 kg.m/s
Aplicando o teorema do impulso temos: T=p'B-pB => p'B=T + pB
Da decomposição temos: T = 60i + 20.73 j N.s => pB=20i-20Vãj kg.m/s p'B= 80i kg.m/s
Após se desprender do fio o corpo descreve um lançamento horizontal com seu momento linear
variado.
I O D
p=Pxi + pyj => p= 80i - (M.vy)j => p= 80i - (4.72.g.h )j => p=80i-(4.^2.g^)j =>

p = 80i - 32 73 j kg.m/s
Ap = p-p'B => Ap = 80i - 32 73 j - 80i => Ap = -3273 j kg.m/s
Módulo: 3273 kg.m/s Direção: Vertical Sentido: Para baixo.
b) Aplicando a equação do alcance do lançamento horizontal, temos:
Í2.H . P'B 2 3R 80 3.2,6 => Ax = 16 73 m
Ax = Vox.J---- => AX = —2- Ax = —
V g m 9 2 4 V 10
x- R.cos 30° =16.73 x-3,2^3 =16.^3 x = 19,2 73 m

263
_______________________________[tementes tia física-Mecânica !-Impulso eQuantitiatietie Movimento
22) (ITA-08) Na figura, um gato de massa m encontra-se parado próximo a uma das extremidades
de uma prancha de massa M que se flutua em repouso na superfície de um lago. A seguir, o gato
salta e alcança uma nova posição na prancha, à distância L. Desprezando o atrito entre a água e
a prancha, sendo e o ângulo entre a velocidade inicial do gato e a horizontal, e g a aceleração da
gravidade, indique qual deve ser a velocidade u de deslocamento da prancha logo após o salto.

| gLM gLM I gLM


a) u = b) u = c) u =
f M'}
1 + — msenôcosG
r ma
11 + — 2msen20 II + —|2msen0
\ mJ l mJ
I gLM | 2gLM
d) u = e) u =
1 + — 2MtanO lí 1 + — tamO
\ mJ
Solução: Alternativa D
Mu
Pela conservação da quantidade de movimento: mvcos 0 = Mu => v =---------
mcos0
Se t é o tempo total que o gato fica no ar, o tempo de subida do movimento vale t/2, logo:
_ . 2vsen0 . 2Musen0 2Mux .
vsen 0 = g(t/2) => t =----------- => t =------------- =-------tg0
g m cos 0 m
Em um intervalo t a prancha percorre um espaço u.t. Logo, pela equação do alcance:
v2sen20 . 2Mu2 . . M2u2(2.sen0.cos0) . 2Mu2 . . 2M2u2
L-u.t --------- => L tge =
tg0 ^-5----- 5------ => L = tg0 + —— tge =>
g------------------- m---------------- gm cos 0---------------------- m m
2u2 tgOM „ M gLm
1+— L =>
gm m
1 + — |2Mtg0
m)

23) (ITA-OO) Um corpo de massa m desliza sem atrito sobre uma superfície plana (e inclinada de
um ângulo a em relação a horizontal) de um bloco de massa M sob à ação da mola, mostrada na
figura. Esta mola, de constante elástica k e comprimento natural C, tem suas extremidades
respectivamente fixadas ao corpo de massa m e ao bloco. Por sua vez, o bloco pode deslizar
sem atrito sobre a superfície plana e horizontal em que se apóia. O corpo é puxado até uma
posição em que a mola seja distendida elasticamente a um comprimento L (L>C), tal que, ao
ser liberado, o corpo passa pela posição em que a força elástica é nula. Nessa posição o módulo
da velocidade do bloco é:

M
a(
Superfície de apoio

264
FlementosdaFísIca-Mecânlcal-Impulsoe Quantidade de Movimento

2m ^k(L - C)2 - mg(L - C)sm(a) 2m ~k(L - C)2 — mg(L — C)sen(a)


(A)^ (B)^
M2[l + sen2(a)] M2[l + rg2(a)]

2m — k(L - C)2 - mg(L - C) sen(a) 2m ^L-C)2


(C) (D)
(m + M) [(m + M)tg2(a)+ M] 1 M2[l + tg2(a)]
(E)0
Solução: Alternativa C

V,

Perceba que as únicas forças que possuem componente horizontal, forças elástica e a normal
trocada entre os corpos, são forças internas. Assim, a componente horizontal da resultante das
forças externas é nula, fazendo com que ocorra a conservação da quantidade de movimento na
direção horizontal:
, Mv ; ii) v’y = (v’x + v)tg a = í— + v]tga (M + m)v tg a .

i) mv'x = Mv => v X —
~
m 1 m ) m
iii) Como a linha pontilhada representa a mola: h = (L - C)sena
iv) Pela conservação da energia mecânica:
k(Ax)~2 mv'2 Mv2
k(Ax)
Epe0= ECf, + ECf2 +Epg( => — 2 - = + ^- + mgh =>
2 2
k(L-C)2 m(v 1 x 2+ v
v 1y 2 ) Mv2
- + —— + mg(L -C)sena
2 2
M2v2 (M + m)2v2 tg2 a
k(L-C)2 m + Mv2 + 2mg(L - C) sen a
m2
M2 (M + m)2 tg2 a ! M
v2 —+ k(L-C)2-2mg(L-C)sena =>
m m ) '
v2(M + m)[(M + m)tg2 a + M] = R(L _ c)2 _ 2mg(L _ c)sena
m
r1
2m ^k(L-C)2-mg(L-C)sena

(M + m)[(M + m)tg2 a + M]

265
Fiementos da Física-Mecânica !-Impulso e Quantidade de Movimento
Exercícios de Embasamento d) a meia distância entre o centro Oi e o ponto
de fixação.
E1) (UFPE-13) A figura a seguir mostra um e) a meia distância entre o centro O2 e o ponto
conjunto de objetos pontuais com massas de fixação.
iguais, dispostos ao longo de uma reta. A
distância entre os objetos 1 e 2 é 4L, enquanto E5) (Cesgranrio) Seis peças de um jogo de
que a distância entre os objetos 2 e 3 é igual a dominó estão dispostas como na figura. Dos
16L. Calcule a posição do centro de massa do pontos indicados (F, G, H, I, J) o que melhor
conjunto, medida a partir do objeto , em localiza o centro de massa desse conjunto é:
unidades de L.
1 2 3
r
r-
41 16L
•c
----
E2) (UFC) Um conjunto de três partículas, •/
todas de igual massa m, está situado na
origem de um sistema de coordenadas
cartesianas xy. Em dado instante, uma delas é •/
I
atirada na direção x, com velocidade constante
de módulo Vx = 9,0 m/s e outra é atirada na a)F b) G c) H d)l e) J
direção y, com velocidade constante de
módulo Vy = 12,0 m/s, ficando a terceira em E6) (UFC-99) Quatro discos, 1, 2, 3 e 4, todos
repouso na origem. Determine o módulo da de mesmo raio R = 20 cm, e de massas m, = 1
velocidade do centro de massa do conjunto. kg, m2 = 2 kg, m3 = 3 kg, e m4 = 4 kg estão
arrumados no plano horizontal, xy, conforme
E3) (UFPE) Duas partículas, de massa M, = M mostra a figura a seguir.
e M2 = M/2, estão presas por uma haste de • •)' icmj

comprimento L = 48 cm e massa desprezível,


conforme a figura.
M, M,
80
L

Qual a distância, em centímetros, do centro de Disco 1 Disco 2


60
massa do sistema em relação à posição da '"i '”2
partícula de massa M-i?
40
E4) (Unesp) Duas esferas homogêneas, de
raios R, e R2 e massas m, e m2, foram fixadas
Disco 4 Disco 3
uma à outra de modo a formar um sistema 20
"’4 '”3
rígido, indicado na figura a seguir.

0 20 40 60 80 x(cm)
A distribuição de massa em cada disco é
homogênea. As coordenadas (x, y) do centro
O. O2 de massa desse conjunto de discos são dadas,
\^2 em centímetros, pelo par ordenado:
m2 \ a) (40, 40) b) (20, 32) c) (20, 60)
m} d) (40, 32) e) (40, 20)

E7) (UFPE-02) A figura mostra uma estrutura


Sendo R3 = 2R2 e rr^ = m2/2, o centro do vertical formada por três barras iguais,
sistema assim constituído encontra-se: homogêneas e de espessuras desprezíveis.
a) no centro da esfera maior. Se o comprimento de cada barra é 90 cm,
b) no centro da esfera menor. determine a altura, em centímetros, do centro
c) no ponto de fixação das esferas. de massa do sistema, em relação ao solo.

266
Elementos da Física-Mecânica!-Impulso e Quantidade de Movimento
-~i-- homenagem ao Brasil. Durante os fogos,
suponha que um rojão com defeito, lançado
obliquamente, tenha explodido no ponto mais
alto de sua trajetória, partindo-se em apenas
dois pedaços que, imediatamente após a
explosão, possuíam quantidades de
movimento p, ep2.
Considerando-se que todos os movimentos
ocorrem em um mesmo plano vertical, assinale
I a(s) proposição(ões) que apresenta(m) o(s)
V
par(es) de vetores p, e p2 fisicamente
possível(eis).
E8) (UFSC-07) Na situação apresentada na 01. 08.
figura a seguir desconsidere o efeito do atrito.
Estando todas as partes em repouso no início,
P2 oo-^
=0
Pl
uma pessoa puxa com sua mão uma corda
que está amarrada ao outro barco. Considere 02. 16.
que o barco vazio (B) tenha a metade da
massa do barco mais a pessoa que formam o
conjunto (A).
04.
1 P2

E11) (Enem PPL-14) Durante um reparo na


estação espacial internacional, um
Assinale a(s) proposição(ões) CORRETA(S).
cosmonauta, de massa 90kg, substitui uma
(01) Após a pessoa puxar a corda, ambos os
bomba do sistema de refrigeração, de massa
barcos se moverão com a mesma velocidade.
360kg, que estava danificada. Inicialmente, o
(02) Após o puxar da corda, o módulo da
cosmonauta e a bomba estão em repouso em
velocidade de B será o dobro do módulo da
relação à estação. Quando ele empurra a
velocidade de A.
(04) É impossível fazer qualquer afirmação bomba para o espaço, ele é empurrado no
sentido oposto. Nesse processo, a bomba
sobre as velocidades das partes do sistema ao
adquire uma velocidade de 0,2m s em relação
se iniciar o movimento.
à estação. Qual é o valor da velocidade
(08) Após o puxar da corda, as quantidades de
escalar adquirida pelo cosmonauta, em relação
movimento dos barcos apresentarão
à estação, após o empurrão?
dependência entre si.
a) 0,05m/s b) 0,20m/s c) 0,40m/s
(16) Ao se iniciar o movimento, a energia
d) 0,50m/s e) 0,80m/s
cinética de A é sempre igual à energia cinética
de B.
E12) (UFU-05) Um skatista, sabendo que sua
massa é de 45 kg, deseja saber a massa de
E9) (Unesp-04) Uma bola de futebol de massa
sua irmãzinha menor. Sendo ele um bom
m, em repouso na marca do pênalti, é atingida
conhecedor das leis da Física, realiza o
pela chuteira de um jogador e deixa a marca
seguinte experimento: ele fica sobre um skate
com velocidade v. A chuteira permanece em
e coloca sua irmãzinha sentada em outro
contato com a bola por um pequeno intervalo
skate, distante 40 m de sua posição, conforme
de tempo At. Nessas condições, a intensidade
figura a seguir.
da força média exercida pela chuteira sobre a
bola é igual a
a) 1/2 mv2At. b) mv2/2At. c) m(At)2/2v.
d) mvAt. e) mv/At.

E10) (UFSC-06) Durante as festividades


comemorativas da Queda da Bastilha, na
Uma corda muito leve é amarrada no skate da
França, realizadas em 14 de julho de 2005,
irmãzinha e o skatista exerce um puxão na
foram lançados fogos de artifício em

267
___________________________________ Mementos da Fís/ca-Mecânica!-Impulso e Quantidade de Movimento
corda, trazendo o skate e a irmãzinha em sua fc(J)
direção, de forma que ambos se encontram a
10 m da posição inicial do skatista. 9
Sabendo-se que cada skate possui massa de
1 kg e, desprezando o peso da corda e o atrito
das rodas dos skates com o chão, após alguns
cálculos o skatista conclui que a massa de sua
irmãzinha é de
a) 11,25 kg. b) 5,1 kg. 4
c) 15,0 kg. d) 14,3 kg.

E13) (Unesp-08) Um atleta, com massa de


1
80kg, salta de uma altura de 3,2m sobre uma
cama elástica, atingindo exatamente o centro 0 1 2 3 v(m/s)
da cama, em postura ereta, como ilustrado na
figura ao lado. Determine, em kg.m/s, a quantidade de
movimento desse corpo quando atinge a
velocidade de 5 m/s.

E16) (UERJ-11) Um corpo de massa igual a


6,0 kg move-se com velocidade constante de
0,4 m/s, no intervalo de 0 s a 0,5 s. Considere
que, a partir de 0,5 s, esse corpo é
impulsionado por uma força de módulo
Devido à sua interação com a cama, ele é constante e de mesmo sentido que a
lançado novamente para o alto, também em velocidade, durante 1,0 s. O gráfico abaixo
postura ereta, até a altura de 2,45m acima da ilustra o comportamento da força em função do
posição em que a cama se encontrava. tempo.
Considerando que o lançamento se deve F(N)
exclusivamente à força de restituição da cama 12,0
elástica e que a interação do atleta com a
cama durou 0,4s, calcule o valor médio da i
força que a cama aplica ao atleta. Considere g !
= 10m/s2.
I
I
E14) (UECE-14) Uma esfera de massa m é
lançada do solo verticalmente para cima, com
velocidade inicial V, em módulo, e atinge o 0 0,5 1,5 í(s)
solo 1 s depois. Desprezando todos os atritos,
a variação no momento linear entre o instante Calcule a velocidade do corpo no instante t =
do lançamento e o instante imediatamente 1,5 s.
antes do retorno ao solo é, em módulo,
a) 2mV. b) mV. E17) (UESPI-10) Na brincadeira de bola de
c) mV2/2. d) mV/2. gude, uma pequena bola de vidro em
movimento (bola A) colide com outra bola de
E15) (UERJ-10) Em uma aula de física, os vidro inicialmente parada sobre uma superfície
alunos relacionam os valores da energia horizontal (bola B). O gráfico a seguir ilustra o
cinética de um corpo aos de sua velocidade. O módulo da força que uma bola exerce sobre a
gráfico abaixo indica os resultados outra durante a colisão. Desprezando o atrito
encontrados. das bolas com a superfície e considerando que
a bola A tem massa de 5 g, a variação na
velocidade da bola A devido à colisão com a
bola B, tem módulo:

268
ílementos tia física - Mecânica !-Impulso e Quantidade de Movimento
F(N) Considere que:
- a resistência ao movimento causada pelo ar e
o atrito entre as garrafas com os fios sejam
1 desprezíveis;
- o tempo que o bólido necessita para
deslocar-se de um extremo ao outro do
brinquedo seja igual ou superior a 0,60 s.
Dessa forma, iniciando a brincadeira com o
bólido em um dos extremos do brinquedo, com
_____ _ velocidade nula, a velocidade de chegada do
0 bólido ao outro extremo, em m/s, é de
10'J 2/10'3 t(s)
A) 10 cm/s B) 20 cm/s C) 30 cm/s a) 16. b) 20. c) 24. d) 28. e) 32.
D) 40 cm/s E) 50 cm/s
E20) (UFPE-13) Uma partícula de massa 0,2
E18) (Puc/SP-05) O gráfico representa a força kg move-se ao longo do eixo x. No instante t =
resultante sobre um carrinho de supermercado 0, a sua velocidade tem módulo 10 m/s ao
de massa total 40 kg, inicialmente em repouso. longo do sentido positivo do eixo. A figura a
seguir ilustra o impulso da força resultante na
♦ F(N)
direção x agindo sobre a partícula. Qual o
30 módulo da quantidade de movimento da
partícula (em kg.m/s) no instante t = 15s?
I(kg.m/s)'' j

t(s)
10

A intensidade da força constante que produz o


mesmo impulso que a força representada no
gráfico durante o intervalo de tempo de 0 a 25
s é, em newtons, igual a t(s)
a) 1,2 b) 12 c)15 d) 20 e) 21

E19) (FGV-10) Um brinquedo muito simples de


construir, e que vai ao encontro dos ideais de
redução, reutilização e reciclagem de lixo, é E21) (Unesp-12) Ao lançar um pacote de 4kg,
retratado na figura. um rapaz o empurra em linha reta, a partir do
repouso, sobre uma superfície horizontal,
3 exercendo sobre ele uma força F também
horizontal, mantendo-o movimento
acelerado por 2,Os.
A brincadeira, em dupla, consiste em mandar o
bólido de 100 g, feito de garrafas plásticas, um
para o outro. Quem recebe o bólido, mantém
suas mãos juntas, tornando os fios paralelos,
enquanto que, aquele que o manda, abre com
vigor os braços, imprimindo uma força variável,
conforme o gráfico.
F(N)4

8,0— O gráfico mostra como varia a intensidade da


T resultante das forças (R) que atuam sobre o
pacote durante os 2,Os em que ele foi
4,0— empurrado.

0
í—
—0,20 -I—
0,40 0,60 t(s)

269
Elementos da física - Mecânica!-Impulso e Quantidade de Movimento
R(N) resgatado chegou à nave com uma velocidade
4,0-- de 6 m/s na vertical. Considere que a massa
da nave é de 650kg, a do personagem
resgatado de 80kg
' Ie a do piloto de 70kg.
I I
a) Quais as componentes horizontal e vertical
da velocidade da nave imediatamente após o
I_____________________________ I______________

0,0 0,8 2,0 t(s) resgate?


Sabendo que o coeficiente de atrito cinético b) Qual foi a variação da energia cinética total
entre o pacote e a superfície vale 0,2 e que g = nesse resgate?
10m/s2, determine o módulo da velocidade
atingida pelo pacote ao final dos 2,Os e a E24) (Unicamp-07) Suponha que o esquilo do
filme “A Era do Gelo” tenha desenvolvido uma
intensidade da força F exercida pelo rapaz
técnica para recolher nozes durante o percurso
entre 0,8s e 2,Os.
para sua toca. Ele desliza por uma rampa até
atingir uma superfície plana com velocidade de
E22) (Unesp-15) Enquanto movia-se por uma
10m/s. Uma vez nessa superfície, o esquilo
trajetória parabólica depois de ter sido lançada
passa a apanhar nozes em seu percurso. Todo
obliquamente e livre de resistência do ar, uma
o movimento se dá sobre o gelo, de forma que
bomba de 400 g explodiu em três partes, A, B
o atrito pode ser desprezado. A massa do
e C, de massas mA = 200 g e mB = mc = 100 g.
esquilo é de 600g e a massa de uma noz é de
A figura representa as três partes da bomba e
40g.
suas respectivas velocidades em relação ao
a) Qual é a velocidade do esquilo após colher
solo, imediatamente depois da explosão.
5 nozes?
i vertical
l b) Calcule a variação da energia cinética do
I conjunto formado pelo esquilo e pelas nozes
à VB = 200 m/s entre o início e o final da coleta das 5 nozes.

B E25) (Fuvest-12)

Vc = 400 m/s v
horizontal O

T VA = 100 m/s
i
I
I
Maria Luisa
]

I Maria e Luísa, ambas de massa M, patinam no


I vertical gelo. Luísa vai ao encontro de Maria com
Analisando a figura, é correto afirmar que a velocidade de módulo V. Maria, parada na
bomba, imediatamente antes de explodir, tinha pista, segura uma bola de massa m e, num
velocidade de módulo igual a certo instante, joga a bola para Luísa. A bola
a) 100m/s e explodiu antes de atingir a altura tem velocidade de módulo v, na mesma
máxima de sua trajetória. direção de V. Depois que Luísa agarra a bola,
b) 100m/s e explodiu exatamente na altura as velocidades de Maria e Luísa, em relação
máxima de sua trajetória.
ao solo, são, respectivamente,
c) 200m/s e explodiu depois de atingir a altura
máxima de sua trajetória. NOTE E ADOTE
d) 400m/s e explodiu exatamente na altura V e v são velocidades em relação ao solo.
máxima de sua trajetória. Considere positivas as velocidades para a direita.
e) 400m/s e explodiu depois de atingir a altura Desconsidere efeitos dissipativos.
máxima de sua trajetória. a) 0; v-V
b) -v; v + V/2
E23) (Unicamp-05) No episódio II do filme c) -mv/M; M V/m
Guerra nas Estrelas, um personagem d) -mv/M; (mv - MV)/(M + m)
mergulha em queda livre, caindo em uma nave e) (MV/2 - mv)/M; (mv - MV/2)/(M + m)
que se deslocava horízontalmente a 100m/s
com os motores desligados. O personagem

270
_ ________________________________ Elementos da Física-Mecânica !-Impulso e Quantidade de Movimento
E26) (Fuvest-15) Um trabalhador de massa m
está em pé, em repouso, sobre uma E30) (AFA-11) Analise as afirmativas abaixo
plataforma de massa M. O conjunto se move, sobre impulso e quantidade de movimento.
sem atrito, sobre trilhos horizontais e retilíneos, I - Considere dois corpos A e B deslocando-se
com velocidade de módulo constante v. Num com quantidades de movimento constantes e
certo instante, o trabalhador começa a iguais. Se a massa de A for o dobro de B,
caminhar sobre a plataforma e permanece com então, o módulo da velocidade de A será
velocidade de módulo v, em relação a ela, e metade do de B.
com sentido oposto ao do movimento dela em II - A força de atrito sempre exerce impulso
relação aos trilhos. Nessa situação, o módulo sobre os corpos em que atua.
da velocidade da plataforma em relação aos III - A quantidade de movimento de uma
trilhos é luminária fixa no teto de um trem é nula para
(2rri4-M)v (2m 4- M)v (2m 4- M)v um passageiro, que permanece em seu lugar
a) b) c) durante todo o trajeto, mas não o é para uma
miM M m
(M-m)v (m 4- M)v pessoa na plataforma que vê o trem passar.
d) e) IV - Se um jovem que está afundando na areia
M M-m movediça de um pântano puxar seus cabelos
para cima, ele se salvará.
E27) (UFPE-99) Um canhão dispara uma bala
São corretas
cuja velocidade imediatamente após o disparo c) apenas III e IV.
a) apenas I e III.
é igual a 84 m/s. Devido à conservação da
b) apenas I, II e III. d) todas as afirmativas.
quantidade de movimento, o canhão recua
com a velocidade de 1,0 m/s. Calcule a razão E31) (ITA) Dadas 3 partículas e suas
entre a energia cinética da bala e a energia respectivas posições, m (x; y), em que m é a
cinética do canhão imediatamente após o massa em quilogramas, x e y as posições em
disparo.
metros, tais que 2 (3; 6), 4 (4; 4), 2 (1; 2).
- y (cm)
E28) (PUC/SP-05) O rojão representado na
figura tem, inicialmente, ao cair, velocidade
vertical de módulo 20m/s. Ao explodir, divide-
se em 2 fragmentos de massas iguais cujas 6
velocidades têm módulos iguais e direções que
formam entre si um ângulo de 120°. S
Dados: 4
sen30° = cos60° = 0,50; eD

cos30° = sen60° = 0,87 •C


2

o 2 4 6 x icm)
Indique qual dos pontos do gráfico representa
o centro de massa do sistema.
a) A b) B c) C d) D e) E

E32) (ITA-80) Uma bola de 1,0 x 10'1 kg tem


velocidade v, sendo v = 11 m/s, no instante
0 módulo da velocidade, em m/s, de cada
que é golpeada por um bastão e obrigada a
fragmento, imediatamente após a explosão,
será voltar com velocidade igual a -v. Supondo
a) 10 b) 20 c) 30 d) 40 e) 50 que o bastão esteve em contato com a bola
durante 3 x 10'2 s, calcular o valor médio da
E29) (AFA-99) Uma bola de 0,40 kg, força exercida pelo bastão sobre a bola.
movimentando-se horizontalmente com a) F = 73,3 N b)F = 3,7x10N
velocidade 14 m/s, é rebatida com uma força c) F = 36,6 N d) F = 3,67 x 10 N
média de 1000 N. Supondo-se que a colisão e)F = 7x10N
tenha durado 27 ms, a velocidade final da bola,
em m/s, é E33) (ITA-76) Um objeto, inicialmente em
a) 14,0. b) 32,5. c) 53,5. d) 81,5. repouso, explode em duas partes, A e B. com

271
___________________________________ fíementos da física-Mecânica!-Impulso e Quantidade de Movimento
massa M e 3M, respectivamente. Num ™1 ,
determinado instante t, após a explosão, a '”2
parte B está a 6,00 m do local da explosão.
Designando-se por x a distância entre A e B,
no instante y, e desprezando-se a influência de
outros corpos, pode-se afirmar que :
A) x = 18, 0 m
B) x = 8,0 m ^<30° 607\
/Z/Z/Z/////////////////////////
C) x = 24,0 m
Determine, em m/s, o módulo da componente
D) não é possível calcular x , pois t não foi
horizontal da velocidade do centro de massa,
dado
no instante t = 12^3 s. Considere os planos
E) N. D. R. A.
sem atrito e suficientemente longos de modo a
E34) (ITA-88) Um plano inclinado de ângulo a garantir que os blocos ainda estarão sobre
e massa M encontra-se em repouso numa eles no instante considerado. Considere g = 10
mesa horizontal perfeitamente lisa. Uma m/s2.
joaninha de massa m inicia a subida deste
plano inclinado a partir da mesa. Ela mantém F3) (UFC) Dois homens A e B, ambos de
em relação ao plano sua velocidade u massa M, estão nas extremidades de uma
constante. Determinar a velocidade do plano plataforma homogênea, de comprimento L =
inclinado. 2,16 m e massa 5M, que pode se deslocar
sobre uma superfície horizontal plana sem
atrito. O homem A joga uma bola de massa
M/5 para o homem B, que a segura
Exercícios de Fixação firmemente. Determine, em centímetros, 0
deslocamento da plataforma com relação à
F1) (UFC) A figura ao lado mostra uma peça posição inicial.
metálica plana, de espessura e densidade
uniformes. A parte horizontal tem comprimento F4) (UFPR-13) Recentemente, foi publicada
L e largura D e os ramos verticais têm em um jornal a seguinte ocorrência: um
comprimento C e largura D, cada um deles. Se homem pegou uma sacola plástica de
L = 98 cm e D = 16 cm, determine o valor do supermercado, encheu com um litro de água e
comprimento C, em centímetros, sabendo que abandonou-a do oitavo andar de um prédio. A
o centro de massa da peça está sobre a linha sacola caiu sobre um automóvel que estava
MN. Veja a figura. estacionado no nível da rua. Admitindo que
i, D D cada andar do prédio tenha uma altura de 2,5
I __________ I” “ m e que a sacola de água tenha sido freada
pelo capô do carro em aproximadamente 0,01
s, calcule o módulo da força normal média de
C c frenagem exercida pelo capô sobre a sacola.
Despreze a resistência do ar, o peso da sacola
vazia e correções referentes ao tamanho do
M N carro e ao fato de a sacola não se comportar
D exatamente como um corpo rígido.

F5) (FGV-07) Ao acender um isqueiro uma


L
J pessoa faz com que seu dedão exerça uma
força variável direcionada a três ações
F2) (UFC) Dois pequenos blocos, um de distintas:
massa m, e outro de massa m2 = 2m,, são I. É preciso vencer a força de atrito estático
abandonados simultaneamente no instante t = entre o rolete e a pedra a ele pressionada.
0 na parte superior de dois planos inclinados, II. Superado o atrito estático, a força aplicada
conjugados, como mostra a figura abaixo. não mais necessita ser de tamanho tão
elevado e, portanto, pode ser reduzida. Ainda
em contato com o rolete, o dedão desce e
começa a abaixar a alavanca que libera 0 gás.

272
_________________________________ Hementos da Física-Mecânica !-Impulso e Quantidade de Movimento
III. Uma vez livre do rolete e com a alavanca F7) (UFBA-13) Ao saltar-se de um lugar alto, é
que libera o gás completamente pressionada, comum dobrar os joelhos enquanto se encosta
a força é mantida constante durante o tempo no solo. Isso é feito de modo instintivo, a fim
que for necessário se ter a chama acesa. de minimizar a força de interação entre o chão
0 gráfico mostra, hipoteticamente, a e o corpo, diminuindo o impacto sobre a
intensidade da força exercida por uma pessoa articulação do joelho. Desprezando a
no ato de acender um isqueiro, para cada ação resistência do ar e considerando uma pessoa
descrita. de massa igual a 60,0kg caindo de uma altura

PI F(H)A de 80,0cm, em um local cujo módulo da


aceleração da gravidade é de 10m/s2, calcule

:0t
a diferença, em módulo, da força de impacto
entre o chão e o corpo, com e sem dobrar os
joelhos, sabendo que o tempo do impacto sem
dobrar os joelhos é de 0,25s e que, dobrando-

0 -I---- H 4---- I— os, é de 1,0 segundo.
0,5 1,0 tfcO
ações: I II iiT F8) (UFTM-12) Em um recente acidente de
Nessas condições, o impulso da força exercida trânsito, uma caminhonete de 1,6 tonelada, a
pelo dedão sobre o rolete do isqueiro e sobre a 144 km/h, atingiu outro veículo, em uma grave
alavanca que libera o gás até seu completo colisão frontal, e conseguiu parar somente a
abaixamento, tem intensidade, em N.s, de 25 metros de distância do abalroamento. A
a) 0,05. b)0,10. c)0,15. intensidade média da força resultante que agiu
d) 0,20. e) 0,25. sobre a caminhonete, do ponto do impacto ao
de paragem, foi, em newtons, igual a
F6) (Unesp-04) Um corpo de 6,0 kg, a) 51 200. b) 52 100. c) 65 000.
deslocando-se com velocidade v na direção e d) 72 400. e) 75 000.
sentido de um eixo x e livre de forças externas,
explode, separando-se em dois pedaços, A e F9) (UFG-08) Partindo da data de hoje,
B, de massas mA e mB, respectivamente. Após considere que, daqui a mil anos, um avião,
a explosão, A e B passam a se deslocar no voando a uma altura H com uma velocidade
plano xOy, afastando-se do ponto O com horizontal vh, sofra uma pane que o faz perder
velocidades v sua propulsão e, por isso, comece a cair com
vA
A e vB, respectivamente,
aceleração constante g. Um dos dispositivos
segundo as direções representadas
de segurança com que o avião será dotado
esquematicamente por linhas pontilhadas na
permitirá que, após perder 80% de sua altura,
figura.
seja ejetado verticalmente para baixo o
contêiner de bagagens e combustível, cuja
massa é 2/3 da massa total do avião. A
velocidade da parte ejetada é igual a 3/2 da
velocidade vertical deste, imediatamente antes
da ejeção. Considere que todas as velocidades
citadas são dadas em relação a um referencial
*x inercial fixo na Terra. Desprezando a
6,0 kg 0
resistência do ar e a ação de forças externas
na ejeção, calcule:
a) a redução percentual da velocidade vertical
do avião;
b) a razão entre a distância horizontal
a) Sendo v o módulo de v e sabendo que os percorrida pelo avião com o mecanismo de
módulos das componentes vetoriais de vA e segurança ativado e a distância que ele
vB na direção de x valem, respectivamente, percorrería sem ativar este dispositivo.
v/2 e 2v, determine as massas mA e mB.
F10) (Unifesp-14) Uma empresa de demolição
b) Sendo vAy e vBy, respectivamente, os
utiliza um guindaste, extremamente massivo,
módulos das componentes de vA e vB, na
que se mantém em repouso e em equilíbrio
direção de y, determine a razão vAy/vBy. estável no solo durante todo o processo. Ao
braço superior fixo da treliça do guindaste,

273
___ _______________________________ Elementos da Física-Mecânica !-Impulso e Quantidade de Movimento
ponto O, prende-se um cabo, de massa Antes de o gatilho ser acionado, os carrinhos e
desprezível e inextensível, de 10 m de a mola moviam-se juntos, sobre uma superfície
comprimento. A outra extremidade do cabo é plana horizontal sem atrito, com energia
presa a uma bola de 300 kg que parte do mecânica de 3,75J e velocidade de 1m/s, em
repouso, com o cabo esticado, do ponto A. relação à superfície. Após o disparo do gatilho,
e no instante em que a mola está totalmente
distendida, o carrinho B perde contato com ela
e sua velocidade passa a ser de 1,5m/s,
também em relação a essa mesma superfície.
Nas condições descritas, calcule a energia
potencial elástica inicialmente armazenada na
"---O--" mola antes de o gatilho ser disparado e a
^Çbooooo II ___ Ll—.^ velocidade do carrinho A, em relação à
Sabe-se que a trajetória da bola, contida em superfície, assim que B perde contato com a
um plano vertical, do ponto A até o ponto B, é mola, depois de o gatilho ser disparado.
um arco de circunferência com centro no ponto
O; que o módulo da velocidade da bola no F12) (Unicamp-13) As nuvens são formadas
ponto B, imediatamente antes de atingir a por gotículas de água que são facilmente
estrutura do prédio, é de 2 m/s; que o choque arrastadas pelo vento. Em determinadas
frontal da bola com o prédio dura 0,02s; e que situações, várias gotículas se juntam para
depois desse intervalo de tempo a bola para formar uma gota maior, que cai, produzindo a
instantaneamente. Desprezando a resistência chuva. De forma simplificada, a queda da gota
do ar e adotando g = 10 m/s2, calcule, em ocorre quando a força gravitacional que age
newtons: sobre ela fica maior que a força do vento
a) o módulo da força resultante média que atua ascendente. A densidade da água é págua =
na bola no intervalo de tempo de duração do 1,0.103 kg/m3.
choque. a) O módulo da força, que é vertical e para
b) o módulo da força de tração no cabo no cima, que certo vento aplica sobre uma gota
instante em que a bola é abandonada do esférica de raio r pode ser aproximado por
repouso no ponto A. vento FVento = b x r, com b = 1,6 * 10-3 N/m.
Calcule o raio mínimo da gota para que ela
F11) (Unesp-13) Um brinquedo é constituído comece a cair.
por dois carrinhos idênticos, A e B, de massas b) O volume de chuva e a velocidade com que
iguais a 3kg e por uma mola de massa as gotas atingem O o solo são fatores
desprezível, comprimida entre eles e presa importantes na erosão.
erosão, O volume é
apenas ao carrinho A. Um pequeno dispositivo, usualmente expresso pelo índice
também de massa desprezível, controla um pluviométrico, que corresponde à altura do
gatilho que, quando acionado, permite que a nível da água da chuva acumulada em um
mola se distenda. recipiente aberto e disposto horizontalmente.
Calcule o impulso transferido pelas gotas da
chuva para cada metro quadrado de solo
horizontal, se a velocidade média das gotas ao
chegar ao solo é de 2,5 m/s e o índice
pluviométrico é igual a 20 mm. Considere a
colisão como perfeitamente inelástica.

F13) (Unicamp-05) No episódio II do filme


antes do disparo Guerra nas Estrelas, um personagem
1,5m/s mergulha em queda livre, caindo em uma nave
que se deslocava horizontalmente a 100 m/s
com os motores desligados. O personagem
resgatado chegou à nave com uma velocidade
de 6 m/s na vertical. Considere que a massa
da nave é de 650 kg, a do personagem
resgatado de 80 kg e a do piloto de 70 kg.
depois do disparo

274
__________________________________ Mementos da Física - Mecânica /- Impulso e Quantidade zfe Movimento
a) Quais as componentes horizontal e vertical
da velocidade da nave imediatamente após o
resgate?
b) Qual foi a variação da energia cinética total .. Vi
nesse resgate?

F14) (Unicamp-10) O lixo espacial é composto Saída de água


por partes de naves espaciais e satélites fora Saída de agua
de operação abandonados em órbita ao redor
da Terra. Esses objetos podem colidir com
satélites, além de pôr em risco astronautas em
atividades extraveiculares. Considere que
durante um reparo na estação espacial, um
astronauta substitui um painel solar, de massa
mp = 80kg, cuja estrutura foi danificada. O
astronauta estava inicialmente em repouso em Entrada de água a
relação à estação e ao abandonar o painel no 30 litros/s
espaço, lança-o com uma velocidade vp =
0,15m/s. a) Qual é a energia potencial gravitacional, em
a) Sabendo que a massa do astronauta é ma = relação à superfície da água, de um piloto de
60kg, calcule sua velocidade de recuo. 60 kg, quando elevado a 10 metros de altura?
b) O gráfico ao lado mostra, de forma b) Considere que o volume de água por
simplificada, o módulo da força aplicada pelo unidade de tempo que entra na mangueira na
astronauta sobre o painel em função do tempo superfície da água é o mesmo que sai nos
durante o lançamento. Sabendo que a jatos do colete, e que a bomba retira água do
variação de momento linear é igual ao impulso, mar a uma taxa de 30 litros/s. Lembre-se que o
cujo módulo pode ser obtido pela área do Impulso I de uma força constante F, dado pelo
gráfico, calcule a força máxima Fmax. produto desta força pelo intervalo de tempo At
Ff de sua aplicação T = FAt, é igual, em módulo,
à variação da quantidade de movimento AQ do
objeto submetido a esta força. Calcule a
^max diferença de velocidade entre a água que
passa pela mangueira e a que sai nos jatos
quando o colete propulsor estiver mantendo o
piloto de m = 60 kg em repouso acima da
superfície da água. Considere somente a
massa do piloto e use a densidade da água
como p = 1 kg/litro.

0 H- t(s) F16) (Fuvest-05) Num espetáculo de fogos de


0,3 0,6 0,9
artifício, um rojão, de massa Mo = 0,5 kg, após
seu lançamento, descreve no céu a trajetória
F15) (Unicamp-15) Jetlev é um equipamento
indicada na figura. No ponto mais alto de sua
de diversão movido a água. Consiste em um
trajetória (ponto P), o rojão explode, dividindo-
colete conectado a uma mangueira que, por
se em dois fragmentos, A e B, de massas
sua vez, está conectada a uma bomba de
iguais a Mq/2. Logo após a explosão, a
água que permanece submersa. O aparelho
velocidade horizontal de A, VA, é nula, bem
retira água do mar e a transforma em jatos
como sua velocidade vertical.
para a propulsão do piloto, que pode ser
elevado a até 10 metros de altura (ver figura y(m)+
abaixo). 60
50
40
30
20
10
- x(m)
10 20 30 40 50 60 70 80

275
Mementos da Física-Mecânica !-Impulso e Quantidade de Movimento
NOTE E ADOTE: F19) (UEL-03) ____________
A massa do explosivo pode ser considerada Como o sistema Arrow protege Israel dos mísseis Scud do i
desprezível.
a) Determine o intervalo de tempo To, em
segundos, transcorrido entre o lançamento do
rojão e a explosão no ponto P.
b) Determine a velocidade horizontal VB, do t

fragmento B, logo após a explosão, em m/s.


O
c) Considerando apenas o que ocorre no Radares israelenses
momento da explosão, determine a energia Eo determinam a trajetória
do missil e localizam seu
fornecida pelo explosivo aos dois fragmentos A AVIÃO y ponto de partida. Aviões
e B, em joules. podem ser enviados para
atacar a plataforma de j
lançamento 1
F17) (Fuvest-99) Sobre a parte horizontal da i MÍSSIL • i MÍSSIL
| SCUD RADAR t- ' ARROW
superfície representada na figura, encontra-se | ISRAELENSE '4-
parado um corpo B de massa M, no qual está
presa uma mola ideal de comprimento natural (Revista Veja, n. 1773, 16 out. 2002.)
Lo e constante elástica k. Os coeficientes de Durante o lançamento de um míssil (Scud ou
atrito estático e dinâmico, entre o corpo B e o Arrow), há uma queima de combustível para
plano, são iguais e valem p. Um outro corpo A, que os gases, provenientes dessa queima,
também de massa M, é abandonado na parte sejam ejetados para fora do míssil, provocando
inclinada. O atrito entre o corpo A e a uma variação da velocidade. A variação da
superfície é desprezível. Determine: velocidade do míssil Av, devido
a) A máxima altura h0, na qual o corpo A pode exclusivamente à queima do combustível,
ser abandonado, para que, após colidir com o depende da massa inicial mu da massa final
corpo B, retorne até a altura original h0. mf, que atinge após a queima do combustível;
b) O valor da deformação x da mola, durante a e de v,, a velocidade de ejeção dos gases em
colisão, no instante em que os corpos A e B relação ao foguete, dada pela relação:
tem a mesma velocidade, na situação em que
o corpo A é abandonado de uma altura H > h0. Av = -v Inf—1.e
(Despreze o trabalho realizado pelo atrito
durante a colisão). Sobre o lançamento desses mísseis, considere
as seguintes afirmativas:
A
I. A relação de massas, representada pelo
quociente m,/mf, cresce exponencialmente à
B
S____ ,
AWkWWWWkx
h0 medida que se procura aumentar a velocidade
final atingida pelo míssil.
II. A velocidade final atingida pelo míssil é
muito inferior à que resultaria da relação
F18) (UFPE-06) Dois blocos A e B, de massas
vf - v, = -v In í— |, pois não estão sendo
mA = 0,2 kg e mB = 0,8 kg, respectivamente,
estão presos por um fio, com uma mola ideal \mí 7
comprimida entre eles. A mola comprimida consideradas as forças externas, tais como a
armazena 32 J de energia potencial elástica. resistência do ar e a força-peso gravitacional.
Os blocos estão inicialmente em repouso, III. Durante o lançamento do míssil, o momento
sobre uma superfície horizontal e lisa. Em um linear do míssil se conserva.
dado instante, o fio se rompe liberando os IV. As leis de Newton não se aplicam ao
blocos. Calcule a velocidade do bloco A, em movimento de um míssil, pois trata-se de um
m/s. sistema de massa variável.
mola São corretas apenas as afirmativas:
a) I e II. b) II!
III e IV. c) I e III.
d) II e IV. e) I e IV.
| |
B:
-1 F20) (EN-12) Dois veículos A e B percorrem a
mesma trajetória retilínea e horizontal (eixo
dos X). O veículo A (da frente), de massa mA =
'fio
20 kg, está sob a ação da força resultante

276
_ ________________________________ Elementos da física - Mecânica /- Impulso e Quantidade de Movimento
F(a> = 8,0.i(N)e o veículo B (detrás), de massa b) nenhum dos blocos escapa da região (1);
c) os dois blocos acabam de atingir a região
mB = 30 kg, está sob a ação da força resultante
(3) com energias cinéticas iguais;
F(b> = 9,0.T(N). No instante t = 0, temos: o d) o bloco B vai de (1) para (3), chegando ao
módulo da velocidade do veículo A é duas patamar da região (3) com cerca de 50 joules
vezes maior do que o módulo da velocidade do de energia cinética, enquanto que o bloco A
veículo B e a velocidade de A em relação a B é vai para a esquerda, voltando em seguida para
2,0 T ( m / s ). No instante t = 5,0 s, o módulo a direita indo atingir também a região (3) com
da velocidade (em m/s) do centro de massa do cerca de 50 joules de energia cinética;
sistema (A + B) é e) ao final, os dois blocos ficarão parados na
a) 4,5 b)4,0 c) 3,6 d) 3,2 e) 3,0 região (3).

F21) (ITA-61) Uma granada é lançada de um F23) (ITA-78) Um corpo de massa igual a 2,0
ponto 0, formando um ângulo A com a Kg acha-se em movimento retilíneo. Num certo
horizontal, como mostra a figura. No ponto de trecho de sua trajetória faz-se agir sobre ele
altura máxima da trajetória, ao explodir, as uma força que tem a mesma direção do
direções e sentidos (indicados pelas setas) movimento e que varia com o tempo, conforme
que seus fragmentos podem tomar, são: a figura abaixo. Neste trecho e nestas
condições, pode-se afirmar que a variação da
velocidade “Av” do coroo será dada oor:
F(n)

40

20

O
3"

-20
I) quando ela explode em duas partes.
II) quando ela explode em três partes/ A) Av = 2,5 m/s; B) Av = 5,0 m/s;
III) quando ela não explode. C) Av = 8,0 m/s; D) Av = 2,0 m/s;
A) a, 5 B) e, (j, S C) e E) Av = 4,0 m/s.
D)p, E)p, S F)a, S.E,
G) Nenhuma das direções acima. F24) (ITA-81) No barco da figura há um
homem de massa 60 kg subindo uma escada
F22) (ITA-66) A superfície cujo perfil está solidária ao barco e inclinada de 60° sobre o
esquematizado na figura mostra três regiões plano horizontal. Sabe-se que os degraus da
planas, horizontais. A região (2) está 2,00m escada estão distanciados de 20 cm um do
acima de (1); e a (3), 1,00m acima de (1). Os outro e que o homem galga um degrau por
blocos A e B, cada um dos quais tem massa segundo. A massa total do sistema barco
de 5,0 kg , estão inicialmente na região (1), mais escada é 300 kg. Sabendo que
separados, mas não ligados, por uma mola inicialmente o barco e o homem estavam em
comprida que armazena 120 joules de energia repouso em relação à água, podemos concluir
potencial elástica. Supondo que esses blocos que o barco passará a mover-se com
podem se mover sem atrito sobre a superfície velocidade de:
e que a aceleração da gravidade vale 10,0
m/seg2, pode-se afirmar que, depois que a
mola se expandir:
(2}


(3)
( ) A. 10 cm/s ; ( ) B. 2,0 cm/s ;
( ) C. 2,5 cm/s ; ( ) D. 10 cm/s ;
(1)
( ) E. 1,66 cm/s;
a) o bloco A fica oscilando na região (1),
enquanto que o bloco B atinge a região (3) F25) (ITA-88) As massas m, = 3,0 kg e m2 =
com cerca de 50 joules de energia cinética; 1,0 kg, foram fixadas nas extremidades de

277
__________________________________ Fiementos da Física-Mecânica !-Impulso e Quantidade de Movimento
uma haste homogênea, de massa desprezível de 25 m acima do chão. No mesmo instante,
e 40 cm de comprimento. Este sistema foi uma segunda bola, com massa de 0,25 kg, é
colocado verticalmente sobre uma superfície lançada verticalmente para cima, a partir do
plana, perfeitamente lisa, conforme mostra a chão, com uma velocidade inicial de 15 m/s.
figura, e abandonado. A massa m, colidirá As duas bolas movem-se ao longo de linhas
com a superfície a uma distância x do ponto muito próximas, mas que não se tocam. Após
P dada por: 2,0 segundos, a velocidade do centro de
massa do sistema constituído pelas duas bolas
m,
é de:
0,5Kg

( ) A. x = 0
_____ Q
(no ponto P)
40 c-m

^//Z/zZ/ZÓ
rq
I í
25 m
( ) B. x = 10 cm
i iv
(
(
(
)
)
)
C.
D.
E.
x
x
x
=
=
=
20
30
40
cm
cm
cm
—Lm 0,25 Kg
(A) 11 m/s, para baixo. (B) 11 m/s, para cima.
F26) (ITA-95) A figura mostra o gráfico da (C) 15 m/s, para baixo. (D) 15 m/s, para cima.
força resultante agindo numa partícula de (E) 20 m/s, para baixo.
massa m, inicialmente em repouso.
F29) (ITA-05) Dois corpos esféricos de massa
F, M e 5M e raios R e 2R, respectivamente, são
liberados no espaço livre. Considerando que a
ti b única força interveniente seja a da atração
o t gravitacional mútua, e que seja de 12R a
f2 distância de separação inicial entre os centros
dos corpos, então, o espaço percorrido pelo
No instante t2 a velocidade da partícula, V2 corpo menor até a colisão será de
será: a) 1,5 R.
a) v2 = [(Fi + F2) t, - F212] / m b) 2,5 R.
b) V2 = [( F, - F2) t, - F212] / m c) 4,5 R.
c) V2 = [( F-i - F2) t, + F212] / m d) 7,5 R.
d) V2 = ( F, t, - F2 t2)/ m e) 10,0 R.
e) V2 = [(t2-t1)(F1-F2)]/2m
12R
F27) (ITA-98) Uma massa m em repouso
divide-se em duas partes, uma com massa F30) (OBF-06) Uma bola de chumbo de massa
2m/3 e outra com massa m/3. Após a divisão, mB igual a 5kg é lançada com uma velocidade
a parte com massa m/3 move-se para a direita vB que faz com que ela caia e fique imobilizada
com uma velocidade de módulo v,. Se a dentro de um carrinho, conforme mostrado no
massa m estivesse se movendo para a desenho. O carrinho tem massa mc igual a
esquerda com velocidade de módulo v antes 10kg e se move com velocidade constante vc =
da divisão, a velocidade da parte m/3 depois 5 m/s.
da divisão seria: 10m
4--------
a) Para a esquerda _ VB
O------
b) (v, - v) para a esquerda.
c) (vt - v) para a direita.
vc
d) Para a direita.

e) (v, + v) para a direita. ! \ Í


o 0 ......
ô
F28) (ITA-00) Uma bola de 0,50 kg é De posse desses dados:
abandonada a partir do repouso a uma altura

278
Mementos da física-Mecânica !-Impulso e Quantidade de Movimento
a) calcule o valor da velocidade vB com que a A2) (UFTM-12) Num trecho plano e horizontal
bola colide com o carrinho; de uma estrada, um carro faz uma curva
b) calcule a velocidade v com que o carrinho mantendo constante o módulo da sua
se movimentará após ter recebido a bola de velocidade em 25 m/s. A figura mostra o carro
chumbo. em duas posições, movendo-se em direções
que fazem, entre si, um ângulo de 120°.
F31) Um projétil disparado por um canhão, no Considerando a massa do carro igual a 1000
ponto mais alto de sua trajetória (a uma kg, pode-se afirmar que, entre as duas
distância A do canhão medida posições indicadas, o módulo da variação da
horizontalmente), explode dividindo-se em dois quantidade de movimento do veículo, em (kg.
fragmentos iguais. Um dos fragmentos é m)/s, é igual a
arremessado para trás com a mesma
velocidade que tinha o projétil antes de
explodir. A que distância do canhão cairá o
segundo fragmento?

F32) Um homem de massa M está de pé sobre


uma tábua de comprimento L, que se encontra
em repouso numa superfície sem atrito. O
homem caminha de um extremo ao outro da
tábua. Que distância percorreu o homem em
relação ao solo se a massa da tábua é M/4?
tV
F33) Na superfície de um lago encontra-se
uma canoa, que está situada
perpendicularmente a linha da margem a tem a
proa dirigida a mesma. A distância entre a proa
a) 10000. b) 12500. c) 25000.
da canoa e a margem é igual a 0,75 metros.
No momento inicial a canoa estava imóvel. O d) 12500x/2. e) 25000^2
homem que se encontra na canoa, passa da
proa para a popa. Atracará a canoa na A3) (UFES-10) Uma mola ideal de constante
margem, sendo o seu comprimento 2 metros? elástica k lança dois blocos unidos por um
A massa da canoa é M = 140 kg, e a massa do dispositivo de massa desprezível. O bloco
homem é m = 60 kg. mais próximo da mola tem massa Meo outro
tem massa 3M. Após o lançamento, os blocos
se movem sobre uma superfície plana,
horizontal e lisa.
Exercícios de Aprofundamento M 3M
Xr
A1) (UFC-96) Numa placa retangular de rnir
100cm x 200cm, são cortados setores I r
circulares, todos de mesmo raio, resultando na A) Sabendo que a mola estava comprimida de
peça mostrada na figura. A placa tem Xo antes do lançamento, determine o módulo
espessura uniforme e é construída de um da velocidade dos blocos após o lançamento.
material homogêneo. Determine, em
centímetros, as coordenadas x e y, do centro Em um determinado instante, após o
de massa da peça. lançamento, o dispositivo (explosivo) que une
y(«n)
os blocos é acionado, lançando o bloco de
ioo-I _________________ massa M de volta contra a mola.
M 3M
k

11 ........
30- ♦H wi
B) Sabendo que o bloco de massa M, ao

50 100 150 200 x(cm) retornar, comprime a mola de — , determine


4
os módulos das velocidades dos blocos de

279
Flementos da Física-Mecânica i-Impulso e Quantidade de Movimento
massa M e de massa 3M imediatamente após eixo x), ficando com velocidade de 30 m/s. A
a separação. O bloco de massa 3M, após a energia potencial inicialmente armazenada na
separação, continua movendo-se no mesmo mola, em joules, era de
sentido até chegar a uma região da superfície
y
não lisa AB, muito extensa.
M 3M

mm
k
a A B
O () t > ()

a) 2570 b) 2640 c) 2940 d) 3750


>x

C) Sabendo que o coeficiente de atrito cinético


entre a região não lisa e o bloco de massa 3M A7) (EN-15) Um artefato explosivo é lançado
é p, determine a distância percorrida por esse do solo com velocidade inicial v0 fazendo um
bloco na região não lisa. ângulo de 30° com a horizontal. Após 3,0
segundos, no ponto mais alto de sua trajetória,
<4) (UFI
A4) (UFBA-08) o artefato explode em duas partes iguais,
F —>v?.
| m —♦
sendo que uma delas (fragmento A) sofre
m, m, apenas uma inversão no seu vetor velocidade.
Desprezando a resistência do ar, qual a
Na figura, o carrinho de massa m, = 10,0kg distância, em metros, entre os dois fragmentos
move-se com velocidade v-io = 3,0m/s. Em quando o fragmento A atingir o solo?
certo momento, lança-se, horizontalmente, Dados: sen 30° = 0,5
sobre ele um bloco de massa m2 = 2,0kg, com cos 30° = 0,9
velocidade inicial v2o = 5,0m/s. A força de atrito g = 10m/s2
entre o bloco e o carrinho faz com que, após a) 280 b) 350 c)432 d) 540 e) 648
algum tempo, ocorra o repouso relativo entre
ambos. Desprezando as perdas de energia A8) (ITA-92) No dispositivo da figura, bolas de
ocasionadas pelos atritos com o ar e entre o gude de 20 g cada uma estão caindo, a partir
carrinho e o solo, determine a velocidade final do repouso, de uma altura de 1 metro, sobre a
do conjunto e a perda da energia dissipada plataforma de uma balança. Elas caem a
pelo atrito entre o carrinho e o bloco. intervalos de tempos iguais At e após o choque
estão praticamente paradas, sendo
A5) (UFPE-00) Dois carros, A e B, inicialmente imediatamente retiradas da plataforma.
em repouso, podem mover-se livremente na Sabendo que o ponteiro da balança indica, em
direção x. A massa do carro A é 4,0kg e a do média, 20 kg, e que a aceleração da gravidade
carro B, 2,0kg. Eles estão unidos, vale 10m/s2, podemos afirmar que a freqüência
comprimindo uma mola, conforme a figura. de queda é:
Quando a corda que os mantém unidos é o o o o
queimada e se rompe, o carro A se desloca
com a velocidade de 3,0 m/s em relação ao 1o
solo. Qual a energia, em J, que estava 1 m
armazenada na mola entes do rompimento da
corda?
A
B —
jJ J 4 j wwrjj! t j, ! a) V20 bolas por segundo
///Z////Z///////////// ////////
b) 20>/5 bolas por segundo
c) 1/60 bolas por segundo
A6) (AFA-01) Dois carrinhos A e B de massas d) 103V5 bolas por segundo
mA = 8 kg e mB = 12 kg movem-se com e) 102 bolas por segundo
velocidade v0 = 9 m/s, ligados por um fio ideal,
conforme a figura. Entre eles existe uma mola A9) (ITA-85) Um atleta de massa 60,0 kg
comprimida, de massa desprezível. Num dado carregando um corpo de 15,0 kg dá um salto
instante, o fio se rompe e o carrinho A é de inclinação 60 °, em relação ao plano
impulsionado para a frente (sentido positivo do horizontal com velocidade inicial 10,0 m/s. Ao

280
__________________________________ ílementos da física-Mecânica !-Impulso e Quantidade de Movimento
atingir a altura máxima lança horizontalmente A) a joaninha não irá se deslocar. Somente o
para trás o corpo com velocidade 2,00 m/s em anel adquirirá um movimento de rotação em
relação ao centro-de-massa do sistema torno de seu centro de simetria.
formado por ele próprio mais o corpo. B) a joaninha descreverá órbitas circulares em
Adotando para a aceleração da gravidade o torno do centro do anel, enquanto que o anel
valor g = 10,0 m/s2, podemos afirmar que o girará em sentido contrário em torno do seu
atleta ganhará em alcance horizontal a centro.
distância: C) a joaninha e o centro de massa (C.M.) do
A) 0,8773 m B) - 0,2573 m C) 0,2573 m sistema descreverão respectivamente órbitas
D) 1,2573 m E) zero. circulares de raios r = R e R Cm = mR/(m + M).
D) o centro de massa (C.M.) do sistema
A10) (ITA-72) Uma bola de tênis, de massa permanecerá em repouso, enquanto que a
igual a 100 g, é atirada contra uma parede, joaninha descreverá órbitas circulares de raio r
onde chega horizontalmente com a velocidade = MR/(m + M).
de 20 m/s. Refletindo na parede ela volta com E) nenhuma das afirmações acima está
a mesma velocidade horizontal. Sabendo-se correta.
que a força média devida à parede que atua
sobre a bola durante o impacto é de 40 N, qual A13) (ITA-02) Uma rampa rolante pesa 120 N
é, aproximadamente, a variação da quantidade e se encontra inicialmente em repouso, como
de movimento que a bola sofre na vertical mostra a figura. Um bloco que pesa 80N,
devido a ação da gravidade, no intervalo de também em repouso, é abandonado no ponto
tempo do impacto ? 1, deslizando a seguir sobre a rampa. O centro
A) 4, 0 kg . m.s ~1 D) 0. 04 kg . m.s de massa G da rampa tem coordenadas:
B) 0, 4 kg . m.s -1 E) 10 kg . m.s "1 2b c
C) 0, 1 kg . m.s ’1 Xr =— e y
’■ G =
—— . São dados ainda: a =
G 3 '^3
15,0m e sen a = 0,6. Desprezando os
A11) (ITA-91) Segundo um observador possíveis atritos e as dimensões do bloco,
acoplado a um referencial inercial, duas pode-se afirmar que a distância percorrida pela
partículas de massa mA e mB possuem rampa no solo, até o instante em que o bloco
velocidades vA e vB, respectivamente. Qual a atinge o ponto 2, é:
quantidade de movimento pA que um
observador preso ao centro de massa do
sistema mede para a partícula A ?
a) Pa= mA vA
b) Pa= mA (vA-vB)
2
C)pa = ma.m O
Ma+M,

d) pa = f MA.MB ÍVa-Vb) a) 16,0 m b) 30,0 m c) 4,8 m


Ima+Mb d) 24,0 m e) 9,6 m
e) Nenhuma das anteriores.
A14) (ITA-78) Um garoto pode deslizar sobre
A12) (ITA-86) Sobre uma superfície um escorregador solidário com um barco, a
perfeitamente lisa, encontra-se em repouso um partir de um altura H. O plano do escorregador
anel de massa M e raio R. Sobre este anel forma um ângulo 30° com o plano horizontal. A
encontra-se em repouso uma joaninha de massa m do garoto é igual a metade da massa
massa “m”. Se a joaninha caminhar sobre o M do conjunto barco/escorregador. Supondo
anel, podemos afirmar que : que o sistema inicialmente esteja em repouso
M e desprezando os atritos, no instante em que o
garoto atingir o ponto A, a velocidade do barco
será dada por:

281
[/ementas da Física-Mecânica !-Impulso e Quantidade de Movimento
M p
m=T
W-- a
A
o '

Despreze a resistência do ar e calcule:


a) o valor das coordenadas do ponto de
3gH explosão;
a) v b) v = 0 c) v =
14 b) as coordenadas de posição A(xa;Va) do
fragmento “a” no instante em que o fragmento
e) v = ^7gH “b”, 1,0 segundo após a explosão, toca o solo
em um ponto B(xB;yB), cuja posição é dada
pelas coordenadas (3000 m; 300 m);
A15) (ITA-13) Uma rampa maciça de 120 kg c) o valor, em N, da força F da explosão,
inicialmente em repouso, apoiada sobre um constante, de duração 1 ms e que atuou no
piso horizontal, tem sua declividade dada por fragmento A. (deixar indicada a raiz quadrada)
tan6 = %. Um corpo de 80kg desliza nessa
rampa a partir do repouso, nela percorrendo 15 A18) (Olimpíada de Física do Ceará-97) Uma
m até alcançar o piso. No final desse percurso, explosão divide uma pedra que se encontrava
e desconsiderando qualquer tipo de atrito, a inicialmente em repouso em três partes de
velocidade da rampa em relação ao piso é de massas ith = m2 = 20 kg e m3 = 40 kg.
aproximadamente Supondo que os fragmentos de massas iguais
a) ( ) 1 m/s. b) ( ) 3 m/s. c) ( ) 5 m/s. se movam com velocidade de 20 m/s nos
d) ( ) 2 m/s. e) ( ) 4 m/s sentidos mostrados na figura, determine o
módulo e o sentido da velocidade do terceiro
A16) (IME-04) Um tanque de guerra de massa fragmento.
M se desloca com velocidade constante v0.
Um atirador dispara um foguete frontalmente
contra o veículo quando a distância entre eles
é D. O foguete de massa m e velocidade X
constante vf colide com o tanque, alojando-se
em seu interior. Neste instante o motorista freia
com uma aceleração de módulo a. Determine:
1-o tempo t transcorrido entre o instante em
que o motorista pisa no freio e o instante em A19) (Seletiva IPhO-02) Um bloco cúbico de
que o veículo pára; massa M, arestas L, com um pequeno furo de
2 - a distância a que, ao parar, o veículo formato circular, de raio L em relação ao
estará do local de onde o foguete foi ponto O, encontra-se sobre uma superfície
disparado. plana. Uma partícula de massa m, velocidade
v é atirada frontalmente contra o cubo, de
A17) (OBF-06) O canhão mostrado dispara forma a penetrar neste. Desprezando todas as
uma granada de massa m = 6,00kg da posição possíveis forças de atrito e sabendo-se que a
O(xo;yo) = (0 m; 0 m) que atinge seu ponto partícula atinge uma altura h acima do cubo,
mais alto na posição P(xP;yP) de coordenadas calcule o valor da velocidade inicial com que a
(3000 m;1125 m). Decorridos 20,Os após o partícula foi atirada.
disparo, a granada explode e seus fragmentos O
“a” e “b” de massas iguais a ma = 2,00 kg e mb
= 4,00kg, respectivamente, caem segundo
trajetórias coplanares à trajetória anterior à L
explosão. m
v

A20) Na figura, os blocos 1 e 2 têm massas


respectivamente iguais a 2,0 kg e 4,0 kg e
acham-se inicialmente em repouso sobre um
282
____________________________ Elementos da física - Mecânica / - Impulso e Quantidade de Movimento
plano horizontal e liso. Entre os blocos existe vem logo atrás, num carrinho idêntico, à
uma mola leve de constante elástica 1,5.102 mesma velocidade. Para salvar a mocinha, o
N/m, comprimida de 20 cm e impedida de mocinho pula de um carrinho para o outro, com
distender-se devido a uma trava. Num dado uma velocidade de 6 m/s em relação ao
instante, a trava é liberada e a mola, ao carrinho que deixa para trás. Calcule a
distender bruscamente, impulsiona os blocos velocidade de cada um dos carrinhos depois
que, depois de percorrerem as distâncias que a mocinho já atingiu o carrinho da frente.
indicadas, colidem com os anteparos. Não
considerando a resistência do ar, determine a A24) O balconista de uma mercearia, para
relação entre os tempos gastos pelos blocos 1 atender a um cliente que pediu 200 g de creme
e 2 para atingirem os respectivos anteparos. de leite fresco, coloca o recipiente vazio sobre
L anteparo 2 uma balança de mola, acerta o zero e despeja
o creme sobre o recipiente desde uma altura
(1) (2) de 75 cm. Depois de 2 s, com a balança
marcando 200 g, o balconista, mais que
depressa, retira o recipiente de cima da
2d <-
balança. Que quantidade de creme o cliente
realmente leva?
A21) Na figura a seguir está representado O
perfil de um carrinho, no interior do qual há um A25) Uma rã de massa m está parada no
plano inclinado. Neste plano estão cinco extremo de uma tábua de massa M e
esferas idênticas apoiadas na trava T. comprimento L. A tábua está flutuando na
Inicialmente o sistema está em repouso. superfície de um lago. A rã salta, formando um
Retirando-se a trava, as esferas se movem e ângulo a em relação à horizontal na direção da
saem pela abertura A. Depois que todas as tábua. Qual deve ser a velocidade inicial da rã
esferas passam por A, qual a velocidade v0, para que depois do salto a rã encontre-se
escalar do carrinho? A massa de cada esfera é no outro extremo da tábua?
igual a 1/5 da massa do carrinho.
A26) Duas lâminas, cujas massas são iguais a
m, estão ligadas através de uma mola de
coeficiente de rigidez k. A lâmina superior foi
2H comprimida para baixo, o suficiente, para que
A a deformação da mola fosse igual a x, sendo
depois liberada. Determinar a que altura
'zzzzzzzz/}Jbzzzzzzzzzzzzzzz)zzzzzzzzzzzzzzz elevar-se-á depois disso o centro de massas
do sistema.
m i i
A22) Um corpo A de massa m, desloca-se na
direção de outro B, como se vê na figura k
abaixo. O corpo B de massa igual M está em
repouso e o corpo A possui uma velocidade
inicial v0. Se desprezarmos qualquer atrito e
considerarmos a aceleração da gravidade g, a
altura máxima que o corpo A subirá em B é:
A27) A figura abaixo corresponde a um
sistema que se compõe de um corpo de massa
B Vo m, um prisma triangular deitado, de massa M e
{ IA um plano horizontal sem atrito dotado de uma
W////z7/zX^
cavidade D. O plano inclinado forma com a
horizontal um ângulo 0. É dada a distância CD
A23) No fundo de uma mina abandonada, O = a. Abandonando o corpo em repouso, este
vilão, levando a mocinha como refém, é desliza ao longo do plano e cai na cavidade D.
perseguido pelo mocinho. O vilão, de 70 kg, Determinar a altura h do plano inclinado.
leva a mocinha de 50 kg, dentro de um
carrinho de minério de 540 kg, que corre com
atrito desprezível sobre um trilho horizontal, à
velocidade de 10 m/s. O mocinho, de 60 kg,

283
ílementos tia física - Mecânica /- Impulso e Quantidade de Movimento
horizontal e está encostado em uma parede
A vertical. Um pequeno corpo de massa m2
desliza sem atrito desde sua posição inicial,
h como mostra a figura. Determine a velocidade
máxima adquirida pelo bloco de massa nh,
C ______ devido à interação com o corpo de massa m2.

I
L—
A28) Duas superfícies inclinadas móveis w2
possuem a mesma massa M e são
suavemente conjugadas com o solo horizontal.
Um bloco de massa m é largado de uma altura
h sobre a superfície inclinada móvel da
esquerda. Qual a máxima altura alcançada mi
pelo bloco na superfície inclinada da direita? O
atrito deve ser desprezado.
A31) Os corpos, de massas M, e M2, ligados
por um fio inextensível, que passa pela roldana
A, deslizam sobre as superfícies laterais de
h uma cunha retangular, cuja base BC se apoia
num plano horizontal liso. Calcular o
M M deslocamento da cunha pelo plano horizontal
durante a descida da carga M-, de uma altura h
= 10 cm. A massa da cunha é M = 4Mt = 16M2.
Menosprezar as massas do fio e da roldana.

A29) A figura mostra a mão de um jardineiro


segurando o bico de uma "mangueira" de regar
jardins e o jato de água da mesma batendo em
uma parede e sendo espalhado
perpendicularmente ao bico da mangueira.
Supondo o escoamento igual a 1,0 kg de água
por segundo, a velocidade da água no interior
da mangueira vE igual a 0,25 m/s e a
velocidade da água ao sair pelo bico vs igual a A32) Um corpo de massa M, com uma
2,0 m/s, pede-se determinar: superfície semicilíndrica de raio R, está em
repouso sobre um solo horizontal liso, em um
local onde a aceleração da gravidade é g.
Determine a mínima velocidade v que deve ser
I
VS imposta a um corpo de massa m de modo que
í LJ este consiga chegar ao topo do corpo de
A. massa M.

muv ) M |2Z?

a) o valor da força horizontal que o jardineiro A33) Um bloco de massa M, onde foi cavada
exerce para equilibrar a força associada à em sua superfície superior uma semiesfera de
mudança de velocidade da água no bico da raio R, está repousando sobre uma superfície
"mangueira"; horizontal sem atrito, em um local onde a
b) o valor da força de reação exercida pela gravidade vale g. Um ponto material, de massa
parede contra o jato de água. m, é abandonado, sem velocidade inicial, em
um ponto P, ponto mais alto do bloco e
A30) Um bloco simétrico de massa m,, com tangenciando a superfície semiesférica.
um furo hemisférico de raio r na superfície
superior, repousa sobre uma superfície

284
Elementos da física - Mecânica /- Impulso e Quantidade de Movimento
p na mesma direção que a parede dianteira do
carrinho. Determine o valor de D para que este
experimento seja realizado com sucesso.
n. Considere que ao perder contato com o
carrinho o corpo possui somente velocidade
M horizontal. Despreze as dimensões do corpo m
e das rodinhas do carrinho.
a) Determine a velocidade do bloco em função
— i—</ m
do ângulo <p.
b) Determine a força normal que o bloco M h
exerce sobre o ponto material em função do M
ângulo <p.

A34) A partir de um certo instante t = 0 uma


força F = kt (k uma constante) é aplicada sobre
um pequeno corpo de massa m que repousava
sobre um plano liso horizontal. A direção
permanente da força forma um ângulo a com a
horizontal. Determine a velocidade do corpo no A37) Um carrinho, de massa M, é composto di
momento em que este perde contato com o duas partes, uma em formato de um quarto de
solo. círculo de raio R e outra reta, conforme indica
F a figura. A passagem da parte curva para a
parte reta é suave. Um corpo de massa m é
abandonado de uma altura 2R acima da parte
m reta do carrinho. Assuma que o corpo m atinge
o carrinho tangencialmente a sua parte curva.
Suponha também que não existe atrito entre o
corpo mea parte curva do carrinho e que o
A35) Um homem de massa M está sentado coeficiente do corpo m com a parte reta do
sobre um monte de gelo de forma hemisférica carrinho seja igual a p.
de raio R, conforme indica a figura abaixo. Se
ele começa a deslizar a partir do repouso
m■ A

(considere o gelo sem atrito) determine a


distância entre o ponto onde o homem atinge o
2R
solo e o centro do hemisfério.
\R

I li
I
M A

O corpo m desliza sobre a superfície do


carrinho, parando apenas em sua extremidade
esquerda. Sendo g a aceleração da gravidade
local, calcule o valor do comprimento total da
carrinho.

A36) Um garoto deseja construir um carrinho A38) Um corpo de massa 400 g está preso a
de madeira para testar um dispositivo um fio de comprimento 1,5 m, girando num
mecânico. Para tanto, ele possui todas as plano horizontal em MCU com <o = ™ rad/s,
madeiras que serão usadas para montar as
paredes do carrinho, faltando apenas ajustar o constituindo um pêndulo cônico. Uma força
valor da altura D da parede lateral dianteira. tangente à trajetória passa então a ser
Os demais comprimentos estão indicados na aplicada sobre o corpo até que ele adquira a
figura abaixo. O garoto deseja colocar um n /o A
velocidade angular de —~ rad/s, atingindo
corpo de massa m sobre o carrinho (cuja
massa total é M) de modo que depois que o um novo MCU. Determine o trabalho desta
corpo se desprenda do carrinho acabe caindo força neste intervalo de tempo, sabendo que g
dentro de um buraco que inicialmente estava = 10m/s2.

285
Mementos da física-Mecânica i-Impulso e Quantidade de Movimento
a) 0,6 J b) 0,9 J c)1,3J a) 2 m/s b) 4,/Õj3 m/s c) 7Í1 m/s
d)1,5J e)1,9J d) 273 m/s e) 4 m/s

A39) Um canhão está rigidamente fixado em A42) Três corpos esféricos A, B e C, de


um carrinho que se encontra em repouso massas respectivamente iguais a M, 3M e 4M,
sobre um trilho sem atrito. Ao ser disparada, a raios respectivamente iguais a R., 2R e R, e
bala de massa m ganha velocidade v (em cargas respectivamente iguais a QA, Qb e Qc.
relação ao solo). Sendo a aceleração da são liberados no espaço livre. Considere que
gravidade g e a massa do conjunto carrinho- as únicas forças intervenientes sejam a
canhão M, determine em função de a, v, m, M gravitacional e a elétrica. Após a liberação, A e
o tamanho mínimo do carrinho para que a bala B se movem ao encontro um do outro, A para
caia sobre o mesmo. a direita e B para a esquerda. Seja 6R a
v distância de separação inicial entre A e B, bem
como entre B e C. Sabendo que, no instante
/?.' em que A e B colidem, o deslocamento de A é
o dobro do de C, determine, nesse momento, o
__ -LT7.. deslocamento de C:
A c
A40) Na figura, a partir do repouso, cai água, a
4M
um fluxo de 2,5 L/s, sobre um bloco, de uma R| ,
altura de 4 m acima do solo. O bloco, de
massa 1 kg, está fixado numa mola de
constante elástica 250 N/m e comprimento 6R 6R
natural 1 m. Sabendo que g = 10 m/s2, que o
a) 2,25R para a esquerda
choque entre a água e o bloco é elástico, que
b) 4,5R para a esquerda
a água não se acumula sobre o bloco e que o
c) 2,25R para a direita
bloco está em equilíbrio, determine a
d) 1,5R para a direita
deformação da mola (despreze as dimensões
e) 0,75R para a direita
do bloco):

1 4m
A43) O vagão da figura tem massa M e os
atritos nas rodas podem ser desprezados. O
pêndulo preso ao teto do vagão tem massa m
e comprimento L. Inicialmente, na posição
mostrada na figura, pêndulo e vagão estão em
repouso. Largando o pêndulo pede-se
determinar:
a) a velocidade máxima atingida pelo vagão;
A41) Um carro de massa M pode se mover b) o trabalho da tração do fio sobre o pêndulo
sem atrito em trilhos horizontais. Sobre o carro desde o instante em que foi largado até o carro
fixou-se um pêndulo simples (uma bola de assumir sua velocidade máxima.
massa m, pendurada em uma corda de
comprimento igual a 2,4 m). No momento V <?
inicial, o carro e o pêndulo estavam em
repouso e a corda foi inclinada num ângulo de
60° em relação à vertical. Determine a
velocidade do carro no instante em que a
corda fizer um ângulo igual a 0o com a linha
vertical, sabendo que M = 2m e que g = 10
O _C>
W/7>//////////////////7////y/7.
m/s2.
A44) Um grande cubo de isopor, de massa M
e aresta L = 30,0 metros, repousa sobre uma
M superfície horizontal perfeitamente lisa. Um
projétil de massa m = (1/5)M é disparado
y-------- ‘TY
horizontalmente contra o cubo atingindo-o com
7/7/77/777/M7//////////á///////////////7 velocidade v = 300 m/s, perpendicularmente a
uma de suas faces. O projétil atravessa o cubo

286
Mementos da Física - Mecânica !-Impulso e Quantidade de Movimento
e sai perpendicular à face oposta, com posição mais baixa, o corpo terá uma
velocidade u = 200 m/s. Suponha que a força velocidade relativa ao tubo, em cm/s, igual a
de resistência (atrito) que atua sobre o projétil,
enquanto ele atravessa o cubo, é constante.

M V.= 0 Aí 11

m
antes depois_______
Calcule a distância D que o cubo percorre
enquanto dura a travessia. __________________________ X
a) 1,00 m b) 1,25 m c) 1,50 m 777 l i l i ! I i i i ! ! 1 i ! t ! Y
d) 1,75 m e) 2,00 m A)-11,3. B)-206. C)11,3.
D) 206. E) 194.
A45) Um caixote tendo uma massa de 50 kg
repousa apoiado contra um bloco fixo que A48) (IME-18) Um veículo de combate tem,
impede o caixote de se mover plano abaixo. como armamento principal, um canhão
Se o coeficiente de atrito entre o plano e o automático eletromagnético, o qual está
caixote é p = 0,3, determine o tempo municiado com 50 projéteis. Esse veículo se
necessário para a força F conferir ao caixote desloca em linha reta, inicialmente, em
uma velocidade de 2 m/s plano acima. F velocidade constante sobre um plano
sempre age paralelamente ao plano e tem um horizontal. Como o veículo está sem freio e
módulo F = (300t) N, onde t é medido em descontrolado, um engenheiro sugeriu
segundos. executar disparos a fim de reduzir a velocidade
F do veículo. Após realizar 10 disparos na
mesma direção e no mesmo sentido da
velocidade inicial do veículo, este passou a se
5
deslocar com metade da velocidade inicial.
Diante do exposto, a massa do veículo, em kg,
30°
é:
Dados:
a) 5,72 s b) 18,91 s c) 0,05 s • velocidade inicial do veículo: 20 m/s;
d) 1,2 s e) 2,06 s • velocidade do projétil ao sair do canhão: 800
m/s; e
A46) Sobre um pequeno bloco de massa m • massa do projétil: 2 kg.
que se encontra em repouso passa a atuar Observação:
uma força cujo módulo depende do tempo • não há atrito entre o plano horizontal e o
segunda a expressão F = kt, onde k é uma veiculo.
constante e t é o tempo. Suponha que a força (A) 1.420 (B) 1.480 (C) 1.500
F sempre mantém o ângulo 0 com relação à (D) 1.580 (E) 1.680
horizontal. Calcule o trabalho realizado pela
força F desde o inicio de sua aplicação até o
instante em que o bloco perde contato com o
piso liso.

P m

7ZZZZZZZZZZZZZZZZZZ/

A47) (ITA-18) Um tubo fino de massa 1225 g e


raio r = 10,0 cm encontra-se inicialmente em
repouso sobre um plano horizontal sem atrito.
A partir do ponto mais alto, um corpo de massa
71,0 g com velocidade inicial zero desliza sem
atrito pelo interior do tubo no sentido anti-
horário, conforme a figura. Então, quando na

287
Elementos da física - Mecânica /- Colisão Mecânica

CONCEITOS FUNDAMENTAIS

Um choque entre dois corpos que ocorre em um intervalo de tempo relativamente muito
pequeno e durante o qual os dois corpos exerçam forças relativamente grandes entre si recebe a
denominação de colisão mecânica ou impacto mecânico ou choque mecânico. A reta normal às
duas superfícies de contato é denominada de linha de choque. Esta reta contém as forças
trocadas pelos corpos devido à colisão, uma força em cada corpo. Se a linha de choque coincide
com reta que une os centros dos corpos a colisão é classificada como central. Uma colisão central
será direta quando os vetores velocidades dos corpos antes do impacto pertençam à linha de
choque. Por outro lado, uma colisão central será oblíqua quando pelo menos um dos vetores
velocidades iniciais dos corpos não pertençam à linha de choque. Uma colisão será classificada
como excêntrica quando a linha de choque não coincidir com a reta que une os centros dos
corpos. Note que a colisão entre dois corpos perfeitamente esféricos será sempre central.

— Vi
linha de linha de V2
choque choque linha de
choque
linha que' ■
une os
colisão central colisão central colisão centros
direta obliqua excêntrica

Suponha que dois corpos estejam se movimentando sobre uma mesma reta em sentidos
opostos. Imediatamente antes do impacto a velocidade do corpo de massa m, é v,, enquanto que
a velocidade do corpo de massa m2 é v2. Toda colisão pode ser dividida em duas fases. A fase
de deformação ocorre desde o instante em que os corpo iniciam o contato da colisão até o
instante t* em que a força trocada pelos corpos devido ao impacto é máxima. Neste instante t* a
velocidade dos corpos é igual e será denominada de ü. Se os corpos não forem extremamente
rígidos, por exemplo duas bolas de futebol, os corpos apresentam uma máxima deformação no
instante t* na região de contato. A partir do instante t* até o instante em que os corpos perdem
contato ocorre a fase de restituição.

Início da fase Término da fase de deformação e Término da fase


de deformação início da fase de restituição de restituição

Em uma colisão cada corpo exerce no outro uma força de contato. Estas forças atuam ao
longo da linha de choque, formando um par de forças ação e reação, ou seja, possuem mesmo
módulo, mesma direção e sentidos opostos. Na figura abaixo F12 é a força que o corpo 1 exerce
no corpo 2 e F21 é a força que o corpo 2 exerce no corpo 1. Assim: F12 - -F21.

288
__ _______ _______________________________ FlementostiaFísica-Mecãnlcal-Colisão Mecânica
Estas forças atuam durante um intervalo de tempo muito pequeno, normalmente menos de
1 décimo de segundo, porém atingem uma intensidade máxima relativamente grande. O gráfico
abaixo mostra como varia o módulo da força de contato que atua em um dos corpos em função do
tempo. Como são iguais os módulos das forças de contato que cada corpo exerce no outro, o
gráfico abaixo pode representar a variação temporal do módulo de F12 ou de F21.

F“
Fmax -

*t

A área compreendida entre o gráfico e o eixo do tempo, desde o instante inicial até o
instante t* (em que a força é máxima), ou seja, durante a fase de deformação, é o módulo do
impulso de deformação | Td |. A área compreendida entre o gráfico e o eixo do tempo, desde o
instante t* até o instante final t(, ou seja, durante a fase de restituição, é o módulo do impulso de
restituição | Tr |.
Como F12 = -F21 segue que os impulsos de deformação e de restituição são iguais, em
módulo, para cada um dos corpos participantes da colisão: | Td11=| Td21 e | Tr11=| |. Vetorialmente
falando tem-se que Td1 = -Td2 e Tr1 = -T^. Deste modo, os módulos do impulso de deformação e
do impulso de restituição da força de contato, calculados sobre todo o sistema, sempre são iguais
a zero. Logo, toda vez que forem citados os impulsos de deformação e restituição entende-se que
fazem referência a apenas um dos corpos que colidem.

Caso não haja conversão de energia interna em energia cinética durante o processo de
colisão (como em uma explosão, por exemplo), tem-se que | Td | > | Tr |.

COLISÃO CENTRAL DIRETA

Sabe-se que em uma colisão central direta os vetores velocidades dos corpo
imediatamente antes do choque pertencem à linha de choque. Como a força que atua em cada
corpo devido ao impacto também pertence à linha de choque, tem-se que as velocidades dos
corpos imediatamente depois do choque também pertencem à linha de choque.
Considere a situação apresentada abaixo, onde duas esferas de massas m, e m2 estão se
movendo sobre um solo horizontal liso. Imediatamente antes do choque a velocidade da esfera de
massa m, é v,, enquanto que a velocidade da esfera de massa m2 é v2.

X Yí

Durante o choque entre as esferas atuam sobre cada uma das esferas três forças: peso,
normal com o solo e força de contato com a outra esfera, como indicado na figura abaixo.

289
Elementos tia física-Mecânica!-Colisão Mecânica

Analisando o sistema formado pelos dois corpos tem-se que a força resultante neste
sistema = P1 + N, + F21 + P2 + N2 + F21. Como existe equilíbrio vertical em ambos os
sistema vale FR sistema
corpos segue que P, + N, = 0 e P2 +N2 = 0. Sabe-se também que as forças de contato formam um
par de forças ação e reação: F21 + F12 = 0. Deste modo conclui-se que FR sistema = 0, ou seja, o
sistema formado por estes dois corpos é mecanicamente isolado e, consequentemente, tem-se a
conservação da quantidade de movimento nos instantes imediatamente antes do choque e
imediatamente depois do choque:

^0 sistema Qf sistema m., vd + m2v2 = m^!'+ m2v2'

Como o movimento dos corpos é unidimensional, em relação ao eixo x adotado, tem-se


que esta última equação fica melhor descrita da seguinte maneira:

m, | v, | -m2 | v2 |= -md ] v, ’ | +m21 v2' |

Outra maneira de escrever a equação de conservação da quantidade de movimento do


sistema é deixar o sinal embutido em cada componente de velocidade:

m-iV, + m2v2 = m1 vd'+ m2v2', onde Vt > 0, v2 < 0, v/ < 0 e v2' > 0

Esta última opção é mais vantajosa quando não é conhecido o sentido de todos os vetores
velocidade depois do choque. Assim, quando uma velocidade assumir valor positivo significa que
seu vetor está no mesmo sentido do eixo adotado e quando uma velocidade assumir valor
negativo tem-se que seu vetor está em sentido contrário ao do eixo adotado. Nesta obra será
adotado este último método, com os sinais embutidos nas variáveis velocidades.
Todo que foi exposto é válido para a situação proposta do choque entre dois corpos que se
movem sobre uma superfície lisa. Entretanto, existe uma infinidade de situações em que dois
corpos colidem de forma central e direta, não sendo possível descrever todas as situações desta
categoria de choque. Perceba que outras forças externas poderiam estar atuando sobre os
corpos, além do peso e da normal. Um dos corpos, por exemplo, poderia estar preso a uma mola.
Outro corpo poderia estar oscilando preso a um fio. O importante é saber quando ocorre, em uma
colisão, a conservação da quantidade de movimento. As condições são as mesmas apresentadas
no capítulo anterior, agora adaptadas para a situação da colisão entre dois corpos. Logo, para que
ocorra conservação da quantidade de movimento durante o choque entre dois corpos basta que
ocorra uma das condições seguintes:

1) Sistema Mecanicamente Isolado:


Independentemente de outras condições, se a soma das forças externas que atuam sobre
o sistema (formado apenas pelos dois corpos que colidem) for igual a zero então existe a
conservação da quantidade de movimento entre o instante inicial do choque e o instante final do
choque.

T = jFRdt = internas
+ ^externas. dt => í-ÍCf.ixtemas ) ^t
Claramente, se X^extemas = 0 segue que T = 0.
É importante ressaltar que o sistema deve ser formado apenas pelos corpos que colidem.
Qualquer outro elemento, como o chão, uma mola, um fio,... deve ser descartado do sistema.

290
Elementos da física - Mecânica /- Colisão Mecânica
2) Tempo de interação nulo:
Quanto mais rígidos foram os corpos que colidem menor é o tempo de interação devido ao
choque. Pode-se admitir, por exemplo, que o impacto entre duas esféricas metálicas possui um
tempo interação que tende a zero. Neste caso, o impulso total durante o choque é nulo e assim
ocorre a conservação da quantidade de movimento. Observe que a suposição que o tempo de
interação é nulo possui apenas sentido teórico. Na prática, por mais que os corpos sejam
extremamente rígidos, sempre existirá um tempo de interação entre os corpos que estão
colidindo, por menor que seja este tempo. Como as forças trocadas devido ao choque são
relativamente grandes, mesmo com um tempo pequeno de choque o impulso devido ao choque
acaba assumindo um valor que não pode ser desconsiderado.

Observe o exemplo seguinte para entender melhor quando ocorre a conservação da


quantidade de movimento em um choque. Suponha que uma esfera 1 é abandonada desde uma
altura h sobre o solo, enquanto que uma esfera 2 é lançada verticalmente para cima sobre a
mesma vertical do movimento da esfera 1. Obviamente tem-se um sistema que não é
mecanicamente isolado, uma vez que a força resultante sobre o sistema é igual à soma das forças
peso dos corpos 1 e 2.

y
As esferas colidem no ar, em um ponto onde a velocidade do corpo 1 é v, com sentido
descendente e a velocidade do corpo 2 é v2 com sentido ascendente. As forças F12 e F21 formam
um par de forças ação e reação, ou seja, F12 + F21 = 0 . Suponha que os corpos fiquem em contato
durante o choque, contando as fases de deformação e restituição, um tempo At. O impulso total
sobre o sistema formado pelos dois corpos é dado por:

T = J F12dt + j F21dt + P,At + P2At = j (F^-rFÇ^) dt + (P, + P2 )At T=(Pl+P2)At

Perceba que P, e P2 são vetores não nulos com mesma direção e sentido, ou seja,
P, + P2 * 0. Assim, o impulso sobre o sistema será nulo apenas se supormos que At = 0. Como já
comentado anteriormente, na prática, sempre tem-se At * 0. Porém, em modelo teóricos pode-se
admitir que se os corpos são extremamente rígidos o tempo de interação no choque é desprezível
e pode-se assumir At = 0, fazendo com que exista a conservação da quantidade de movimento,
mesmo com o sistema não sendo mecanicamente isolado.

291
Fiementos da Física-Mecânica!-Colisão Mecânica
COEFICIENTE DE RESTITUIÇÃO

Considere uma colisão central direta entre dois corpos de massas rm e m2, que formam
um sistema mecanicamente isolado. Define-se coeficiente de restituição de um choque como
sendo a razão entre os módulos dos impulsos de restituição e deformação atuantes sobre um dos
I S restituição I
corpos: e
I i deformação I

É importante destacar que estes impulsos devem ser calculados sobre apenas um dos
corpos participantes do impacto, uma vez que tanto o impulso de deformação quanto o impulso de
restituição do sistema possuem módulo igual a zero. Esta expressão do coeficiente de restituição
pode ser desenvolvida de modo a ficar em função apenas das velocidades inicial e final dos
corpos no choque. Para tanto, suponha a situação da colisão central direta dos corpos de massas
m, e m2 apresentada no início do capitulo, onde as velocidades dos corpos imediatamente antes
do choque, no instante de máxima força de contato e imediatamente depois do choque estão
indicadas na figura seguinte.

Início da fase Término da fase de deformação e Término da fase


de deformação início da fase de restituição de restituição

Como todos os vetores em questão (velocidades, quantidades de movimento e impulsos)


são paralelos, pode-se adotar um eixo unidimensional x paralelo à linha de movimento dos corpos
para medir a intensidade de todos estes vetores. Assim, se um vetor possuir mesmo sentido que
este eixo x, sua componente neste eixo será positiva. Porém, se um vetor possuir sentido
contrário ao do eixo x, sua componente neste eixo será negativa. Por exemplo, na situação
apresentada na última figura, adotando um eixo unidimensional x com sentido da esquerda para a
direita, este eixo medirá que a componente v-, de v1 é positiva e a componente v2 de v2 é
negativa. Como o eixo x é unidimensional, todas as expressões serão escritas escalarmente, com
os sinais embutidos nas respectivas componentes. Assim, pode-se algebrizar o impulso de
deformação sobre cada um dos corpos como segue abaixo:

ld = ldi = - Id2 => ld = AQi = - AQ2 => ld = miu - rmv! = m2v2 - m2u
+m2v2
rmu + m2u = nw + m2v2 u
m1 + m2

Substituindo este valor de u para calcular ld:

m1v1 + m2v2 + m2v2 - rj: <-m2v/ nyn^v-j-v^


ld = rrijU - miv, = m. -V1
m, +m2 +m.2 nr^ +m2

O impulso de restituição sobre cada um dos corpos pode ser escrito da seguinte maneira:

Ir = l<-i = —!« => lr = AQi'= - AQ2’ => lr = m,Vi’- mru = m2u - m2v2’ =>
, , '+ m,v,'
m,u + m2u = m,Vi + m2v2 => u =——1------ A-
m1 +m2

Substituindo este valor de u para calcular lr:

292
flementos da física-Mecânica !-Colisão Mecânica
+ " m2v2''
m,v,'+ m2v2' _m,m2(v,v2')
Ir= rrhv/ - 171,11 = m, v, = mi
m, + m2 m1 + m-2 m, + m2

Deste modo, o coeficiente de restituição também pode ser expresso da seguinte forma:

(v,'-v2')
I ! restituição I | lr I «í + m2
e= e IV1' I
I ’ deformação i Hd I I v2 ~ v, I
(v2-v,)

Inicialmente perceba que v2 - v, * 0, uma vez que se v, = v2 os corpos nunca colidirão.


Note também que, independentemente dos sentidos das velocidades, |v,’ - v2’| é o módulo da
velocidade relativa final (de afastamento) e |v2 - v,| é o módulo da velocidade relativa inicial (de
aproximação) entre os corpos. Assim, pode-se afirmar que:

I V relativa final I OU £ I V relativa de afastamento I


e
I V relativa inicial I I ^relativa de aproximação I

Como estudou-se até o momento apenas a colisão central direta, todas as velocidades
envolvidas no cálculo do coeficiente de restituição são medidas em um eixo unidimensional
paralelo à linha de movimento dos corpos. No item referente ao estudo da colisão central obliqua
será apresentado como é realizado o cálculo do coeficiente de restituição para um choque em que
os vetores velocidades não sejam paralelos à linha de choque.

TIPOS DE COLISÃO

Os choques centrais diretos podem ser classificados com relação ao valor do impulso de
restituição.

Colisão Perfeitamente Elástica


Uma colisão central direta é classificada como perfeitamente elástica quando o impulso de
restituição é igual ao impulso de deformação: Trestituiçâ0 = Tdefonnaçâo .
Neste caso, tem-se que | Trestituiçâ0 |=| Tdefomiaçao | e assim se9ue 9ue ° coeficiente de
restituição de uma colisão perfeitamente elástica é igual a 1: e = 1.
Suponha um sistema mecanicamente isolado formado por dois corpos, de massas m, e
m2, que se deslocam sobre uma mesma reta, com velocidades respectivamente iguais a v, e v2.
Imediatamente depois do choque verifica-se que as velocidades dos corpos são v,' e v2',
conforme indicado na figura seguinte.

imediatamente imediatamente
antes do choque depois do choque

Desde que o sistema é mecanicamente isolado então ocorre a conservação da quantidade


de movimento:

293
Elementos da Física-Mecânica!-Colisão Mecânica
n^v, + m2v2 m-jV,'+ m2v2'

Como os corpos se movimentam sobre uma mesma reta pode-se adotar um eixo
unidimensional x, paralelo à reta de movimento dos corpos. Assim, todas as velocidades são
paralelas ao eixo x, fazendo com que a expressão da conservação da quantidade de movimento
possa ser reescrita da seguinte maneira:

m!V1 + m2V2 = miV1'+m2V2 ' (1)'

onde os sinais estão embutidos nas variáveis velocidades, ou seja, se v, possui o mesmo sentido
de x então Vt > 0 e se v2 possui sentido contrário de x então v2 < 0.
Como a colisão é perfeitamente elástica o coeficiente de restituição é igual a 1:

I Vjv2 ’ I 1 _ V2 Vi '
e= => V2’- V/ = V, - V2 (2)
IV2-VJ vi-v2

Perceba que, dados m1y m2, Vi e v2, de posse das equações (1) e (2) sempre é possível
calcular os valores de v/ e v2’, uma vez que as equações (1) e (2) formam um sistema de
equações lineares 2x2 possível e determinado nas variáveis v/ e v2’. Desta forma, se a colisão
central direta for classificada como perfeitamente elástica e se forem fornecidos os valores das
massas dos corpos e das velocidades iniciais, sempre é possível determinar as velocidades dos
corpos imediatamente depois do choque, bastando escrever as equações de conservação da
quantidade de movimento e do coeficiente de restituição, que vale 1.

Conservação da energia em uma colisão perfeitamente elástica


Do exposto no item anterior sabe-se que em uma colisão central perfeitamente elástica
pode-se escrever as duas expressões:

rr^v, + m2v2 =m1v1'+m2v2' (1)


v2’- Vj’= Vi - v2 (2)

Da equação (1) segue que: m 1 (v, - v^) = m2(v2’ - v2) (3)

Da equação (2) segue que: v4 + v/ = v2 + v2’ (4)

Multiplicando lado a lado as equações (3) e (4) obtém-se:

mi(Ví - v/Xvt + v,’) = m2(v2’ - v2)(v2 + v2’) => m^,2 - tr^v, 12 = m2v2,2-m2v22
mivi2 . —
m2v22* m2'■
m.v/2 ! - v2 12
m^2 + m2v22 = m^, ,2+ m2v2 <2
2 2 2 F
o 1o membro desta última equação representa a energia cinética do sistema no instante
imediatamente anterior ao choque e o 2o membro representa a energia cinética do sistema
imediatamente depois do choque. Assim, pode-se afirmar que em um choque central
perfeitamente elástico existe a conservação da energia cinética do sistema. Como em uma colisão
o tempo de interação dos corpos é muito pequeno, pode-se desprezar a variação da energia
potencial gravitacional entre os instantes anterior e posterior ao choque. Deste modo, pode-se
assumir que a energia mecânica é composta apenas da modalidade energia cinética. Portanto,
pode-se enunciar que em um choque central perfeitamente elástico ocorre a conservação da
energia mecânica do sistema.
De tudo que foi exposto, pode-se afirmar que em um choque central perfeitamente elástico
de dois corpos de massas ni! e m2, com velocidades iniciais v, e v2 e finais v/ e v2’,
respectivamente, pode-se escrever as seguintes expressões relacionando estas grandezas:

294
F/ementos tia Física-Mecânica !-Colisão Mecânica
n^v-i + m2v2 = n^v, '+ m2v2'
v2’ - v/ = v, - v2
2 m2v22 mivi '2
m1v1 , m2v22
+ 2~
2 ~~~ 2~

Perceba que, dados os valores de m,, m2, v, e v2, a determinação de v,' e v2’ é dada pela
resolução de um sistema formado por apenas duas das três equações. Isto ocorre uma vez que
qualquer uma das três equações pode ser determinada pela manipulação algébrica das outras
duas. Por simplicidade, deve dar preferência pelas equações da conservação da quantidade de
movimento e do coeficiente de restituição, que são as únicas lineares com relação às velocidades.

Colisão Inelástica
Uma colisão central direta é denominada de inelástica quando o impulso de restituição é
I ' restituição I
nul0: 'restituição = 0 • Como e então o coeficiente de restituição do choque também é
I ' deformação I

nulo. Desde que também pode-se expressar o coeficiente de restituição do choque em função das
velocidades imediatamente anterior e imediatamente posterior ao choque, a saber e = ,
Ivz-vj
então e = 0 se e somente se v,’ = v2’ = v’. Desta forma, em uma colisão central inelástica as
velocidades finais dos corpos são iguais. A figura abaixo ilustra esta situação:

imediatamente imediatamente
antes do choque depois do choque

Supondo que o sistema é mecanicamente isolado ocorre a conservação da quantidade de


movimento:

m,v, + m2v2
m1v1 + m2 v2 = rr^v'+ m2 v' n^v, + m2v2 = (m, + m2 )v' => v‘
m, +m2

Diferentemente do que ocorre com o choque perfeitamente elástico, onde é necessário


resolver um sistema de equações para determinar as velocidades finais dos corpos, no choque
inelástico basta escrever a equação devido à conservação da quantidade de movimento para
determinar a velocidade dos corpos imediatamente depois do choque.
Note que para um choque central ser classificado como inelástico não é obrigatório que os
corpos estejam “grudados” depois do choque, basta que a velocidade relativa dos corpos depois
do choque seja nula.

Dissipação de energia em uma colisão inelástica


Em uma colisão central inelástica não ocorre a conservação da energia mecânica. O valor
da energia mecânica dissipada no sistema formado pelos dois corpos devido ao choque é
calculado abaixo:

>2
_miv m2v'2 m^2 m2v22 (m1 + m2)v’2-m1v12 -m2v22
Q = ECf - Ec0 —+ 2 —
2 2 2
\2
n^v, +m2v2 2 2 (m^+m^)2 2 2
(m, + m2)| - rr^vf - m2v2 - n^vf - m2v2
m, + m2 m, +m2
Q =-------------- T” 2

295
____________ Elementos tia Física-Mecânica!-Colisão Mecânica
,2.
jpfof + + 2m1m2v1v2 - npfof - m1m2v2 - m^v2 - jpjvf
2
_ -mim2 (v,2 + v22 - 2v,v2)
Q
2(01., + m2) 2(m1+m2)
n_ m1m2(v1-v2)2
2(m1+m2)

Inicialmente deve-se observar que v-, * v2, uma vez que se Vf = v2 os corpos não colidiríam.
Além disso, como (Vt - v2)2 > 0 e as massas são sempre positivas, segue que Q < 0. Assim,
conclui-se que em um choque central inelástico a energia mecânica do sistema não se conserva,
sempre diminuindo: Emt < Em0.

Colisão Parcialmente Elástica

Uma colisão central direta é classificada como parcialmente elástica quando o módulo do
impulso de restituição assume um valor não nulo menor que o módulo do impulso de deformação:
'restituição l<l 'deformação I *

Suponha um choque central descrito pela imagem abaixo:

v^-_
mi JT12

imediatamente imediatamente
antes do choque depois do choque

Assumindo que o sistema formado por mt e m2 é mecanicamente isolado, existe a


conservação da quantidade de movimento:

m-jV.] + m2v2 = m1v1'+ m2v2'

Como o choque é unidimensional pode-se escrever:

+m2v2 = m1v1'+m2v2' (1),

onde os sinais estão embutidos nas variáveis velocidades, ou seja, se v, possui o mesmo sentido
de x então v, > 0 e se v2 possui sentido contrário de x então v2 < 0.
IJ reatituição I entã0 segue qUe o < e < 1. Assim,
Como 0 <| Trestituiçâ0 l<'l ^deformação I ® ®
I ' deformação I

adotando os sentidos das velocidades apresentados na figura anterior, pode-se afirmar que:

pJV1 -V2'l v2Vf'


C — ----------------------------- v2'-*!’ = e^-vj (2)
Pz-vJ Vl-v2

O sistema formado pelas expressões (1) e (2) permite determinar os valores de v/ e v2’,
que são as velocidades de m, e m2, respectivamente, imediatamente depois do choque. Deve-se
ressaltar que, dados Vt, v2, e, mi e m2, o sistema formado pelas equações (1) e (2) é sempre
linear 2x2 nas variáveis v/ e v2’, sendo sempre possível e determinado, ou seja, sempre
admitindo uma solução.

296
__ ___________ __________________________________ Mementos tia Física - Mecânica /- Colisão Mecânica
Dissipação de energia em uma colisão parcialmente elástica
Como em uma colisão central parcialmente elástica tem-se 0 < e < 1:

V2 Vl'
0< <1 => v2 vi' Vi-V2 v, + v,’ > v2* + v2 (3)
Vi-V2

Desenvolvendo a expressão (1) obtém-se: m^v, - v,’) = m2(v2’ - v2) (4)


Multiplicando, lado a lado, as expressões (3) e (4):

rn-iív! - v/jívi + vV) > m2(v2’ - v2)(v2’ + v2) => n^v,2 - it^v,12 >m2v2,2-m2v22
2 2 mivi'2 ' m2v2'2
™ivi , m2v2
m^,2 + m2v22 > m^ 12+ m2v212
2 2 2 2

Como a variação da energia potencial em um choque pode ser desprezada, o 1o membro


desta última equação é a energia mecânica do sistema imediatamente antes do choque, enquanto
que o 2o membro é a energia mecânica do sistema imediatamente depois do choque. Assim, em
uma colisão central parcialmente elástica tem-se Em0 > Emf, ou seja, não ocorre a conservação
da energia mecânica, sendo seu valor final sempre menor que o inicial.

Quando analisa-se a classificação dos choques quanto ao coeficiente de restituição, é


possível afirmar que o único tipo de choque que existe a conservação da energia mecânica é o
choque perfeitamente elástico. No choque inelástico e no choque parcialmente elástico não ocorre
a conservação da energia mecânica, com a energia mecânica final sempre inferior ao valor da
energia mecânica inicial.

COLISÃO CENTRAL OBLÍQUA

Considere uma situação em que dois corpos, de massa rm e m2, se desloquem em um


plano horizontal com vetores velocidades v, e v2, respectivamente. Suponha que em
determinado momento ocorra uma colisão entre eles, de modo que os vetores velocidades v, e
v2 não sejam simultaneamente paralelos à linha de choque. A figura abaixo ilustra todo o
esquema . Assuma que no momento da colisão a linha de choque coincida com a reta que passa
pelo centros de gravidade dos corpos.

“Y

x
■»-------------------------------

linha de linha de
choque choque

imediatamente antes do choque imediatamente depois do choque

Todos os elementos descritos acima são suficientes para classificar esta colisão como
central oblíqua. Para simplificar o estudo desta colisão deve-se adotar um sistema de eixos xy de
modo que o eixo x coincida com a linha de choque e o eixo y seja perpendicular à esta direção,
com a origem do sistema exatamente no ponto onde ocorre a colisão. Perceba que as direções e
sentidos das velocidades indicados na figura não particularizam a analise que será efetuada, uma
vez que os sinais das componentes das velocidades nos eixos x e y estarão embutidos nos
valores de cada componente. Analisando o sistema em cada um dos eixos:

297
Elementos tia física - Mecânica /- Colisão Mecânica
Eixo x
Como a força trocada pelos corpos durante o choque é um vetor paralelo ao eixo x, todas
as componentes das velocidades, antes e depois do choque, no eixo x se comportam como em
um choque central direto neste eixo. Deste modo, pode-se escrever tanto a conservação da
quantidade de movimento e quanto a expressão do coeficiente de restituição do choque, utilizando
apenas as componentes das velocidades no eixo x:

miv1x + m2v2x = m^n’ + m2v2x’ (1)


e (2)i
|v2x-v1x|

lembrando que se a colisão é perfeitamente elástica tem-se e = 1, se a colisão é parcialmente


elástica tem-se 0 < e < 1 e se o choque for inelástico tem-se e = 0. Analisando os módulos
envolvidos na expressão do coeficiente de restituição para a situação proposta na figura pode-se
escrever que:

c v2x'~Vlx'
v1x-v2x

Eixo y
Observe que no eixo y os corpos não trocam força durante o choque. Como não há força,
não há aceleração no eixo y. Sem aceleração não existe alteração das componentes da
velocidades de cada corpo no eixo y, analisando as velocidades imediatamente antes e
imediatamente depois da colisão. Logo, pode-se escrever que:

v1y = v1y’ e v2y = v2y’ (3)

Pode-se afirmar que existe conservação da quantidade de movimento no eixo y? Sim,


existe, para demonstrar basta multiplicar cada expressão acima pela sua respectiva massa e
somar lado a lado as duas expressões obtidas:

m!V1y = n^v-i/ e m2v2y = m2v2y’ => miV1y + m2v2y = n^v-j/ + m2v2y’ => QOy = Qfy

Apesar desta última expressão garantir que existe a conservação da quantidade de


movimento no eixo y (fato que já era esperado, uma vez que foi assumido que o sistema era
mecanicamente isolado), as expressões descritas em (3) da conservação das componentes das
velocidades no eixo y, de cada corpo, são resultados bem mais abrangentes que a expressão da
conservação da quantidade de movimento em y.
Por fim, perceba que a manipulação algébrica das expressões (1), (2) e (3) sempre
permitem determinar os vetores velocidades imediatamente depois do choque em uma colisão
central oblíqua. Acompanhe os exemplos resolvidos para entender como este processo é
realizado na prática.
De tudo que foi exposto sobre a colisão central oblíqua é possível afirmar que sua
classificação como perfeitamente elástica, parcialmente elástica ou inelástica é realizada levando-
se em consideração as componentes das velocidades iniciais e finais apenas no eixo que é
paralelo à linha de choque.

298
Elementos da Física - Mecânica /- Colisão Mecânica
Exemplos

1) (ITA-90) Um projétil de massa m e velocidade v atinge um objeto de massa M, inicialmente


imóvel. O projétil atravessa o corpo de massa M e sai dele com velocidade v/2. O corpo que foi
atingido desliza por uma superfície sem atrito, subindo uma rampa até a altura h. Nestas
condições podemos afirmar que a velocidade inicial do projétil era de:

M
m v h
m -= • 2
'777777777777777777777777777777777777777/

a)v =—J2gh b) v = 2. 2—gh c) v = 2.I—gh


m v m Nm
d) v = ^/Sgh e) v = 2-jg’n

Solução: Alternativa A
I) Qo = Qf => mv = mv/2 + Mv’ => v’ = mv/2M
2M pr-r
IQ Em1 “ Em2 Mv’2/2 = Mgh => mV/4M2 = 2gh v2 = 8M2gh/m2 v =— J2gh
m

2) (ITA-98) Uma bala de massa 10 g é atirada horizontalmente contra um bloco de madeira de 100
g que está fixo, penetrando nele 10 cm até parar. Depois, o bloco é suspenso de tal forma que se
possa mover livremente e uma bala idêntica à primeira é atirada contra ele. Considerando a força
de atrito entre a bala e a madeira em ambos os casos como sendo a mesma, conclui-se que a
segunda bala penetra no bloco a uma profundidade de aproximadamente:
a) 8,0 cm. b) 8,2 cm. c) 8,8 cm. d) 9,2 cm. e) 9,6 cm.
Solução: Alternativa D
Sabe-se que o trabalho da força de atrito é igual à variação da energia mecânica:
mv2 mv2
AE,m => -Fat.d = 0—— => Fat.d = -y- (1)
1a experiência: WFai = AE

Como o sistema é mecanicamente isolado, pode-se aplicar a conservação da quantidade de


movimento na 2a experiência:
mv
m.v = (M + m)v’ => v ’ = —;— (2)
M+m
Na 2a experiência o trabalho da força de atrito também é igual a variação da energia mecânica:
^-S4^ + F.,.d. (3)
Substituindo os resultados (1) e (2) em (3):
mv2 (M + m)m2v2 mv2d’ 1 m £ H- d M
d = d--------- d' = (10)^5. = 91 cm
2 2(M + m)2 + 2d ~(M + m) + d M+m 110
2d

3) (ITA-09) Considere uma bola de basquete de 600g a 5m de altura e, logo acima dela, uma de
tênis de 60g. A seguir, num dado instante, ambas as bolas são deixadas cair. Supondo choques
perfeitamente elásticos e ausência de eventuais resistências, e considerando g = 10m/s2, assinale
o valor que mais se aproxima da altura máxima alcançada pela bola de tênis em sua ascensão
após o choque.
a) 5m. b) 10m. c) 15m. d) 25m. e) 35m.
Solução: Alternativa E
Velocidade de cada bola imediatamente antes do choque com o solo:
vt2 = - 2gh => V-| = - 10 m/s

299
______________________________________ _____ Elementos tia física-Mecânica!-Colisão Mecânica
Como o choque da bola de basquete com o solo é elástico, então sua velocidade imediatamente
depois do choque com o solo será 10 m/s. Assim, a velocidade da bola de basquete antes do
choque com a bola de tênis é 10 m/s, enquanto que a velocidade da bola de tênis antes do
choque é - 10 m/s.
Q0 = Qf => mbv, - mtv, = mbvb + mtvt => 10vb + vt = 90 (1)
Como o choque é elástico o coeficiente de restituição vale 1: e = V‘^ V- = 1 => v, - vb = 20 (2)

290 ,
Resolvendo o sistema formado pelas equações (1) e (2) obtém-se vt = ----- m/s.
11
Assim: v2 = 2gH => H = 34,75 m

4) (ITA-06) Animado com velocidade inicial v0, o objeto X, de massa m, desliza sobre um piso
horizontal ao longo de uma distância d, ao fim da qual colide com o objeto Y, de mesma massa,
que se encontra inicialmente parado na beira de uma escada de altura h. Com o choque, o objeto
Y atinge o solo no ponto P. Chamando pk o coeficiente de atrito cinético entre o objeto X e o piso,
g a aceleração da gravidade e desprezando a resistência do ar, assinale a expressão que dá a
distância d.
s +* d

h
Y
P
s2g -1 (
a)d = —1- vg-
2íxkkgg(< 2h J
c)d = ^ tÍvo-s®
2nkg^ V2hJ
2Pkgl 2h 2g

1 Í2vg-^S
d) d = — e)d = S^.fvo_s
2pk
2ukgg 0 2h Hkg ( V 2h J
Solução: Alternativa: A

Quando o objeto Y cai o tempo de queda é dado por: t = /—


V9
Na direção x o movimento é uniforme e vale x = vx.t, onde vx é a mesma velocidade de quando
abandona o último degrau, então: S => v=síS
Supondo que o choque é perfeitamente elástico, essa velocidade é a mesma do objeto x no
instante do choque e porTorriceli:
v2 = Vq - 2ad, onde a = pkg, pois FR = ma = Fat => ma = pkmg.

Substituindo temos: S2— = v„-2pK.gd => d = ■- - ■ (Vp


2h 2pKg 2h

5) (ITA-15) Nêutrons podem atravessar uma fina camada de chumbo, mas tem sua energia
cinética absorvida com alta eficiência na água ou em materiais com elevada concentração de
hidrogênio. Explique este efeito considerando um nêutron de massa m e velocidade v0 que efetua
uma colisão elástica e central com um átomo qualquer de massa M inicialmente em repouso.
Solução
I - Através da conservação do momento linear e pela condição imposta na questão, podemos
supor que
(eq.l) mv0 = mv’o + Mv’, onde v’o e v’ representam a velocidade do nêutron e do átomo logo após
o choque entre os mesmos.

300
__ _____________ ________ _____________ _________ Mementos da física-Mecânica!-Colisão Mecânica
II - Como a colisão é elástica, temos que pela definição do coeficiente de restituição:
vV 'n
1 =-------- - => v = v0 + Vo que substituindo na eq. I:
vo
mv0 mv'o + M(v0 + v’o) => (m M).v'o = (m - M)v0 _ (m - M) £tomo
v’o
(m + M) 0
apresentar massa igual a do nêutron tem-se v’o = 0, que implica em total transferência de energia
para o átomo, que é exatamente o que ocorre quando se tem uma elevada concentração de
hidrogênio, já que mH = mN

6) (ITA-08) A figura mostra uma bola de massa m que cai com velocidade v1 sobre a superfície
de um suporte rígido, inclinada de um ângulo e em relação ao plano horizontal. Sendo e o
coeficiente de restituição para esse impacto, calcule o módulo da velocidade v2 com que a bola é
ricocheteada, em função de v^ 0 e e. Calcule também o ângulo a.
mO

Solução:
No cálculo do coeficiente de restituição leva-se em consideração apenas as componentes das
velocidades na direção da linha de choque, ou seja, a direção perpendicular ao plano inclinado:
v2 sena v1 sen a
v^osO v2 e.cos0
Vi cosa
Como na direção paralela ao suporte não há aceleração: v^en 0 = v2cos a
V2 sen0
Assim: -sena tga = e.cot0 => a = arc tg (e.cot 0)
e.cos0 sen0
Por outro lado: sena =—e.cos0 e cosa = — sen0
v2 v2

Uma vez que sen2a + cos2a = 1 => -ye2.cos2 G + ^sen2 0 = 1 => v2 = v.Ve2 cos2 0 + sen2 0
v2 v2

7) (ITA-03) Quando solto na posição angular de 45° (mostrada na figura), um pêndulo simples de
massa m e comprimento L colide com um bloco de massa M. Após a colisão, o bloco desliza
sobre uma superfície rugosa, cujo coeficiente de atrito dinâmico é igual a 0,3. Considere que após
a colisão, ao retornar, o pêndulo alcança uma posição angular máxima de 30°. Determine a
distância pelo bloco em função dem, Me L.

m M

301
Fiementos da Física-Mecânica!-Colisão Mecânica
Solução:
As velocidades antes (v0) e depois (v,) do choque do corpo de massa m podem ser calculadas
pela conservação da energia mecânica:

i) m.g.L(1 - cos 45°) = m.v02/2 vo =

ii) m.Vj2^ = m.g.L(1 - cos 30°) => Vi=j2g.L 1 2 = Vg.L(2->/3)

Pela conservação da quantidade de movimento:


m (7g.L(2-V2) + 7g.L(2-5/3))
m.Vo = - m.v4 + M.v2 => v2 = m(v0 + v-,)/M => v, =—------------------------------------------ -
M
m Vgü (5/2 -5/2 + 5/2->/3)
V2
2 M
Depois do choque, a força resultante em M é a força de atrito: M.a = M.g.p => a = g.p
Portanto, a distância percorrida d é tal que:
m2 .g.L (2-V2+2-73 + 2^/(2 - 5/2)(2-5/3
v22 = 2.a.d d=^L
2.g.p
d
2.g.pM2
i2.l(4-5/2-5/3
m' + 2-7(2- V2)(2-V3)j
d=—
0,6.M2

8) (IME-80) Um carro esporte, pesando 5 kN e deslocando-se a 108 km/h, choca-se com um


furgão pesando 15 kN e com velocidade de 36 km/h, nas condições da figura. Os dois veículos,
cujos motores deixam de funcionar no instante do choque, ficam presos um ao outro, e deslocam-
se, após a colisão, 15 m até parar. Determine o módulo da força constante que travou os veículos.

90°

□t
Solução:
Conservação da quantidade de movimento:
I Qo |=|Qf I => V(mcvc)2 +(mfvf)2 =(mc + mf)v' => 7(5000.30)2 +(15000.10)2 =20000.v' =>

15OOOO5/2 =20000.v' => v’=Í^-m/s


2
Pela equação de Torriceli: v’2 = 2.a.Ax => — = 2.a.15 => a =— m/s2
2 4
Assim: F = (mc + mf).a => F = 2000.(15/4) => F = 7,5 kN

9) (IME-83) Um projétil de massa m, com velocidade v, choca-se com o bloco de massa M,


suspenso por um fio de comprimento R, conforme mostra a figura. Depois da colisão, projétil cai
verticalmente e o bloco descreve uma circunferência completa, no plano vertical. Determinar a
velocidade mínima do projétil, antes da colisão, em função de M, m, g e R, para que o bloco
descreva a trajetória prevista.

302
Mementos da Física - Mecânica!- Colisão Mecânica

Solução:

i) Conservação da quantidade de movimento no choque: mv = Mv’ => v' = -^^


M
ii) Para que o corpo de massa M faça o loop é necessário que no ponto mais alto da trajetória:
Fcp=P => ^- = Mg => v”2 = gR
R
iii) Pela conservação da energia mecânica:
Mv’2 Mv"2 .. „D „ m2v2
— = —+ Mg2R x2 x 2 => — gR + 4gR => v

w
10) (IME-96) A figura mostra um hemisfério oco e liso, cujo plano equatorial é mantido fixo na
horizontal.
mi m2
A B

7777777/r77/i7777777.
Duas partículas de massas m-, e m2 são largadas no mesmo instante, de dois pontos
diametralmente opostos, A e B, situados na borda do hemisfério. As partículas chocam-se e, após
0 choque rrh sobe até uma altura th e m2 sobe até uma altura h2. Determine o coeficiente de
restituição do choque. Sabe-se que th = R/2 e h2 = R/3, onde R é o raio do hemisfério.
Solução:
Pela conservação da energia mecânica tem-se que a velocidade v de cada partícula em função da
altura h que alcança depois do choque é dada por v = -J2gh .
{2gR 2gR
Assim’ e = ^afastamenl° — v 1+ v 2 _ y 2________
3 V3+V2
^aproximação ^1 + ^2 ^2gR+j2gR 2V6

11) (IME-80) Um bloco com 10 kg de massa está apoiado sobre o plano horizontal e ligado à
parede através da mola de constante elástica de 10 N/m e massa desprezível. Um projétil de 20 g
de massa e com velocidade de 750 m/s choca-se com o bloco, ficando no interior do mesmo. O
coeficiente de atrito entre o bloco e o plano é 0,2. Determine a amplitude do movimento final do
sistema.
4
A7

Solução:
Conservação da quantidade de movimento:
m.v = (M + m).v’ => 0,02.750 = (10 + 0,02).^ => v’ = 1,5 m/s
Pelo princípio do trabalho - energia:

303
__________________________________________________ Mementos da física-Mecânica!-Colisão Mecânica
... ... r- r- kx2 ... . _ (M + m)v12
Wfei + Wfa, = ECf- Ec0 => —— -(M + m)gpx = 0------- — =>

10x2 + 2.10,02.10.0,2.x = 10,02.1,52 => 5x2 + 20,04x - 11,2725 = 0


Resolvendo esta equação de 2o grau obtém-se, aproximadamente, x s 0,5 m

12) (IME-90) Um bloco C desliza com velocidade constante sobre o trecho horizontal da pista e
choca-se com o bloco D, de mesma massa, inicialmente em repouso. Em conseqüência, o bloco
D desloca-se e ao passar no ponto mais alto B não exerce qualquer esforço sobre a pista. O bloco
C continua em movimento e chega a subir na parte curva da pista até uma altura de 0,2 m em
relação ao trecho horizontal. Desprezando a resistência do ar e o atrito entre as superfícies,
determine a velocidade do bloco C antes do choque.
Dados: g = 10 m/s2; r = 2,88m

Solução:
Velocidade de C imediatamente depois do choque: v'c ^/2.g.hc : 72.10.0,2 =2,0 m/s
mD.v"p
No ponto B a normal em D é zero: PD = Fcpb => mD.g = => v"2 = gr = 28,8
r
Conservação da energia mecânica no movimento do corpo D:

EmB = EmA
mD.g.2r + ]H^l = 22ÇÍ => 57,6+14,4=^- => v’D = 12 m/s
Conservação da quantidade de movimento no choque entre os corpos C e D:
Qo = Q, => mvc = mv’c + mv’D => vc = 2,0 + 12 => vc=14m/s

13) (IME-91) A figura mostra um bloco P de massa 10 kg que parte do repouso em A e desce o
plano inclinado com atrito cujo coeficiente cinético é p = 0,2. Em B, o bloco P choca-se com o
bloco Q de massa 2 kg, inicialmente em repouso. Com o choque, Q desloca-se na pista horizontal,
desliza sobre sua parte semicircular e vai cair sobre o ponto B. Sabendo que as partes horizontal
e semicircular da pista não têm atrito e que o coeficiente de restituição entre P e Q é 0,8,
determine a altura h. D

Dados:
g = 10 m/s2
' I h
r = 2,5 m
x = 2VTl m Q e
6 = 45° c X B
<
Obs: Despreze a resistência do ar e as dimensões dos blocos.
Solução:
Após o lançamento as equações de movimento do corpo Q em um sistema de eixos com origem
em D são:
..i)x . gt2 x2 x íg 2VT1 [7Õ
i) x = vDtte ~- => 2y = 4r = g-j- => vD=-^=-^-^
e y = ^-

Conservação da energia mecânica do corpo Q nos pontos C e D:


mv„ mv! .. 2
n) mg2r + -^- = —=> 100 + 44 = vB2 vB = 12 m/s

Conservação da quantidade de movimento do choque entre os corpos P e Q


iü) mpvp = mpv’p + mqv’q => 10vp = 10v’p + 2.12 => vp - v’p = 2,4 (1)

304
[lementos da física-Mecânica!-Colisão Mecânica
Coeficiente de restituição:
v' — v' 12 — v'
iv) e = -3------3. => 0,8 = ——^ => 0,8vp + v’p=12 (2)
vp vp
Resolvendo o sistema formado pelas equações (1) e (2) obtém-se: vp = 8 m/s
Movimento de P ao longo do plano inclinado:
, h
v) v2 = 2aAe = 2g(sen45°-iicos45°)----------- 64 = 20(1 -0,2)h => h = 4,0m
p sen 45°

14) (IME-04) Um tanque de guerra de massa M se desloca com velocidade constante v0. Um
atirador dispara um foguete frontalmente contra o veiculo quando a distância entre eles é D. O
foguete de massa m e velocidade constante v, colide com o tanque, alojando-se em seu interior.
Neste instante o motorista freia com uma aceleração de módulo a. Determine:
1 - o tempo t transcorrido entre o instante em que o motorista pisa no freio e o instante em que o
veículo pára;
2 - a distância a que, ao parar, o veículo estará do local de onde o foguete foi disparado.
Solução:
1. Da conservação no momento linear, temos que: p0 = p, Mv0 - mv, = (M + m)v’
, Mvn - mv, . , . , . , . , . .
v = —--------- - , onde v representa a velocidade do tanque apos a colisão.
M+m
O motorista ao pisar no freio retarda o movimento do tanque, gastando um tempo até parar de:
Av 0-v' v' t, 1 f Mv0 -mvf
-a -a a a M+m
D
2. Cálculo do tempo até o instante da colisão: Asreiativo = vveiativa t => D = (v0 + vf)t => t=
v0+v,
D
3. O deslocamento do tanque para o referido tempo é dado por: AsT = v0.t AsT vo
V0+Vf
Após a colisão, o deslocamento do tanque será:
.2
v,2 = v02 + 2.a.As’T => 0 = v'2 - 2.a.As’T => As'T=— —22X1
2a M+m
Como não sabemos se o tanque ultrapassa ou não a posição do atirador:
2
1 fMv0-mv,
d = |D — (AsT + As’t)| => d D- D- V° -
v0+vf 2a M+m
2
D.v, 1 ( Mv0 - mv,
d=
v0 + v, 2a M+m

15) (ITA-11) Um objeto de massa m é projetado no ar a 45° do chão horizontal com uma
velocidade v. No ápice de sua trajetória, este objeto é interceptado por um segundo objeto, de
massa M e velocidade V, que havia sido projetado verticalmente do chão. Considere que os dois
objetos “se colocam” e desprezando qualquer tipo de resistência aos movimentos, determine a
distância d do ponto de queda dos objetos em relação ao ponto de lançamento do segundo objeto
Solução:
v2 sen2 45° v2
Cinemática: Hmáx=
2g 4g
Conservação da quantidade de movimento: Q, = Qf => (Qi)x = (Q<)x e (Qi)y=(Q,)y
Sendo u = velocidade após a colisão:
mv mv
(Qi)x = (Qf)x => -yj=(m + M)ux ux = V2(M + m)

305
Elementos da Física - Mecânica /- Colisão Mecânica
M
(Q|)y - (Qf)y => MV = (m + M)uy uv = --------.V
m+M

y = Hmax + Uy.t-|t2 y= Hmáx +uy(—)-^. x2


=>
X = Vyt
Uxy c.
K)!
i n V2 PJMV g x2.2(m + M)2
Logo: 0=—+ V2----- x--.-------- —----- => 0
4g m v 2 m v 4g m v m vV2
2 ,2
.2 . //^MV,2 . f v2 | í. M 1 => a = 2H V i2 (< M
m
A = b -4ac = (v2----- ) + 4 — g1 + — + 1+—
m v i 4g i i m
— v2 l m) v m
2 2
.^MV 2 M V i2 T-i M
+ i+—
-b ± VÃ m v t m v m
y =—--------- => y = T"
2a „ i. M
-2g 1 + —
1 m v2
2 2 2^
V2^V
y=
m v 1 l.mJVv; v mj
v2
„2g (.1 + —
Mf

16) (IME-07) Um pêndulo com comprimento L = 1m, inicialmente em repouso, sustenta uma
partícula com massa m = 1kg. Uma segunda partícula com massa M = 1kg movimenta-se na
direção horizontal com velocidade constante v0 até realizar um choque perfeitamente inelástico
com a primeira. Em função do choque, o pêndulo entra em movimento e atinge um obstáculo,
conforme ilustrado na figura. Observa-se que a maior altura alcançada pela partícula sustentada
pelo pêndulo é a mesma do ponto inferior do obstáculo. O fio pendular possui massa desprezível
e permanece sempre esticado. Considerando a aceleração da gravidade g = 10 m/s2 e a
resistência do ar desprezível, determine:
a) a velocidade v0 da partícula com massa M antes do choque;
b) a força que o fio exerce sobre a partícula de massa m imediatamente após o fio bater no
obstáculo.
ni
0,2 L
e=60°

L obstáculo

Solução:
2mV2
a) Pela conservação da energia mecânica: = 2mg.h' => V2 = 2,g . 0,8 L => V = 4m/s
2
Aplicando a conservação da quantidade de movimento:
Mv0 = (m+m).V => Vo = 2.V => Vo = 8m/s
2mV'2
b) Após o choque tem-se: T - 2mg cos60° = ---Ã*— ( Eq. I )
R
Após o choque ocorre a conservação da energia mecânica:

306
fiementos da física-Mecânica 1-Colisão Mecânica
.2
2mV2 2mV
—+ 2mgh' => V’2 = V2-2.g.h’ => V’2 = 16 - 2. 10.0,5 V’2 = 6 m2/s2
2 2
Substituindo na Eq. I, temos: T -210^ = y^
=> T = 20 + 10 => T = 30 N

17) (ITA-13) Uma pequena bola de massa m é lançada de um ponto P contra uma parede vertical
lisa com uma certa velocidade v0, numa direção de ângulo a em relação à horizontal. Considere
que após a colisão a bola retorna ao seu ponto de lançamento, a uma distância d da parede,
como mostra a figura. Nestas condições, o coeficiente de restituição deve ser
m
a) ( ) e = gd / (Vq sen 2a - gd)
b) ( ) e = 2gd / (Vq cos2a-2gd)
c) ( ) e = 3gd / (2v(j sen 2a - 2gd)
d) ( ) e = 4gd / (v§ cos2a - gd)
Ol
e) ( ) e = 2gd / (v§ tan 2a - gd)
h 4
d
Solução: Alternativa A
d
No ponto de impacto tem-se que:: vx = v0.cos a, d = vx.t = v0.cos a.t => t
v0.cosa
Como a força devido ao choque é normal à parede, então o módulo de vy não altera, apenas c
módulo de vx é alterado: e = ^- => Vxf =evxo => vxf = e.v0.cos a
vx0
d
Como a componente horizontal da velocidade é constante: d = vx(.t’ => t' =
e.v0.cosa
As componentes verticais das velocidades em P (na ida e na volta) são iguais, uma vez que não
age sobre o corpo nenhuma outra aceleração vertical além da gravidade: vy0 = vy( = v0.sen a

—= vu
^--v
De P até a parede: h = v0.sena.t-^ 0.sena.t'-^|-
.sena.t => |(t'+t)(t'-t) = v0.sena(t'-t) =>

f+t = ^v°-sena d _2v0.sena d 2.Vp sen a. cos a - gd


g e.v0.cosa g v0.cosa v0.g.cosa

e = ___ g.rf
Vq sen2a-gd

18) (ITA-13) Num plano horizontal x x y, um projétil de massa m é lançado com velocidade v, na
direção 0 com eixo x, contra o centro de massa de uma barra rígida, homogênea, de comprimento
L e massa M, que se encontra inicialmente em repouso a uma distância D de uma parede,
conforme a figura. Após uma primeira colisão elástica com a barra, o projétil retrocede e colide
elasticamente com a parede. Desprezando qualquer atrito, determine o intervalo de valores 0 para
que ocorra uma segunda colisão com a barra, e também o tempo decorrido entre esta e a anterior
na parede.
M
m v
'o' 'r - L/2
------- *- X

D L/2

Solução:

307
Elementos da Física-Mecânica !-Colisão Mecânica

M Conservação da quantidade de movimento na direção x:


m
m.v.cos 0 = M.u - m.v’x
u + v'x
p Como o choque é elástico: 1 = u = v.cos 0 - v’x
v.cos© ”
U2 Logo: m.v.cos 0 = M(v.cos 0 - v’x) - m.v’x
(M-m)vcos0
v’x(M + m) = M.v.cos 0 - m.v.cos 0 => v'x
M+m
(D
u = v.cos 0 - v’x =>
. M.v.cos0-m.v.cos0 M.v. cos 0 + m.v. cos 0 - M.v. cos 0 + m.v. cos 0 2m.v.cos0
u = v. cos 0--------------------------------- (2)
M+m M+m M+m
Na direção y não existe força trocada devido ao choque: v’y = v.sen 0
De modo a ocorrer o 2° choque do projétil com a barra deve-se ter:
2D
d + 2D = v’x.t => u.t + 2D = v’x.t => 2D = t(v'x - u) => t =
v'x-u
L 2D
Como L/2 > v’y.t => t<-=- => : (3)
2v'y vx—u 2v y
(M-m)vcos0 2m.v.cos0i (M-3m)v.cos0
Assim: v'x-u = (4)
M+m M+m M+m
2D(M + m) L (M-3m).L.
Substituindo (4) em (3): <------------- => tge<
(M-3m)v.cos0 2.v.sen0 4D(M + m)

Analisando o em x: D + d = v’x.At => D + u.t = v’x.At => d+-2Uu = v'x.At (5)


v'x-u
4m.v.cos0
M+m (M-m)v cos0
Substituindo (1), (2) e (4) em (5): D 1 + At
(M-3m)v.cos0 M+m
M+m
(M-m)vcosG At = Dfl + _±!L_ M+m
At
D(M + m)2
M+m l M-3m M-3m (M - 3m)(M - m)v. cos 0

19) (IME-81) Uma bola de bilhar atinge a tabela S da mesa no ponto B, com velocidade v e
coeficiente de restituição igual a 0,5. Considerando a bola com uma partícula, determine o ângulo
a e a velocidade que seguirá a bola, após o seu segundo contato com a tabela.

Solução:
O cálculo do coeficiente de restituição é válido somente para as
componentes das velocidades perpendiculares às superfícies das
respectivas tabelas S. Na direção paralela à S não existe alteração
da componente da velocidade.
Assim, no 1o choque pode-se afirmar que:
i) v.cos 15° = v’.cos p (1)
v'.senp
ü) e = -V ' Sf”fB- => v.sen 15° = 2.v’.sen p (2)
v.sen 15°
No 2o choque:

308
Flementos da Física-Mecânica !-Colisão Mecânica
i) e = 90° - p
ii) v’.cos 0 = v”.cos a => v’.sen p = v”.cos a (3)
... „ v ”.sena v".sena
v".sena
ii) e =------------- =-------------- => v’.cos p = 2.v”.sen a (4)
... —o
v'.sen0 v'.cosp
De(1)e(2): =-ccos
os1515°° = seP1^ 1
=> tgp = —tg15°
v cosp 2.senp
v"_senp_ cosp
De (3) e (4): => cota = 2tgp = tg15° => a = 90°-15° = 75°
v' cosa 2. sen a

20) (IME-08) A figura abaixo ilustra um pequeno bloco e uma mola sobre uma mesa retangular de
argura d, vista de cima. A mesa é constituída por dois materiais diferentes, um sem atrito e outro
com coeficiente de atrito cinético p igual a 0,5. A mola tem uma de suas extremidades fixada no
oonto A e a outra no bloco. A mola está inicialmente comprida de 4cm, sendo liberada para que o
bloco oscile na região sem atrito na direção y. Depois de várias oscilações, ao passar pela
posição na qual tem máxima velocidade, o bloco é atingindo por uma bolinha que se move com
relocidade de 2 m/s na direção x e se aloja nele.O sistema é imediatamente liberado da mola e se
desloca na parte áspera da mesa. Determine:
a) o vetor quantidade de movimento do sistema bloco + bolinha no instante em que ele é liberado
da mola;
b) a menor largura e o menor comprimento da mesa para que o sistema pare antes de cair.
d/2 t i
Dados: comprimento da mola = 25 cm;
Constante elástica da mola = 10 N/cm; A
Massa da bolinha = 0,2 kg; . n = 0,5 H=0
Massa do bloco = 0,4kg;
Aceleração da gravidade = 10 m/s2;

y
mesa vista de cima
Solução:
a) a) Uma vez que os valores medidos da direção y aumentam para baixo e diminuem para cima,
então a situação de máxima velocidade ocorre quando a mola não está deformada e o bloco está
descendo (afastando-se do ponto A).
Conservação de energia: = -m^1 => (1000)(0,04)2 = (0,4^/ => v1 = 2 m/s

Como se trata de um choque, há a conservação da quantidade de movimento, o vetor quantidade


de movimento do sistema depois do choque é igual ao vetor quantidade de movimento antes do
choque:
Q = mbobnha-Vbohnha+mbloco-Vbloco => Q = 0,4.1 + 0,8. j

b) Depois que o sistema entra na região com atrito o módulo da aceleração pode ser calculado
por:
F,« = Fat => m.a = m.g.p => a = 5,0 m/s2
Conservação da quantidade de movimento em x: Qox = (mboünha + mbioco)Vfx =>
2
0,4 = 0,6.Vfx => vfx = — m/s
3
Conservação da quantidade de movimento em y: QOy = (mboünha + mbioco)vty =>
0,8 = 0,6.Vfy => Vfy = m/s

Se 0 é o ângulo que a linha do movimento final faz com o eixo x, então:

309
Elementos da Física - Mecânica /- Colisão Mecânica

cosO = —
tg0 = —= 2 => sen0 =------ e cos0
vfx 5 5
Desta forma, as componentes das acelerações são:
i) ax = a.cos 0 => ax = 5/5 m/s2
ii) ay = a.sen 0 => ay = 2^5 m/s2

Deslocamento em x: vox2 = 2.ax.x => | = 2(5/5 )x ; 2V5


x =------ m
45
Deslocamento em y: voy2 = 2.ay.y =>
16
— = 2(2>/5)y
4^5
• y =------ m
45
Assim, a menor largura e o menor comprimento são
..d . 4^5
1) — = x => d =------ m
' 2 45
16J5+45
ü) <? = y + í0 ---------------- m
45 4 180

21) (IME-85) Duas bolas de bilhar, de mesmo tamanho e massa colidem, no plano horizontal, com
as velocidades de aproximação e os sentidos mostrados na figura. Sabendo-se que o coeficiente
de restituição é igual a 0,80, determinar:
a) as velocidades de separação das duas bolas;
b) a percentagem de energia mecânica dissipada no choque.
Ay
X
----------------- ►
V1 = 5 m/s / 45°
V2 = 10 m/s

Solução:
a) Como a força devido ao choque é paralela ao eixo x conclui-se que:
i) v'iy = °
ii) v’2y = v2y = v2.sen 45° = 5x/2 m/s
Qox = Qfx => mv, - mv2x = mv’1x + mv’2x => v'1x+v'2x =5-5x/2 (1)
O cálculo do coeficiente de restituição envolve apenas as componentes das velocidades na
mesma direção da força trocada pelo choque, ou seja, as componentes das velocidades ao longo
do eixo x:
c _ v'2x~V'lx V'2x~ V'lx
=> 0,8 V2x -v'ix 4 + 45/2 (2)
Vl+V2x 5 + 5V2
9-5/2
Somando as equações (1) e (2): 2v ’2x = 9-5/2 V'2x = m/s => v’2x = 3,79 m/s
2
Subtraindo as equações (1) e (2): 2v'1x =1-9x/2 => v'
v'.1x=-— m/s => v’1x 2 - 5,86 m/s

Os sinais nulo de v’1y e negativo de v’1x indicam que o corpo 1, após o choque, se desloca ao
longo do eixo x da direita para a esquerda, enquanto que os sinais positivos de v’2y e v’2x indicam
que o corpo 2, após o choque, se desloca no 1o quadrante do sistema de eixo xy adotado.
mv? mv? m.25 m.100
b) Ec0 = ----- L +---- í- =------- +---------- = I 62,5.m
’ 2 2 2 2
mv'2 mv'? mv mv’2 m(v,2x + v 2y) m(5,86)2 + m[(3.79)^(5^yi_4935ni
Ec( = -------- L 4-
+----- £. — li. 4
2----- 2---------
2 2-------------- 2 2

310
flementos da física-Mecânica!-Colisão Mecânica
%Ed Ec0 - Ecf 62,5m-49,35m
100% 100% s 21%
Ec0 62,5m

22) (ITA-92) Um objeto de massa M é deixado cair de uma altura h. Ao final do 1° segundo de
lueda o objeto é atingido horizontalmente por um projétil de massa m e velocidade v, que nele se
aloja. Calcule o desvio x que objeto sofre ao atingir o solo, em relação ao alvo pretendido.
3) V^h/g (M + m)v d) (A/2h/g v
m
m
3) #vg -------- v e) (1 - 72h/g)(M + m)v
M+m
m
') (72h7g-l) -------- v
M+m
Solução:
:sta questão foi propositadamente colocada como última do capítulo pois não possui alternativa
lorreta.
No 1 s de queda o objeto cai uma altura y dada por:
: m •
gt2 g
y=— => y=-
2 7 2
Vi y
A velocidade Vi do objeto imediatamente antes do choque é
Vx
V m
dada por: v, = gt => v, = g
. vy
Uma vez que existe conservação da quantidade de
movimento na direção horizontal:
h-y ... mv
mv = (M + m)vx => v =--------
M+m
O ponto chave desta questão é quanto à variação da
D quantidade de movimento na direção vertical. Observe que o
enunciado afirma que o projétil possui apenas velocidade
horizontal ao colidir com o objeto. Desta forma, caso não
X *1 colidisse com o objeto, o projétil continuaria com velocidade
lorizontal. Assim, após o choque e o projétil se alojando no objeto, este é responsável por
icelerar verticalmente o projétil para baixo. Assim, o objeto exerce no projétil uma força vertical
:om sentido para baixo. Pela 3a Lei de Newton o projétil exerce no objeto uma força vertical com
sentido para cima, diminuindo sua velocidade vertical imediatamente antes do choque. Assim,
sxiste a variação da quantidade de movimento vertical do objeto devido ao choque. Considerando
]ue o At do choque tende a zero segue que ocorre a conservação da quantidade de movimento
ambém na direção y: Mv, = (M + m)vy => v
Vy =

^ela equação horária na direção y:


3 = h-y-vyt-^ => h_9__M9-t- 9^ = 0 2Mg
=> gt2 + t - 2h + g = O
y 2 2 M+m 2 M+ m
2
2Mg 2Mg
± + 4g(2h-g)
1=
M+m M+m
t=
M
M+m '
f+^-1- M M+ m
2g g

'omo a velocidade é constante no eixo x: x = vx.t


mv
X —----------
M+m
M
M+m '
f^-1-
g
M
M+m

311
Elementos Ca Física-Mecânica!-Colisão Mecânica
Exercícios de Embasamento exercida por 6 s sobre o objeto, na mesma
direção de seu movimento. Em seguida, o
E1) (ENEM-14) O pêndulo de Newton pode ser objeto colide frontalmente com um obstáculo e
constituído por cinco pêndulos idênticos tem seu movimento invertido, afastando-se
suspensos em um mesmo suporte. Em um com velocidade de 3 m/s. O módulo do
dado instante, as esferas de três pêndulos são impulso exercido pelo obstáculo e a variação
deslocadas para a esquerda e liberadas, da energia cinética do objeto, durante a
deslocando-se para a direita e colidindo colisão, foram, respectivamente,
elasticamente com as outras duas esferas, que a) 26 Ns e -91 J. b) 14 Ns e-91 J.
inicialmente estavam paradas. c) 26 Ns e -7 J. d) 14 Ns e-7 J.
[ e) 7 Ns e -7 J.

E3) (UFRGS-15) Na figura abaixo, estão


representados dois pêndulos simples, X e Y,
de massas iguais a 100 g. Os pêndulos, cujas
hastes têm massas desprezíveis, encontram-
se no campo gravitacional terrestre. O pêndulo
Y encontra-se em repouso quando o pêndulo X
O movimento dos pêndulos após a primeira é liberado de uma altura h = 0,2 m em relação
colisão está representado em: a ele. Considere o módulo da aceleração da
gravidade g = 10 m/s2.

a)

b)
z--jh
Após a colisão, X e Y passam a mover-se
juntos, formando um único pêndulo de massa
200 g. Se v é a velocidade do pêndulo X no
instante da colisão, o módulo da velocidade do
c) pêndulo de massa 200 g, imediatamente após
a colisão, é
a) 2v . b) V2v. c) v.
d) v/>/2. e) v/2 .

d) E4) (FMTM-13) Em uma colisão frontal entre


duas esferas, A e B, a velocidade de A varia
com o tempo, como mostra o gráfico.
antes I durante depois
da colisão | a colisão da colisão

2,0
e)

E2) (UFRGS-14) Um objeto de massa igual a 2


kg move-se em linha reta com velocidade 0,0 —►
constante de 4 m/s. A partir de um certo 2,0 I2,1 t(s)
instante, uma força de módulo igual a 2 N é -0,5--

312
Elementos da física-Mecânica !-Colisão Mecânica
Sabendo que a massa da esfera A é de 100 g, colisão, a velocidade horizontal final da
o módulo da força média que ela exerce sobre partícula 1 é v1( = 4,5 m/s.
a esfera B durante essa colisão, em newtons, Utilizando a aceleração da gravidade g = 10
é igual a m/s2, calcule
a) 2,5. b) 1,5. c) 3,5. a) a velocidade horizontal da partícula 1 antes
d) 0,5. e)4,5. da colisão.
b) a velocidade horizontal da partícula 2 após a
E5) (FMTM-13) Duas crianças brincam com colisão e a altura máxima que ela atinge.
massas de modelar sobre uma mesa Apresente os cálculos.
horizontal e fazem duas esferas, A e B, de
massas iguais. Em seguida, lançam as esferas E7) (UESPI-11) Uma pequena esfera está
que passam a rolar sobre a mesa, em direções presa na extremidade de uma haste rígida de
perpendiculares entre si, conforme comprimento 45 cm, articulada no ponto O (ver
representado na figura. Após a colisão no figura). Ao ser liberada do repouso, com a
ponto P, as esferas permanecem grudadas haste horizontal, a esfera descreve o
uma na outra, movendo-se juntas após o movimento mostrado na figura, colidindo,
choque. quando a haste se encontra na vertical, com
um bloco inicialmente parado sobre uma
aO superfície horizontal. Considere a aceleração
da gravidade 10 m/s2. Se a esfera, de massa
B 100 g, entra em repouso com a colisão, qual a
velocidade do bloco de massa 200 g após o
Sabendo que imediatamente antes da colisão choque?
as esferas têm velocidades VA = VB = 2 m/s, o (Despreze as forças dissipativas e a massa da
módulo da velocidade do conjunto formado haste, e considere a bola e o bloco como
pelas duas esferas juntas, em m/s, partículas materiais.)
imediatamente depois da colisão é igual a O
a) -jl .b) VÕ . c) 8.
d) 2. e)4.

6) (UEL-14) Analise as figuras a seguir.


Figura 1 I5
M

A) 1,5 m/s B) 2,5 m/s C) 3,5 m/s


D) 4,5 m/s E) 5,5 m/s
Figura 2
E8) (UFBA-09) Buscando melhorar a
segurança de seus veículos, as fábricas de
automóveis fazem testes de impacto, a fim de
avaliar os efeitos sobre a estrutura dos carros
h e sobre seus ocupantes. Como resultado
dessa iniciativa, as pesquisas têm conduzido à
r- () construção de carros com carroceria menos
rígida, que se deformam mais facilmente em
.1/ m caso de colisão. Em um teste realizado, um
Uma partícula 1 com massa M, inicialmente veículo de 1000,0kg, movendo-se com
velocidade igual a 72,0km/h e dirigido por
em repouso, que está a uma altura de h = 1,25
controle remoto foi arremessado contra uma
m, desliza sem atrito por uma calha, como
parede de concreto. A colisão, completamente
esquematizado na Figura 1. Essa partícula
inelástica, durou 0,05 segundos.
colide elasticamente com a partícula 2 com
massa m, inicialmente em repouso. Após a Analise a decisão dos fabricantes de produzir
automóveis com carroceria menos rígida e

313
Elementos tia Física-Mecânica!-Colisão Mecânica
calcule a intensidade da força média exercida em que uma partícula de massa m, = 0,2 kg
pela parede sobre esse veículo. colide com um alvo que inicialmente estava em
repouso, conforme a figura.
E9) (Unesp-01) Uma esfera de aço de massa
0,20 kg é abandonada de uma altura de 5,0 m,
yt
atinge o solo e volta, alcançando a altura
máxima de 1,8 m. Despreze a resistência do ar
e suponha que o choque da esfera com o solo
ocorra durante um intervalo de tempo de 0,050
vly Vi
s. Levando em conta esse intervalo de tempo,
determine: mL
a) a perda de energia mecânica e o módulo da V0 X
variação da quantidade de movimento da
esfera; v2y V2
2
b) a força média exercida pelo solo sobre a
esfera. Kj'
Adote g = 10 m/2 . Após a colisão, obteve-se como resultado que
as componentes y das velocidades são
E10) (UFBA-11) Uma esfera rígida de massa respectivamente Viy = 5 m/s e v2y = - 2 m/s.
m, = 0,5kg, presa por um fio de comprimento L Neste caso, a massa do alvo em kg é:
= 45,0cm e massa desprezível, é suspensa em a) 0,08 b) 0,2 c) 0,5
uma posição tal que, como mostra a figura, o d) 0,8 e) 1,25
fio suporte faz um ângulo de 90° com a direção
vertical. Em um dado momento, a esfera é E12) (UFES-09) Um bloco A é lançado em um
solta, indo se chocar com outra esfera de plano horizontal com velocidade de módulo vA
massa m2 = 0,5kg, posicionada em repouso no = 4,0 m/s. O bloco A tem massa mA = 2,0 kg e
solo. colide frontalmente com uma esfera B de
m, massa mB = 5,0 kg. Inicialmente, a esfera
o- L íãT
' I
I
encontra-se em repouso e suspensa por um fio
ideal de comprimento L, fixo em O, como
I mostra a figura abaixo. Após a colisão, a
I
I esfera atinge uma altura máxima de hB = 0,20
I
I m. Os atritos do bloco A e da esfera B com a
I
I superfície são desprezíveis.
I [
I
I

Considerando o diâmetro das esferas


desprezível e o choque entre elas
perfeitamente elástico, determine a velocidade
A
7A
das esferas após o choque, supondo todas as
forças dissipativas desprezíveis, o módulo da
aceleração da gravidade local igual a 10m/s2 e I
Com essas informações,
I
o coeficiente de restituição e = ———, em A) determine o módulo da velocidade da esfera
Vi-v2 B, imediatamente após a colisão;
que v’t e v’2 são as velocidades finais das B) determine o módulo e o sentido da
esferas e Vt e v2 as velocidades iniciais. velocidade do corpo A, após a colisão;
C) determine a diferença entre a energia
E11) (UFG-14) Uma experiência comum cinética do sistema, antes e após a colisão;
utilizando um acelerador de partículas consiste D) responda se a colisão foi ou não
em incidir uma partícula conhecida sobre um perfeitamente elástica. Justifique a sua
alvo desconhecido e, a partir da análise dos resposta.
resultados do processo de colisão, obter
informações acerca do alvo. Um professor, E13) (Unifesp-08) A figura representa um
para ilustrar de formas simplificada como esse pêndulo balístico usado em laboratórios
processo ocorre, propôs a seguinte situação didáticos.

314
Elementos da Eislca-Mecânica!-Colisão Mecânica
a) a velocidade do pêndulo com a pedra
engastada, imediatamente após a colisão.
b) a altura máxima atingida pelo pêndulo com
a pedra engastada e a tensão T na corda
neste instante.

pêndulo na---- E15) (Unifesp-12) Um corpo esférico, pequeno


posição final n.
lançador
e de massa 0,1 kg, sujeito a aceleração
gravitacional de 10m/s2, é solto na borda de
uma pista que tem a forma de uma depressão
hemisférica, de atrito desprezível e de raio
20cm, conforme apresentado na figura. Na
A esfera disparada pelo lançador se encaixa parte mais baixa da pista, o corpo sofre uma
em uma cavidade do bloco preso à haste — colisão frontal com outro corpo, idêntico e em
em consequência ambos sobem até ficarem repouso. ____________
presos por atrito em uma pequena rampa, o
que permite medir o desnível vertical h do õ
centro de massa do pêndulo (conjunto bloco-
esfera) em relação ao seu nível inicial. Um
aluno trabalha com um equipamento como
esse, em que a massa da esfera é mE = 10g, a Q
massa do bloco é mB = 190g e a massa da
haste pode ser considerada desprezível. Em Considerando que a colisão relatada seja
um ensaio experimental, o centro de massa do totalmente inelástica, determine:
conjunto blocoesfera sobe h = 10cm. a) O módulo da velocidade dos corpos, em
a) Qual a energia potencial gravitacional m/s, imediatamente após a colisão.
adquirida pelo conjunto bloco-esfera em b) A intensidade da força de reação, em
relação ao nível inicial? newtons, que a pista exerce sobre os corpos
b) Qual a velocidade da esfera ao atingir o unidos no instante em que, após a colisão,
bloco? atingem a altura máxima.
Suponha que a energia mecânica do conjunto
bloco-esfera se conserve durante o seu E16) (Unesp-15) Uma esfera de borracha de
movimento e adote g = 10m/s2. tamanho desprezível é abandonada, de
determinada altura, no instante t = 0, cai
E14) (Unifesp-11) Uma pequena pedra de 10g verticalmente e, depois de 2 s, choca-se contra
é lançada por um dispositivo com velocidade o solo, plano e horizontal. Após a colisão, volta
horizontal de módulo igual a 600m/s, incide a subir verticalmente, parando novamente, no
sobre um pêndulo em repouso e nele se instante T, em uma posição mais baixa do que
engasta, caracterizando uma colisão aquela de onde partiu. O gráfico representa a
totalmente inelástica. O pêndulo tem 6,0kg de velocidade da esfera em função do tempo,
massa e está pendurado por uma corda de considerando desprezível o tempo de contato
massa desprezível e inextensível, de 1,0m de entre a esfera e o solo.
comprimento. Ele pode girar sem atrito no V (m/s)
plano vertical, em torno da extremidade fixa da 20
corda, de modo que a energia mecânica seja
conservada após a colisão.

1,0 m T
fora de escala 0
t (s)

-18

v = 600 m/s Desprezando a resistência do ar e adotando g


= 10 m/s2, calcule a perda percentual de
energia mecânica, em J, ocorrida nessa
Considerando g = 10,0m/s2, calcule colisão e a distância total percorrida pela

315
Elementos da Física-Mecânica!-Colisão Mecânica
esfera, em m, desde o instante t = 0 até o O tempo de interação da bola de vôlei com o
instante T. tórax da pessoa é o dobro do tempo de
interação da bola de golfe.
E17) (Unicamp-06) Em uma auto-estrada, por A área média de contato da bola de vôlei com
causa da quebra de uma ponta de eixo, a roda o tórax é 10 vezes maior que a área média de
de um caminhão desprende-se e vai em contato da bola de golfe.
direção à outra pista, atingindo um carro que a) Antes das colisões, a quantidade de
vem em sentido oposto. A roda é lançada com movimento da bola de golfe é maior que a da
uma velocidade de 72km/h, formando um bola de vôlei.
ângulo de 30° com a pista, como indicado na b) Antes das colisões, a energia cinética da
figura abaixo. A velocidade do carro antes da bola de golfe é maior que a da bola de vôlei.
colisão é de 90km/h; a massa do carro é igual c) Após as colisões, a velocidade da bola de
a 900kg e a massa da roda do caminhão é golfe é maior que a da bola de vôlei.
igual a 100kg. A roda fica presa ao carro após d) Durante as colisões, a força média exercida
a colisão. pela bola de golfe sobre o tórax da pessoa é
Pista maior que a exercida pela bola de vôlei.
e) Durante as colisões, a pressão média
exercida pela bola de golfe sobre o tórax da
ímB—* pessoa é maior que a exercida pela bola de
Gramado vôlei.
30°

O E19) (AFA-99) Uma esfera de aço de massa


0,5 kg, amarrada a uma corda de 70 cm de
Pista comprimento, é solta quando a corda está na
ANTES horizontal, conforme figura abaixo. Na parte
Pista
mais baixa de sua trajetória, colide
elasticamente com um bloco de aço de massa
2,5 kg, inicialmente em repouso sobre uma
superfície sem atrito. A velocidade do bloco,
Gramado após a colisão, em m/s, é, aproximadamente

a) 0,86.
b) 1,23.
Pista c) 2,50.
d) 3,20.
DEPOIS
a) Imediatamente após a colisão, qual é a
componente da velocidade do carro na direção
transversal à pista? E20) (AFA-00) Uma série de n projéteis, de 10
b) Qual é a energia cinética do conjunto carro- gramas cada um, é disparada com velocidade
roda imediatamente após a colisão? v = 503 m/s sobre um bloco amortecedor, de
Se for necessário, use: sen30° = 0,5, cos30° = massa M = 15 kg, que os absorve
0,87. integralmente. Imediatamente após, o bloco
desliza sobre um plano horizontal com
E18) (Fuvest-13) Compare as colisões de uma velocidade V = 3 m/s. Qual o valor de n?
bola de vôlei e de uma bola de golfe com o a) 4 b) 6 c) 7 d) 9
tórax de uma pessoa, parada e em pé. A bola
de vôlei, com massa de 270g, tem velocidade E21) (EN-13) Um bloco A, de massa mA = 1,0
de 30m/s quando atinge a pessoa, e a de kg, colide frontalmente com outro bloco, B, de
golfe, com 45g, tem velocidade de 60m/s ao massa mB = 3,0 kg, que se encontrava
atingir a mesma pessoa, nas mesmas inicialmente repouso. Para que os blocos
condições. Considere ambas as colisões sigam grudados com velocidade 2,0 m/s, a
totalmente inelásticas. É correto apenas o que energia total dissipada durante a colisão, em
se afirma em: joules, deve ser
Note e adote: a) 24 b) 32 c) 36 d) 48 e) 64
A massa da pessoa é muito maior que a
massa das bolas. E22) (AFA-12) De acordo com a figura abaixo,
As colisões são frontais. a partícula A, ao ser abandonada de uma

316
Elementos Ha física - Mecânica /- Colisão Mecânica
altura H, desce a rampa sem atritos ou ( ) B - depois do choque a velocidade do
resistência do ar até sofrer uma colisão,
perfeitamente elástica, com a partícula B que centro de massa independe de — e é
m2
possui o dobro da massa de A e que se
constante.
encontra inicialmente em repouso. Após essa
( ) C - como as partículas se movimentam
colisão, B entra em movimento e A retorna,
juntas após o choque, o centro massa passa a
subindo a rampa e atingindo uma altura igual a
ter, depois do choque, uma velocidade em
A- módulo não nula e maior do que | v, |.
( ) D - como uma das partículas está parada
antes do choque, o centro de massa terá
sempre uma velocidade | V | < | v, |.
( ) E - como uma das partículas está parada
antes do choaue, o centro de massa terá uma
a) H b) H/2 c) H/3 d) H/9 velocidade | V | = I v, |.

E23) (ITA-76) Uma esfera de massa m, com


E26) (ITA-95) Uma massa m, em movimento
velocidade v, colide elasticamente em colisão retilíneo com velocidade de 8.0.10'2 m/s colide
frontal, com outra esfera de massa 2m frontal e elasticamente com outra massa m2
inicialmente em repouso. Após o choque as em repouso e sua velocidade passa a ser
massa m e 2m têm, respectivamente, 5,0.10' 2 m/s. Se a massa m2 adquire a
velocidades íf, e v2, dadas por: velocidade de 7,5.10'2 m/s podemos afirmar
A) V, = 0 vv2 = S. que a massa m4 é:
_7
B) V,= -_ a)10m2 b)3,2m2 c) 0,5 m2
2 d) 0,04 m2 e) 2,5 m2
C) v, = - =
3
D) V, = f = li
3
E27) (ITA-78) Consideram-se dois pêndulos
simples dispostos conforme a figura ao lado.
E) n. D. R. A.
Abandonando o da esquerda, na posição
indicada, o mesmo colidirá com o outro; após a
E24) (ITA-75) Uma partícula de massa th, tem
colisão, as duas esferas dos pêndulos
velocidade V} dirigida para outra partícula de caminharão aderidas uma à outra. Para tal
massa m2 com velocidade V2 = 0. Depois do sistema, pode-se afirmar que:
choque ot, tem velocidade K/que faz um
ângulo 0, com em, adquire uma velocidade
de P/que faz um ângulo 02 com Pj. Podemos
afirmarque:
A - m, V,' sen ô, = m2 V2' cos 02
B - m, V,' cos 6, = m, V2' cos 02
= Pj' + w2y2' CO
D- Pj' sen 0, = m2 V2 sen 62 A) em qualquer instante de tempo a
E-NDA. quantidade de movimento é conservada, mas
a energia mecânica não;
E25) (ITA-79) Num dado referencial inercial, B) não é possível resolver este problema, pois
uma partícula de massa m, com velocidade Vj a energia mecânica não é conservada e,
choca-se com uma partícula de massa m2 que devido à ação gravitacional, a quantidade de
está parada. Após interação, as duas movimento também não se conserva;
partículas movimentam-se juntas. Pode-se C) somente a componente horizontal da
afirmar que: quantidade de movimento, no instante da
( ) A - antes do choque a velocidade do centro colisão, é conservada;
de massa do sistema das duas partículas era D) tanto a energia mecânica como a
m, + m2 - quantidade de movimento são conservadas.
v = ----- :-V,- E) Nenhuma das afirmações acima é correta.
m.

317
Elementos tia Física-Mecânica!-Colisão Mecânica
Exercícios de Fixação 16) O deslocamento horizontal do conjunto
bloco-projétil é de V2LH-H2 .
F1) (UEM-14) Em um experimento, uma das
extremidades de uma mola ideal de constante F3) (UESPI-12) Em um acidente de trânsito, os
elástica k0 e de comprimento Lo está presa a carros A e B colidem no cruzamento mostrado
uma parede vertical. Um bloco de massa m0, nas figuras 1 e 2 a seguir. Logo após a colisão
que se desloca com velocidade constante em perfeitamente inelástica, os carros movem-se
linha reta sobre uma superfície plana e sem ao longo da direção que faz um ângulo de 0 =
atrito, choca-se contra a outra extremidade 37° com a direção inicial do carro A (figura 2).
livre da mola, comprimindo-a por uma distância Sabe-se que a massa do carro A é o dobro da
Lq/2, até parar completamente. O bloco é massa do carro B, e que o módulo da
então projetado novamente em sua trajetória velocidade dos carros logo após a colisão é de
original, devido à ação da mola. Com base 20 km/h. Desprezando o efeito das forças de
nessas informações, analise as alternativas atrito entre o solo e os pneus e considerando
abaixo e assinale o que for correto. sen(37°) = 0,6 e cos(37°) = 0,8, qual é a
01) A energia potencial elástica acumulada na velocidade do carro A imediatamente antes da
mola, enquanto ela está totalmente colisão?
L2
comprimida, é de k0 —. vista de cima Figura 1
do cruzamento
02) Enquanto a mola está sendo comprimida
pelo bloco, sua força elástica realiza um
trabalho negativo sobre o bloco. carro A
= ------------ -q
04) O bloco é projetado pela mola a uma

velocidade de v0 = Lo
V2m0 ' À
08) A mola realiza um trabalho positivo de

sobre o bloco, para fazê-lo retornar à


ó
|i carro B

sua trajetória inicial. Figura 2


16) A variação da quantidade de movimento do
bloco durante o experimento é nula. JÍÍ...
F2) (UEM-14) Um projétil de massa mp é
disparado contra um bloco de madeira de
massa MB, suspenso por dois fios de
comprimento L presos ao teto de um
laboratório de balística. Antes da colisão, o
bloco estava em repouso e, após a colisão, o A) 24 km/h B) 39 km/h C) 63 km/h
projétil é alojado no interior do bloco e ambos D) 82 km/h E) 92 km/h
sobem a uma altura H em relação à posição
inicial do bloco. Considere g como o módulo da F4) (UFBA-07)
aceleração gravitacional, despreze o atrito com
o ar e assinale o que for correto. 40
01) A colisão entre o bloco e o projétil é
elástica.
02) A velocidade do conjunto bloco-projétil £
£ 20
imediatamente após a colisão é ^2gH .
04) A velocidade do projétil antes da colisão
. f —Ma-b +1
era de 2gH .
0
0.06
lmp . Intervalo de tempo da colisão (s)

08) O módulo do trabalho realizado pela força O gráfico mostra aproximadamente a força, em
peso do conjunto bloco-projétil é de (mp + função do tempo, que uma parede vertical
MB)gH. exerce sobre uma bola de borracha de massa

318
Elementos da Física-Mecânica!-Colisão Mecânica
30g que se movimenta horizontalmente, desde
o instante em que a bola toca na parede até o
instante em que se separam. Considerando a F.,
colisão perfeitamente elástica, calcule, a partir 1
da análise do gráfico, o impulso que a parede
transmite à bola e, com esse valor, determine
a velocidade inicial da bola.

F5) (Unifesp-09) Uma pequena esfera A, com


massa de 90g, encontra-se em repouso e em
contato com a mola comprimida de um
dispositivo lançador, sobre uma mesa plana e
horizontal. Quando o gatilho é acionado, a l
mola se descomprime e a esfera é atirada a) Considere que um ônibus de massa M =
horizontalmente, com velocidade de 2,0m/s, 9000kg, viajando a 80km/h, colide na traseira
em direção frontal a uma outra esfera B, com de um carro de massa ma = 1000kg que se
massa de 180g, em repouso sobre a mesma encontrava parado. A colisão é inelástica, ou
mesa. No momento da colisão, as esferas se seja, carro e ônibus seguem grudados após a
conectam e passam a se deslocar juntas. O batida. Calcule a velocidade do conjunto logo
gráfico mostra a intensidade da força elástica após a colisão.
da mola em função de sua elongação. b) Além do excesso de velocidade, a falta de
241---------- ----------- ----- ----- ----- ----- ---------- -----
manutenção do veículo pode causar acidentes.
Por exemplo, o desalinhamento das rodas faz
20
com que o carro sofra a ação de uma força
16 lateral. Considere um carro com um pneu
dianteiro desalinhado de 3o, conforme a figura
3 12 acima, gerando uma componente lateral da
8,0 força de atrito FL em uma das rodas. Para um
carro de massa mb = 1600kg, calcule o módulo
4,0 da aceleração lateral do carro, sabendo que o
o* módulo da força de atrito em cada roda vale Fal
0 0,010 0,020 0,030 0,040 0,050 0,060 = 8000N.
x (m) Dados: sen3° = 0,05 e cos3° = 0,99.
Considerando que as esferas não adquirem
movimento de rotação, que houve F7) (Fuvest-07) Uma bola chutada
conservação da quantidade de movimento na horizontalmente de cima de uma laje, com
colisão e que não há atrito entre as esferas e a velocidade Vo, tem sua trajetória parcialmente
mesa, calcule: registrada em uma foto, representada no
a) a energia cinética da composição de esferas desenho abaixo. A bola bate no chão, no ponto
AB após a colisão. A, voltando a atingir o chão em B, em choques
b) quanto a mola estava comprimida no parcialmente inelásticos.
instante em que o gatilho do dispositivo
lançador é acionado. Foto
Vo
F6) (Unicamp-12) O tempo de viagem de
qualquer entrada da Unicamp até a região
central do campus é de apenas alguns
minutos. Assim, a economia de tempo obtida,
desrespeitando-se o limite de velocidade, é
muito pequena, enquanto o risco de acidentes
aumenta significativamente. H, = 3,2m

H2 = 1,8 m

1
I 1,6m r8
A D=?

319
Elementos tia física-Mecânica!-Colisão Mecânica
Nos choques, a velocidade horizontal da bola Imediatamente após a caça, o vetor velocidade
não é alterada. Desconsidere a resistência do u do gavião, que voa segurando o melro, forma
ar, o atrito e os efeitos de rotação da bola. um ângulo a com o plano horizontal tal que tg
a) Estime o tempo T, em s, que a bola leva até a é aproximadamente igual a
atingir o chão, no ponto A. a) 20. d) 0,3.
b) Calcule a distância D, em metros, entre os b) 10. e)0,1.
pontos A e B. c) 3.
c) Determine o módulo da velocidade vertical
da bola VA, em m/s, logo após seu impacto F10) (Fuvest-13) Uma das hipóteses para
com o chão no ponto A. explicar a extinção dos dinossauros, ocorrida
há cerca de 60 milhões de anos, foi a colisão
F8) (Fuvest-09) Para testar a elasticidade de de um grande meteoro com a Terra.
uma bola de basquete, ela é solta, a partir de Estimativas indicam que o meteoro tinha
uma altura Ho, em um equipamento no qual massa igual a 1016kg e velocidade de 30km/s,
seu movimento é monitorado por um sensor. imediatamente antes da colisão. Supondo que
Esse equipamento registra a altura do centro esse meteoro estivesse se aproximando da
de massa da bola, a cada instante, Terra, numa direção radial em relação à órbita
acompanhando seus sucessivos choques com desse planeta em torno do Sol, para uma
o chão. A partir da análise dos registros, é colisão frontal, determine
possível, então, estimar a elasticidade da bola, a) a quantidade de movimento Pj do meteoro
caracterizada pelo coeficiente de restituição imediatamente antes da colisão;
CR. O gráfico apresenta os registros de alturas, b) a energia cinética Ec do meteoro
em função do tempo, para uma bola de massa imediatamente antes da colisão;
M = 0,60 kg, quando ela é solta e inicia o c) a componente radial da velocidade da Terra,
movimento com seu centro de massa a uma Vr, pouco depois da colisão;
altura Ho = 1,6 m, chocando-se sucessivas d) a energia Ed, em megatons, dissipada na
vezes com o chão. O coeficiente de restituição, colisão.
CR = VR/V|, é a razão entre a velocidade com Note e adote:
que a bola é rebatida pelo chão (VR) e a A órbita da Terra é circular.
velocidade com que atinge o chão (V,), em Massa da Terra: 6.1024 kg.
cada choque. Esse coeficiente é 1 megaton = 4.1015 J é a energia liberada pela
aproximadamente constante nas várias explosão de um milhão de toneladas de
colisões. A partir dessas informações estime o trinitrotolueno.
coeficiente de restituição CR dessa bola,
utilizando a definição apresentada abaixo. F11) (EN-11) Dois pêndulos constituídos por
fios de massas desprezíveis e de comprimento
F9) (Fuvest-11) Um gavião avista, abaixo dele, L = 2,0 m estão pendurados em um teto em
um melro e, para apanhá-lo, passa a voar dois pontos próximos de tal modo que as
verticalmente, conseguindo agarrá-lo. esferas A e B , de raios desprezíveis , estejam
Imediatamente antes do instante em que o muito próximas, sem se tocarem. As massas
gavião, de massa MG = 300 g, agarra o melro, das esferas valem mA = 0,10 Kg e mB = 0,15
de massa Mm = 100 g, as velocidades do Kg. Abandona-se a esfera A quando um o fio
gavião e do melro são, respectivamente, VG = forma um ângulo de 60° com a vertical,
80 km/h na direção vertical, para baixo, e VM = estando a esfera B do outro pêndulo na
24 km/h na direção horizontal, para a direita, posição de equilíbrio. Sabendo que, após a
como ilustra a figura abaixo. colisão frontal, a altura máxima alcançada pelo
VG centro de massa do sistema , em relação à
posição de equilíbrio, é de 0,40m , o
coeficiente de restituição da colisão é
Dado : |g| = 10,0m/s2

VM
a
u

320
flementos da Física - Mecânica /- Colisão Mecânica
//////////// 0,40 s, atinge um ponto, no chão, a uma
distância D8 =2,0 m, ao longo da direção
horizontal, a partir da extremidade da mesa.
Supondo que nesse choque não tenha havido
conservação de energia cinética e que os
L L blocos tenham iniciado a queda no mesmo
instante:
B

a) zero b) 0,25 c) 0,50 d) 0,75 e)1,00

F12) (Mackenzie-06) Sobre uma mesa


disposta horizontalmente, na qual o atrito é ■<-------- Db------- ►

desprezível, encontra-se parada uma bola


indeformável, de massa M. Uma segunda bola a) Determine a distância horizontal DA, em
indeformável, de massa m, é lançada contra a metros, ao longo da direção horizontal, entre a
primeira, atingindo-a com a velocidade v. posição em que o bloco A atinge o chão e a
Imediatamente após o choque, as bolas extremidade da mesa.
seguem por direções perpendiculares entre si b) Represente, no sistema de eixos da folha de
e com velocidades, respectivamente, iguais a resposta, a velocidade vertical Vv de cada um
Vi e v2. A direção de v2 forma 60° com a dos blocos, em função do tempo, após o
direção de v e a relação entre seus módulos é: choque, identificando por A e B cada uma das
a) v2 = v/3 b) v2 = v/V3 c) v2 = v/2 curvas.
d) v2 = v e) v2 = 2v
F15) (Fuvest-05) Em uma canaleta circular,
plana e horizontal, podem deslizar duas
F13) (Fuvest-01) Um objeto A, de massa M = 4,0
pequenas bolas A e B, com massas MA = 3 MB,
kg, é largado da janela de um edifício, de uma
que são lançadas uma contra a outra, com
altura Ho = 45 m. Procurando diminuir o impacto
igual velocidade Vo, a partir das posições
de A com o chão, um objeto B, de mesma
indicadas. Após o primeiro choque entre elas
massa, é lançado um pouco depois, a partir do
(em 1), que não é elástico, as duas passam a
chão, verticalmente, com velocidade inicial V0B.
movimentar-se no sentido horário, sendo que a
Os dois objetos colidem, a uma altura de 25 m,
bola B mantém o módulo de sua velocidade
com velocidades tais que |VA| = |VB|. Com o
Vo. Pode-se concluir que o próximo choque
impacto, grudam-se, ambos, um no outro,
entre elas ocorrerá nas vizinhanças da posição
formando um só corpo AB, de massa 2M, que cai
1
atingindo o chão. i
I
a) Determine, a energia mecânica Q, em J, 8x
dissipada na colisão. 1
b) Determine a energia cinética Ec, em J, Vo/ v°
imediatamente antes de AB atingir o chão.

05
7“aP Õ-3
c) Construa, no sistema de coordenadas da folha
de respostas, o gráfico dos módulos das
velocidades em função do tempo para A, B e AB, '4
6
considerando que V0B = 30 m/s. Identifique, i

respectivamente, com as letras A, B e AB, os 5
gráficos correspondentes. a) 3 b) 5 c)6 d) 7 e) 8

F14) (Fuvest-02) Em um jogo, um pequeno F16) (UFC-02) A figura ao lado mostra uma
bloco A, de massa M, é lançado com calha circular, de raio R, completamente lisa,
velocidade Vo = 6,0 m/s sobre a superfície de em posição horizontal. Dentro dela há duas
uma mesa horizontal, sendo o atrito bolas, 1 e 2, idênticas e em repouso no ponto
desprezível. Ele atinge, no instante to =0, o A. Ambas as bolas são disparadas,
bloco B, de massa M/2, que estava parado simultaneamente, desse ponto: a bola 1, para
sobre a borda da mesma mesa, ambos indo ao a direita, com velocidade v-t = 6n m/s e a bola
chão. Devido ao choque, o bloco B, decorridos

321
Mementos tia Física - Mecânica!- Colisão Mecânica
2, para a esquerda, com velocidade v2 = 2n em repouso sobre ela, conforme mostra a
m/s. As colisões entre as bolas são figura (a). Após o choque da bola de basquete
perfeitamente elásticas. Indique onde ocorrerá com o solo, e em seguida com a bola de
a quarta colisão entre as bolas, após o disparo pingue-pongue, esta última atinge uma altura
delas. muito maior do que sua altura inicial.
Ibl

a) Entre os pontos A e B a) Para h = 80cm, calcule a velocidade com


b) Exatamente no ponto A que a bola de basquete atinge o solo.
c) Entre os pontos C e D Despreze a resistência do ar.
d) Exatamente no ponto C b) Abandonadas de uma altura diferente, a
e) Exatamente no ponto D bola de basquete, de massa M, reflete no solo
e sobe com uma velocidade de módulo V =
F17) (UFRJ-07) A figura 1 a seguir mostra um 5,0m/s. Ao subir, ela colide com a bola de
pêndulo constituído por um fio ideal de pingue-pongue que está caindo também com V
comprimento L, com uma extremidade presa a = 5,0m/s, conforme a situação representada na
um ponto fixo P, e por uma partícula de massa figura (b). Considere que, na colisão entre as
m presa à outra extremidade. O pêndulo está bolas, a energia cinética do sistema não se
inicialmente em repouso com o fio esticado na conserva e que, imediatamente após o
posição horizontal. Após ter sido abandonado choque, as bolas de basquete e pingue-
do repouso, o pêndulo desce e colide com pongue sobem com velocidades de V’b =
outra partícula de massa m, que está em 4,95m/s e V’p = 7,0m/s, respectivamente. A
repouso sobre uma superfície lisa, no ponto partir da sua própria experiência cotidiana,
mais baixo de sua trajetória. No choque, as faça uma estimativa para a massa da bola de
partículas se grudam de modo que o pêndulo pingue-pongue, e, usando esse valor e os
continua seu movimento com as duas presas dados acima, calcule a massa da bola de
em sua extremidade, como mostra a figura 2. basquete.
------- ®P F19) (Fuvest-08)
u

lll B
A v
O
figura 1 figura 2 ■ 1,6m I.Sm *

Suponha que todo o movimento ocorra em um Duas pequenas esferas iguais, A e B, de


plano vertical. mesma massa, estão em repouso em uma
a) Calcule, em função de L e do módulo da superfície horizontal, como representado no
aceleração da gravidade g, a velocidade da esquema acima. No instante t = 0 s, a esfera A
partícula presa à extremidade do pêndulo, é lançada, com velocidade Vo = 2,0 m/s, contra
imediatamente antes da colisão. a esfera B, fazendo com que B suba a rampa à
b) Calcule o valor máximo do ângulo 9 que o frente, atingindo sua altura máxima, H, em t =
pêndulo faz com a vertical após a colisão. 2,0 s. Ao descer, a esfera B volta a colidir com
A, que bate na parede e, em seguida, colide
F18) (Unicamp-08) Um experimento novamente com B. Assim, as duas esferas
interessante pode ser realizado abandonando- passam a fazer um movimento de vai e vem,
se de certa altura uma bola de basquete com que se repete.
uma bola de pingue-pongue (tênis de mesa)

322
Mementos da Física - Mecânica /- Colisão Mecânica
Os choques são elásticos. Tanto o atrito entre sua vez,a esfera de massa m encontra-se
as esferas e o chão inicialmente em repouso na posição A,
quanto os efeitos de rotação devem ser suspensa por um fio inextensivel e de massa
desconsiderados. desprezível. Após a colisão, percorre a
a) Determine o instante tA, em s, no qual trajetória circular ABCD de raio igual ao
ocorre a primeira colisão entre A e B. comprimento L do fio, Despreze o atrito no pivô
b) Determine o período T, em s, de um ciclo do O e a resistência do ar. Para que a esfera de
movimento das esferas. massa m percorra a trajetória circular, o valor
mínimo do módulo da velocidade Vo, antes da
F20) (AFA-01) Duas esferas A e B, de massas colisão, antes da colisão é
respectivamente iguais a 4 kg e 2 kg, Dado g é a aceleração da gravidade.
percorrem a mesma trajetória retilínea,
C
apoiadas num plano horizontal, com .....
velocidades de 10 m/s e 8 m/s, Y ’■••••
respectívamente, conforme a figura. Após a ;
ocorrência de um choque frontal entre elas, as l:
esferas movem-se separadamente e a energia
dissipada na colisão vale 162 J. Os módulos D; L

r
0 L j B
das velocidades de A e de B, após a colisão,
em m/s, valem, respectivamente,
L
Va Vb m0 v„ m
& B,
A
a) 8 e 6 b) 2 e7 c) 1 e8 d) 1 e 10
a) VgT b) 75g.L
F21) (EN-00) Uma bola de massa m, = M e
d) 2v'5gT e) 2^/lOg.L
velocidade v0 = 10 m/s choca-se com uma
outra bola de massa m2 = 1,5M que se
encontra em repouso na extremidade do topo F23) (ITA-60) A massa do bloco na figura
abaixo é 2,0 kg; a massa da bala é 2,0.10 3 kg.
do prédio do Corpo de Aspirantes da Escola
Naval, de altura h = 20m. Sabendo-se que A porcentagem da energia cinética da bala que
após o choque a bola de massa m2 é lançada é transformada em energia cinética do
horizontalmente e atinge o solo num ponto a conjunto é aproximadamente a:
dezesseis metros do prédio, conforme indica a ( ) A. 10%
figura abaixo, determine a velocidade da bola ( ) B. 0%
de massa m, (modulo, direção e sentido) logo ( )C. 0,1%
após a colisão. ( ) D. 100%
(Despreze os atritos e a resistência do ar e ( ) E. Nenhum dos valores a - -
considere g = 10 m/s2)
m, v« = 10 m/s
O enunciado que se segue refere-se às duas

□ questões abaixo.
Um bloco de massa m = 4,00 kg desliza sobre
um plano horizontal sem atrito e choca-se com
uma mola horizontal de massa desprezível, e
I I 20 J I constante elástica K = 1,00 n/m, presa a uma

□ □ parede vertical. Se a compressão máxima da


mola é de 2,00 cm.:

16 m
----- *
JzZzzzZz/z/zz/zzzzz
F22) (EN-12) A esfera de massa m0 tem o
módulo da sua velocidade reduzida a zero na
colisão frontal e inelástica (ou parcialmente F24) (ITA-69)
elástica) com a esfera de massa m = 2m0. Por

323
Mementos tia Física-Mecânica!-Colisão Mecânica
( ) A. a velocidade com que o bloco se afasta
IOmp
da mola, uma vez cessada a interação, é 1,00 V B
x 10'2 m/s; - ►—om»

( ) B, a energia cinética se conserva durante a ....................................


interação;
( ) C. a quantidade de movimento do bloco é
a mesma durante a interação; B
( ) D. a energia potencial do bloco é máxima
o
para uma compressão de 1,00 cm da mola; ___________________ A_________
( ) E. nenhuma das afirmações é correta.
A) -8.2 ms'’ B ) + 8.2 ms
F25) (ITA-69) e se o tempo de interação é de C ) 9,1 ms'1 D ) 110 ms ’’
1,0 segundos a força média (em relação ao E ) Indeterminado, pois não são conhecidas as
posições e velocidades iniciais dos projéteis.
tempo), que atua sobre o bloco será:
a)4,00x10'2N b)8,O0x1O’2N
F29) (ITA-81) Considere um sistema bate-
c)2,00x10’1N d)4,00x10’1N
e) 8,00x10’1 N estacas desses usados em construção civil.
Seja H a altura de queda do martelo que tem
massa mM e seja mE a massa da estaca a
F26) (ITA-71) Uma bola de golfe é deixada cair
ser gravada. Desejamos aumentar a
de uma altura H sobre uma superfície plana,
penetração a cada golpe e para isso podemos
horizontal e rígida. Supondo que colisão com a
alterar H ou mM . Considere o choque
superfície é perfeitamente elástica e que a
inelástico e despreze o atrito com o ar. Qual
força de atrito com o ar é constante em toda a
das afirmações está correta:
trajetória e igual a 10% da força da gravidade,
a) duplicando a altura de queda do martelo
a bola voltará a uma altura aproximadamente
também duplicamos sua velocidade no
igual a:
instante do impacto;
A) 0,90 H; B)0,10H;
b) duplicando a massa do martelo estaremos
C) 0,92 H; D) 0,82 H;
duplicando a energia cinética do sistema
E) Nenhum dos valores acima é correto.
martelo mais estaca imediatamente após o
choque;
F27) (ITA-71) A partir do resultado que você
c) a energia cinética do sistema é, após o
obteve na questão anterior, e supondo que a
choque, menor quando duplicamos a massa
bola continue pulando, após quantos pulos ela
do que duplicamos a altura de queda;
atinge aproximadamente a altura máxima de
-g? (Iog2s0,301) d) o fato de modificarmos H ou mM não altera
o poder de penetração da estaca;
A) 4; B) 6; C) 8; D) 10; E) 12. e) duplicando a massa do martelo estaremos
duplicando a quantidade de movimento do
F28) (ITA-86) Dois projéteis de igual massa m 0 e sistema após o choque.
mesma velocidade, movem-se em sentidos opostos
e colidem simultaneamente com um bloco de F30) (ITA-87) Um bloco de madeira de massa
madeira de massa 10 m 0, conforme a figura. O M está oscilando horizontalmente sobre uma
bloco, inicialmente em repouso, pode deslizar sem mesa sem atrito, sob a ação de uma mola de
atrito sobre a superfície em que se apoia. O projétil constante elástica K. A amplitude de sua
A, que se desloca para a direita, fica aprisionado ao
oscilação é A.
bloco, enquanto que o projétil B, que se desloca
para a esquerda, atravessa o bloco, e mantém a Quando a elongação da mola é máxima, o
sua direção original. A velocidade do projétil B, bloco é atingido por uma bala de massa m,
após atravessar o bloco de madeira é 100 m s ’1. viajando horizontalmente. A bala se engasta
Podemos afirmar que a velocidade final do bloco de instantaneamente no bloco e a amplitude do
madeira será da ordem de : movimento passa a ser 2A. Pedem-se:

* tn v
I
(a) a velocidade v da bala antes de atingir o
bloco.

324
ílementos da Física - Mecânlcal-Colisão Mecânica
(b) a máxima velocidade que o sistema rígida de comprimento L, inextensível e de
atingirá após o choque. massa desprezível. A seguir, a partícula I
(c ) a quantidade de calor gerada no choque, desloca-se na direção de II com velocidade
supondo que toda a energia dissipada se uniforme VB, que forma um ângulo 0 com a
transforme em calor. haste. Desprezando qualquer tipo de
Sâo dados: M, K, A, m. resistência ou atrito, pode-se afirmar que,
imediatamente após a colisão (elástica) das
F31) (ITA-88) Uma bola de massa m é
partículas.
lançada, com velocidade inicial v0 , para o
interior de um canhão de massa M, que se Q
acha inicialmente em repouso sobre uma ^90°
superfície lisa e sem atrito, conforme mostra a
figura. O canhão é dotado de uma mola, que
estava distendida, fica comprimida ao máximo e L X
e a bola fica aderida ao sistema, mantendo a
mola na posição de compressão máxima. II
Supondo que a energia mecânica do sistema
permaneça constante, a fração da energia
cinética inicial da bola que ficará armazenada
em forma de energia potencial elástica será a) a partícula II se movimenta na direção
igual a:
definida pelo vetor VB.
M b) o componente y do momento linear do
m sistema é conservado.
v. c) o componente x do momento linear do
O- sistema é conservado.
I
d) a energia cinética do sistema é diferente do
'////////////////////z seu valor inicial.
a) m/M b) M/m c) M/(m + M) e) N.D.A.
d) m/(m + M) e) 1,0
F34) (ITA-05) Um vagão-caçamba de massa M
F32) (ITA-88) A figura a seguir esquematiza o se desprende da locomotiva e corre sobre
estudo das colisões unidimensionais trilhos horizontais com velocidade constante v
= 72,0 km/h (portanto, sem resistência de
®—*---- ®
®--------© qualquer espécie ao movimento). Em dado
m m m/2 instante, a caçamba é preenchida com uma
A partícula (A) de massa m com uma carga de grãos de massa igual a 4M,
velocidade inicial v0 colide com a partícula (B) despejada verticalmente a partir do repouso de
também de massa m que se acha em repouso. uma altura de 6,00 m (veja figura). Supondo
A colisão é perfeitamente elástica. Após a que toda a energia liberada no processo seja
primeira colisão, a partícula (B) colide com a integralmente convertida em calor para o
partícula (C) de massa m/2. Que se acha em aquecimento exclusivo dos grãos, então, a
repouso. No processo anteriormente descrito, quantidade de calor por unidade de massa
calcular: recebido pelos grãos é
a) a velocidade vCM do centro de massa deste a) 15 J/ kg.
grãos 4M
sistema de partículas. b) 80 J / kg.
b) a velocidade vB da partícula B após a c) 100 J/kg. l l l l lV
colisão perfeitamente elástica com a partícula d) 463 J / kg. M
C. e) 578 J / kg. o
F33) (ITA-03) Sobre um plano liso e horizontal
repousa um sistema constituído de duas F35) (ITA-07) Uma bala de massa m e
partículas, I e II, de massas M e m, velocidade v0 é disparada contra um bloco de
respectivamente. A partícula II é conectada a massa M, que inicialmente se encontra em
uma articulação O sobre o plano por meio de repouso na borda de um poste de altura h,
uma haste que inicialmente é disposta na conforme mostra a figura. A bala aloja-se no
posição indicada na figura. Considere a haste bloco que, devido ao impacto, cai no solo.

325
Elementos da Física-Mecânica!-Colisão Mecânica
Sendo g a aceleração da gravidade, e não k
havendo atrito e nem resistência de qualquer
outra natureza, o módulo da velocidade com X VíO
que o conjunto atinge o solo vale
B
__ ^2 I d
mv0 + 2gh
a)
m+M
vo m, + m2

'lli.
in
a) b) x = m,
b)
y (m + m)2 m, + m2
c) x = d) m,
c) Jvg
V

M e) x = __ SL__ d
(m, + m,)
d) 7vo +2gh
I 2 F38) (ITA-82) Um martelo bate-estacas
e) J^ + 2gh funciona levantando um corpo de pequenas
Im + M
dimensões e de massa 70,0 kg acima do topo
de uma estaca de massa 30,0 kg. Quando a
F36) (ITA-88) Uma haste rígida e de massa
altura do corpo acima do topo da estaca é de
desprezível possui presas em suas
1,00 m, ela afunda de 0,500 m do solo.
extremidades duas massas idênticas m. Este
Supondo uma aceleração da gravidade de
conjunto acha-se sobre uma superfície
10,0 m.s-2 e considerando o choque inelástico,
horizontal perfeitamente lisa (sem atrito). Uma
podemos concluir que a força média de
terceira partícula também de massa m e
velocidade $ desliza sobre esta superfície resistência à penetração da estaca é de:
A) 1,96 x 103N
numa direção perpendicular à haste e colide B) 2,96 x 103N
inelasticamente com uma das massas da C) Não é possível determiná-la se não
haste, ficando colocada à mesma após a forem dadas as dimensões da estaca
colisão. Podemos afirmar que a velocidade do D) 29,0 x 103N
centro de massas VCM (antes e após a E) 29,7 x 103N
colisão), bem como o movimento do sistema
após a colisão serão : F39) (ITA-04) Atualmente, vários laboratórios,
Çm utilizando vários feixes de laser, são capazes
de resfriar gases a temperaturas muito
próximas do zero absoluto, obtendo moléculas
e átomos ultrafrios. Considere três átomos
ultrafrios de massa M, que se aproximam com
velocidades desprezíveis. Da colisão tripla
resultante, observada de um referencial
VCM (antes) Vcw(após) MOV, subsequente situado no centro de massa do sistema, forma-
do sistema se uma molécula diatômica com liberação de
a) 0 0 Circular e uniforme certa quantidade de energia B. Obtenha a
b) 0 v/3 Translacional e rotacional velocidade final do átomo remanescente em
c) 0 v/3 Só translacional função de BeM.
d) v/3 v/3 Translacional e Rotacional
e) v/3 0 Só rotacional F40) (IME-73) Uma bola de 0,1 kg de massa é
deixada cair de uma altura de 10 m. ao se
F37) (ITA-83) Um corpo A de massa igual a chocar com o chão (horizontal) ela sofre uma
mi é abandonado no ponto 0 e escorrega variação de quantidade de movimento de 2,52
por uma rampa. No plano horizontal, choca-se m.kg/s. Determine o coeficiente de restituição
com outro corpo B de massa igual a m2 que entre a bola e o chão. Use g = 9,8 m/s2.
estava em repouso. Os dois ficam grudados e
continuam o movimento na mesma direção até F41) (IME-75) Três corpos de massas iguais a
atingir uma outra rampa na qual o conjunto 0,02 utm, cada um, movendo-se a 400 m/s,
pode subir. Considere o esquema da figura e atingem simultaneamente um bloco de
despreze o atrito. Qual a altura x que os madeira, em repouso, de massa igual a 1 utm.
corpos atingirão na rampa? As trajetórias individuais dos três corpos estão
num mesmo plano vertical, conforme a figura.

326
Mementos da Física - Mecânica !- Colisão Mecânica
Calcular a velocidade do sistema bloco e três representada na figura acima, sem atrito,
corpos logo após a colisão. colide com outro corpo puntiforme de massa
mB, que se encontrava inicialmente em
repouso no ponto B. Sabendo que este choque
a = 30° é perfeitamente inelástico e que o corpo
resultante deste choque atinge o ponto C,
ponto mais alto da rampa, com a menor
velocidade possível mantendo o contato com a
Bloco
rampa, a velocidade inicial do corpo no ponto
A, em m/s, é
F42) (IME-76) O choque entre 2 esferas A e B, Dados:
levou ao traçado do gráfico abaixo. Sendo • raio da rampa circular: 2 m;
dados: mA = 0,15 kg, mB = 0,2 kg e t2 - ti = • aceleração da gravidade g: 10m/s2;
0,001 s, determinar • massa mA: 1kg;
1) o coeficiente de restituição • massa mB: 1kg.
2) a natureza da choque a) 10 b) 20 c)4VÍ5 d) 10>/5 e) 8^5
3) a força de impulsão

kg.m/s F45) (OBF-04) No esquema da figura, a


bolinha de massa m é abandonada em
3 A repouso e desliza, sem girar, sobre o plano
inclinado sem atrito, desde a altura h, até ser
B
2 encaçapada pelo buraco na superfície
inclinada.
A t(s)
-1 h
B

F43) (IME-88) Um pêndulo A, de peso PA = 10


N, é solto com velocidade nula de uma posição
Considerando que nao há atrito entre o bloco
horizontal e oscila livremente até a posição
de massa Mea superfície horizontal, o bloco
vertical, atingindo o pêndulo B, de peso PB =
17 N, que está inicialmente em repouso. Os M
a) move-se inicialmente para a esquerda e
pêndulos tem o mesmo comprimento / = 0,45
depois retorna à sua posição inicial.
m. Devido ao choque (com coeficiente de
b) adquire uma velocidade para a direita
restituição e = 0,8), o pêndulo B oscila até uma
proporcional à altura h.
altura h desde a sua posição inicial. Calcule
esta altura h. Considere g = 10 m/s2. c) move-se para a esquerda e pára.

••A»
d) adquire uma velocidade para a esquerda
I I I I I 1
proporcional à altura h.
e) permanece imóvel o tempo todo.

F46) (OBF-07) Uma esfera de aço de massa


m, = 200 g, está presa à extremidade de uma
F44) (IME-15) corda de comprimento t = 45 cm, e que tem
fixa a outra extremidade. A esfera é
abandonada sob a ação de seu peso quando a
corda está na horizontal. No ponto mais baixo
de sua trajetória a esfera colide elasticamente
com um bloco de aço de massa m2 = 1,8 kg,
inicialmente em repouso, sobre uma superfície
horizontal, cujo coeficiente de atrito vale p =
0,2.
a) Qual a velocidade dos corpos
B imediatamente após a colisão?
Um corpo puntiforme de massa mA parte de b) Quanto o bloco se desloca sobre a
ponto A, percorrendo a rampa circular superfície horizontal até atingir o repouso?

327
flementos da fisica-Mecânica!-Colisão Mecânica
F51) Considere n pêndulos idênticos feitos de
F47) (OBF-04) Uma bola, de massa igual a massa de moldar, destas que mantêm as
100 g, é abandonada de uma altura de 1,25 m, deformações a que são submetidas. Elevando-
bate no chão e torna a subir até a altura de se um dos pêndulos da extremidade e a uma
0,80 m. Desprezando a resistência do ar, altura h acima do nível mais baixo e
determine: abandonando-se, este colide com as outras.
a) o coeficiente de restituição; Calcule a altura atingida pelo centro de massa
b) o impulso do chão sobre a bola; do conjunto após a colisão.
c) a força máxima exercida pelo chão sobre a
bola, considerando que a colisão dure 20 ms e F52) Uma bala de 5 g incide sobre um pêndulo
que a variação da força com o tempo seja balístico de massa igual a 2 kg com uma
como no gráfico abaixo. velocidade de 400 m/s, atravessa-o e emerge
do outro lado com uma velocidade de 100 m/s.
Calcule a altura de elevação do pêndulo,
desprezando a elevação durante o tempo que
z a bala leva para atravessá-lo.

F53) Duas esferas, de massas m e m/4, estão


suspensas por fios sujeitos a um mesmo
ponto. A esfera de massa m/4 é desviada de
t(s) tal modo que se eleva até uma altura H e é
solta. Depois do choque esta esfera se eleva
F48) A que altura volta um corpo, largado de até uma altura h. O choque é central.
uma altura h, após chocar-se com o solo, Determinar a quantidade de energia que é
sendo que o choque possui coeficiente de transformada em calor.
restituição igual a e? Determine também o
tempo gasto para que a bola pare de pular.

F49) Uma bolinha de um pêndulo simples, de Exercícios de Aprofundamento


comprimento /, tem massa m e é largada, sem
velocidade inicial, com o fio do pêndulo na A1) (UFES-07) Um corpo de massa m está em
horizontal. Ao passar pelo ponto mais baixo de contato com uma mola de constante elástica k
sua trajetória, a bolinha colide frontal e que sofre uma deformação x. O sistema é
elasticamente com um carrinho de massa 2m, liberado e o corpo é lançado, indo se chocar
inicialmente em repouso e apoiado em um frontalmente com outro corpo de massa M em
trilho que é horizontal naquela região. Depois repouso, preso a uma haste rígida de
do choque, o carrinho se desloca sem atrito ao comprimento L e massa desprezível. A haste
longo do trilho e sobe até uma determinada está presa por um pino na outra extremidade
altura máxima em relação ao trecho horizontal de forma a poder girar sem atrito. Após o
do trilho. Qual o valor dessa altura máxima? choque, os corpos permanecem unidos, e a
haste e os corpos giram com velocidade
F50) Uma partícula de massa m colide angular constante. Os movimentos ocorrem
horizontalmente com uma placa que se sobre uma mesa plana, e não há atrito entre os
encontra presa a uma mola de constante corpos e a mesa. Determine:
elástica K, conforme a figura. Após a colisão a
partícula permanece presa à placa
comprimindo a mola. Sendo Xo a compressão
máxima sofrida pela mola, determine a
velocidade da partícula imediatamente antes
da colisão considerando: a) massa desprezível
para a placa; b) massa m para a placa.
m v


ÍW A) a velocidade com que o corpo de massa m
é lançado pela mola;
B) a velocidade dos corpos logo após a
colisão;

328
Elementos ãa Física-Mecânica!-Colisão Mecânica
y
C) a tração na haste enquanto os corpos
giram.
X

A2) (UFES-13) Um bloco de massa M = 0,30 j R


h*
kg é abandonado do alto de uma rampa de <i> v> <2> Z
declividade a = 45°. Ele parte da altura H / / / ^'7777'77'7 ,
=16m, indo atingir outro bloco de massa m = a) 1,0 b) 1,2 c) 2,5 d) 2,0 e) 3,0
0,10 kg , inicialmente em repouso na base da
rampa (ver figura). Os blocos se grudam por A4) (Fuvest-98) Um brinquedo é constituído
causa de velcro afixado nas partes que se por um cano (tubo) em forma de 3/4 de arco de
tocam no choque, indo o conjunto, em seguida, circunferência, de raio médio R, posicionado
subir uma segunda rampa de declividade 0 = num plano vertical, como mostra a figura. O
30° . A rampa inicial, à direita na figura, é desafio é fazer com que o bola 1, ao ser
áspera, e o coeficiente de atrito cinético entre abandonada de uma certa altura H acima da
sua superfície e o bloco vale m = 0,80. Já a extremidade B, entre pelo cano em A, bata na
rampa à esquerda é perfeitamente polida, não bola 2 que se encontra parada em B, ficando
havendo atrito entre ela e os blocos. Nos nela grudada, e ambas atinjam juntas a
cálculos a seguir, despreze a pequena extremidade A. As massas das bolas 1 e 2 são
curvatura da trajetória dos blocos nas junções M, e M2, respectivamente. Despreze os efeitos
entre as bases das rampas e o piso horizontal. do ar e das forças de atrito.
Despreze também o atrito no piso horizontal, a a) Determine a velocidade v com que as duas
resistência do ar e as dimensões dos blocos. bolas grudadas devem sair da extremidade B
do tubo para atingir a extremidade A.
(sem atrito) b) Determine o valor de H para que o desafio
seja vencido.
■■■■ i
ni

Determine
A) a velocidade do bloco de massa M,
imediatamente antes de se chocar com o bloco
em repouso;
B) a velocidade do conjunto de blocos,
imediatamente após o choque;
C) a altura máxima atingida pelo conjunto de
blocos ao subir a rampa da esquerda; A5) (UFC-04) Um grande cubo de isopor, de
D) o intervalo de tempo gasto pelo conjunto na massa M e aresta L = 30,0 metros, repousa
subida da rampa da esquerda. sobre uma superfície horizontal perfeitamente
lisa. Um projétil de massa m = (1/5)A4 é
A3) (EN-12) Uma pista é composta por um disparado horizontalmente contra o cubo
trecho retilíneo longo horizontal seguido do atingindo-o com velocidade v = 300 m/s,
trecho circular vertical de raio R (conforme a perpendicularmente a uma de suas faces. O
figura abaixo). O carrinho (1) (partícula), de projétil atravessa o cubo e sai perpendicular à
massa mi = 1,0 kg e velocidade Vi= 5,0.f face oposta, com velocidade u = 200 m/s.
(m/s), colide com o carrinho (2) (partícula), de Suponha que a força de resistência (atrito) que
massa m2 = 2,0 kg, em repouso no trecho atua sobre o projétil, enquanto ele atravessa o
retilíneo. Despreze os atritos. O coeficiente da cubo, é constante.
restituição do choque vale 0,80. Após a
colisão, o carrinho (2) sobe o trecho circular \l u
vertical e, num certo instante, passa pela vez m
na posição A, de altura hA = R, com velocidade depois |
tal que o módulo da força normal da pista a) Qual a velocidade V do cubo após ser
sobre o carrinho é igual ao módulo do seu atravessado pelo projétil?
peso. Nesse instante, o módulo da velocidade b) Quanto tempo durou a travessia?
(em m / s) do carrinho (2) em relação ao c) Calcule a distância D que o cubo percorre
carrinho (1) é enquanto dura a travessia.

329
Elementos da Física-Mecânica!-Colisão Mecânica
módulo da quantidade de movimento do anti-
A6) (Unicamp-97) Jogadores de sinuca e bilhar neutrino.
sabem que, após uma colisão não frontal de b) Qual é a velocidade do núcleo de hélio após a
duas bolas A e B de mesma massa, estando a desintegração? A massa do núcleo de hélio é 5,0
bola B inicialmente parada, as duas bolas saem x W27 kg.
em direções que formam um ângulo de 90° .
Considere a colisão de duas bolas de 200g, A8) (UFSC-02) Em uma partida de sinuca,
representada na figura a seguir. A se dirige em resta apenas a bola oito a ser colocada na
direção a B com velocidade V = 2,0 m/s caçapa. O jogador da vez percebe que, com a
formando um ângulo a com a direção y tal que disposição em que estão as bolas na mesa,
sen a = 0,80. Após a colisão, B sai na direção y. para ganhar a partida ele deve desviar a bola
a) Calcule as componentes x e y das velocidades oito de 30 graus, e a bola branca de pelo
de A e B logo após a colisão. menos 60 graus, para que a mesma não entre
b) Calcule a variação da energia (cinética de na caçapa oposta, invalidando sua jogada.
translação) na colisão. Então, ele impulsiona a bola branca, que colide
NOTA: Despreze a rotação e o rolamento das elasticamente com a bola oito, com uma
bolas. velocidade de 5 m/s, conseguindo realizar a
jogada com sucesso, como previra, vencendo
a partida. A situação está esquematizada na
figura abaixo. Considere as massas das bolas
como sendo iguais e despreze qualquer atrito.

A7) (Unicamp-02) A existência do neutrino e do


anti-neutrino foi proposta em 1930 por Wolfgang
Pauli, que aplicou as leis de conservação de
quantidade de movimento e energia ao processo Considerando o sistema constituído pelas duas
de desintegração p. O esquema abaixo ilustra bolas, assinale a(s) proposição(ões) CORRE­
esse processo para um núcleo de trítio, H3 (um TAS).
isótopo do hidrogênio), que se transforma em um 01. Devido à colisão entre a bola branca e a
núcleo de hélio, He3, mais um elétron, e“, e um bola oito, a quantidade de movimento do
anti-neutrino, v. O núcleo de trítio encontra-se sistema de bolas não é conservada.
02. A velocidade da bola branca, após a
inicialmente em repouso. Após a desintegração,
colisão, é de 2,5m/s.
o núcleo de hélio possui uma quantidade de
movimento com módulo de 12 x 10'24 kg m/s e o 04. A energia cinética da bola oito, após a
colisão, é maior do que a energia cinética da
elétron sai em uma trajetória fazendo um ângulo
bola branca, antes da colisão.
de 60° com o eixo horizontal e uma quantidade
de movimento de módulo 6,0 x 10'24 kg m/s. 08. Após a colisão, a quantidade de
movimento total, na direção perpendicular à
direção de incidência da bola branca, é nula.
16. A energia cinética da bola branca, após a
colisão, é três vezes menor que a energia
He3 60° cinética da bola oito.
32. Como a colisão é elástica, a energia
cinética da bola branca, antes da colisão, é
maior do que a soma das energias cinéticas
das bolas branca e oito, após a colisão.
64. Desde que não existam forças externas
v \ atuando sobre o sistema constituído pelas
bolas, a quantidade de movimento total é
a) O ângulo a que a trajetória do anti-neutrino faz
conservada no processo de colisão.
com o eixo horizontal é de 30°. Determine o

330
fiementas da física - Mecânica /- Colisão Mecânica
A9) (ITA-81) O bloco 1 de massa igual a 1,0 kg atrito. Após a colisão, o movimento é mantido
e velocidade de 8,0 m/s colide com um bloco pela mesma força F, tal que o bloco de massa
idêntico 2, inicialmente em repouso. Após a m permanece unido ao de massa M em
colisão ambos os blocos ficam grudados e equilíbrio vertical, devido ao coeficiente de
sobem a rampa até comprimir a mola M de atrito estático pe existente entre os dois blocos.
0,10 m. Desprezando os atritos e considerando
g = 10 m/s2; h = 0,50 m e 6 = 30°, pergunta-se
qual o valor da constante da mola?

_ M M
T
I m I
0,10 m
V i = 8,0 m/s
h Considerando g a aceleração da gravidade e
B, B, 32. a velocidade instantânea do primeiro bloco
a) 1,2 x 103 N.m'1 ; b) 1,0 x 103 N.m’1 ; logo antes da colisão, a potência requerida
c)6,4x 103 N.m’1 ; d)3,2x103 N.m'1 ; para mover o conjunto, logo após a colisão, tal
e) 1,1 x 102 N.m’1 ; que o bloco de massa m não deslize sobre o
outro, é dada pela relação:
A10) (ITA-85) Dois blocos de massas m, e m2 g(M - m)V0 gmV0 gMV0
b) c)
estão ligados por um fio inextensível que Me Me pe(M + m)
passa por uma polia, com atrito desprezível, gmVp gMV0
d) e)
sendo m1 > m2. O corpo mf repousa pe(M + m) Me
inicialmente sobre um apoio fixo. A partir de
altura h deixa-se cair sobre m2 um corpo de A12) (ITA-72) Três bolas rígidas idênticas, de
massa m3, que gruda nele. Sabendo-se que massa igual a 0, 20 kg estão sobre uma mesa;
m-i > m2 + m3, pode-se afirmar que a altura duas delas estão paradas e a terceira dirige-se
máxima atingida por m1 será: com velocidade v0 = 2,0 m /s para uma colisão
com as outras duas, conforme a figura. A mesa
não oferece atrito ao deslocamento as bolas
(não há rotação das mesmas). Da
m3 configuração de velocidade abaixo, qual delas
deve representar o que ocorre com as bolas
m, “t após o choque ? Os vetores estão em escala.
h

m2
2 ■ T,
m3 | m.] + m2 +m3 h
a)
^m2 +m3 I m1 m2 — m3
m2(mi+m2+m3)h
b)
(m1-m2 -m3)3

C) ------------------- ----------------------- h
(m1 + m2 + m3 )(m1 - m2 - m3)
d)h

e) --------- ------------ 7»
(rrq +m2 +m3)

A11) (ITA-99) Um bloco de massa M desliza


por uma superfície horizontal sem atrito,
empurrado por uma força F, como mostra a A13) (ITA-72) No problema anterior, os
figura abaixo. Esse bloco colide com outro de módulos das velocidades das bolas após o
massa m em repouso, suspenso por uma choque são respectivamente:
argola de massa desprezível e também em

331
Elementos tia Física - Mecânica /- Colisão Mecânica
v, (m/s) v2 (m/s) v3 (m/s) c) Verdadeira, sua velocidade era 75 km/h.
A) 0,66 0,66 0,66 d) Verdadeira, sua velocidade era de 60 km/h.
B) 2,0 1,0 1,0 e) Não é possível calcular sua velocidade com
C) 0,40 1,38 1,38 os dados fornecidos.
D) 1,38 0,40 0,40
E) 1,0 2,0 1,0 A16) (IME-90) Um bloco C desliza com
velocidade constante sobre o trecho horizontal
A14) (ITA-08) Numa brincadeira de aventura, o da pista e choca-se com o bloco D, de mesma
garoto (de massa M) lança-se por uma corda massa, inicialmente em repouso. Em
amarrada num galho de árvore num ponto de consequência, o bloco D desloca-se e ao
altura L acima do gatinho (de massa m) da passar no ponto mais alto B não exerce
figura, que pretende resgatar. Sendo g a qualquer esforço sobre a pista. O bloco C
aceleração da gravidade e H a altura da continua em movimento e chega a subir na
plataforma de onde se lança, indique o valor parte curva da pista até uma altura de 0,2 m
da tensão na corda, imediatamente após o em relação ao trecho horizontal. Desprezando
garoto apanhar o gato para aterrisá-lo na outra a resistência do ar e o atrito entre as
margem do lago. superfícies, determine a velocidade do bloco C
antes do choque.
Dados: g = 10 m/s2; r = 2,88 m

A17) (IME-90) Uma bola cai de uma altura H =


a) Mg^1+^ b) (M + m)g 1-í^l 2H 5 m e saltita sobre uma placa rígida na
k M ) L superfície da terra. Um pesquisador observa
que o tempo decorrido entre o início de sua
C) Mgfl-^j d)(M+m)gíl-f^2^ queda e o instante em que a bola atinge a
+ my L altura máxima após dois choques com a placa
é de 3,24 segundos. Desprezando-se as
e) (m + M)g V M J resistências e admitindo que os choques
L
tenham o mesmo coeficiente de restituição,
determine:
A15) (IME-72) O carro A foi abalroado pelo a) o coeficiente de restituição dos choques;
caminhão B, de massa igual ao triplo da sua. O b) a altura máxima após o 2o choque.
caminhão se deslocava com velocidade de 36 Dado: g = 10 m/s2.
km/h. Após o choque, que se deu no ponto P,
os dois veículos, unidos, se deslocaram em A18) (IME-95) Em uma fábrica de bombons,
linha reta até Q. O motorista do carro declarou tabletes de balas caem continuamente sobre o
que sua velocidade no instante do choque era prato de uma balança, que originariamente
inferior a máxima permitida, 80 km/h. Esta indicava leitura nula. Eles caem de uma altura
afirmação é falsa ou verdadeira? de 1,8 m à razão de 6 por segundo.
Determine a leitura da escala da balança, ao
fim de 10s, sabendo que cada tablete tem
massa de 10g e as colisões são
o
completamente inelásticas.
A NOTA: Despreze a resistência do ar.
Considere g = 10 m/s2

n0r
a) Falsa, sua velocidade era de 90 km/h.
b) Falsa, sua velocidade era de 108 km/h.
A19) (IME-09) Dois corpos A e B encontram-se
sobre um plano horizontal sem atrito. Um
observador inercial O está na origem do eixo x.
Os corpos A e B sofrem colisão frontal
perfeitamente elástica, sendo que,

332
Elementos da Física-Mecânica!-Colisão Mecânica
inicialmente, o corpo A tem velocidade vA = t&A) (IME-79) Um bloco com 10 kg de massa
2m/s (na direção x com sentido positivo) e o está apoiado sobre o plano horizontal e ligado
corpo B está parado na posição x = 2m. à parede através da mola de constante elástica
Considere um outro observador inercial O’, que de 10 N/m e massa desprezível. Um projétil de
no instante da colisão tem a sua posição 20 g de massa e com velocidade de 750 m/s
coincidente com a do observador O. Se a choca-se com o bloco, ficando no interior do
velocidade relativa de O’ em relação a Oé v0' mesmo. O coeficiente de atrito entre o bloco e
= 2 m/s (na direção x com sentido positivo), o plano é 0,2. Determine a amplitude do
determine em relação a O’: movimento final do sistema.
a) as velocidades dos corpos A e B após a
colisão;
b) a posição do coro A dois segundos após a
colisão.
Dados:
massa de A = 100g; massa de B = 200g. A25) (IME-09)
Px(kgms)' t
A20) (IME-72) Dois corpos de massas m, = 20 A
kg e m2 = 10 kg, deslocam-se sobre um plano 4
B
horizontal sem atrito, como mostra a figura. 3
Sabe-se que | u, | = 5 m/s ; | ü2 |= 20 m/s e 2
que após o choque os corpos invertem os 1
sentidos das respectivas velocidades e A
0
deslocam-se com | v, |= 10 m/s e | v2 l= 10 B T(10’s)
m/s. Admitindo, que outro choque, os mesmo Py(kgm s) j k
corpos deslocam-se com as velocidades
| ü, | = 10 m/s e | ti, | = 5 m/s, com os sentidos A
2
da figura, as velocidades v1 e v2 após o
1
choque serão: A, B
0
T(10-’s)
.. rn,n-£'...... -1
B
-2

A21) (IME-77) Deixa-se cair livremente, de um Duas partículas A e B de massas mA = 0,1 kg e


altura h, um martelo de massa Mi sobre uma mB = 0,2 kg sofrem colisão não frontal. As
estaca de massa M2. O coeficiente de componentes x e y do vetor quantidade de
restituição do choque é desprezível. Admiti-se movimento em função do tempo são
que o solo ofereça uma resistência apresentadas nos gráficos acima.
proporcional à penetração da estaca em cada Considere as seguintes afirmativas:
golpe do martelo. Determine a expressão de h, I. A energia cinética total é conservada.
para a qual a estaca penetra, com um só II. A quantidade de movimento total é
golpe, uma distância e (h » e) conservada.
III. O impulso correspondente à partícula B é 2i
A22) (IME-78) Uma bola de aço, com massa + 4j.
de 0,02 kg, colide verticalmente contra um IV. O impulso correspondente à partícula A é -
bloco de aço, fixo ao solo, atingindo-o com 3i + 2j.
velocidade de 20 m/s. Sendo 0,8 o coeficiente As afirmativas corretas são apenas:
de restituição, calcule a altura atingida pela a) I e II b) I e III c) II e III d) II e IV e) III e IV
bola após a colisão. Use g = 10 m/s2.
A26) (IME-11) A figura apresenta duas massas
A23) (IME-79) Uma esfera com massa de 1 kg m, = 5 kg e m2 = 20 kg presas por um fio que
colide frontalmente, com velocidade de 10 m/s, passa por uma roldana. As massas são
com outra esfera de mesma massa e em abandonadas a partir do repouso, ambas a
repouso. Durante a colisão há perda de 25 J uma altura h do solo, no exato instante em que
de energia cinética. Determine o coeficiente de um cilindro oco de massa m = 5 kg atinge m,
restituição do par de esferas. com velocidade v = 36 m/s, ficando ambas
coladas.

333
___ Mementos da Física-Mecânica!-Colisão Mecânica
A28) Demonstre que, no caso de um choque
'/////////, elástico entre uma partícula material de massa
m, com outra de massa m2 que está
inicialmente em repouso:
Vlf
mi

V|l m2 z^0,
Cilindro oco v.l -►

m2
m\
] [ mi v2f

a) o ângulo máximo segundo o qual m2 pode


desviar-se pelo choque é dado por
I h
cos2 9 = 1- m2
, de modo que 0 < 01 < n/2.
m,
quando m, > m2.
b) 0, + 02 = tt/2, quando m-, = m2.
c) 0, pode tomar todos os valores entre 0 e n,
Determine a altura /?, em metros, para que quando m, < m2.
chegue ao solo com velocidade nula.
Dado: A29) Dois cubos estão situados bem juntos
• Aceleração da gravidade: g = 10 m/s2 sobre uma superfície horizontal lisa. A aresta
Observação: de cada cubo é L e a massa M. Um dos cubos
• A roldana e o fio são ideais. é atingido por uma bala de massa m, que se
a) 5,4 b) 2,7 c) 3,6 d) 10,8 e)1,8 move em direção da linha que une os centros
dos cubos. Considerando que a força de
A27) (IME-12) resistência horizontal, resultante do movimento
da bala, é constante e igual a F, determinar em
piso rugoso
que limites deve encontrar-se a velocidade
piso liso inicial da bala, para que a mesma passe pelo
Mi
primeiro bloco, mas fique encaixada no
v, = 1 m/s segundo.
-> —1------------------ > .v
60°^y
V m
/
M M
v,= 2 m/s

Duas bolas, 1 e 2, movem-se em um piso


fFL(2M + m) lFL(4M + 3m)
perfeitamente liso. A bola 1, de massa rm = 2 a)
Mm V Mm
kg, move-se no sentido da esquerda para
direita com velocidade v, = 1 m/s. A bola 2, de fFL(M + m) FL(2M + m)
b)
massa m2 = 1 kg, move-se com ângulo de 60o Mm 0 Mm
com o eixo x, com velocidade v2 = 2 m/s. FL(2M + m)
Sabe-se que o coeficiente de atrito cinético Mm
entre as bolas e o piso rugoso é 0,10.sec2p e a
FL(3M + 2m7
aceleração gravitacional é 10 m/s2. Ao
colidirem, permanecem unidas após o choque Mm
e movimentam-se em um outro piso rugoso, , /fl ífl
e) J— ív0 < J—
conforme mostra a figura. A distância V M V m
percorrida, em metros, pelo conjunto bola 1 e
bola 2 até parar é igual a A30) Uma bola cai livremente desde uma
a) 0,2 b) 0,5 c) 0,7 d) 0,9 e)1,2 altura h sobre um plano inclinado que forma
um ângulo a com a horizontal. Encontrar o

334
Fiementos da Física - Mecânica /- Colisão Mecânica
A A33) Uma partícula de massa m2, desloca-se
valor da razão —-, que corresponde à razão
A2 com velocidade escalar e colide
entre os dois primeiros alcances do movimento frontalmente com uma partícula estacionária
da bola ao longo do plano inclinado. Considere de massa m2. Depois da colisão, as
que os choques da bola com o plano são velocidades das partículas são v1f e v2f. Provar
perfeitamente elásticos. que a condição para v1f ser positiva, isto é,
para que a primeira partícula continuar a
9i deslocar-se na mesma direção inicial, é mt/m2
> e, onde e é o coeficiente de restituição.
hl.z
A34) Duas partículas, cujas massas são m e
2m, possuem quantidades de movimentos
iguais a Q e Q/2 e se movem em direções
perpendiculares entre si. Estas partículas se
chocam e invertem seus valores de quantidade
de movimento. Determinar a perda de energia
mecânica devido ao choque.

A31) Um disco com massa de 5kg, movendo- A35) Três esferas de raios iguais, porém de
massa diferentes, estão suspendidas por fios
se a 2m/s, aproxima-se de outro disco idêntico
cujas longitudes são iguais, de maneira que se
estacionário sobre uma superfície de gelo, sem
atrito. Após a colisão, o primeiro disco passa a encontram em contato e formam uma linha
reta. A esfera de massa m( é desviada de tal
se mover com uma velocidade Vi a 30° com a
modo que se eleva até uma altura H e se solta.
direção original de seu movimento; o segundo
Que massas m2 e m3 devem ter as outras
disco passa a se mover com velocidade v2 a
esferas, para que depois das colisões da
60°, conforme mostra na figura.
primeira com a segunda e da segunda com a
terceira, as quantidades de movimentos das
três esferas sejam iguais? A que altura se
elevarão? Considerar que todos os choques
v = 2m/s \ 30°
5kg são perfeitamente elásticos.
60°

H
f
O tipo de colisão e os valores de v3 e v2 estão A36) Uma bola é solta de uma altura h acima
indicados no item: do patamar de uma escada e salta para baixo,
a) 1,73 m/s - 1 m/s - inelástica. sobre os degraus do lande de escada.
b) 1,73 m/s - 1 m/s - elástica. Designando por e o coeficiente de restituição,
c) 1 m/s - 1,73 m/s - elástica. determine o valor de h para que a bola salte
d) 1 m/s - 1,73 m/s - inelástica. sempre à mesma altura acima de cada degrau.
e) 2 m/s - 1,73 m/s - inelástica.

A32) Uma partícula, com velocidade escalar


inicial v0, colide elasticamente com uma
segunda partícula estacionária e é desviada
num ângulo a. N velocidade escalar depois da
colisão é v. A segunda partícula recua, com
velocidade fazendo um ângulo <9 com a direção
inicial do movimento da primeira partícula.
Mostrar que tan0 =—^in-a—.
v0-v.cosa

335
Elementos Ha Física-Mecânica!-Colisão Mecânica
A37) Uma bola de aço é largada em A, bate na A42) Duas esferas de massas mi e m2
placa de aço rígida e salta para o ponto C. movem-se em direções perpendiculares com
Sabendo que o coeficiente de restituição é velocidades v, e v2 respectivamente. As duas
0,80, determine a distância d. esferas colidem e como resultado passam a se
______________ A
mover grudadas. Determine a quantidade Q de
calor liberada pela colisão.
""O----- v

i-
1,2 m

A43) Uma esfera atinge obliquamente um solo


15°/ horizontal com velocidade v e salta do plano
com velocidade v\=vyÍ2/2. Determinar o
I* d ângulo de incidência a e o ângulo de reflexão
p, se o coeficiente de restituição do choque é e
A38) Uma partícula de massa m, movendo-se = V3/3 .
com velocidade v, colide frontal e
elasticamente com outra partícula de massa M. A44) (OBF-06) Uma partícula alfa (núcleo de
Mostre que, se M está em repouso e é muito
hélio), movimentando-se na direção e sentido
maior do que m, a variação relativa da energia do semieixo horizontal positivo, colide contra
cinética de m é JE</EC « - 4(m//W) um núcleo de oxigênio em repouso. Após o
choque, cada constituinte do sistema, partícula
A39) Verificou-se experimentalmente que, na alfa e núcleo de oxigênio, passam a se mover
colisão frontal de duas esferas sólidas, tais com direções respectivamente iguais a 53° e a
como duas bolas de bilhar, as velocidades 300° em relação à direção e sentido do
depois da colisão são relacionadas com as de deslocamento anterior da partícula alfa. Se a
antes pela expressão v’f — v'2 = — e(v, - v2) massa do núcleo mN de oxigênio é 4 vezes
onde e tem valor entre zero e um e é chamado maior que a da partícula alfa mP, determine a
coeficiente de restituição. Esse resultado foi razão entre os módulos da velocidade vP da
descoberto por Newton e tem validade partícula e a do núcleo de oxigênio, vN, após a
somente aproximada. Ademais, a quantidade colisão.
de movimento é conservada na colisão. Prove
o seguinte: A45) (OBF-09) Uma pequena esfera metálica
a) as velocidades depois da colisão são dadas de massa m foi abandonada juntamente com
por uma bola de borracha de massa M, esférica,
v’? = [vf(mf - m^) + v2m2(1 + e)]/(m1 + m2) e de raio R, conforme a figura.
v2 = [1/7/777(1 + e) + v2(m2- m1e)]/(m1 + m2)
b) a variação de energia cinética do sistema é
- (1 - e2)m;m2(v7 - v2)2/2(m7 + m2)
g
A40) Um corpo atinge o solo sob um ângulo a,
com a vertical, e com uma velocidade v, e
ricocheteia segundo um ângulo fl, e velocidade
v’. Se e é o coeficiente de restituição do A massa M é muito menor que meo volume
choque, prove que da esfera metálica é desprezível quando
tg (J= (1/e)tg a e v’= v(e2cos2 a+ sen2 a)1/2. comparado ao da bola de borracha.
Considerando que: os movimentos dos centros
A41) Mostrar que numa colisão perfeitamente de massa da esferinha e da bola estão sempre
elástica entre duas partículas de massas na mesma vertical; o sistema se choca contra
iguais, com uma delas inicialmente em o solo e todos os choques envolvidos são
repouso, a energia transferida à partícula perfeitamente elásticos; a distância na vertical
inicialmente estacionária é (sen2 0)EO, onde Eo percorrida pela esferinha é muito maior que a
é a energia inicial e 0 é o ângulo de desvio da deformação da bola de borracha; é desprezível
partícula incidente. a resistência do ar em questão, determine:

336
Elementos da física-Mecânica!-Colisão Mecânica
a) A velocidade aproximada com que a perfeitamente inelástica? Compare essa
esferinha se separa da bola na subida. velocidade com aquela obtida no item (a).
b) A distância vertical percorrida pela esferinha Explique o resultado.
na subida em função da distância percorrida
pela mesma, na descida. A49) (OBF-07) Em uma mesa de bilhar estão
dispostas três bolas idênticas de massa m =
A46) (OBF-12) Uma bola com coeficiente de 200g, em repouso. Um jogador toca a bola 1
restituição e é atirada com uma velocidade com um taco, exercendo sobre ela uma força
horizontal v0. Determine a que distância, d, a de módulo F = 5N, paralela à superfície da
bola atinge o ponto P. Despreze a resistência mesa. Após 0,2s da aplicação da força, a bola
do ar. 1 atinge a bola 2. Observa-se que, após o
choque, as bolas 1 e 2 passam a mover-se em
direções que formam 60° e 30°,
respectivamente, com a direção original da
H bola 1. Após certo tempo a bola 2 colide com a
bola 3 e fica em repouso, enquanto a bola 3
P segue em direção à caçapa, atingindo-a em
0,3s. Considerando os choques elásticos e
<- d
desprezando o atrito entre as bolas e a
superfície, pergunta-se:
A47) (OBF-13) Com base na figura, as duas a) A velocidade da bola 3 ao atingir a caçapa;
esferas à direita estão inicialmente em repouso b) A distância percorrida pela bola 3 até atingir
e a esfera da esquerda incide sobre a do a caçapa.
centro com velocidade v0.
Vo A50) (OBF-06) A figura representa um vagão
-------------- ►
A, em repouso, que contém em seu interior um
automóvel B, também em repouso. As massas
de ambos são iguais, os freios do automóvel
1111 1111 ui? estão soltos e pode-se considerar que para
Supondo que as colisões sejam frontais e esta situação não há atritos apreciáveis entre
elásticas, mostre que se mu > m2 há duas B e A. Num instante qualquer o vagão A é
posto em movimento retilíneo com velocidade
colisões m, < m2 há três colisões.
igual a 1,00m/s e, após alguns instantes,
A48) (OBF-07) Dois carrinhos A e B idênticos ocorre uma colisão entre a parede do vagão
estão ligados rigidamente por uma barra e contra o para-choque do automóvel.
juntos (carrinho A + carrinho B + barra) têm a Considerando que o coeficiente de restituição
massa de 4 kg. Dois carrinhos C e D, de ao choque devido às propriedades das
massas Mc = 2 kg e Mo = 2 kg, são colocados paredes do vagão e as dos para-choques do
em repouso entre os carrinhos A e B , a iguais automóvel é igual a 0,50,
distâncias.
barra rígida

d d d
Sabendo que a velocidade de A e B é de 3 m/s
para a direita, e considerando que o atrito
entre as rodas dos carrinhos e o solo é
desprezível, responda:
a) Se a colisão entre A e C for perfeitamente
inelástica (C fica “grudado” em A) e, entre C e
D for perfeitamente elástica, qual será a
velocidade final do sistema sabendo que a
a) calcule a velocidade do automóvel
colisão entre D e B também será perfeitamente
relativamente ao vagão imediatamente após a
inelástica (D fica “grudado” em B)?
primeira colisão entre eles.
b) Qual a velocidade final do sistema
b) Choques do automóvel B contra as paredes
considerando a colisão entre A e C
do vagão A se sucederão, ora de um lado, ora
perfeitamente inelástica, e entre C e D também
de outro. Após um número muito elevado de

337
[lementos da Física - Mecânica /- Colisão Mecânica
colisões, calcule, relativamente ao solo, para poste ao local onde a bala atinge o solo. Que
quanto tenderá a velocidade do automóvel B. parte da energia cinética inicial da bala é
perdida como forma de calor? Utilizar g = 9,8
A51) (OBF-04) Uma partícula de massa m, m/s2.
localizada sobre uma plataforma horizontal de
altura H = 50 cm é lançada com uma
velocidade ü0 e atinge, a uma altura h = 5 cm, t
uma cunha de massa M = 5m, que pode 5 m
deslizar sem atrito numa superfície horizontal.
A colisão é perfeitamente elástica e a partícula
é rebatida da cunha com uma velocidade 20m >
horizontal, enquanto que a cunha adquire -------- — x *
velocidade V = 6 m/s para a direita. Determine
o módulo de ü0 e a distância d, a partir da
A55) (IPhO-91) Na figura, uma esfera de raio
base da plataforma, que a partícula atinge o R está girando com velocidade angular
solo. constante sobre um eixo horizontal. A esfera é
m Ua abandonada de uma altura h, medida desde
seu ponto mais baixo até a superfície
horizontal. Depois de chocar-se com o solo, a
M
esfera acaba subindo uma altura ah, também
medida desde seu ponto mais baixo até a
H
V superfície horizontal. Considere que a esfera
ficou um breve intervalo de tempo em contato
com o solo antes de subir devido ao choque. O
coeficiente de atrito entre a esfera e o solo é p.
d^
Considere g como sendo a aceleração da
gravidade.
A52) (OBF-04) Uma bola de massa m com a) Calcule a tangente do ângulo de reflexão 0;
velocidade v sofre uma colisão elástica frontal b) Calcule o alcance horizontal x do movimento
com outra de mesmo tamanho, mas de massa entre o primeiro e o segundo choque da esfera
M, inicialmente parada. A bola de massa M, com o solo.
tendo adquirido momento, choca-se em
seguida, frontal e elasticamente, com outra
bola idêntica àquela que a atingiu, também
inicialmente parada. Qual deve ser o valor de
M para que ao final dos dois choques a última
bola tenha velocidade igual a 8/9 da
velocidade que a primeira bola tinha antes dos /vV------- “,h
choques? Considere a superfície em que se
encontram as três bolas, plana e de atrito
desprezível.
JD...O x >

A53) (Seletiva OIF-02) Considere uma colisão


frontal completamente elástica entre uma bola
de massa m e outra de massa M, que se A56) Um mesatenista usa sua raquete para
encontra em repouso. Calcule a energia que a bater em uma bolinha de tênis de mesa. Antes
bola de massa m transfere à bola de massa M, do choque, tanto a bolinha quanto a raquete
em função da razão das massas k = m/M. possuem movimento apenas vertical, com a
bolinha descendo e a raquete subindo. Devido
A54) (IPho-67) Uma bola de massa M = 0,2 kg ao choque a bolinha atinge uma altura de 40
repousa sobre um poste vertical de altura h = 5 cm em relação ao ponto de impacto. Suponha
m. Uma bala de massa m = 0,01 kg, viajando a que o choque é parcialmente elástico com
uma velocidade de v0 = 500 m/s na horizontal, coeficiente de restituição igual a 0,5. Supondo
atinge e atravessa o centro da bola. A bola que a massa da raquete é muito maior que a
choca-se com o solo a uma distância de d = 20 massa da bolinha, determine a velocidade da
m do pé do poste. Determine a distância x do raquete imediatamente antes do choque.

338
fíementos da física - Mecânica /- Colisão Mecânica
a) 8,4 m/s b) 2,8 m/s c) 5,6 m/s A59) Uma partícula de massa m, que se move
d) 11,2 m/s e) 16,8 m/s com velocidade v = V3 m/s, choca-se
elasticamente com outra partícula em repouso,
A57) Uma pista de skate possui o perfil
cuja massa é m/2, passando a ter um
mostrado na figura. O skatista A, de massa 60
movimento que forma com a direção inicial um
kg, parte do repouso do ponto 1 para fazer a
ângulo de 30°. Determine o ângulo p com a
sua apresentação. A altura do ponto 1 é 1,8 m
horizontal.
e a aceleração da gravidade vale 10 m/s2. A
manobra de A, no entanto, tem uma falha, e o
skatista acaba por colidir com um fiscal B, de
massa 60 kg, que está no nível do solo. O
,x^30°
fiscal B observa A chegando, tenta fugir a 1
m/s para a direita, mas é atingido e ambos,
O m/2
skatista e fiscal, caem no chão, deslizando até
parar. Sabendo-se que o coeficiente de atrito
cinético médio entre as roupas de A e o chão é
0,4 e entre as roupas de B e o chão é 0,5 e
que após o repouso A distava 200 cm, a) p = 60°. b) p = arctg—. c) p = 45°.
aproximadamente, de B, podemos afirmar que 6
o coeficiente de restituição associado à colisão
d) p = arc tg — . e) p = 30°.
de A e B vale: 4
A
1
v0a=o A60) No dispositivo abaixo temos, sobre um
plano horizontal, dois discos, de dimensões
desprezíveis, que podem movimentar-se sem
1,8 m atrito sobre o plano. Os discos estão presos,
, 200 cm
B >4----------------------
por fios inextensíveis, a pontos fixos do plano.
: a : B
O disco 1 possui massa m e o fio que o
2 3 4 mantém preso ao plano possui comprimento
a) 0,2 b) 0,4 c) 0,6 d) 0,8 e) 1,0 L,, enquanto que e o disco 2 possui massa 2m
e seu fio comprimento L2, onde 3L2 = 2Lv
A58) A figura mostra um bloco apoiado em Aplica-se um pequeno impulso sobre o disco 1
uma extremidade de uma barra de de modo que ele assume velocidade v,
comprimento L. O sistema se dirige em direção percorrendo uma semi-circunferência até
a uma parede perpendicular ao solo. atingir o disco 2, que inicialmente estava
Considere que mbarra = 3mb|0C0 e que a parado. Considerando que o choque é central
aceleração da gravidade local é g. O e que o coeficiente de restituição é e = 0,8,
coeficiente de atrito entre o bloco e a barra é p, determine quantas voltas dará o disco 2 antes
enquanto que não existe atrito entre a barra e de chocar-se com o disco 1 pela segunda vez.
o solo. Qual é o menor valor de v de tal forma
que depois do choque elástico da barra contra
a parede o pequeno bloco caia da barra?
Despreze o tempo de duração do choque da X ___ X
barra. L) l2

8l a)4 b) 1 c) 9 d) 6 e) 3

p _____ r
WS//SSS//SSSSSS/SS/7///%'.
V
A61) Uma haste rígida, de comprimento L e de
massa desprezível, possui presas em suas
extremidades duas massas, 2m e 3m,
conforme a figura. Este conjunto acha-se sobre
uma superfície horizontal perfeitamente lisa.

a)2J¥ b)
HgL
3
pgf
C)
2ngL
3
Uma terceira partícula de massa m e
velocidade v desliza sobre esta superfície
numa direção perpendicular à haste e colide
3ggL
d) e)
2 2 inelasticamente com a massa 3m, ficando
colada à mesma após a colisão. Sabendo que

339
Elementos tia física - Mecânica 1- Colisão Mecânica
a massa 2m tem velocidade nula em relação
superfície, imediatamente após a colisão,
determine a velocidade angular da haste após
a colisão.
Q2m

m Q\
v
O —► -K)3m
A)— B) — C) —
' 4L 2L 8L

D) — E) —
6L 4L

A62) Uma esfera B está presa a um fio A64) Os discos A e B, cujas massas são mA e
inextensível. Uma outra esfera idêntica A é mB, respectivamente, podem deslizar
abandonada desde o repouso, quando apenas livremente sobre uma superfície horizontal sem
tangencia o fio, e atinge a esfera com atrito. O disco B está me repouso quando é
velocidade v0. Supondo que a colisão entre as golpeado por um disco A que move com
esferas é perfeita elástica e que inexiste atrito, velocidade v0 em uma direção que forma um
determine a velocidade de cada esfera ângulo 0 com a linha de impacto. Se e é o
imediatamente após a colisão. coeficiente de restituição entre os discos,
demonstre que a componente n da velocidade
de A depois do impacto é:
ÕU a) positiva se mA < emB;
b) negativa se mA > emB;
c) zero se mA = emB.
A

í A

A63) Uma esfera colide contra uma parede fixa


sem atrito de formato irregular, como indicado
na figura. A incidência ocorre sob um ângulo 0
em relação à normal à superfície no ponto de
impacto. Sabe-se que a é o ângulo de reflexão
da esfera, também em relação à normal.
a) Demonstre que a > 0.
b) Demonstre que a porcentagem de energia
cinética dissipada é 100(1 - e2)cos2 0.

340
ENERGIA CINÉTICA DE ROTAÇÃO

No capítulo sobre energia mecânica, foi enunciado que a modalidade da energia associada
a movimento é a energia cinética, enquanto que a modalidade associada a posição é a energia
potencial. Quando uma partícula de massa m apresenta, em um determinado referencial, módulo
y2
de velocidade v, sabe-se que a energia cinética da partícula é ——. Suponha agora que um

ventilador de teto está girando com velocidade angular co. Como calcular a energia cinética deste
ventilador? O centro de massa do ventilador está parado em relação ao teto, porém a energia
cinética do ventilador não é zero, uma vez que as pás do ventilador possuem massa e velocidade
tangencial. Observe a figura abaixo, onde estão destacados três diferenciais de massas, m,, m2 e
m3, e suas respectivas velocidades tangenciais.

“ v3

------

v2 ss
Assim, a energia cinética de um corpo em rotação é dada por:

E - miv? , m2v2 , , "Vn .. y m,vi2


c 2 2 "■ 2 tT 2 '

onde Vi é a velocidade tangencial da partícula de massa m,. Entretanto, esta soma não é simples
de ser executada, pois V| não é igual para todas as partículas, dependendo de sua posição. Como
a velocidade angular de cada partícula é <o, pode-se escrever que V| = cop, onde r é a distância da
partícula de massa mj ao eixo de rotação do corpo. Logo, a energia cinética de rotação vale:

c -èí 2 & 2 2^ J

O termo entre parênteses depende de como a massa está distribuída em relação ao eixo
de rotação do corpo. Denomina-se esta grandeza de momento de inércia do corpo, sendo
representada pela letra I. O momento de inércia depende da geometria do corpo e do eixo em
torno do qual o corpo está rotacionando. Assim, pode-se afirmar que:

i=£miii2 c Ico2
e rot 2
1=1

Perceba que, no sistema internacional de unidades, a unidade de I é kg.m2. É possível


calcular o valor do momento de inércia, utilizando integral, dos corpos com geometria mais
comum, como corpos esféricos, cilindros, placas retangulares ou hastes, porém este cálculo foge
do objetivo deste livro. A seguir o leitor poderá verificar uma tabela com o valor do momento de
inércia de alguns objetos.

341
Elementos da Física-Mecânica!-Apêndice-. Dinâmica da Rotação

Exemplos de Momento de Inércia

Ri

Longa haste fina com


Aro ou casca o eixo de rotação que
Cilindro sólido Cilindro oco
cilíndrica^ = MR2 passa pelo fim
ou disfc^ = ±.\IR2 1 CM = l-Vf (á,2 + R} )
U/Z2

Longa Haste fina com Placa retangular Esfera sólida Esfera oca
eixo de rotação que
passa pelo centro. Icm = 1L.V/(rz2+Z>2 I CM = j.\ÍR2 1 CM = 1MR2
1CM = 1^’

TEOREMA DOS EIXOS PARALELOS

Sabe-se que o cálculo do momento de inércia de um corpo depende de sua geometria e do


eixo ao qual é desejado calcular o momento de inércia. A tabela do item anterior exibe o valor do
momento de inércia de alguns objetos em relação a um eixo que passa pelo centro de massa.
Entretanto, um corpo pode rotacionar em torno de eixo que não passe pelo seu centro de massa.
Neste caso, o momento de inércia do corpo é distinto do valor tabelado para o centro de massa.

O teorema dos eixos paralelos afirmar que o momento de inércia I em relação ao eixo AA’,
paralelo a um eixo que passa pelo centro de massa BB’ vale:

I = lCM + Md2,

onde M é a massa do corpo e d é a distância entre os eixos AA’ e BB’.

Por exemplo, pode-se aplicar o teorema dos eixos paralelos para calcular o momento de
inércia de um disco de raio R em relação a um eixo perpendicular a seu plano, passando por sua

342
_______________________________________ Mementos da Física-Mecânica!-Apêndice: Dinâmica da Rotação
borda. É conhecido que o momento de inércia de um disco de massa M e raio, em relação a um

eixo que passa pelo centro de massa do disco, é ICm = —• Portanto:

3MR2
I = lCM + MR2= MR2 =
2

torque

Suponha que uma partícula de massa m está girando, em uma trajetória circular, em torno
de um eixo perpendicular ao plano de rotação. Adote que o raio da trajetória circular é r. A
componente tangencial da força resultante é dada por:

F, = m.at = m.a.r,

onde a é a aceleração circular instantânea da partícula.

Sabe-se que o torque resultante sobre a partícula vale:

t = Ft.r = m.a.r2 = (m.ri^a

O termo mr2 é o momento de inércia do partícula em relação ao eixo de rotaçao. Logo, o


torque resultante sobre a partícula é igual a

t = Ia

Esta expressão pode ser estendida para um corpo rígido girando em torno de um eixo fixo,
uma vez que um corpo pode ser entendido como um conjunto de partículas.

Vamos agora aplicar o que já foi estudado neste capítulo para resolver uma das situações
mais clássicas da Física, que é a Máquina de Atwood, considerando agora que a roldana possui
massa e apresenta movimento de rotação.

mi[_|
m2

Suponha que a roldana seja um disco de massa M e raio R. O momento de inércia relativo

ao eixo que passa por seu centro é ——. O movimento das massas mi e m2 é apenas de
translação e o da roldana é apenas de rotação. Para que a roldana se mova é necessário que
exista uma força de atrito fa entre a corda e ela. Como a corda não desliza na roldana, esta força
é de atrito estático. Na situação clássica, em que a roldana não possui massa, as trações em cada
lado da corda são iguais, porém, considerando a massa da roldana e sua rotação, isto não ocorre
mais. As forças nas extremidades da corda não são mais iguais porque a roldana reage
exercendo uma força de atrito sobre a corda, de sentido oposto ao movimento dela relativo à
corda. Observe a figura a seguir para entender melhor a decomposição das forças que atuam nos
corpos.

343
Elementos da Física-Mecânica l-Apéntlice: Dinâmica tia Rotação
fa -fa
t2
,N Tx

P
Ti
T2
roldana corda Pi P2

As forças aplicadas à roldana são o seu peso P, a reação normal ao peso N exercida pelo
seu suporte, a força de atrito fa que a corda exerce sobre ela e a reação R à força de atrito
exercida pelo suporte da roldana. Como a roldana não possui movimento de translação:

N-Mg = 0 e fa - R = 0

Aplicando a segunda lei de Newton às massas (considere um eixo vertical com sentido
positivo para cima):

T, -01,9 = +01,3 e T2 -m2g —0I2 a

A segunda lei de Newton aplicada à corda dá:

T2-T,-fa =mca = 0

porque foi desprezada a massa da corda em relação às outras massas.

A equação t = Ia descreve o movimento de rotação da roldana em torno de um eixo que


passa pelo seu centro de massa. Tomando o sentido horário como sendo positivo, tem-se, para o
movimento de rotação da roldana:

faR = la = l^ , a
r = faRsen(n/2) = Ia => T2-T, =1.—

Subtraímos membro a membro as equações da segunda lei de Newton:

1.^ - (m2 - m, )g = -(m, + m2 )a m2 -m,


(L ~ T,) - (m2 - m, )g = -(m, + m2 )a a= 9
m, +m2 +(I/R2)

Como o momento de inércia da roldana é I = ^-


2

m2 -m,
a=
m, +m2 +(M/2) 9

Se compararmos esta aceleração com o valor da situação clássica (roldana sem massa e

sem rotação), que é a = IHulTi L verifica-se que a aceleração para o caso de considerar a
m, +m2)
massa e a rotação da roldana é menor que a do caso clássico. A razão disso é que a inércia do
sistema aumenta considerando a massa e a rotação da roldana e a presença da massa dela no
denominador indica isso claramente.

344
_______________________________________ Elementos da Física-Mecânica!-apêndice: Dinâmica da notação
TRABALHO, ENERGIA CINÉTICA E POTÊNCIA

mv2
Sabe-se que a energia cinética associada ao movimento de translação é ECBan, =——,
onde m é a massa do corpo e v sua velocidade, e que a energia cinética associada ao movimento
I 2
de rotação é Ecrot = —, onde I é o momento de inércia do corpo e co sua velocidade angular.
Desta forma, um corpo que possui simultaneamente movimento de translação e de rotação
apresenta energia cinética total igual a

mv2 Ico2
*-c total ~

Para o caso de um corpo que está apenas em rotação, o trabalho de uma determinada
força F pode ser calculada como (lembre-se que apenas a componente tangencial da força realiza
trabalho sobre um corpo que está em rotação pura):

W = jF,dr = |Ftrd0 w = Jtde

Se o torque for constante, segue que W = t(ef -0O)

Assim, se um corpo possui simultaneamente movimento de translaçao e de rotação, o


trabalho de uma determinada força é

W = |Fdr + |Td0

Caso a força e o torque sejam constantes:

W = F.d.cos0 + t(0f —0O)

O teorema do trabalho energia também é válido para um corpo rígido em rotação. Se o


movimento for apenas de rotação (co * 0 e v = 0), segue que:

W = Ecrot( Ec rot 0

Se o torque for constante: r(0f -0o) = -y--~^-

Caso ocorra movimento conjugado de rotaçao e translação, o teorema do trabalho energia


fica da forma:

W = Ecf-Ec0

Se o torque e a força forem constantes:

r . A za a \ mv? mvo '“o


F.d.cos0 + T(0f-0O) = -y~+~^ Y~~2

A energia mecânica também pode ser para o caso de um corpo rígido em movimento de
rotação e translação:

345
Mementos tia física-Mecânica i-âpéntiice: Dinâmica tia Rotação
_ mv2 Ico2 ,__ l,
Em “ Ec + Ep — Ec trans 4" Ep rol + EP= —+ —+mgh

Na ausência de forças dissipativas, tem-se a conservação da energia mecânica:

mv? Ico? . mv? Ico? .


_o. + -_o. + mgh0 =_L + _L + mghf

A taxa de realização de trabalho com o tempo é a potência liberada pela força:

_ dW d0
P =------- = T----- = TCO
dt dt

Uma aplicação prática da conservação da energia mecânica de corpo rígido em translação


e rotação é o movimento de um cilindro, de massa m e raio R, sobre um plano inclinado áspero de
altura h. Note que é necessário que exista atrito entre o cilindro e o plano inclinado, pois senão o
cilindro não apresentaria movimento de rotação. Além disso, essa força de atrito é estática, uma
vez que não existe movimento de arrasto entre os corpos.

A decomposição das forças está na figura acima. A normal ao plano inclinado e a


componente do peso, perpendicular ao plano inclinado, não realizam trabalho sobre o corpo. A
força de atrito está aplicada no eixo instantâneo de rotação do corpo, onde a velocidade dele,
relativamente ao plano, é nula; portanto, como não há deslocamento do corpo em relação ao
plano, o trabalho da força de atrito é nulo. Assim, a única força que realiza trabalho sobre o corpo
é a componente do peso paralela ao plano, que é conservativa. Pela conservação da energia
mecânica:

mv2 Ico2 mv? Ico? , mv2 Ico2


EmO — Emf => _o + + mgh0 = —L +-A- + mghf mgh = —+ —

mR2
Sabendo-se que I = —— e v = coR:

. mv2 1 mR2 v2 V2 V2 3v2


mgh = -—- + — ------- T => gh=-+- => gh = —
2 2 2 R2

346
Exercícios de Embasamento
E1) D E2) B E3) E
E4) E E5) B E6) C
E7) C E8) E E9) 40.103N
E10) C E11) C E12) E
E13) B E14) E E15) 1,5 N
E16) A E17) c) 2,5 m/s2 E18) A
E19) B E20) C E21) C
E22) D E23) E E24) D
E25) A E26) E E27) C

Exercícios de Fixação
F1) B F2) A F3) D
F4) a) (Ma + MB)a; b) ^M^g2 +M|a2 ; c) tg 0 = a/g F5) D
F6) FVFW F7) D F8) D
F9) C F10) A F11) D
F12) C F13) A F14) E
F15) 4 m/s2
F16) a)a = p = 0;b)a = 0ep = 5 m/s2; c) a = 5,0 m/s2 e p = 15 m/s2
F17) 5,0 m/s2 e 30N F18) E F19) 1,2 cm,3
F20) D F21) B
F22) a) a = 2 m/s2; b) T = 15 N; c) T = 24 N F23) A
F24) D F25) B F26) E
F27) B F28) A F29) D
F30) 3,2 s F31) A F32) D
F33) 2^2 kgf F34) are sen 2/3 e 20 N F35) 9N
F36) a) 720 N/m; b) aA = 0, aB = 5,0 m/s2 e ap = 2,5 m/s2; c) 40 N F37) E
F38) C F39) 480 N F40) 5N
-2x
F41) —g

Exercícios de Aprofundamento
A1) D A2) D A3) 15,7 s
A4) T = - •v=//2
24(//g)
a( = (m2 - m3)2g/(m22 + m2 + 6m2m3), a2 = (m2 - 4m3}gl(m2 4m2m3) e
A5) a3 = (m2 - 4m2)gl(m2 + 4m2m3)
'2(H-h)(2M + m) , h I 2M + m A7) (m, - m2)2g/(m, + m2)
A6) t =
mg 2mg(H-h)
A8) 15° A9) A A10) D
aíov a (m1m2 + m2m3 - 4m1m3 )g
A11) E A12) a2 ------------------------1---------
' ' m,m2 + m2m3 + 4m1m3
A13) a) aA + aB = 2ac; b) aA = 4 m/s2, aB = 6 m/s2, ac = 5 m/s2; c) 60 N
A14) aA = 1,51 m/s2, aB = 6,04 m/s2, TA = 90,6 N e TB = 22,6 N
A15) a) aA = 12,11 m/s5 e aB = 0,831 m/s2; b) 74,8 N
.. 6r|h
A16) — A17) a, = a2 = g
r] + 4
8g
T=
A18) a) aA = 4aB; b) aA = 8 m/s2 e aB = 2 m/s2; c) 6 N A19) 1 _i_ + 1
m, m2 m8
M+m ,
A20) g/33 A21) mB ---------- kx
mg-kx

347
Mementos da Física-Mecânica!-Gabaritos
. 2M
A22) a) f = ; b) fmax = 1 +------------
2 M2 -M,
A23) 8 = 0 ou e = 90° e T = 150 N A24) 800 N
A25) 2s A26) a) aA = 240/41 m/s2 e aB = 192/41 m/s2
2x/3
A27) a) abx = —— g, ab) = 9 e ap = Ai;b)VHãE;c) füT
íb) A28) t =
12472 d
9 ,y 3 9 3 V g V 195 g
a. cos a mgsenOcosO
A29) 1/6 A30) tg6 = A31)
g + a.sena mcos2 0 + Msen2 0
m,sena + m2 senp
A32) 2a sen A33) A34) A
m, cosa + m2 cosp ’’
A35) E A36) 55 N
_ m,(m,+m2+M)g _ m,m2g
e a2 =
' m^m, +m2 +M) + m2(m1 +M) m^m, +m2 +M) + m2(m, +M)
A38) E A39) E A40) D
A41) A A42) A A43) C
A44) 4 m/s2 A45) C A46) 22,5m/s2
+[m2(m3 -mj-mf]
A47) C A48) g
(m, + m2 )(m, + m3) + n^m.,
mg sen 0 cos 0
A49) g/2 A50)
M + msen2 0

Exercícios de Embasamento
E1) B E2) E E3) WFFF
E4) C E5) D E6) C
E7) A E8) D E9) A
E10) C E11) D E12) B
E13) E E14) E E15) A
E16) 0,75 E17) WFV E18) 42 N
E19) 9,0 N E20) b) peFM/g - m0 E21) 02
E22) E23) a) 2,5 N; b) >/3/6 E24) B
—. ■ 2^3 . ^3
E25) 720 me 15000 kg E26) a) ; b) E27) a) 1.2.104 N;b) 4,0 m/s2
' ' 3 ’ 3
E28) D E29) B E30) C
E31) C E32) A E33) C
E34) B E35) B E36) a) 10/3 m/s2; b) 894 m

Exercícios de Fixação
F1) C F2) a) 0,5 m/s2; b) 4,0 m/s2
F3) Tac = 60 N, Tbde = 30 N e Fat = 10 N F4) WFFF
F5) 2,5 m F6) D F7) 01
F8) 04 F9) 6,6 N para cima do plano inclinado
F10) A F11) B F12) B
F13) a)100N; b) 0,448 F14) B F15) B
F16) D F17) C F18) B
F19) A F20) E F21) A
F22) E F23) E F24) A
F25) a)1,5s; b) 5,6 m F26) (V3-V2)/(V2+1)
F27) tg a = n, F = (m.g.p) / 7l + p7
:

F28) a) 0,6g; b) 22,5 kg; c) 0,7g para m2 e m3, 0,4g para mít 88,2 N
F29) F = mg(tg 6 + p)/(1 - p.tgO) F30) v= vod1l(d-di)
F31) A F32) 60° F33) C

348
Elementos da Física-Mecânica !-Gabaritos
F34) I2gh M~m(sene + ílCosB)
/2gh F35) A F36) B
V M+m
F37) pe = 0,6 e pc = 0,5 F38) a) 1,39 m; b) 0,56 s; c) 0,76 s; d) 3,69 m/s
F39) 3,54 kg < m2 < 10,6 kg F40) a) aA = 0,698 m/s2 e aB = 0,233 m/s2; b) 79,8 N.

Exercícios de Aprofundamento
A1) 25 kgf A2) 4075 kg A3) E
A4) B A5) D A6) B
A7) E A8) A
A9) a) 2,26 m/s2; b) 2,3 N; c) T = 0, com os corpos se movimentando em contato um com o outro.
A10) Fat = [M/(M + m)]Fcos a - mg sen a A11) / = 2/,/2/(/,+ /2)
A12) (R + r)gp/r A13) g/3 A14) D
A15) A 15-73
A16) 5- m/s2
8
A17) a) arc tg 3/4; b) 21 mg; c) 84/107 A18) A
. kx - mag(sen9 + pcos9) . . mag(sen9 + gcos9)
lyi __ ) uy
ma + mb k
A20) t = -J21M/[F- p.g(M + m)] F> /j(M + m)g
sena + p. cosa
A21) P = arc tg p F = mg A22) 9N
7W
A23) empurrando para a direita a menor aceleração é 9(1-M)
1+p
A24) F = 2mg(1 + p.cos 9 + sen 9)cos (rt/4 - 9/2) A25) B

A26) 0,6 A27) A28) D

A29) 156,4 N A30) b) a = g(2p + 1)^ A31) a) 1,0 m/s2; b) 0,3 s


A32) Á —
Arr = í:__________________ 2/>(»>, +|,,;)_________________
' V g(scna + se>i/J) [mjíênã- pcosa)-iii,(scnp + zrcosp)]
A33) a) 0,986 m/s2; b) 51,7 N; c) 1,794 m/s2; d) 58,2 N.
A34) a) 1,96 m/s2; b) 39,1 N.
A35) a) 33,6 N; b) aA = 4,76 m/s2, aB = 3,08 m/s2, ac = 1,401 m/s.2:
2íMm
A36) a) 60 cm; b) p = arc tg 3/16 A37) Fal =
(M-m)t2
1 2.h.cosa
A38) A39) 0,33 s A40) D
g. senp.sen(p-a)
A41) 0,45 m/s2 A42) D A43) B
A44) D A45) D A46) E

Exercícios de Embasamento
E1) C E2) E E3) E
E4) b) 6,0 m/s E5) D E6) C
E7) b) 0,80 Hz E8) C C C E C E E E9) A
E10) 56,4 m/s E11) 04 E12) 01
E13) a) 8000 N; b) 2000 N E14) CC E15) 96 m
E16) a) 8 m/s; b) 128 N E17) B
E18) a) 4 m/s; b) 0,8 m/s2; c) 551 N e 756 N
E19) a) 3,3 rad/s; b) 3 s; c) 3 rad/s; d) 4,1 rad E20) A
E21) C E22) B E23) A
E24) A E25) D E26) v2/gR
E27) 240/n rpm E28) 1 rad/s E29) a) 12,5 N; b) 7,5 N

349
Elementos tia Física-Mecânica!-Gabaritos
Exercícios de Fixação
F1) E F3) a) 10 rad/s; b) 7,2 s F4) a) 6 m/s; b) 100 N

F5) CECCCE D6) a)


#b) e o peso aparente diminuiu

F7) b) 4,2 s; c) Mo > 180^2 kg F8) a) 200 N; b) 0,40 m


F9) b)V3Õ rad/s F10) D F11) A
F12) A F13) B F14) D
F15) E F16) D F17) B

F18) B F19) pest F20) C


gR
F21) b) VgRtgO F22) a) T, = 2,4 N, T2 = 2,0 N e T3 = 1,2 N; b) corda da bola 1
£cos0
F23) a)
1 g .
: b) 2rr.
fcosO ’
F24) R =
mv2
4
g 4 k 2

Exercícios de Aprofundamento
___________ D2___________
A1) mg cos 9 + A2) 9,9 m/s'.2
2£cos:9| L(l-cos9) + Z>tg9]
A3) a) 2,5 Mg; b) 2; c) 2,5 voltas/s A4) B
A5) D A6) 7,1 cm A7) cos 0 = 3g/2w2/
- —, -»•
A8) T5up = —^g+—w2lj
m( 3 2, , _ VÕmC w2 l~ gtga
inf 2 ' -g A9) w =
4 VLsena + d
jR cos 9-sen 9
A10) a)/? = U(/()t-4^2m/2)b) T= ArrmkLofVfk - A^mf2) A11)
cos9 + p.sen9
- 1 g(sen9 + p.cosO) g(sen9-p.cos9)
A12) a) — b)—
' 2rt \ R(cos0-p.senS) 2rt \ R(cos0 + p.senO)
A13) /J = (cos á)(g - >v2/?sen a)/(g.
_.sen a+ w2Rcos2 a)
A14) / = (kl0 — mgcos ci)i(k - nmirsen2 á)
(p.sena + cosajg
A15) l = A16) F = m-^g2 + <o4í2 sen2 a
sena(sena + gcosa)w2
r0(k2 -2kma>2)
A17) A18) A
k2 + (ma2 )2 - 3kmra2

A19) a) ^/gLsenOo tg0o ; b) mgsen60 ------------------------ 1 A20) a) 62,01 m; b) 72 m/s


gLsen6otg0o
A21) B A22) 6,65 N
g mb + M(a + b) kl0
A23) coscp = A24) A25) E
co2 mb2 + M(a + b)2 2n(k-mf2)
A26) B A27) D A28) E

A29) B A30) -^-1 A30) E


meo
A31) A A32) C A33) 5 m

A34) B
1
A35) -1 V3mg
R\3j3
R\3V3 3

350
Flementos da Física-Mecânica !-Gabaritos

Exercícios de Embasamento
E1) C E2) B E3) B
E4) 70 J E5) A E6) C
E7) D E8) A E9) B
E10) E E11) E E12) C
E13) E E14) 30 m3/s E15) 7J
E16) 18 J E17) a) 2t + 3; b) 800 J E18) C
E19) E E20) 4.104W
E21) a) 20 m/s2; b) 33000 N; c) 1.32.106 W
E22) a) 5.0.103 m3/s; b) 2000 mm
E23) a)6,75.106J; b) 0,9 CV
E24) a)pe = 0,15 pic = 0,1; b)100J; c) 0 E25) E
E26) a)-2,4 J; b) 2 m/s E27) A E28) 8.105J
E29) C E30) B E31) C
E32) C E33) B E34) C
E35) 8,6% e 5,3.102kW

Exercícios de Fixação
F1) a) 400 N; b) 2 m/s F2) a) g/5; b) - 4/3 F3) D
F4) a) - 0,2 J; b) 2 m/s F5) 0,8 g/cm3; b) 243,9 J F6) a) 1000 W; b) 225 g
F7) 1,5 m F8) a) V3 ; b) 3; c) 3
lMogRo
F9) a) 27tMogRo; b) - 2npxmv02; c)
V Mm
F10) a) 200 s; b) 8000 N; c) 0,2 m/s2 F11) C
F12) B F13) B F14) A
F15) D F16) B F17) A
F18) C F19) C F20) pi = (P'2 - P2)1/2/P
F21) E F22) 0,4 m/s
F23) a) T = 4,5.10“ N; b) P = 7.5.105 W F24) B
F25) P, = P(sen a/pi + cos a - 1) F26) 3,96
F27) a) 8,575 kW b) 2573 J c) Sim. 257 W
F28) a) - picMgí; b) - kí2/2; c) 0; d) 0; e) - (picMgí + kí2/2); f) (1/pi2M2g2 + VqIcM -picMg)/k
F29) E F30) A F31) E

Exercícios de Aprofundamento
A1) VgL/2 A2) IVP = p.g.A[h2 - (h,2 + h22 + /j32)]/2
A3) a) P = 188,7 CV; b) P = 290,3 CV A4) - n(n - 1 )mgt/2
A5) W = (2m + 5pV/3)gH/2 = 2,9 kJ A6) 6,71 m
A7) a) 58,9 kW; b) 52,9 kW A8) pV
pV,(gh V,2/2S2)
t(gh + V/72S A9) 2,84 m
A10) 204 s A11) pi = 1/2e W = (V2-l)mga
A12) 1,981 m/s A13) a) 2.67.103 J; b) 1.19.103 J
A14) 10 m/s A15) 75 J e 5,9 m/s A16) mg(h/2-3a)
A17) 14,7 kW A18) b) mgh + mgípi A19) B
A20) a) 2.3.102 n; b) 3.4.102 J; c) 1 m, W,peso = -3,4.102J, W,tração = 2,3.102 J eW, = 1.1-102J
A21) D A22) B A23) E
A24) C A25) C A26) A
A27) mk4t2/8 A28) 32 W A29) a) 500 J; b) 10 m/s
A30) a) 300 N; 36000 J A31) B A32) -56J
A33) E A34) E A35) C
Í2
A36) E A37) E A38) - ■ gh • (4 - senO - pi • cos 0)
V5
A39) A A40) A A41) B
m1m2co2R2
A42)
2(m, +m2)

351
Elementos da Física-Mecânica!-Gabaritos

Exercícios de Embasamento
E1) FWFV E2) D
E3) a) 3,6.103 N; b) 87,5 me5,4.10sJ
E4) b) - 4,9 m/s; c) 1,2 J; d) 0,6 m E5) 34 m/s e 29 kJ
E6) a) 4 m/s; b) 0,6 m E7) D E8) E
E9) B E10) A
.2
E11) a) 375 J e 150 kg.m/s; b) 270 N; c) 5 m/s' E12) 0,5 m
E13) D E14) E E15) B
E16) E E17) C 2a(d-L)m
E18) Ax =
k
E19) E E20) C E21) B
E22) a) 10 m/s; b) 8 m/s; c) 40 cm E23) 4,1 m
E24) a) 3 J; b) 1 cm e 4 cm E25) 20 m/s
E26) a) - 1,2 m/s e 1,8 m; b) 20 m/s

Exercícios de Fixação
F1) B F2) B F3) D
F4) a) 3600 N; b) 1785,72 N F5) a) 2v10 m/s; b) 35 m F6) a) 7.10 6 m2; b) 20 m/s
F7) a) 1/7.105 N; b)-1,01.101°J
F8) a) 10 m/s; b) 4,25 m; c) 8,7 m F9) 19
F10) a) 5 m; b) 6 m; c) Não. F11) A F12) D
F13) D F14) A F15) C
F16) A F17) E F18) D
F19) A F20) B F21) B
F22) D F23) B F24) A
F25) E F26) C F27) B
F28) D F29) C F30) C
F31) B F32) 77gí/2 F33) C
R__ 'l
F34) a) ^SgR • b) hA = — F35) 0,25 m
2^1-p.cotO J

F36) a) 60 m; b) 50 m/s2 F37) a) 1 m; b)


10V3
m/s F38) 100 N/m
3
F39) a) NA = 6 N e NB = 4,5 N; b) 1,84 m F40) H-2pd
'8mgR
F41) F42) a) 7,4 cm b) O bloco para c) 72,5 %
k
F43) 7,59 m/s F44) a) 6,86 m/s; b) 2,33 m F45) b = h/2 e c = g
----- .. mg
F46) Tmáx - Tmi„ = 6mg F47) N = -^ 3h-2h, F48) 2,13 m/s
R 0 g

Exercícios de Aprofundamento
2
2mgH D .. ( D2
1+ : b) mg ——senO
A1) D A2) a) —k 2H l Zrlrx

A3) a) 2 m/s; b) 2,5 N; c) 5 N; d) 7,5 N A4) cos0 = —


3
A5) F F F F V F A6) C
A7) a) 2,37 m/s; b) 0,187 m; c) 0,485 m A8) E
A9) a) (3/5, 4/5); b) y = 39/40 A10) C

A11) 0 = arccos 2 H-1 , para r < h < 2,5r A12) A


3l r
A13) E A14) A A15) 0,33 m

352
Elementos da física-Mecânica!-Gabaritos
A17) v = ^gR(2 + 2cosa + ^l-|
A16) [2g(H-/) mg2/k]1'2

A18) d = 0,6/ A19) 3,32 m/s A21) 50R/27


A22) t = 47,4 m
h iA 33
h \i 3 2mgh í 3 + COSP^
A23) a) cosa ==—-7 ; |-—; b) a tensão aumenta de
| —+ cosp
t l
A24) a) 2L/3; b) vx = (16gL/27)i1/2 vy = (20gL/27)1/2 A25) 4 m/s
.— . -73 _
A26) a) ^g; b)
mg A27) vn = ./ (1 + 2.cos0 + 2.cos26)
' 2 T VCOS0
A28) C A29) D

A30) a) e ; b) 5mg; c) g/3 e 2g/3 A31) C

A32) D A33) - 1,2m J A34) 5 m/s


A35) 85,7% A36) cos a = 1/3 A37) 6 m/s
A38) B A39) D A40) 6 rad/s e 6 m

Exercícios de Embasamento
E1) 4L E2) 5,0 m/s E3) 16 cm
E4) C E5) D E6)
E7) 60 cm E8) FVFVF E9) E
E10) VFFVF E11) E E12) D
E13) 3800 N E14) A E15) 10
E16) 2,4 m/s E17) B E18) E
E19) C E20) 18 kg.m/s E21) 1,6 m/s e 12 N
E22) B E23) a) 0,6 m/s; b) - 361296 J
E24) a) 7,5 m/s; b) - 7,5 J E25) D E26) A
E27) 0,0119 E28) D E29) C
E30) B E31) B E32) A
m.U.cosa
E33) E E34)
M+m
Exercícios de Fixação
F1) 28 cm F2) 30 m/s F3) 6 cm
F4) 2010 N F5) E
F6) a) 4,0 e 2,0 kg; b) 1/2 F7) 720 N F8) A
F9) a) 100%; b) 0,675 F10) a) 30000 N; 1440 N F11) 0,75 Je 0,5 m/s
F12) r>2 10"%; b) 50 N.s F13) a) vx = 90 m/s, vy = 0,6 m/s; b) - 361296 J
F14) a) 0,2 m/s; b) 20 N. F15) a) 600 J; b) 20 m/s
F16) a) 3,0 s; b) 40m/s; c) 100J F17) a) (M2gp2)/(2.m,k2)
F18) 16 m/s F19) A F20) A
F21) G F22) C F23) E
F24) E F25) B F26) C
F27) 0 F28) C F29) D
F30) a) 14,1 m/s; b) 6,67 m/s F31) d 4A F32) L/5
F33) Não atracará

Exercícios de Aprofundamento
A1) (100, 50) cm A2) E
3xo . 9kx§
A3) A4) - 1,67 m/se 53,3 J
4 VM ’ 32pMg
A5) 54 J A6) C A7) D
A8) D A9) C A10) A
A11) D A12) D A13) C

353
Elementos da física - Mecânica!- Gabaritos

A14) NULA A15) C

l í Mv0 -mvr D.vr 1 /'MVo-mVf


A16) a) t' = - ; b) d =
a M+m v0 + vr 2a M +m
A17) a) 1000 m; b) (6600 m; 2235 m); c) (29696400)',1/2 A18) 10 m/s e 30°
A19) [2(M + m)(L + h)g/M]1/2
‘" A20) t,/t2 = 1/3 A21) v = (3g/7/2)1'2

vg 1-
A22) -5-
_mj
A23) 9,4 m/s (de trás), 10,4 m/s (da frente)
2g m + M)
' g-L
A24) 167 g A25) A26)
(m/M + 1).sen2a 8mg 4 8k
A27) h = (m + M).a.tg G/M A28) hm„ = h[MI(M + m)]2 A29) a) 1,75 N; b) 2,0 N
2m2>/2gr
A30) Vimax — A31) A cunha se deslocará à direita de 3,77 cm
rn-f + m2

[(5M + 4m)
A32) M y

I 2m2________ [3M + (2 + cos2 <p)m]Mm


A33) a) gRsen3q>; b) gsentp
(M + m)(M + m cos2 <p) (M + m cos2 <p)2
2
A34)
mg cosa 4V23+5V5 _
A35) -------------------R A36)
m2L2
2ksen2a 27 4M(M + m)h
v2 sen 2a 71 + rn'!
A37) + A38) C A39)
g l +mJ
A40) 20 cm A41) A A42) E
I 2gL ■
A43) a) m. ;b) A44) B
M(M + m) ’ ' M+m

1 m3g4 cot2 0
A45) E A46) - A47) D
8 k2 sen20
A48) B

Exercícios de Embasamento
E1) C E2) A E3) E
E4) A E5) A E6) a) 5 m/s; b) 4,5225 m
E7) A E8) 4.105N
E9) a) 6,4 J e 3,2 kg.m/s; b) 66 N E10) v’, = 0 e v’2 = 3,0 m/s
E11) C E12) a) 2 m/s; b) 1 m/s, para a esquerda; c) - 5 J; d) não
E13) a) 0,2 J; b) 20^2 m/s E14) a) 1 m/s; b) 57,1 N E15) a) 1 m/s; b) 1,5 N
E16) 19% e 36,2 m E17) a) 1 m/s; b)215988,8J E18) E
E19) B E20) D E21) B
E22) D E23) C E24) C
E25) D E26) E E27) C

Exercícios de Fixação
F1) FVFFF F2) FWW F3) A
F4) 23 m/s F5) a) 6.10"’2J;b) 0,03 m F6) a) 72 km/s; b) 0
F7) a) 0,8 s; b) 2,4 m; c) 6,0 m/s F8) 0,5
F9) b
F10) a) 3.1O20 kg.m/s; b) 4,5.1024 J; c) 5.10 -5 m/s; d)1,125.1O0 Mton
F11) E F12) C F13) a) 1600 J; b) 2000 J
F14) a)1,4m F15) B F16) B

354
Elementos da Física-Mecânica !-Gabaritos
F17) a) 72gL ; b) arc cos 3/4 F18) a) 4 m/s; b) se m = 2,5 g então M = 600 g
F19) a) 0,8 s; b) 4,0 s F20) D
F21) Iv/I = 2 m/s, direção horiz, da direita para a esquerda F22) D
F23) C F24) A F25) A
F26) D F27) B F28) 9,1 m/s
F29) E F30) a) A,/3k(M + m) / m; b) 2A^k / (M + m); c) 3MkA2/2m
F31) C F32) a) 2vo/5; b) Vq/3 F33) C
F34) C F35) A F36) D

F37) C F38) 2.96.103 N

F40) 0,8 F41) 20,6 m/s


F42) 1)e = 0,4 2) elástico 3) F = 3.103N F43) 0,2 m
F44) B F45) C
F46) a) v,’ = 0,6 m/s e v2’ = 2,4 m/s; b) 9 cm
F47) a) 0,8; b) 0,9 kg.m/s; c) 90 N
F48) H = e2h t= (2h/g)'l2(-[ + e)/(1 - e) F49) (2/3)2/
F50) h/n2 F51) a) v, (K/mx0)1/2 b)
b) vv22 = 2(K/mx00)) 1*
= 2(K/mx
F52) 2,0 cm F53) Q = (1/16)mg(3H-5h + 27Sh)

Exercícios de Aprofundamento

mk
A1) a) x & b) xJ-___________ . m kx2
• Q)_____________
! c)
Vm i + M),22 ’
y (m m+M t
M.) a) 8,0 m/s; b) 6,0 m/s; c) 1,8 m; d) 1,2 s A3) D
A4) a) (gR/2)1'2; b) R[(M, + M2)/M,]2/4
A5) a) 20 m/s; b) 1/8 s; c) 1,25 m
A6) vAx =1,6 m/s vBy =1,2 m/s
A7) a) 6V3.10’24 kg.m/s; b) 2,4.103 m/s A8) ECECCEC
A9) B A10) C A11) E
A12) D A13) C A14) D
A15) B A16) 14 m/s A17) a) 0,8; b) 2,048 m
A18) 636 g A19) a) v’a( = - 8/3 m/s e v’M = - 2/3 m/s; b) - 10/3 m

A20) 1 m/s e 13 m/s A21) h = tke " 2(M’ + M2 )9]


2M,g
A22) 12,8 m A23) 0 A24) 0,5 m
A25) D A26) A A27) B
A29) A A30) 1/2 A31) B
A34) AE = 3Q2/16m A35) m2 = m,/2 m3 = ith/6 H, = H/9 H2 = 4H/9 H3 = 4H
m1m2
A36) [h/(h + d)]1/2 A37) 1,47 m A42) Q = (v?+v^)
2(m, + m2)
A43) a = rr/6 e (3 = rr/4 A44) 4,35
(m-SIVh ... . fm-3MY
2 A46) v0^(1 + 2e)
A45) a) V2g(h-R)

A48) a) 1,5 m/s; b) 1,5 m/s A43) a)

A50) a) 0,5 m/s; b) 0,5 m/s A51) u0 = 17,85 m/s e 4,14 m


4k
A52) M = m/2 A53) AE = E0
(k + 1)2
A54) x=105m e Q = 1156,8 J
1
A55) a) 0 = arc tg J 1+ —L | ; b)x = 4hpa 1 + ~t= A56) B
k va J k va.
A57) D A58) C A59) B
A60) C A61) E A62) 0,721 v0 e O,693vo

355
Marcelo Rufíno de Oliveira nasceu em
1976 em Manáus-AM. Apesar de sua
família ser radicada em Belém, já residir
em 9 cidades, devido às transferências
de trabalho de seus pais. Ingressou err
1994 no Instituto Tecnológico dt
Aeronáutica (ITA), onde graduou-se em
Eng. Mecânica-Aeronáutica em 1999
Desde então trabalha, comc
coordenador e professor de matemática
e física, com turmas preparatórios para
concursos militares e olimpíadas
científicas em Belém. Em 2000 assumir
a coordenação regional da Olimpíada
Brasileira de Matemática no estado d(
Pará, sendo também responsável pela
organização da Olimpíada Paraense d<
Matemática. Em 2002 participou da
banca corretora da Olimpíada d<
Matemática do Cone Sul, realizada en
Fortaleza-CE.
A partir de 2007 se dedicou a escrevei
livros, de matemática e física, voltado:
para concursos militares e olimpíada:
científicas, lançando mais de um:
dezena de títulos, que têm fortalecido <
estudo de muitas gerações de alunos.
Aproveita suas poucas horas vaga:
para cultivar seus hobbies, como ouvi:
um bom rock n’ roll e viajar di
motocicleta com seus amigos do Par:
Moto Clube.

ISBN 978-859 93672-4

llllllllllll
788591 936724

Você também pode gostar